Download as docx, pdf, or txt
Download as docx, pdf, or txt
You are on page 1of 631

PNLE I for Foundation of

Professional Nursing Practice


Text Mode – Text version of the exam
1. The nurse In-charge in labor and delivery unit administered a dose of
terbutaline to a client without checking the client’s pulse. The standard that
would be used to determine if the nurse was negligent is:

A. The physician’s orders.


B. The action of a clinical nurse specialist who is recognized expert in the
field.
C. The statement in the drug literature about administration of terbutaline.
D. The actions of a reasonably prudent nurse with similar education and
experience.
2. Nurse Trish is caring for a female client with a history of GI bleeding, sickle
cell disease, and a platelet count of 22,000/μl. The female client is dehydrated
and receiving dextrose 5% in half-normal saline solution at 150 ml/hr. The
client complains of severe bone pain and is scheduled to receive a dose of
morphine sulfate. In administering the medication, Nurse Trish should avoid
which route?

A. I.V
B. I.M
C. Oral
D. S.C
3. Dr. Garcia writes the following order for the client who has been recently
admitted “Digoxin .125 mg P.O. once daily.” To prevent a dosage error, how
should the nurse document this order onto the medication administration
record?

A. “Digoxin .1250 mg P.O. once daily”


B. “Digoxin 0.1250 mg P.O. once daily”
C. “Digoxin 0.125 mg P.O. once daily”
D. “Digoxin .125 mg P.O. once daily”
4. A newly admitted female client was diagnosed with deep vein thrombosis.
Which nursing diagnosis should receive the highest priority?

A. Ineffective peripheral tissue perfusion related to venous congestion.


B. Risk for injury related to edema.
C. Excess fluid volume related to peripheral vascular disease.
D. Impaired gas exchange related to increased blood flow.
5. Nurse Betty is assigned to the following clients. The client that the nurse
would see first after endorsement?

A. A 34 year-old post operative appendectomy client of five hours who is


complaining of pain.
B. A 44 year-old myocardial infarction (MI) client who is complaining of
nausea.
C. A 26 year-old client admitted for dehydration whose intravenous (IV)
has infiltrated.
D. A 63 year-old post operative’s abdominal hysterectomy client of three
days whose incisional dressing is saturated with serosanguinous fluid.
6. Nurse Gail places a client in a four-point restraint following orders from the
physician. The client care plan should include:

A. Assess temperature frequently.


B. Provide diversional activities.
C. Check circulation every 15-30 minutes.
D. Socialize with other patients once a shift.
7. A male client who has severe burns is receiving H2 receptor antagonist
therapy. The nurse In-charge knows the purpose of this therapy is to:

A. Prevent stress ulcer


B. Block prostaglandin synthesis
C. Facilitate protein synthesis.
D. Enhance gas exchange
8. The doctor orders hourly urine output measurement for a postoperative
male client. The nurse Trish records the following amounts of output for 2
consecutive hours: 8 a.m.: 50 ml; 9 a.m.: 60 ml. Based on these amounts,
which action should the nurse take?
A. Increase the I.V. fluid infusion rate
B. Irrigate the indwelling urinary catheter
C. Notify the physician
D. Continue to monitor and record hourly urine output
9. Tony, a basketball player twist his right ankle while playing on the court and
seeks care for ankle pain and swelling. After the nurse applies ice to the ankle
for 30 minutes, which statement by Tony suggests that ice application has
been effective?

A. “My ankle looks less swollen now”.


B. “My ankle feels warm”.
C. “My ankle appears redder now”.
D. “I need something stronger for pain relief”
10.The physician prescribes a loop diuretic for a client. When administering
this drug, the nurse anticipates that the client may develop which electrolyte
imbalance?

A. Hypernatremia
B. Hyperkalemia
C. Hypokalemia
D. Hypervolemia
11.She finds out that some managers have benevolent-authoritative style of
management. Which of the following behaviors will she exhibit most likely?

A. Have condescending trust and confidence in their subordinates.


B. Gives economic and ego awards.
C. Communicates downward to staffs.
D. Allows decision making among subordinates.
12. Nurse Amy is aware that the following is true about functional nursing

A. Provides continuous, coordinated and comprehensive nursing services.


B. One-to-one nurse patient ratio.
C. Emphasize the use of group collaboration.
D. Concentrates on tasks and activities.
13.Which type of medication order might read “Vitamin K 10 mg I.M. daily × 3
days?”
A. Single order
B. Standard written order
C. Standing order
D. Stat order
14.A female client with a fecal impaction frequently exhibits which clinical
manifestation?

A. Increased appetite
B. Loss of urge to defecate
C. Hard, brown, formed stools
D. Liquid or semi-liquid stools
15.Nurse Linda prepares to perform an otoscopic examination on a female
client. For proper visualization, the nurse should position the client’s ear by:

A. Pulling the lobule down and back


B. Pulling the helix up and forward
C. Pulling the helix up and back
D. Pulling the lobule down and forward
16. Which instruction should nurse Tom give to a male client who is having
external radiation therapy:

A. Protect the irritated skin from sunlight.


B. Eat 3 to 4 hours before treatment.
C. Wash the skin over regularly.
D. Apply lotion or oil to the radiated area when it is red or sore.
17.In assisting a female client for immediate surgery, the nurse In-charge is
aware that she should:

A. Encourage the client to void following preoperative medication.


B. Explore the client’s fears and anxieties about the surgery.
C. Assist the client in removing dentures and nail polish.
D. Encourage the client to drink water prior to surgery.
18. A male client is admitted and diagnosed with acute pancreatitis after a
holiday celebration of excessive food and alcohol. Which assessment finding
reflects this diagnosis?

A. Blood pressure above normal range.


B. Presence of crackles in both lung fields.
C. Hyperactive bowel sounds
D. Sudden onset of continuous epigastric and back pain.
19. Which dietary guidelines are important for nurse Oliver to implement in
caring for the client with burns?

A. Provide high-fiber, high-fat diet


B. Provide high-protein, high-carbohydrate diet.
C. Monitor intake to prevent weight gain.
D. Provide ice chips or water intake.
20.Nurse Hazel will administer a unit of whole blood, which priority
information should the nurse have about the client?

A. Blood pressure and pulse rate.


B. Height and weight.
C. Calcium and potassium levels
D. Hgb and Hct levels.
21. Nurse Michelle witnesses a female client sustain a fall and suspects that
the leg may be broken. The nurse takes which priority action?

A. Takes a set of vital signs.


B. Call the radiology department for X-ray.
C. Reassure the client that everything will be alright.
D. Immobilize the leg before moving the client.
22.A male client is being transferred to the nursing unit for admission after
receiving a radium implant for bladder cancer. The nurse in-charge would take
which priority action in the care of this client?

A. Place client on reverse isolation.


B. Admit the client into a private room.
C. Encourage the client to take frequent rest periods.
D. Encourage family and friends to visit.
23.A newly admitted female client was diagnosed with agranulocytosis. The
nurse formulates which priority nursing diagnosis?

A. Constipation
B. Diarrhea
C. Risk for infection
D. Deficient knowledge
24.A male client is receiving total parenteral nutrition suddenly demonstrates
signs and symptoms of an air embolism. What is the priority action by the
nurse?

A. Notify the physician.


B. Place the client on the left side in the Trendelenburg position.
C. Place the client in high-Fowlers position.
D. Stop the total parenteral nutrition.
25.Nurse May attends an educational conference on leadership styles. The
nurse is sitting with a nurse employed at a large trauma center who states
that the leadership style at the trauma center is task-oriented and directive.
The nurse determines that the leadership style used at the trauma center is:

A. Autocratic.
B. Laissez-faire.
C. Democratic.
D. Situational
26.The physician orders DS 500 cc with KCl 10 mEq/liter at 30 cc/hr. The
nurse in-charge is going to hang a 500 cc bag. KCl is supplied 20 mEq/10 cc.
How many cc’s of KCl will be added to the IV solution?

A. .5 cc
B. 5 cc
C. 1.5 cc
D. 2.5 cc
27.A child of 10 years old is to receive 400 cc of IV fluid in an 8 hour shift. The
IV drip factor is 60. The IV rate that will deliver this amount is:

A. 50 cc/ hour
B. 55 cc/ hour
C. 24 cc/ hour
D. 66 cc/ hour
28.The nurse is aware that the most important nursing action when a client
returns from surgery is:
A. Assess the IV for type of fluid and rate of flow.
B. Assess the client for presence of pain.
C. Assess the Foley catheter for patency and urine output
D. Assess the dressing for drainage.
29. Which of the following vital sign assessments that may indicate
cardiogenic shock after myocardial infarction?

A. BP – 80/60, Pulse – 110 irregular


B. BP – 90/50, Pulse – 50 regular
C. BP – 130/80, Pulse – 100 regular
D. BP – 180/100, Pulse – 90 irregular
30.Which is the most appropriate nursing action in obtaining a blood pressure
measurement?

A. Take the proper equipment, place the client in a comfortable position,


and record the appropriate information in the client’s chart.
B. Measure the client’s arm, if you are not sure of the size of cuff to use.
C. Have the client recline or sit comfortably in a chair with the forearm at
the level of the heart.
D. Document the measurement, which extremity was used, and the
position that the client was in during the measurement.
31.Asking the questions to determine if the person understands the health
teaching provided by the nurse would be included during which step of the
nursing process?

A. Assessment
B. Evaluation
C. Implementation
D. Planning and goals
32.Which of the following item is considered the single most important factor
in assisting the health professional in arriving at a diagnosis or determining
the person’s needs?

A. Diagnostic test results


B. Biographical date
C. History of present illness
D. Physical examination
33.In preventing the development of an external rotation deformity of the hip
in a client who must remain in bed for any period of time, the most
appropriate nursing action would be to use:

A. Trochanter roll extending from the crest of the ileum to the midthigh.
B. Pillows under the lower legs.
C. Footboard
D. Hip-abductor pillow
34.Which stage of pressure ulcer development does the ulcer extend into the
subcutaneous tissue?

A. Stage I
B. Stage II
C. Stage III
D. Stage IV
35.When the method of wound healing is one in which wound edges are not
surgically approximated and integumentary continuity is restored by
granulations, the wound healing is termed

A. Second intention healing


B. Primary intention healing
C. Third intention healing
D. First intention healing
36.An 80-year-old male client is admitted to the hospital with a diagnosis of
pneumonia. Nurse Oliver learns that the client lives alone and hasn’t been
eating or drinking. When assessing him for dehydration, nurse Oliver would
expect to find:

A. Hypothermia
B. Hypertension
C. Distended neck veins
D. Tachycardia
37.The physician prescribes meperidine (Demerol), 75 mg I.M. every 4 hours
as needed, to control a client’s postoperative pain. The package insert is
“Meperidine, 100 mg/ml.” How many milliliters of meperidine should the
client receive?
A. 0.75
B. 0.6
C. 0.5
D. 0.25
38. A male client with diabetes mellitus is receiving insulin. Which statement
correctly describes an insulin unit?

A. It’s a common measurement in the metric system.


B. It’s the basis for solids in the avoirdupois system.
C. It’s the smallest measurement in the apothecary system.
D. It’s a measure of effect, not a standard measure of weight or quantity.
39.Nurse Oliver measures a client’s temperature at 102° F. What is the
equivalent Centigrade temperature?

A. 40.1 °C
B. 38.9 °C
C. 48 °C
D. 38 °C
40.The nurse is assessing a 48-year-old client who has come to the
physician’s office for his annual physical exam. One of the first physical signs
of aging is:

A. Accepting limitations while developing assets.


B. Increasing loss of muscle tone.
C. Failing eyesight, especially close vision.
D. Having more frequent aches and pains.
41.The physician inserts a chest tube into a female client to treat a
pneumothorax. The tube is connected to water-seal drainage. The nurse in-
charge can prevent chest tube air leaks by:

A. Checking and taping all connections.


B. Checking patency of the chest tube.
C. Keeping the head of the bed slightly elevated.
D. Keeping the chest drainage system below the level of the chest.
42.Nurse Trish must verify the client’s identity before administering
medication. She is aware that the safest way to verify identity is to:
A. Check the client’s identification band.
B. Ask the client to state his name.
C. State the client’s name out loud and wait a client to repeat it.
D. Check the room number and the client’s name on the bed.
43.The physician orders dextrose 5 % in water, 1,000 ml to be infused over 8
hours. The I.V. tubing delivers 15 drops/ml. Nurse John should run the I.V.
infusion at a rate of:

A. 30 drops/minute
B. 32 drops/minute
C. 20 drops/minute
D. 18 drops/minute
44.If a central venous catheter becomes disconnected accidentally, what
should the nurse in-charge do immediately?

A. Clamp the catheter


B. Call another nurse
C. Call the physician
D. Apply a dry sterile dressing to the site.
45.A female client was recently admitted. She has fever, weight loss, and
watery diarrhea is being admitted to the facility. While assessing the client,
Nurse Hazel inspects the client’s abdomen and notice that it is slightly
concave. Additional assessment should proceed in which order:

A. Palpation, auscultation, and percussion.


B. Percussion, palpation, and auscultation.
C. Palpation, percussion, and auscultation.
D. Auscultation, percussion, and palpation.
46. Nurse Betty is assessing tactile fremitus in a client with pneumonia. For
this examination, nurse Betty should use the:

A. Fingertips
B. Finger pads
C. Dorsal surface of the hand
D. Ulnar surface of the hand
47. Which type of evaluation occurs continuously throughout the teaching and
learning process?
A. Summative
B. Informative
C. Formative
D. Retrospective
48.A 45 year old client, has no family history of breast cancer or other risk
factors for this disease. Nurse John should instruct her to have mammogram
how often?

A. Twice per year


B. Once per year
C. Every 2 years
D. Once, to establish baseline
49.A male client has the following arterial blood gas values: pH 7.30; Pao2 89
mmHg; Paco2 50 mmHg; and HCO3 26mEq/L. Based on these values, Nurse
Patricia should expect which condition?

A. Respiratory acidosis
B. Respiratory alkalosis
C. Metabolic acidosis
D. Metabolic alkalosis
50.Nurse Len refers a female client with terminal cancer to a local hospice.
What is the goal of this referral?

A. To help the client find appropriate treatment options.


B. To provide support for the client and family in coping with terminal
illness.
C. To ensure that the client gets counseling regarding health care costs.
D. To teach the client and family about cancer and its treatment.
51.When caring for a male client with a 3-cm stage I pressure ulcer on the
coccyx, which of the following actions can the nurse institute independently?

A. Massaging the area with an astringent every 2 hours.


B. Applying an antibiotic cream to the area three times per day.
C. Using normal saline solution to clean the ulcer and applying a protective
dressing as necessary.
D. Using a povidone-iodine wash on the ulceration three times per day.
52.Nurse Oliver must apply an elastic bandage to a client’s ankle and calf. He
should apply the bandage beginning at the client’s:

A. Knee
B. Ankle
C. Lower thigh
D. Foot
53.A 10 year old child with type 1 diabetes develops diabetic ketoacidosis and
receives a continuous insulin infusion. Which condition represents the
greatest risk to this child?

A. Hypernatremia
B. Hypokalemia
C. Hyperphosphatemia
D. Hypercalcemia
54.Nurse Len is administering sublingual nitrglycerin (Nitrostat) to the newly
admitted client. Immediately afterward, the client may experience:

A. Throbbing headache or dizziness


B. Nervousness or paresthesia.
C. Drowsiness or blurred vision.
D. Tinnitus or diplopia.
55.Nurse Michelle hears the alarm sound on the telemetry monitor. The nurse
quickly looks at the monitor and notes that a client is in a ventricular
tachycardia. The nurse rushes to the client’s room. Upon reaching the client’s
bedside, the nurse would take which action first?

A. Prepare for cardioversion


B. Prepare to defibrillate the client
C. Call a code
D. Check the client’s level of consciousness
56.Nurse Hazel is preparing to ambulate a female client. The best and the
safest position for the nurse in assisting the client is to stand:

A. On the unaffected side of the client.


B. On the affected side of the client.
C. In front of the client.
D. Behind the client.
57.Nurse Janah is monitoring the ongoing care given to the potential organ
donor who has been diagnosed with brain death. The nurse determines that
the standard of care had been maintained if which of the following data is
observed?

A. Urine output: 45 ml/hr


B. Capillary refill: 5 seconds
C. Serum pH: 7.32
D. Blood pressure: 90/48 mmHg
58. Nurse Amy has an order to obtain a urinalysis from a male client with an
indwelling urinary catheter. The nurse avoids which of the following, which
contaminate the specimen?

A. Wiping the port with an alcohol swab before inserting the syringe.
B. Aspirating a sample from the port on the drainage bag.
C. Clamping the tubing of the drainage bag.
D. Obtaining the specimen from the urinary drainage bag.
59.Nurse Meredith is in the process of giving a client a bed bath. In the middle
of the procedure, the unit secretary calls the nurse on the intercom to tell the
nurse that there is an emergency phone call. The appropriate nursing action is
to:

A. Immediately walk out of the client’s room and answer the phone call.
B. Cover the client, place the call light within reach, and answer the phone
call.
C. Finish the bed bath before answering the phone call.
D. Leave the client’s door open so the client can be monitored and the
nurse can answer the phone call.
60. Nurse Janah is collecting a sputum specimen for culture and sensitivity
testing from a client who has a productive cough. Nurse Janah plans to
implement which intervention to obtain the specimen?

A. Ask the client to expectorate a small amount of sputum into the emesis
basin.
B. Ask the client to obtain the specimen after breakfast.
C. Use a sterile plastic container for obtaining the specimen.
D. Provide tissues for expectoration and obtaining the specimen.
61. Nurse Ron is observing a male client using a walker. The nurse determines
that the client is using the walker correctly if the client:

A. Puts all the four points of the walker flat on the floor, puts weight on the
hand pieces, and then walks into it.
B. Puts weight on the hand pieces, moves the walker forward, and then
walks into it.
C. Puts weight on the hand pieces, slides the walker forward, and then
walks into it.
D. Walks into the walker, puts weight on the hand pieces, and then puts all
four points of the walker flat on the floor.
62.Nurse Amy has documented an entry regarding client care in the client’s
medical record. When checking the entry, the nurse realizes that incorrect
information was documented. How does the nurse correct this error?

A. Erases the error and writes in the correct information.


B. Uses correction fluid to cover up the incorrect information and writes in
the correct information.
C. Draws one line to cross out the incorrect information and then initials
the change.
D. Covers up the incorrect information completely using a black pen and
writes in the correct information
63.Nurse Ron is assisting with transferring a client from the operating room
table to a stretcher. To provide safety to the client, the nurse should:

A. Moves the client rapidly from the table to the stretcher.


B. Uncovers the client completely before transferring to the stretcher.
C. Secures the client safety belts after transferring to the stretcher.
D. Instructs the client to move self from the table to the stretcher.
64.Nurse Myrna is providing instructions to a nursing assistant assigned to
give a bed bath to a client who is on contact precautions. Nurse Myrna
instructs the nursing assistant to use which of the following protective items
when giving bed bath?

A. Gown and goggles


B. Gown and gloves
C. Gloves and shoe protectors
D. Gloves and goggles
65. Nurse Oliver is caring for a client with impaired mobility that occurred as a
result of a stroke. The client has right sided arm and leg weakness. The nurse
would suggest that the client use which of the following assistive devices that
would provide the best stability for ambulating?

A. Crutches
B. Single straight-legged cane
C. Quad cane
D. Walker
66.A male client with a right pleural effusion noted on a chest X-ray is being
prepared for thoracentesis. The client experiences severe dizziness when
sitting upright. To provide a safe environment, the nurse assists the client to
which position for the procedure?

A. Prone with head turned toward the side supported by a pillow.


B. Sims’ position with the head of the bed flat.
C. Right side-lying with the head of the bed elevated 45 degrees.
D. Left side-lying with the head of the bed elevated 45 degrees.
67.Nurse John develops methods for data gathering. Which of the following
criteria of a good instrument refers to the ability of the instrument to yield the
same results upon its repeated administration?

A. Validity
B. Specificity
C. Sensitivity
D. Reliability
68.Harry knows that he has to protect the rights of human research subjects.
Which of the following actions of Harry ensures anonymity?

A. Keep the identities of the subject secret


B. Obtain informed consent
C. Provide equal treatment to all the subjects of the study.
D. Release findings only to the participants of the study
69.Patient’s refusal to divulge information is a limitation because it is beyond
the control of Tifanny”. What type of research is appropriate for this study?
A. Descriptive- correlational
B. Experiment
C. Quasi-experiment
D. Historical
70.Nurse Ronald is aware that the best tool for data gathering is?

A. Interview schedule
B. Questionnaire
C. Use of laboratory data
D. Observation
71.Monica is aware that there are times when only manipulation of study
variables is possible and the elements of control or randomization are not
attendant. Which type of research is referred to this?

A. Field study
B. Quasi-experiment
C. Solomon-Four group design
D. Post-test only design
72.Cherry notes down ideas that were derived from the description of
an investigation written by the person who conducted it. Which type
of reference source refers to this?

A. Footnote
B. Bibliography
C. Primary source
D. Endnotes
73.When Nurse Trish is providing care to his patient, she must remember
that her duty is bound not to do doing any action that will cause the
patient harm. This is the meaning of the bioethical principle:

A. Non-maleficence
B. Beneficence
C. Justice
D. Solidarity
74.When a nurse in-charge causes an injury to a female patient and the
injury caused becomes the proof of the negligent act, the presence of the
injury is said to exemplify the principle of:
A. Force majeure
B. Respondeat superior
C. Res ipsa loquitor
D. Holdover doctrine
75.Nurse Myrna is aware that the Board of Nursing has quasi-judicial
power. An example of this power is:

A. The Board can issue rules and regulations that will govern the practice
of nursing
B. The Board can investigate violations of the nursing law and code
of ethics
C. The Board can visit a school applying for a permit in collaboration with
CHED
D. The Board prepares the board examinations
76. When the license of nurse Krina is revoked, it means that she:

A. Is no longer allowed to practice the profession for the rest of her life
B. Will never have her/his license re-issued since it has been revoked
C. May apply for re-issuance of his/her license based on certain conditions
stipulated in RA 9173
D. Will remain unable to practice professional nursing
77.Ronald plans to conduct a research on the use of a new method of
pain assessment scale. Which of the following is the second step in
the conceptualizing phase of the research process?

A. Formulating the research hypothesis


B. Review related literature
C. Formulating and delimiting the research problem
D. Design the theoretical and conceptual framework
78. The leader of the study knows that certain patients who are in
a specialized research setting tend to respond psychologically to
the conditions of the study. This referred to as :

A. Cause and effect


B. Hawthorne effect
C. Halo effect
D. Horns effect
79.Mary finally decides to use judgment sampling on her research. Which
of the following actions of is correct?

A. Plans to include whoever is there during his study.


B. Determines the different nationality of patients frequently admitted and
decides to get representations samples from each.
C. Assigns numbers for each of the patients, place these in a fishbowl and
draw 10 from it.
D. Decides to get 20 samples from the admitted patients
80. The nursing theorist who developed transcultural nursing theory is:

A. Florence Nightingale
B. Madeleine Leininger
C. Albert Moore
D. Sr. Callista Roy
81.Marion is aware that the sampling method that gives equal chance to
all units in the population to get picked is:

A. Random
B. Accidental
C. Quota
D. Judgment
82.John plans to use a Likert Scale to his study to determine the:

A. Degree of agreement and disagreement


B. Compliance to expected standards
C. Level of satisfaction
D. Degree of acceptance
83.Which of the following theory addresses the four modes of adaptation?

A. Madeleine Leininger
B. Sr. Callista Roy
C. Florence Nightingale
D. Jean Watson
84.Ms. Garcia is responsible to the number of personnel reporting to her.
This principle refers to:
A. Span of control
B. Unity of command
C. Downward communication
D. Leader
85.Ensuring that there is an informed consent on the part of the patient before
a surgery is done, illustrates the bioethical principle of:

A. Beneficence
B. Autonomy
C. Veracity
D. Non-maleficence
86.Nurse Reese is teaching a female client with peripheral vascular
disease about foot care; Nurse Reese should include which instruction?

A. Avoid wearing cotton socks.


B. Avoid using a nail clipper to cut toenails.
C. Avoid wearing canvas shoes.
D. Avoid using cornstarch on feet.
87.A client is admitted with multiple pressure ulcers. When developing
the client’s diet plan, the nurse should include:

A. Fresh orange slices


B. Steamed broccoli
C. Ice cream
D. Ground beef patties
88.The nurse prepares to administer a cleansing enema. What is the
most common client position used for this procedure?

A. Lithotomy
B. Supine
C. Prone
D. Sims’ left lateral
89.Nurse Marian is preparing to administer a blood transfusion. Which
action should the nurse take first?

A. Arrange for typing and cross matching of the client’s blood.


B. Compare the client’s identification wristband with the tag on the unit of
blood.
C. Start an I.V. infusion of normal saline solution.
D. Measure the client’s vital signs.
90.A 65 years old male client requests his medication at 9 p.m. instead of
10 p.m. so that he can go to sleep earlier. Which type of nursing intervention is
required?

A. Independent
B. Dependent
C. Interdependent
D. Intradependent
91.A female client is to be discharged from an acute care facility
after treatment for right leg thrombophlebitis. The Nurse Betty notes that
the client’s leg is pain-free, without redness or edema. The nurse’s
actions reflect which step of the nursing process?

A. Assessment
B. Diagnosis
C. Implementation
D. Evaluation
92.Nursing care for a female client includes removing elastic stockings
once per day. The Nurse Betty is aware that the rationale for this intervention?

A. To increase blood flow to the heart


B. To observe the lower extremities
C. To allow the leg muscles to stretch and relax
D. To permit veins in the legs to fill with blood.
93.Which nursing intervention takes highest priority when caring for a
newly admitted client who’s receiving a blood transfusion?

A. Instructing the client to report any itching, swelling, or dyspnea.


B. Informing the client that the transfusion usually take 1 ½ to 2 hours.
C. Documenting blood administration in the client care record.
D. Assessing the client’s vital signs when the transfusion ends.
94.A male client complains of abdominal discomfort and nausea
while receiving tube feedings. Which intervention is most appropriate for
this problem?

A. Give the feedings at room temperature.


B. Decrease the rate of feedings and the concentration of the formula.
C. Place the client in semi-Fowler’s position while feeding.
D. Change the feeding container every 12 hours.
95.Nurse Patricia is reconstituting a powdered medication in a vial.
After adding the solution to the powder, she nurse should:

A. Do nothing.
B. Invert the vial and let it stand for 3 to 5 minutes.
C. Shake the vial vigorously.
D. Roll the vial gently between the palms.
96.Which intervention should the nurse Trish use when administering
oxygen by face mask to a female client?

A. Secure the elastic band tightly around the client’s head.


B. Assist the client to the semi-Fowler position if possible.
C. Apply the face mask from the client’s chin up over the nose.
D. Loosen the connectors between the oxygen equipment and humidifier.
97.The maximum transfusion time for a unit of packed red blood cells
(RBCs) is:

A. 6 hours
B. 4 hours
C. 3 hours
D. 2 hours
98.Nurse Monique is monitoring the effectiveness of a client’s drug
therapy. When should the nurse Monique obtain a blood sample to measure
the trough drug level?

A. 1 hour before administering the next dose.


B. Immediately before administering the next dose.
C. Immediately after administering the next dose.
D. 30 minutes after administering the next dose.
99.Nurse May is aware that the main advantage of using a floor stock
system is:

A. The nurse can implement medication orders quickly.


B. The nurse receives input from the pharmacist.
C. The system minimizes transcription errors.
D. The system reinforces accurate calculations.
100. Nurse Oliver is assessing a client’s abdomen. Which finding should
the nurse report as abnormal?

A. Dullness over the liver.


B. Bowel sounds occurring every 10 seconds.
C. Shifting dullness over the abdomen.
D. Vascular sounds heard over the renal arteries.
Answers and Rationales
1. Answer: (D) The actions of a reasonably prudent nurse with similar education
and experience. The standard of care is determined by the average degree
of skill, care, and diligence by nurses in similar circumstances.
2. Answer: (B) I.M. With a platelet count of 22,000/μl, the clients tends to
bleed easily. Therefore, the nurse should avoid using the I.M. route
because the area is a highly vascular and can bleed readily when
penetrated by a needle. The bleeding can be difficult to stop.
3. Answer: (C) “Digoxin 0.125 mg P.O. once daily” The nurse should always
place a zero before a decimal point so that no one misreads the figure,
which could result in a dosage error. The nurse   should never insert a zero
at the end of a dosage that includes a decimal point because this could be
misread, possibly leading to a tenfold increase in the dosage.
4. Answer: (A) Ineffective peripheral tissue perfusion related to
venous congestion. Ineffective peripheral tissue perfusion related to
venous congestion takes the highest priority because venous
inflammation and clot formation  impede blood flow in a client with deep
vein thrombosis.
5. Answer: (B) A 44 year-old myocardial infarction (MI) client who
is complaining of nausea. Nausea is a symptom of impending myocardial
infarction (MI) and should be assessed immediately so that treatment can
be instituted and further damage to the heart is avoided.
6. Answer: (C) Check circulation every 15-30 minutes. Restraints encircle the
limbs, which place the client at risk for circulation being restricted to the
distal areas of the extremities. Checking the client’s circulation every 15-
30 minutes will allow the nurse to adjust the restraints before injury from
decreased blood flow occurs.
7. Answer: (A) Prevent stress ulcer. Curling’s ulcer occurs as a generalized
stress response in burn patients. This results in a decreased production of
mucus and increased secretion of gastric acid. The best treatment for this
prophylactic use of antacids and H2 receptor blockers.
8. Answer: (D) Continue to monitor and record hourly urine output. Normal
urine output for an adult is approximately 1 ml/minute (60 ml/hour).
Therefore, this client’s output is normal. Beyond continued evaluation, no
nursing action is warranted.
9. Answer: (B) “My ankle feels warm”. Ice application decreases pain and
swelling. Continued or increased pain, redness, and increased warmth are
signs of inflammation that shouldn’t occur after ice application
10. Answer: (B) Hyperkalemia. A loop diuretic removes water and, along with
it, sodium and potassium. This may result in hypokalemia, hypovolemia,
and hyponatremia.
11. Answer:(A) Have condescending trust and confidence in
their subordinates. Benevolent-authoritative managers pretentiously show
their trust and confidence to their followers.
12. Answer: (A) Provides continuous, coordinated and comprehensive nursing
services. Functional nursing is focused on tasks and activities and not on
the care of the patients.
13. Answer: (B) Standard written order. This is a standard written order.
Prescribers write a single order for medications given only once. A stat
order is written for medications given immediately for an urgent client
problem. A standing order, also known as a protocol, establishes
guidelines for treating a particular disease or set of symptoms in special
care areas such as the coronary care unit. Facilities also may institute
medication protocols that specifically designate drugs that a nurse may
not give.
14. Answer: (D) Liquid or semi-liquid stools. Passage of liquid or semi-liquid
stools results from seepage of unformed bowel contents around the
impacted stool in the rectum. Clients with fecal impaction don’t pass hard,
brown, formed stools because the feces can’t move past the impaction.
These clients typically report the urge to defecate (although they can’t
pass stool) and a decreased appetite.
15. Answer: (C) Pulling the helix up and back. To perform an otoscopic
examination on an adult, the nurse grasps the helix of the ear and pulls it
up and back to straighten the ear canal. For a child, the nurse grasps the
helix and pulls it down to straighten the ear canal. Pulling the lobule in any
direction wouldn’t straighten the ear canal for visualization.
16. Answer: (A) Protect the irritated skin from sunlight. Irradiated skin is very
sensitive and must be protected with clothing or sunblock. The priority
approach is the avoidance of strong sunlight.
17. Answer: (C) Assist the client in removing dentures and nail polish. Dentures,
hairpins, and combs must be removed. Nail polish must be removed so
that cyanosis can be easily monitored by observing the nail beds.
18. Answer: (D) Sudden onset of continuous epigastric and back pain. The
autodigestion of tissue by the pancreatic enzymes results in pain from
inflammation, edema, and possible hemorrhage. Continuous, unrelieved
epigastric or back pain reflects the inflammatory process in the pancreas.
19. Answer: (B) Provide high-protein, high-carbohydrate diet. A positive nitrogen
balance is important for meeting metabolic needs, tissue repair, and
resistance to infection. Caloric goals may be as high as 5000 calories per
day.
20. Answer: (A) Blood pressure and pulse rate. The baseline must be
established to recognize the signs of an anaphylactic or hemolytic
reaction to the transfusion.
21. Answer: (D) Immobilize the leg before moving the client. If the nurse
suspects a fracture, splinting the area before moving the client is
imperative. The nurse should call for emergency help if the client is not
hospitalized and call for a physician for the hospitalized client.
22. Answer: (B) Admit the client into a private room. The client who has a
radiation implant is placed in a private room and has a limited number of
visitors. This reduces the exposure of others to the radiation.
23. Answer: (C) Risk for infection. Agranulocytosis is characterized by a
reduced number of leukocytes (leucopenia) and neutrophils (neutropenia)
in the blood. The client is at high risk for infection because of the
decreased body defenses against microorganisms. Deficient knowledge
related to the nature of the disorder may be appropriate diagnosis but is
not the priority.
24. Answer: (B) Place the client on the left side in the Trendelenburg
position. Lying on the left side may prevent air from flowing into the
pulmonary veins. The Trendelenburg position increases intrathoracic
pressure, which decreases the amount of blood pulled into the vena cava
during aspiration.
25. Answer: (A) Autocratic. The autocratic style of leadership is a task-
oriented and directive.
26. Answer: (D) 2.5 cc. 2.5 cc is to be added, because only a 500 cc bag of
solution is being medicated instead of a 1 liter.
27. Answer: (A) 50 cc/ hour. A rate of 50 cc/hr. The child is to receive 400 cc
over a period of 8 hours = 50 cc/hr.
28. Answer: (B) Assess the client for presence of pain. Assessing the client for
pain is a very important measure. Postoperative pain is an indication of
complication. The nurse should also assess the client for pain to provide
for the client’s comfort.
29. Answer: (A) BP – 80/60, Pulse – 110 irregular. The classic signs of
cardiogenic shock are low blood pressure, rapid and weak irregular pulse,
cold, clammy skin, decreased urinary output, and cerebral hypoxia.
30. Answer: (A) Take the proper equipment, place the client in a comfortable
position, and record the appropriate information in the client’s chart. It is a
general or comprehensive statement about the correct procedure, and it
includes the basic ideas which are found in the other options
31. Answer: (B) Evaluation. Evaluation includes observing the person, asking
questions, and comparing the patient’s behavioral responses with the
expected outcomes.
32. Answer: (C) History of present illness. The history of present illness is the
single most important factor in assisting the health professional in
arriving at a diagnosis or determining the person’s needs.
33. Answer: (A) Trochanter roll extending from the crest of the ileum to the mid-
thigh. A trochanter roll, properly placed, provides resistance to the external
rotation of the hip.
34. Answer: (C) Stage III. Clinically, a deep crater or without undermining of
adjacent tissue is noted.
35. Answer: (A) Second intention healing. When wounds dehisce, they will
allowed to heal by secondary intention
36. Answer: (D) Tachycardia. With an extracellular fluid or plasma volume
deficit, compensatory mechanisms stimulate the heart, causing an
increase in heart rate.
37. Answer: (A) 0.75. To determine the number of milliliters the client should
receive, the nurse uses the fraction method in the following equation.
 75 mg/X ml = 100 mg/1 ml
 To solve for X, cross-multiply:
 75 mg x 1 ml = X ml x 100 mg
 75 = 100X
 75/100 = X
 0.75 ml (or ¾ ml) = X
38. Answer: (D) It’s a measure of effect, not a standard measure of weight or
quantity. An insulin unit is a measure of effect, not a standard measure of
weight or quantity. Different drugs measured in units may have no
relationship to one another in quality or quantity.
39. Answer: (B) 38.9 °C. To convert Fahrenheit degreed to Centigrade, use
this formula
 °C = (°F – 32) ÷ 1.8
 °C = (102 – 32) ÷ 1.8
 °C = 70 ÷ 1.8
 °C = 38.9
40. Answer: (C) Failing eyesight, especially close vision. Failing eyesight,
especially close vision, is one of the first signs of aging in middle life (ages
46 to 64). More frequent aches and pains begin in the early late years
(ages 65 to 79). Increase in loss of muscle tone occurs in later years (age
80 and older).
41. Answer: (A) Checking and taping all connections. Air leaks commonly occur
if the system isn’t secure. Checking all connections and taping them will
prevent air leaks. The chest drainage system is kept lower to promote
drainage – not to prevent leaks.
42. Answer: (A) Check the client’s identification band. Checking the client’s
identification band is the safest way to verify a client’s identity because
the band is assigned on admission and isn’t be removed at any time. (If it
is removed, it must be replaced). Asking the client’s name or having the
client repeated his name would be appropriate only for a client who’s alert,
oriented, and able to understand what is being said, but isn’t the safe
standard of practice. Names on bed aren’t always reliable
43. Answer: (B) 32 drops/minute. Giving 1,000 ml over 8 hours is the same as
giving 125 ml over 1 hour (60 minutes). Find the number of milliliters per
minute as follows:
 125/60 minutes = X/1 minute
 60X = 125 = 2.1 ml/minute
 To find the number of drops per minute:
 2.1 ml/X gtt = 1 ml/ 15 gtt
 X = 32 gtt/minute, or 32 drops/minute
44. Answer: (A) Clamp the catheter. If a central venous catheter becomes
disconnected, the nurse should immediately apply a catheter clamp, if
available. If a clamp isn’t available, the nurse can place a sterile syringe or
catheter plug in the catheter hub. After cleaning the hub with alcohol or
povidone-iodine solution, the nurse must replace the I.V. extension and
restart the infusion.
45. Answer: (D) Auscultation, percussion, and palpation.The correct order of
assessment for examining the abdomen is inspection, auscultation,
percussion, and palpation. The reason for this approach is that the less
intrusive techniques should be performed before the more intrusive
techniques. Percussion and palpation can alter natural findings during
auscultation.
46. Answer: (D) Ulnar surface of the hand. The nurse uses the ulnar surface, or
ball, of the hand to asses tactile fremitus, thrills, and vocal vibrations
through the chest wall. The fingertips and finger pads best distinguish
texture and shape. The dorsal surface best feels warmth.
47. Answer: (C) Formative. Formative (or concurrent) evaluation occurs
continuously throughout the teaching and learning process. One benefit is
that the nurse can adjust teaching strategies as necessary to enhance
learning. Summative, or retrospective, evaluation occurs at the conclusion
of the teaching and learning session. Informative is not a type of
evaluation.
48. Answer: (B) Once per year. Yearly mammograms should begin at age 40
and continue for as long as the woman is in good health. If health risks,
such as family history, genetic tendency, or past breast cancer, exist, more
frequent examinations may be necessary.
49. Answer: (A) Respiratory acidosis. The client has a below-normal (acidic)
blood pH value and an above-normal partial pressure of arterial carbon
dioxide (Paco2) value, indicating respiratory acidosis. In respiratory
alkalosis, the pH value is above normal and in the Paco2 value is below
normal. In metabolic acidosis, the pH and bicarbonate (Hco3) values are
below normal. In metabolic alkalosis, the pH and Hco3 values are above
normal.
50. Answer: (B) To provide support for the client and family in coping with
terminal illness. Hospices provide supportive care for terminally ill clients
and their families. Hospice care doesn’t focus on counseling regarding
health care costs. Most client referred to hospices have been treated for
their disease without success and will receive only palliative care in the
hospice.
51. Answer: (C) Using normal saline solution to clean the ulcer and applying a
protective dressing as necessary. Washing the area with normal saline
solution and applying a protective dressing are within the nurse’s realm of
interventions and will protect the area. Using a povidone-iodine wash and
an antibiotic cream require a physician’s order. Massaging with an
astringent can further damage the skin.
52. Answer: (D) Foot. An elastic bandage should be applied form the distal
area to the proximal area. This method promotes venous return. In this
case, the nurse should begin applying the bandage at the client’s foot.
Beginning at the ankle, lower thigh, or knee does not promote venous
return.
53. Answer: (B) Hypokalemia. Insulin administration causes glucose and
potassium to move into the cells, causing hypokalemia.
54. Answer: (A) Throbbing headache or dizziness. Headache and dizziness
often occur when nitroglycerin is taken at the beginning of therapy.
However, the client usually develops tolerance
55. Answer: (D) Check the client’s level of consciousness. Determining
unresponsiveness is the first step assessment action to take. When a
client is in ventricular tachycardia, there is a significant decrease in
cardiac output. However, checking the unresponsiveness ensures whether
the client is affected by the decreased cardiac output.
56. Answer: (B) On the affected side of the client.When walking with clients, the
nurse should stand on the affected side and grasp the security belt in the
midspine area of the small of the back. The nurse should position the free
hand at the shoulder area so that the client can be pulled toward the nurse
in the event that there is a forward fall. The client is instructed to look up
and outward rather than at his or her feet.
57. Answer: (A) Urine output: 45 ml/hr. Adequate perfusion must be
maintained to all vital organs in order for the client to remain visible as an
organ donor. A urine output of 45 ml per hour indicates adequate renal
perfusion. Low blood pressure and delayed capillary refill time are
circulatory system indicators of inadequate perfusion. A serum pH of 7.32
is acidotic, which adversely affects all body tissues.
58. Answer: (D ) Obtaining the specimen from the urinary drainage bag. A urine
specimen is not taken from the urinary drainage bag. Urine undergoes
chemical changes while sitting in the bag and does not necessarily reflect
the current client status. In addition, it may become contaminated with
bacteria from opening the system.
59. Answer: (B) Cover the client, place the call light within reach, and answer the
phone call. Because telephone call is an emergency, the nurse may need to
answer it. The other appropriate action is to ask another nurse to accept
the call. However, is not one of the options. To maintain privacy and
safety, the nurse covers the client and places the call light within the
client’s reach. Additionally, the client’s door should be closed or the room
curtains pulled around the bathing area.
60. Answer: (C) Use a sterile plastic container for obtaining the specimen. Sputum
specimens for culture and sensitivity testing need to be obtained using
sterile techniques because the test is done to determine the presence of
organisms. If the procedure for obtaining the specimen is not sterile, then
the specimen is not sterile, then the specimen would be contaminated and
the results of the test would be invalid.
61. Answer: (A) Puts all the four points of the walker flat on the floor, puts weight
on the hand pieces, and then walks into it. When the client uses a walker, the
nurse stands adjacent to the affected side. The client is instructed to put
all four points of the walker 2 feet forward flat on the floor before putting
weight on hand pieces. This will ensure client safety and prevent stress
cracks in the walker. The client is then instructed to move the walker
forward and walk into it.
62. Answer: (C) Draws one line to cross out the incorrect information and then
initials the change. To correct an error documented in a medical record, the
nurse draws one line through the incorrect information and then initials
the error. An error is never erased and correction fluid is never used in the
medical record.
63. Answer: (C) Secures the client safety belts after transferring to the
stretcher. During the transfer of the client after the surgical procedure is
complete, the nurse should avoid exposure of the client because of the
risk for potential heat loss. Hurried movements and rapid changes in the
position should be avoided because these predispose the client to
hypotension. At the time of the transfer from the surgery table to the
stretcher, the client is still affected by the effects of the anesthesia;
therefore, the client should not move self. Safety belts can prevent the
client from falling off the stretcher.
64. Answer: (B) Gown and gloves. Contact precautions require the use of
gloves and a gown if direct client contact is anticipated. Goggles are not
necessary unless the nurse anticipates the splashes of blood, body fluids,
secretions, or excretions may occur. Shoe protectors are not necessary.
65. Answer: (C) Quad cane. Crutches and a walker can be difficult to
maneuver for a client with weakness on one side. A cane is better suited
for client with weakness of the arm and leg on one side. However, the
quad cane would provide the most stability because of the structure of the
cane and because a quad cane has four legs.
66. Answer: (D) Left side-lying with the head of the bed elevated 45 degrees. To
facilitate removal of fluid from the chest wall, the client is positioned
sitting at the edge of the bed leaning over the bedside table with the feet
supported on a stool. If the client is unable to sit up, the client is
positioned lying in bed on the unaffected side with the head of the bed
elevated 30 to 45 degrees.
67. Answer: (D) Reliability Reliability is consistency of the research
instrument. It refers to the repeatability of the instrument in extracting the
same responses upon its repeated administration.
68. Answer: (A) Keep the identities of the subject secret. Keeping the identities
of the research subject secret will ensure anonymity because this will
hinder providing link between the information given to whoever is its
source.
69. Answer: (A) Descriptive- correlational. Descriptive- correlational study is
the most appropriate for this study because it studies the variables that
could be the antecedents of the increased incidence of nosocomial
infection.
70. Answer: (C) Use of laboratory data. Incidence of nosocomial infection is
best collected through the use of biophysiologic measures, particularly in
vitro measurements, hence laboratory data is essential.
71. Answer: (B) Quasi-experiment. Quasi-experiment is done when
randomization and control of the variables are not possible.
72. Answer: (C) Primary source. This refers to a primary source which is a
direct account of the investigation done by the investigator. In contrast to
this is a secondary source, which is written by someone other than the
original researcher.
73. Answer: (A) Non-maleficence. Non-maleficence means do not cause harm
or do any action that will cause any harm to the patient/client. To do good
is referred as beneficence.
74. Answer: (C) Res ipsa loquitor. Res ipsa loquitor literally means the thing
speaks for itself. This means in operational terms that the injury caused is
the proof that there was a negligent act.
75. Answer: (B) The Board can investigate violations of the nursing law and code
of ethics. Quasi-judicial power means that the Board of Nursing has the
authority to investigate violations of the nursing law and can issue
summons, subpoena or subpoena duces tecum as needed.
76. Answer: (C) May apply for re-issuance of his/her license based on certain
conditions stipulated in RA 9173. RA 9173 sec. 24 states that for equity and
justice, a revoked license maybe re-issued provided that the following
conditions are met: a) the cause for revocation of license has already
been corrected or removed; and, b) at least four years has elapsed since
the license has been revoked.
77. Answer: (B) Review related literature. After formulating and delimiting the
research problem, the researcher conducts a review of related literature to
determine the extent of what has been done on the study by previous
researchers.
78. Answer: (B) Hawthorne effect. Hawthorne effect is based on the study of
Elton Mayo and company about the effect of an intervention done to
improve the working conditions of the workers on their productivity. It
resulted to an increased productivity but not due to the intervention but
due to the psychological effects of being observed. They performed
differently because they were under observation.
79. Answer: (B) Determines the different nationality of patients frequently
admitted and decides to get representations samples from each. Judgment
sampling involves including samples according to the knowledge of the
investigator about the participants in the study.
80. Answer: (B) Madeleine Leininger. Madeleine Leininger developed the
theory on transcultural theory based on her observations on the behavior
of selected people within a culture.
81. Answer: (A) Random. Random sampling gives equal chance for all the
elements in the population to be picked as part of the sample.
82. Answer: (A) Degree of agreement and disagreement. Likert scale is a 5-point
summated scale used to determine the degree of agreement or
disagreement of the respondents to a statement in a study
83. Answer: (B) Sr. Callista Roy. Sr. Callista Roy developed the Adaptation
Model which involves the physiologic mode, self-concept mode, role
function mode and dependence mode.
84. Answer: (A) Span of control. Span of control refers to the number of
workers who report directly to a manager.
85. Answer: (B) Autonomy. Informed consent means that the patient fully
understands about the surgery, including the risks involved and the
alternative solutions. In giving consent it is done with full knowledge and
is given freely. The action of allowing the patient to decide whether a
surgery is to be done or not exemplifies the bioethical principle of
autonomy.
86. Answer: (C) Avoid wearing canvas shoes. The client should be instructed to
avoid wearing canvas shoes. Canvas shoes cause the feet to perspire,
which may, in turn, cause skin irritation and breakdown. Both cotton and
cornstarch absorb perspiration. The client should be instructed to cut
toenails straight across with nail clippers.
87. Answer: (D) Ground beef patties. Meat is an excellent source of complete
protein, which this client needs to repair the tissue breakdown caused by
pressure ulcers. Oranges and broccoli supply vitamin C but not protein. Ice
cream supplies only some incomplete protein, making it less helpful in
tissue repair.
88. Answer: (D) Sims’ left lateral. The Sims’ left lateral position is the most
common position used to administer a cleansing enema because it allows
gravity to aid the flow of fluid along the curve of the sigmoid colon. If the
client can’t assume this position nor has poor sphincter control, the dorsal
recumbent or right lateral position may be used. The supine and prone
positions are inappropriate and uncomfortable for the client.
89. Answer: (A) Arrange for typing and cross matching of the client’s blood. The
nurse first arranges for typing and cross matching of the client’s blood to
ensure compatibility with donor blood. The other options,although
appropriate when preparing to administer a blood transfusion, come later.
90. Answer: (A) Independent. Nursing interventions are classified as
independent, interdependent, or dependent. Altering the drug schedule to
coincide with the client’s daily routine represents an independent
intervention, whereas consulting with the physician and pharmacist to
change a client’s medication because of adverse reactions represents an
interdependent intervention. Administering an already-prescribed drug on
time is a dependent intervention. An intradependent nursing intervention
doesn’t exist.
91. Answer: (D) Evaluation. The nursing actions described constitute
evaluation of the expected outcomes. The findings show that the
expected outcomes have been achieved. Assessment consists of the
client’s history, physical examination, and laboratory studies. Analysis
consists of considering assessment information to derive the appropriate
nursing diagnosis. Implementation is the phase of the nursing process
where the nurse puts the plan of care into action.
92. Answer: (B) To observe the lower extremities. Elastic stockings are used to
promote venous return. The nurse needs to remove them once per day to
observe the condition of the skin underneath the stockings. Applying the
stockings increases blood flow to the heart. When the stockings are in
place, the leg muscles can still stretch and relax, and the veins can fill with
blood.
93. Answer:(A) Instructing the client to report any itching, swelling, or
dyspnea. Because administration of blood or blood products may cause
serious adverse effects such as allergic reactions, the nurse must monitor
the client for these effects. Signs and symptoms of life-threatening
allergic reactions include itching, swelling, and dyspnea. Although the
nurse should inform the client of the duration of the transfusion and
should document its administration, these actions are less critical to the
client’s immediate health. The nurse should assess vital signs at least
hourly during the transfusion.
94. Answer: (B) Decrease the rate of feedings and the concentration of the
formula. Complaints of abdominal discomfort and nausea are common in
clients receiving tube feedings. Decreasing the rate of the feeding and the
concentration of the formula should decrease the client’s discomfort.
Feedings are normally given at room temperature to minimize abdominal
cramping. To prevent aspiration during feeding, the head of the client’s
bed should be elevated at least 30 degrees. Also, to prevent bacterial
growth, feeding containers should be routinely changed every 8 to 12
hours.
95. Answer: (D) Roll the vial gently between the palms. Rolling the vial gently
between the palms produces heat, which helps dissolve the medication.
Doing nothing or inverting the vial wouldn’t help dissolve the medication.
Shaking the vial vigorously could cause the medication to break down,
altering its action.
96. Answer: (B) Assist the client to the semi-Fowler position if possible. By
assisting the client to the semi-Fowler position, the nurse promotes easier
chest expansion, breathing, and oxygen intake. The nurse should secure
the elastic band so that the face mask fits comfortably and snugly rather
than tightly, which could lead to irritation. The nurse should apply the face
mask from the client’s nose down to the chin — not vice versa. The nurse
should check the connectors between the oxygen equipment and
humidifier to ensure that they’re airtight; loosened connectors can cause
loss of oxygen.
97. Answer: (B) 4 hours. A unit of packed RBCs may be given over a period of
between 1 and 4 hours. It shouldn’t infuse for longer than 4 hours because
the risk of contamination and sepsis increases after that time. Discard or
return to the blood bank any blood not given within this time, according to
facility policy.
98. Answer: (B) Immediately before administering the next dose. Measuring the
blood drug concentration helps determine whether the dosing has
achieved the therapeutic goal. For measurement of the trough, or lowest,
blood level of a drug, the nurse draws a blood sample immediately before
administering the next dose. Depending on the drug’s duration of action
and half-life, peak blood drug levels typically are drawn after administering
the next dose.
99. Answer: (A) The nurse can implement medication orders quickly. A floor
stock system enables the nurse to implement medication orders quickly. It
doesn’t allow for pharmacist input, nor does it minimize transcription
errors or reinforce accurate calculations.
100. Answer: (C) Shifting dullness over the abdomen. Shifting dullness over the
abdomen indicates ascites, an abnormal finding. The other options are
normal abdominal findings.
PNLE II for Community Health
Nursing and Care of the Mother and
Child
1. May arrives at the health care clinic and tells the nurse that her last
menstrual period was 9 weeks ago. She also tells the nurse that a home
pregnancy test was positive but she began to have mild cramps and is now
having moderate vaginal bleeding. During the physical examination of the
client, the nurse notes that May has a dilated cervix. The nurse determines
that May is experiencing which type of abortion?
A. Inevitable
B. Incomplete
C. Threatened
D. Septic
2. Nurse Reese is reviewing the record of a pregnant client for her first
prenatal visit. Which of the following data, if noted on the client’s record,
would alert the nurse that the client is at risk for a spontaneous abortion?
A. Age 36 years
B. History of syphilis
C. History of genital herpes
D. History of diabetes mellitus
3. Nurse Hazel is preparing to care for a client who is newly admitted to the
hospital with a possible diagnosis of ectopic pregnancy. Nurse Hazel
develops a plan of care for the client and determines that which of the
following nursing actions is the priority?
A. Monitoring weight
B. Assessing for edema
C. Monitoring apical pulse
D. Monitoring temperature
4. Nurse Oliver is teaching a diabetic pregnant client about nutrition and
insulin needs during pregnancy. The nurse determines that the client
understands dietary and insulin needs if the client states that the second half
of pregnancy require:
A. Decreased caloric intake
B. Increased caloric intake
C. Decreased Insulin
D. Increase Insulin
5. Nurse Michelle is assessing a 24 year old client with a diagnosis of
hydatidiform mole. She is aware that one of the following is unassociated with
this condition?
A. Excessive fetal activity.
B. Larger than normal uterus for gestational age.
C. Vaginal bleeding
D. Elevated levels of human chorionic gonadotropin.
6. A pregnant client is receiving magnesium sulfate for severe pregnancy
induced hypertension (PIH). The clinical findings that would warrant use of the
antidote , calcium gluconate is:
A. Urinary output 90 cc in 2 hours.
B. Absent patellar reflexes.
C. Rapid respiratory rate above 40/min.
D. Rapid rise in blood pressure.
7. During vaginal examination of Janah who is in labor, the presenting part is
at station plus two. Nurse, correctly interprets it as:
A. Presenting part is 2 cm above the plane of the ischial spines.
B. Biparietal diameter is at the level of the ischial spines.
C. Presenting part in 2 cm below the plane of the ischial spines.
D. Biparietal diameter is 2 cm above the ischial spines.
8. A pregnant client is receiving oxytocin (Pitocin) for induction of labor. A
condition that warrant the nurse in-charge to discontinue I.V. infusion of
Pitocin is:
A. Contractions every 1 ½ minutes lasting 70-80 seconds.
B. Maternal temperature 101.2
C. Early decelerations in the fetal heart rate.
D. Fetal heart rate baseline 140-160 bpm.
9. Calcium gluconate is being administered to a client with pregnancy induced
hypertension (PIH). A nursing action that must be initiated as the plan of care
throughout injection of the drug is:
A. Ventilator assistance
B. CVP readings
C. EKG tracings
D. Continuous CPR
10. A trial for vaginal delivery after an earlier caesareans, would likely to be
given to a gravida, who had:
A. First low transverse cesarean was for active herpes type 2 infections;
vaginal culture at 39 weeks pregnancy was positive.
B. First and second caesareans were for cephalopelvic disproportion.
C. First caesarean through a classic incision as a result of severe fetal
distress.
D. First low transverse caesarean was for breech position. Fetus in this
pregnancy is in a vertex presentation.
11.Nurse Ryan is aware that the best initial approach when trying to take a
crying toddler’s temperature is:
A. Talk to the mother first and then to the toddler.
B. Bring extra help so it can be done quickly.
C. Encourage the mother to hold the child.
D. Ignore the crying and screaming.
12.Baby Tina a 3 month old infant just had a cleft lip and palate repair. What
should the nurse do to prevent trauma to operative site?
A. Avoid touching the suture line, even when cleaning.
B. Place the baby in prone position.
C. Give the baby a pacifier.
D. Place the infant’s arms in soft elbow restraints.
13. Which action should nurse Marian include in the care plan for a 2 month
old with heart failure?
A. Feed the infant when he cries.
B. Allow the infant to rest before feeding.
C. Bathe the infant and administer medications before feeding.
D. Weigh and bathe the infant before feeding.
14.Nurse Hazel is teaching a mother who plans to discontinue breast feeding
after 5 months. The nurse should advise her to include which foods in her
infant’s diet?
A. Skim milk and baby food.
B. Whole milk and baby food.
C. Iron-rich formula only.
D. Iron-rich formula and baby food.
15.Mommy Linda is playing with her infant, who is sitting securely alone on
the floor of the clinic. The mother hides a toy behind her back and the infant
looks for it. The nurse is aware that estimated age of the infant would be:
A. 6 months
B. 4 months
C. 8 months
D. 10 months
16.Which of the following is the most prominent feature of public health
nursing?
A. It involves providing home care to sick people who are not confined in
the hospital.
B. Services are provided free of charge to people within the catchments
area.
C. The public health nurse functions as part of a team providing a public
health nursing services.
D. Public health nursing focuses on preventive, not curative, services.
17.When the nurse determines whether resources were maximized in
implementing Ligtas Tigdas, she is evaluating
A. Effectiveness
B. Efficiency
C. Adequacy
D. Appropriateness
18.Vangie is a new B.S.N. graduate. She wants to become a Public Health
Nurse. Where should she apply?
A. Department of Health
B. Provincial Health Office
C. Regional Health Office
D. Rural Health Unit
19.Tony is aware the Chairman of the Municipal Health Board is:
A. Mayor
B. Municipal Health Officer
C. Public Health Nurse
D. Any qualified physician
20.Myra is the public health nurse in a municipality with a total population of
about 20,000. There are 3 rural health midwives among the RHU personnel.
How many more midwife items will the RHU need?
A. 1
B. 2
C. 3
D. The RHU does not need any more midwife item.
21.According to Freeman and Heinrich, community health nursing is a
developmental service. Which of the following best illustrates this statement?
A. The community health nurse continuously develops himself personally
and professionally.
B. Health education and community organizing are necessary in providing
community health services.
C. Community health nursing is intended primarily for health promotion
and prevention and treatment of disease.
D. The goal of community health nursing is to provide nursing services to
people in their own places of residence.
22.Nurse Tina is aware that the disease declared through Presidential
Proclamation No. 4 as a target for eradication in the Philippines is?
A. Poliomyelitis
B. Measles
C. Rabies
D. Neonatal tetanus
23.May knows that the step in community organizing that involves training of
potential leaders in the community is:
A. Integration
B. Community organization
C. Community study
D. Core group formation
24.Beth a public health nurse takes an active role in community participation.
What is the primary goal of community organizing?
A. To educate the people regarding community health problems
B. To mobilize the people to resolve community health problems
C. To maximize the community’s resources in dealing with health
problems.
D. To maximize the community’s resources in dealing with health
problems.
25.Tertiary prevention is needed in which stage of the natural history of
disease?
A. Pre-pathogenesis
B. Pathogenesis
C. Prodromal
D. Terminal
26.The nurse is caring for a primigravid client in the labor and delivery area.
Which condition would place the client at risk for disseminated intravascular
coagulation (DIC)?
A. Intrauterine fetal death.
B. Placenta accreta.
C. Dysfunctional labor.
D. Premature rupture of the membranes.
27.A fullterm client is in labor. Nurse Betty is aware that the fetal heart rate
would be:
A. 80 to 100 beats/minute
B. 100 to 120 beats/minute
C. 120 to 160 beats/minute
D. 160 to 180 beats/minute
28.The skin in the diaper area of a 7 month old infant is excoriated and red.
Nurse Hazel should instruct the mother to:
A. Change the diaper more often.
B. Apply talc powder with diaper changes.
C. Wash the area vigorously with each diaper change.
D. Decrease the infant’s fluid intake to decrease saturating diapers.
29.Nurse Carla knows that the common cardiac anomalies in children with
Down Syndrome (tri-somy 21) is:
A. Atrial septal defect
B. Pulmonic stenosis
C. Ventricular septal defect
D. Endocardial cushion defect
30.Malou was diagnosed with severe preeclampsia is now receiving I.V.
magnesium sulfate. The adverse effects associated with magnesium sulfate
is:
A. Anemia
B. Decreased urine output
C. Hyperreflexia
D. Increased respiratory rate
31.A 23 year old client is having her menstrual period every 2 weeks that last
for 1 week. This type of menstrual pattern is bets defined by:
A. Menorrhagia
B. Metrorrhagia
C. Dyspareunia
D. Amenorrhea
32. Jannah is admitted to the labor and delivery unit. The critical laboratory
result for this client would be:
A. Oxygen saturation
B. Iron binding capacity
C. Blood typing
D. Serum Calcium
33.Nurse Gina is aware that the most common condition found during the
second-trimester of pregnancy is:
A. Metabolic alkalosis
B. Respiratory acidosis
C. Mastitis
D. Physiologic anemia
34.Nurse Lynette is working in the triage area of an emergency department.
She sees that several pediatric clients arrive simultaneously. The client who
needs to be treated first is:
A. A crying 5 year old child with a laceration on his scalp.
B. A 4 year old child with a barking coughs and flushed appearance.
C. A 3 year old child with Down syndrome who is pale and asleep in his
mother’s arms.
D. A 2 year old infant with stridorous breath sounds, sitting up in
his mother’s arms and drooling.
35.Maureen in her third trimester arrives at the emergency room with painless
vaginal bleeding. Which of the following conditions is suspected?
A. Placenta previa
B. Abruptio placentae
C. Premature labor
D. Sexually transmitted disease
36.A young child named Richard is suspected of having pinworms. The
community nurse collects a stool specimen to confirm the diagnosis. The
nurse should schedule the collection of this specimen for:
A. Just before bedtime
B. After the child has been bathe
C. Any time during the day
D. Early in the morning
37.In doing a child’s admission assessment, Nurse Betty should be alert to
note which signs or symptoms of chronic lead poisoning?
A. Irritability and seizures
B. Dehydration and diarrhea
C. Bradycardia and hypotension
D. Petechiae and hematuria
38.To evaluate a woman’s understanding about the use of diaphragm for
family planning, Nurse Trish asks her to explain how she will use the
appliance. Which response indicates a need for further health teaching?
A. “I should check the diaphragm carefully for holes every time I use it”
B. “I may need a different size of diaphragm if I gain or lose weight more
than 20 pounds”
C. “The diaphragm must be left in place for atleast 6 hours after
intercourse”
D. “I really need to use the diaphragm and jelly most during the middle of
my menstrual cycle”.
39.Hypoxia is a common complication of laryngotracheobronchitis. Nurse
Oliver should frequently assess a child with laryngotracheobronchitis for:
A. Drooling
B. Muffled voice
C. Restlessness
D. Low-grade fever
40.How should Nurse Michelle guide a child who is blind to walk to the
playroom?
A. Without touching the child, talk continuously as the child walks down
the hall.
B. Walk one step ahead, with the child’s hand on the nurse’s elbow.
C. Walk slightly behind, gently guiding the child forward.
D. Walk next to the child, holding the child’s hand.
41.When assessing a newborn diagnosed with ductus arteriosus, Nurse Olivia
should expect that the child most likely would have an:
A. Loud, machinery-like murmur.
B. Bluish color to the lips.
C. Decreased BP reading in the upper extremities
D. Increased BP reading in the upper extremities.
42.The reason nurse May keeps the neonate in a neutral thermal environment
is that when a newborn becomes too cool, the neonate requires:
A. Less oxygen, and the newborn’s metabolic rate increases.
B. More oxygen, and the newborn’s metabolic rate decreases.
C. More oxygen, and the newborn’s metabolic rate increases.
D. Less oxygen, and the newborn’s metabolic rate decreases.
43.Before adding potassium to an infant’s I.V. line, Nurse Ron must be sure to
assess whether this infant has:
A. Stable blood pressure
B. Patant fontanelles
C. Moro’s reflex
D. Voided
44.Nurse Carla should know that the most common causative factor of
dermatitis in infants and younger children is:
A. Baby oil
B. Baby lotion
C. Laundry detergent
D. Powder with cornstarch
45.During tube feeding, how far above an infant’s stomach should the nurse
hold the syringe with formula?
A. 6 inches
B. 12 inches
C. 18 inches
D. 24 inches
46. In a mothers’ class, Nurse Lhynnete discussed childhood diseases such
as chicken pox. Which of the following statements about chicken pox is
correct?
A. The older one gets, the more susceptible he becomes to the
complications of chicken pox.
B. A single attack of chicken pox will prevent future episodes, including
conditions such as shingles.
C. To prevent an outbreak in the community, quarantine may be imposed
by health authorities.
D. Chicken pox vaccine is best given when there is an impending outbreak
in the community.
47.Barangay Pinoy had an outbreak of German measles. To prevent
congenital rubella, what is the BEST advice that you can give to women in the
first trimester of pregnancy in the barangay Pinoy?
A. Advice them on the signs of German measles.
B. Avoid crowded places, such as markets and movie houses.
C. Consult at the health center where rubella vaccine may be given.
D. Consult a physician who may give them rubella immunoglobulin.
48.Myrna a public health nurse knows that to determine possible sources of
sexually transmitted infections, the BEST method that may be undertaken is:
A. Contact tracing
B. Community survey
C. Mass screening tests
D. Interview of suspects
49.A 33-year old female client came for consultation at the health center with
the chief complaint of fever for a week. Accompanying symptoms were
muscle pains and body malaise. A week after the start of fever, the client
noted yellowish discoloration of his sclera. History showed that he waded in
flood waters about 2 weeks before the onset of symptoms. Based on her
history, which disease condition will you suspect?
A. Hepatitis A
B. Hepatitis B
C. Tetanus
D. Leptospirosis
50.Mickey a 3-year old client was brought to the health center with the chief
complaint of severe diarrhea and the passage of “rice water” stools. The client
is most probably suffering from which condition?
A. Giardiasis
B. Cholera
C. Amebiasis
D. Dysentery
51.The most prevalent form of meningitis among children aged 2 months to 3
years is caused by which microorganism?
A. Hemophilus influenzae
B. Morbillivirus
C. Steptococcus pneumoniae
D. Neisseria meningitidis
52.The student nurse is aware that the pathognomonic sign of measles is
Koplik’s spot and you may see Koplik’s spot by inspecting the:
A. Nasal mucosa
B. Buccal mucosa
C. Skin on the abdomen
D. Skin on neck
53.Angel was diagnosed as having Dengue fever. You will say that there is
slow capillary refill when the color of the nailbed that you pressed does not
return within how many seconds?
A. 3 seconds
B. 6 seconds
C. 9 seconds
D. 10 seconds
54.In Integrated Management of Childhood Illness, the nurse is aware that the
severe conditions generally require urgent referral to a hospital. Which of the
following severe conditions DOES NOT always require urgent referral to a
hospital?
A. Mastoiditis
B. Severe dehydration
C. Severe pneumonia
D. Severe febrile disease
55.Myrna a public health nurse will conduct outreach immunization in a
barangay Masay with a population of about 1500. The estimated number of
infants in the barangay would be:
A. 45 infants
B. 50 infants
C. 55 infants
D. 65 infants
56.The community nurse is aware that the biological used in Expanded
Program on Immunization (EPI) should NOT be stored in the freezer?
A. DPT
B. Oral polio vaccine
C. Measles vaccine
D. MMR
57.It is the most effective way of controlling schistosomiasis in an endemic
area?
A. Use of molluscicides
B. Building of foot bridges
C. Proper use of sanitary toilets
D. Use of protective footwear, such as rubber boots
58.Several clients is newly admitted and diagnosed with leprosy. Which of the
following clients should be classified as a case of multibacillary leprosy?
A. 3 skin lesions, negative slit skin smear
B. 3 skin lesions, positive slit skin smear
C. 5 skin lesions, negative slit skin smear
D. 5 skin lesions, positive slit skin smear
59.Nurses are aware that diagnosis of leprosy is highly dependent on
recognition of symptoms. Which of the following is an early sign of leprosy?
A. Macular lesions
B. Inability to close eyelids
C. Thickened painful nerves
D. Sinking of the nosebridge
60.Marie brought her 10 month old infant for consultation because of fever,
started 4 days prior to consultation. In determining malaria risk, what will you
do?
A. Perform a tourniquet test.
B. Ask where the family resides.
C. Get a specimen for blood smear.
D. Ask if the fever is present everyday.
61.Susie brought her 4 years old daughter to the RHU because of cough and
colds. Following the IMCI assessment guide, which of the following is a
danger sign that indicates the need for urgent referral to a hospital?
A. Inability to drink
B. High grade fever
C. Signs of severe dehydration
D. Cough for more than 30 days
62.Jimmy a 2-year old child revealed “baggy pants”. As a nurse, using the IMCI
guidelines, how will you manage Jimmy?
A. Refer the child urgently to a hospital for confinement.
B. Coordinate with the social worker to enroll the child in a feeding
program.
C. Make a teaching plan for the mother, focusing on menu planning for her
child.
D. Assess and treat the child for health problems like infections and
intestinal parasitism.
63.Gina is using Oresol in the management of diarrhea of her 3-year old child.
She asked you what to do if her child vomits. As a nurse you will tell her to:
A. Bring the child to the nearest hospital for further assessment.
B. Bring the child to the health center for intravenous fluid therapy.
C. Bring the child to the health center for assessment by the physician.
D. Let the child rest for 10 minutes then continue giving Oresol more
slowly.
64.Nikki a 5-month old infant was brought by his mother to the health center
because of diarrhea for 4 to 5 times a day. Her skin goes back slowly after a
skin pinch and her eyes are sunken. Using the IMCI guidelines, you will
classify this infant in which category?
A. No signs of dehydration
B. Some dehydration
C. Severe dehydration
D. The data is insufficient.
65.Chris a 4-month old infant was brought by her mother to the health center
because of cough. His respiratory rate is 42/minute. Using the Integrated
Management of Child Illness (IMCI) guidelines of assessment, his breathing is
considered as:
A. Fast
B. Slow
C. Normal
D. Insignificant
66.Maylene had just received her 4th dose of tetanus toxoid. She is aware that
her baby will have protection against tetanus for
A. 1 year
B. 3 years
C. 5 years
D. Lifetime
67.Nurse Ron is aware that unused BCG should be discarded after how many
hours of reconstitution?
A. 2 hours
B. 4 hours
C. 8 hours
D. At the end of the day
68.The nurse explains to a breastfeeding mother that breast milk is sufficient
for all of the baby’s nutrient needs only up to:
A. 5 months
B. 6 months
C. 1 year
D. 2 years
69.Nurse Ron is aware that the gestational age of a conceptus that is
considered viable (able to live outside the womb) is:
A. 8 weeks
B. 12 weeks
C. 24 weeks
D. 32 weeks
70.When teaching parents of a neonate the proper position for the neonate’s
sleep, the nurse Patricia stresses the importance of placing the neonate on
his back to reduce the risk of which of the following?
A. Aspiration
B. Sudden infant death syndrome (SIDS)
C. Suffocation
D. Gastroesophageal reflux (GER)
71.Which finding might be seen in baby James a neonate suspected of having
an infection?
A. Flushed cheeks
B. Increased temperature
C. Decreased temperature
D. Increased activity level
72.Baby Jenny who is small-for-gestation is at increased risk during the
transitional period for which complication?
A. Anemia probably due to chronic fetal hyposia
B. Hyperthermia due to decreased glycogen stores
C. Hyperglycemia due to decreased glycogen stores
D. Polycythemia probably due to chronic fetal hypoxia
73.Marjorie has just given birth at 42 weeks’ gestation. When the nurse
assessing the neonate, which physical finding is expected?
A. A sleepy, lethargic baby
B. Lanugo covering the body
C. Desquamation of the epidermis
D. Vernix caseosa covering the body
74.After reviewing the Myrna’s maternal history of magnesium sulfate during
labor, which condition would nurse Richard anticipate as a potential problem
in the neonate?
A. Hypoglycemia
B. Jitteriness
C. Respiratory depression
D. Tachycardia
75.Which symptom would indicate the Baby Alexandra was adapting
appropriately to extra-uterine life without difficulty?
A. Nasal flaring
B. Light audible grunting
C. Respiratory rate 40 to 60 breaths/minute
D. Respiratory rate 60 to 80 breaths/minute
76. When teaching umbilical cord care for Jennifer a new mother, the nurse
Jenny would include which information?
A. Apply peroxide to the cord with each diaper change
B. Cover the cord with petroleum jelly after bathing
C. Keep the cord dry and open to air
D. Wash the cord with soap and water each day during a tub bath.
77.Nurse John is performing an assessment on a neonate. Which of the
following findings is considered common in the healthy neonate?
A. Simian crease
B. Conjunctival hemorrhage
C. Cystic hygroma
D. Bulging fontanelle
78.Dr. Esteves decides to artificially rupture the membranes of a mother who
is on labor. Following this procedure, the nurse Hazel checks the fetal heart
tones for which the following reasons?
A. To determine fetal well-being.
B. To assess for prolapsed cord
C. To assess fetal position
D. To prepare for an imminent delivery.
79.Which of the following would be least likely to indicate anticipated bonding
behaviors by new parents?
A. The parents’ willingness to touch and hold the new born.
B. The parent’s expression of interest about the size of the new born.
C. The parents’ indication that they want to see the newborn.
D. The parents’ interactions with each other.
80.Following a precipitous delivery, examination of the client’s vagina reveals
a fourth-degree laceration. Which of the following would be contraindicated
when caring for this client?
A. Applying cold to limit edema during the first 12 to 24 hours.
B. Instructing the client to use two or more peripads to cushion the area.
C. Instructing the client on the use of sitz baths if ordered.
D. Instructing the client about the importance of perineal (kegel) exercises.
81. A pregnant woman accompanied by her husband, seeks admission to the
labor and delivery area. She states that she’s in labor and says she attended
the facility clinic for prenatal care. Which question should the nurse Oliver ask
her first?
A. “Do you have any chronic illnesses?”
B. “Do you have any allergies?”
C. “What is your expected due date?”
D. “Who will be with you during labor?”
82.A neonate begins to gag and turns a dusky color. What should the nurse do
first?
A. Calm the neonate.
B. Notify the physician.
C. Provide oxygen via face mask as ordered
D. Aspirate the neonate’s nose and mouth with a bulb syringe.
83. When a client states that her “water broke,” which of the following actions
would be inappropriate for the nurse to do?
A. Observing the pooling of straw-colored fluid.
B. Checking vaginal discharge with nitrazine paper.
C. Conducting a bedside ultrasound for an amniotic fluid index.
D. Observing for flakes of vernix in the vaginal discharge.
84. A baby girl is born 8 weeks premature. At birth, she has no spontaneous
respirations but is successfully resuscitated. Within several hours she
develops respiratory grunting, cyanosis, tachypnea, nasal flaring, and
retractions. She’s diagnosed with respiratory distress syndrome, intubated,
and placed on a ventilator. Which nursing action should be included in the
baby’s plan of care to prevent retinopathy of prematurity?
A. Cover his eyes while receiving oxygen.
B. Keep her body temperature low.
C. Monitor partial pressure of oxygen (Pao2) levels.
D. Humidify the oxygen.
85. Which of the following is normal newborn calorie intake?
A. 110 to 130 calories per kg.
B. 30 to 40 calories per lb of body weight.
C. At least 2 ml per feeding
D. 90 to 100 calories per kg
86. Nurse John is knowledgeable that usually individual twins will grow
appropriately and at the same rate as singletons until how many weeks?
A. 16 to 18 weeks
B. 18 to 22 weeks
C. 30 to 32 weeks
D. 38 to 40 weeks
87. Which of the following classifications applies to monozygotic twins for
whom the cleavage of the fertilized ovum occurs more than 13 days after
fertilization?
A. conjoined twins
B. diamniotic dichorionic twins
C. diamniotic monochorionic twin
D. monoamniotic monochorionic twins
88. Tyra experienced painless vaginal bleeding has just been diagnosed as
having a placenta previa. Which of the following procedures is usually
performed to diagnose placenta previa?
A. Amniocentesis
B. Digital or speculum examination
C. External fetal monitoring
D. Ultrasound
89. Nurse Arnold knows that the following changes in respiratory functioning
during pregnancy is considered normal:
A. Increased tidal volume
B. Increased expiratory volume
C. Decreased inspiratory capacity
D. Decreased oxygen consumption
90. Emily has gestational diabetes and it is usually managed by which of the
following therapy?
A. Diet
B. Long-acting insulin
C. Oral hypoglycemic
D. Oral hypoglycemic drug and insulin
91. Magnesium sulfate is given to Jemma with preeclampsia to prevent which
of the following condition?
A. Hemorrhage
B. Hypertension
C. Hypomagnesemia
D. Seizure
92. Cammile with sickle cell anemia has an increased risk for having a sickle
cell crisis during pregnancy. Aggressive management of a sickle cell crisis
includes which of the following measures?
A. Antihypertensive agents
B. Diuretic agents
C. I.V. fluids
D. Acetaminophen (Tylenol) for pain
93. Which of the following drugs is the antidote for magnesium toxicity?
A. Calcium gluconate (Kalcinate)
B. Hydralazine (Apresoline)
C. Naloxone (Narcan)
D. Rho (D) immune globulin (RhoGAM)
94. Marlyn is screened for tuberculosis during her first prenatal visit. An
intradermal injection of purified protein derivative (PPD) of the tuberculin
bacilli is given. She is considered to have a positive test for which of the
following results?
A. An indurated wheal under 10 mm in diameter appears in 6 to 12 hours.
B. An indurated wheal over 10 mm in diameter appears in 48 to 72 hours.
C. A flat circumcised area under 10 mm in diameter appears in 6 to 12
hours.
D. A flat circumcised area over 10 mm in diameter appears in 48 to 72
hours.
95. Dianne, 24 year-old is 27 weeks’ pregnant arrives at her physician’s office
with complaints of fever, nausea, vomiting, malaise, unilateral flank pain, and
costovertebral angle tenderness. Which of the following diagnoses is most
likely?
A. Asymptomatic bacteriuria
B. Bacterial vaginosis
C. Pyelonephritis
D. Urinary tract infection (UTI)
96. Rh isoimmunization in a pregnant client develops during which of the
following conditions?
A. Rh-positive maternal blood crosses into fetal blood, stimulating
fetal antibodies.
B. Rh-positive fetal blood crosses into maternal blood,
stimulating maternal antibodies.
C. Rh-negative fetal blood crosses into maternal blood,
stimulating maternal antibodies.
D. Rh-negative maternal blood crosses into fetal blood, stimulating
fetal antibodies.
97. To promote comfort during labor, the nurse John advises a client to
assume certain positions and avoid others. Which position may cause
maternal hypotension and fetal hypoxia?
A. Lateral position
B. Squatting position
C. Supine position
D. Standing position
98. Celeste who used heroin during her pregnancy delivers a neonate. When
assessing the neonate, the nurse Lhynnette expects to find:
A. Lethargy 2 days after birth.
B. Irritability and poor sucking.
C. A flattened nose, small eyes, and thin lips.
D. Congenital defects such as limb anomalies.
99. The uterus returns to the pelvic cavity in which of the following time
frames?
A. 7th to 9th day postpartum.
B. 2 weeks postpartum.
C. End of 6th week postpartum.
D. When the lochia changes to alba.
100. Maureen, a primigravida client, age 20, has just completed a difficult,
forceps-assisted delivery of twins. Her labor was unusually long and required
oxytocin (Pitocin) augmentation. The nurse who’s caring for her should stay
alert for:
A. Uterine inversion
B. Uterine atony
C. Uterine involution
D. Uterine discomfort
Answers and Rationales
1. Answer: (A) Inevitable. An inevitable abortion is termination of pregnancy
that cannot be prevented. Moderate to severe bleeding with mild cramping
and cervical dilation would be noted in this type of abortion.
2. Answer: (B) History of syphilis. Maternal infections such as syphilis,
toxoplasmosis, and rubella are causes of spontaneous abortion.
3. Answer: (C) Monitoring apical pulse. Nursing care for the client with a
possible ectopic pregnancy is focused on preventing or identifying
hypovolemic shock and controlling pain. An elevated pulse rate is an
indicator of shock.
4. Answer: (B) Increased caloric intake. Glucose crosses the placenta, but
insulin does not. High fetal demands for glucose, combined with the
insulin resistance caused by hormonal changes in the last half of
pregnancy can result in elevation of maternal blood glucose levels. This
increases the mother’s demand for insulin and is referred to as the
diabetogenic effect of pregnancy.
5. Answer: (A) Excessive fetal activity. The most common signs and
symptoms of hydatidiform mole includes elevated levels of human
chorionic gonadotropin, vaginal bleeding, larger than normal uterus for
gestational age, failure to detect fetal heart activity even with sensitive
instruments, excessive nausea and vomiting, and early development of
pregnancy-induced hypertension. Fetal activity would not be noted.
6. Answer: (B) Absent patellar reflexes. Absence of patellar reflexes is an
indicator of hypermagnesemia, which requires administration of calcium
gluconate.
7. Answer: (C) Presenting part in 2 cm below the plane of the ischial
spines. Fetus at station plus two indicates that the presenting part is 2 cm
below the plane of the ischial spines.
8. Answer: (A) Contractions every 1 ½ minutes lasting 70-80
seconds. Contractions every 1 ½ minutes lasting 70-80 seconds,
is indicative of hyperstimulation of the uterus, which could result in injury
to the mother and the fetus if Pitocin is not discontinued.
9. Answer: (C) EKG tracings. A potential side effect of calcium gluconate
administration is cardiac arrest. Continuous monitoring of cardiac activity
(EKG) throught administration of calcium gluconate is an essential part of
care.
10. Answer: (D) First low transverse caesarean was for breech position. Fetus in
this pregnancy is in a vertex presentation. This type of client has no obstetrical
indication for a caesarean section as she did with her first caesarean
delivery.
11. Answer: (A) Talk to the mother first and then to the toddler. When dealing
with a crying toddler, the best approach is to talk to the mother and ignore
the toddler first. This approach helps the toddler get used to the nurse
before she attempts any procedures. It also gives the toddler an
opportunity to see that the mother trusts the nurse.
12. Answer: (D) Place the infant’s arms in soft elbow restraints. Soft restraints
from the upper arm to the wrist prevent the infant from touching her lip
but allow him to hold a favorite item such as a blanket. Because they
could damage the operative site, such as objects as pacifiers, suction
catheters, and small spoons shouldn’t be placed in a baby’s mouth after
cleft repair. A baby in a prone position may rub her face on the sheets and
traumatize the operative site. The suture line should be cleaned gently to
prevent infection, which could interfere with healing and damage the
cosmetic appearance of the repair.
13. Answer: (B) Allow the infant to rest before feeding. Because feeding
requires so much energy, an infant with heart failure should rest before
feeding.
14. Answer: (C) Iron-rich formula only. The infants at age 5 months should
receive iron-rich formula and that they shouldn’t receive solid food, even
baby food until age 6 months.
15. Answer: (D) 10 months. A 10 month old infant can sit alone and
understands object permanence, so he would look for the hidden toy. At
age 4 to 6 months, infants can’t sit securely alone. At age 8 months,
infants can sit securely alone but cannot understand the permanence of
objects.
16. Answer: (D) Public health nursing focuses on preventive, not curative,
services. The catchments area in PHN consists of a residential community,
many of whom are well individuals who have greater need for preventive
rather than curative services.
17. Answer: (B) Efficiency. Efficiency is determining whether the goals were
attained at the least possible cost.
18. Answer: (D) Rural Health Unit. R.A. 7160 devolved basic health services
to local government units (LGU’s ). The public health nurse is an employee
of the LGU.
19. Answer: (A) Mayor. The local executive serves as the chairman of the
Municipal Health Board.
20. Answer: (A) 1. Each rural health midwife is given a population
assignment of about 5,000.
21. Answer: (B) Health education and community organizing are necessary in
providing community health services. The community health nurse develops
the health capability of people through health education and community
organizing activities.
22. Answer: (B) Measles. Presidential Proclamation No. 4 is on the Ligtas
Tigdas Program.
23. Answer: (D) Core group formation. In core group formation, the nurse is
able to transfer the technology of community organizing to the potential or
informal community leaders through a training program.
24. Answer: (D) To maximize the community’s resources in dealing with health
problems. Community organizing is a developmental service, with the goal
of developing the people’s self-reliance in dealing with community health
problems. A, B and C are objectives of contributory objectives to this goal.
25. Answer: (D) Terminal. Tertiary prevention involves rehabilitation,
prevention of permanent disability and disability limitation appropriate for
convalescents, the disabled, complicated cases and the terminally ill
(those in the terminal stage of a disease).
26. Answer: (A) Intrauterine fetal death. Intrauterine fetal death, abruptio
placentae, septic shock, and amniotic fluid embolism may trigger normal
clotting mechanisms; if clotting factors are depleted, DIC may occur.
Placenta accreta, dysfunctional labor, and premature rupture of the
membranes aren’t associated with DIC.
27. Answer: (C) 120 to 160 beats/minute. A rate of 120 to 160 beats/minute in
the fetal heart appropriate for filling the heart with blood and pumping it
out to the system.
28. Answer: (A) Change the diaper more often. Decreasing the amount of time
the skin comes contact with wet soiled diapers will help heal the irritation.
29. Answer: (D) Endocardial cushion defect. Endocardial cushion defects are
seen most in children with Down syndrome, asplenia, or polysplenia.
30. Answer: (B) Decreased urine output. Decreased urine output may occur in
clients receiving I.V. magnesium and should be monitored closely to keep
urine output at greater than 30 ml/hour, because magnesium is excreted
through the kidneys and can easily accumulate to toxic levels.
31. Answer: (A) Menorrhagia. Menorrhagia is an excessive menstrual period.
32. Answer: (C) Blood typing. Blood type would be a critical value to have
because the risk of blood loss is always a potential complication during
the labor and delivery process. Approximately 40% of a woman’s cardiac
output is delivered to the uterus, therefore, blood loss can occur quite
rapidly in the event of uncontrolled bleeding.
33. Answer: (D) Physiologic anemia. Hemoglobin values and hematocrit
decrease during pregnancy as the increase in plasma volume exceeds the
increase in red blood cell production.
34. Answer: (D) A 2 year old infant with stridorous breath sounds, sitting up in his
mother’s arms and drooling. The infant with the airway emergency should be
treated first, because of the risk of epiglottitis.
35. Answer: (A) Placenta previa. Placenta previa with painless vaginal
bleeding.
36. Answer: (D) Early in the morning. Based on the nurse’s knowledge of
microbiology, the specimen should be collected early in the morning. The
rationale for this timing is that, because the female worm lays eggs at
night around the perineal area, the first bowel movement of the day will
yield the best results. The specific type of stool specimen used in the
diagnosis of pinworms is called the tape test.
37. Answer: (A) Irritability and seizures. Lead poisoning primarily affects the
CNS, causing increased intracranial pressure. This condition results in
irritability and changes in level of consciousness, as well as seizure
disorders, hyperactivity, and learning disabilities.
38. Answer: (D) “I really need to use the diaphragm and jelly most during the
middle of my menstrual cycle”. The woman must understand that, although
the “fertile” period is approximately mid-cycle, hormonal variations do
occur and can result in early or late ovulation. To be effective, the
diaphragm should be inserted before every intercourse.
39. Answer: (C) Restlessness. In a child, restlessness is the earliest sign of
hypoxia. Late signs of hypoxia in a child are associated with a change in
color, such as pallor or cyanosis.
40. Answer: (B) Walk one step ahead, with the child’s hand on the nurse’s
elbow. This procedure is generally recommended to follow in guiding a
person who is blind.
41. Answer: (A) Loud, machinery-like murmur. A loud, machinery-like murmur
is a characteristic finding associated with patent ductus arteriosus.
42. Answer: (C) More oxygen, and the newborn’s metabolic rate increases. When
cold, the infant requires more oxygen and there is an increase in metabolic
rate. Non-shievering thermogenesis is a complex process that increases
the metabolic rate and rate of oxygen consumption, therefore, the
newborn increase heat production.
43. Answer: (D) Voided. Before administering potassium I.V. to any client,
the nurse must first check that the client’s kidneys are functioning and
that the client is voiding. If the client is not voiding, the nurse should
withhold the  potassium and notify the physician.
44. Answer: (C) Laundry detergent. Eczema or dermatitis is an allergic skin
reaction caused by an offending allergen. The topical allergen that is the
most common causative factor is laundry detergent.
45. Answer: (A) 6 inches. This distance allows for easy flow of the formula by
gravity, but the flow will be slow enough not to overload the stomach too
rapidly.
46. Answer: (A) The older one gets, the more susceptible he becomes to the
complications of chicken pox. Chicken pox is usually more severe in adults
than in children. Complications, such as pneumonia, are higher in
incidence in adults.
47. Answer: (D) Consult a physician who may give them rubella
immunoglobulin. Rubella vaccine is made up of attenuated German
measles viruses. This is contraindicated in pregnancy. Immune globulin, a
specific prophylactic against German measles, may be given to pregnant
women.
48. Answer: (A) Contact tracing. Contact tracing is the most practical and
reliable method of finding possible sources of person-to-person
transmitted infections, such as sexually transmitted diseases.
49. Answer: (D) Leptospirosis. Leptospirosis is transmitted through contact
with the skin or mucous membrane with water or moist soil contaminated
with urine of infected animals, like rats.
50. Answer: (B) Cholera. Passage of profuse watery stools is the major
symptom of cholera. Both amebic and bacillary dysentery are
characterized by the presence of blood and/or mucus in the stools.
Giardiasis is characterized by fat malabsorption and, therefore,
steatorrhea.
51. Answer: (A) Hemophilus influenzae. Hemophilus meningitis is unusual
over the age of 5 years. In developing countries, the peak incidence is in
children less than 6 months of age. Morbillivirus is the etiology of
measles. Streptococcus pneumoniae and Neisseria meningitidis may
cause meningitis, but age distribution is not specific in young children.
52. Answer: (B) Buccal mucosa. Koplik’s spot may be seen on the mucosa of
the mouth or the throat.
53. Answer: (A) 3 seconds. Adequate blood supply to the area allows the
return of the color of the nailbed within 3 seconds.
54. Answer: (B) Severe dehydration. The order of priority in the management
of severe dehydration is as follows: intravenous fluid therapy, referral to a
facility where IV fluids can be initiated within 30 minutes, Oresol or
nasogastric tube. When the foregoing measures are not possible or
effective, then urgent referral to the hospital is done.
55. Answer: (A) 45 infants. To estimate the number of infants, multiply total
population by 3%.
56. Answer: (A) DPT. DPT is sensitive to freezing. The appropriate storage
temperature of DPT is 2 to 8° C only. OPV and measles vaccine are
highly sensitive to heat and require freezing. MMR is not an immunization
in the Expanded Program on Immunization.
57. Answer: (C) Proper use of sanitary toilets. The ova of the parasite get out of
the human body together with feces. Cutting the cycle at this stage is the
most effective way of preventing the spread of the disease to susceptible
hosts.
58. Answer: (D) 5 skin lesions, positive slit skin smear. A multibacillary leprosy
case is one who has a positive slit skin smear and at least 5 skin lesions.
59. Answer: (C) Thickened painful nerves. The lesion of leprosy is not macular.
It is characterized by a change in skin color (either reddish or whitish) and
loss of sensation, sweating and hair growth over the lesion. Inability to
close the eyelids (lagophthalmos) and sinking of the nosebridge are late
symptoms.
60. Answer: (B) Ask where the family resides. Because malaria is endemic, the
first question to determine malaria risk is where the client’s family resides.
If the area of residence is not a known endemic area, ask if the child had
traveled within the past 6 months, where she was brought and whether
she stayed overnight in that area.
61. Answer: (A) Inability to drink. A sick child aged 2 months to 5 years must
be referred urgently to a hospital if he/she has one or more of the
following signs: not able to feed or drink, vomits everything, convulsions,
abnormally sleepy or difficult to awaken.
62. Answer: (A) Refer the child urgently to a hospital for confinement. “Baggy
pants” is a sign of severe marasmus. The best management is urgent
referral to a hospital.
63. Answer: (D) Let the child rest for 10 minutes then continue giving Oresol more
slowly. If the child vomits persistently, that is, he vomits everything that he
takes in, he has to be referred urgently to a hospital. Otherwise, vomiting is
managed by letting the child rest for 10 minutes and then continuing with
Oresol administration. Teach the mother to give Oresol more slowly.
64. Answer: (B) Some dehydration. Using the assessment guidelines of IMCI,
a child (2 months to 5 years old) with diarrhea is classified as having
SOME DEHYDRATION if he shows 2 or more of the following signs:
restless or irritable, sunken eyes, the skin goes back slow after a skin
pinch.
65. Answer: (C) Normal. In IMCI, a respiratory rate of 50/minute or more is
fast breathing for an infant aged 2 to 12 months.
66. Answer: (A) 1 year. The baby will have passive natural immunity by
placental transfer of antibodies. The mother will have active artificial
immunity lasting for about 10 years. 5 doses will give the mother lifetime
protection.
67. Answer: (B) 4 hours. While the unused portion of other biologicals in EPI
may be given until the end of the day, only BCG is discarded 4 hours
after reconstitution. This is why BCG immunization is scheduled only in
the morning.
68. Answer: (B) 6 months. After 6 months, the baby’s nutrient needs,
especially the baby’s iron requirement, can no longer be provided by
mother’s milk alone.
69. Answer: (C) 24 weeks. At approximately 23 to 24 weeks’ gestation, the
lungs are developed enough to sometimes maintain extrauterine life. The
lungs are the most immature system during the gestation period. Medical
care for premature labor begins much earlier (aggressively at 21 weeks’
gestation)
70. Answer: (B) Sudden infant death syndrome (SIDS). Supine positioning is
recommended to reduce the risk of SIDS in infancy. The risk of aspiration
is slightly increased with the supine position. Suffocation would be less
likely with an infant supine than prone and the position for GER requires
the head of the bed to be elevated.
71. Answer: (C) Decreased temperature. Temperature instability, especially
when it results in a low temperature in the neonate, may be a sign of
infection. The neonate’s color often changes with an infection process but
generally becomes ashen or mottled. The neonate with an infection will
usually show a decrease in activity level or lethargy.
72. Answer: (D) Polycythemia probably due to chronic fetal hypoxia. The small-
for-gestation neonate is at risk for developing polycythemia during the
transitional period in an attempt to decreasehypoxia. The neonates are
also at increased risk for developing hypoglycemia and hypothermia due
to decreased glycogen stores.
73. Answer: (C) Desquamation of the epidermis. Postdate fetuses lose the
vernix caseosa, and the epidermis may become desquamated. These
neonates are usually very alert. Lanugo is missing in the postdate
neonate.
74. Answer: (C) Respiratory depression. Magnesium sulfate crosses the
placenta and adverse neonatal effects are respiratory depression,
hypotonia, and bradycardia. The serum blood sugar isn’t affected by
magnesium sulfate. The neonate would be floppy, not jittery.
75. Answer: (C) Respiratory rate 40 to 60 breaths/minute. A respiratory rate 40
to 60 breaths/minute is normal for a neonate during the transitional
period. Nasal flaring, respiratory rate more than 60 breaths/minute, and
audible grunting are signs of respiratory distress.
76. Answer: (C) Keep the cord dry and open to air. Keeping the cord dry and
open to air helps reduce infection and hastens drying. Infants aren’t given
tub bath but are sponged off until the cord falls off. Petroleum jelly
prevents the cord from drying and encourages infection. Peroxide could
be painful and isn’t recommended.
77. Answer: (B) Conjunctival hemorrhage. Conjunctival hemorrhages are
commonly seen in neonates secondary to the cranial pressure applied
during the birth process. Bulging fontanelles are a sign of intracranial
pressure. Simian creases are present in 40% of the neonates with trisomy
21. Cystic hygroma is a neck mass that can affect the airway.
78. Answer: (B) To assess for prolapsed cord. After a client has an amniotomy,
the nurse should assure that the cord isn’t prolapsed and that the baby
tolerated the procedure well. The most effective way to do this is to check
the fetal heart rate. Fetal well-being is assessed via a nonstress test. Fetal
position is determined by vaginal examination. Artificial rupture of
membranes doesn’t indicate an imminent delivery.
79. Answer: (D) The parents’ interactions with each other. Parental interaction
will provide the nurse with a good assessment of the stability of the
family’s home life but it has no indication for parental bonding.
Willingness to touch and hold the newborn, expressing interest about the
newborn’s size, and indicating a desire to see the newborn are behaviors
indicating parental bonding.
80. Answer: (B) Instructing the client to use two or more peripads to cushion the
area. Using two or more peripads would do little to reduce the pain or
promote perineal healing. Cold applications, sitz baths, and
Kegel exercises are important measures when the client has a fourth-
degree laceration.
81. Answer: (C) “What is your expected due date?” When obtaining the history
of a client who may be in labor, the nurse’s highest priority is to determine
her current status, particularly her due date, gravidity, and parity. Gravidity
and parity affect the duration of labor and the potential for labor
complications. Later, the nurse should ask about chronic illnesses,
allergies, and support persons.
82. Answer: (D) Aspirate the neonate’s nose and mouth with a bulb syringe. The
nurse’s first action should be to clear the neonate’s airway with a bulb
syringe. After the airway is clear and the neonate’s color improves, the
nurse should comfort and calm the neonate. If the problem recurs or the
neonate’s color doesn’t improve readily, the nurse should notify the
physician. Administering oxygen when the airway isn’t clear would be
ineffective.
83. Answer: (C) Conducting a bedside ultrasound for an amniotic fluid index. It
isn’t within a nurse’s scope of practice to perform and interpret a bedside
ultrasound under these conditions and without specialized training.
Observing for pooling of straw-colored fluid, checking vaginal discharge
with nitrazine paper, and observing for flakes of vernix are appropriate
assessments for determining whether a client has ruptured membranes.
84. Answer: (C) Monitor partial pressure of oxygen (Pao2) levels. Monitoring
PaO2 levels and reducing the oxygen concentration to keep PaO2 within
normal limits reduces the risk of retinopathy of prematurity in a premature
infant receiving oxygen. Covering the infant’s eyes and humidifying the
oxygen don’t reduce the risk of retinopathy of prematurity. Because
cooling increases the risk of acidosis, the infant should be kept warm so
that his respiratory distress isn’t aggravated.
85. Answer: (A) 110 to 130 calories per kg. Calories per kg is the accepted way
of determined appropriate nutritional intake for a newborn. The
recommended calorie requirement is 110 to 130 calories per kg of
newborn body weight. This level will maintain a consistent blood glucose
level and provide enough calories for continued growth and development.
86. Answer: (C) 30 to 32 weeks. Individual twins usually grow at the same rate
as singletons until 30 to 32 weeks’ gestation, then twins don’t’ gain weight
as rapidly as singletons of the same gestational age. The placenta can no
longer keep pace with the nutritional requirements of both fetuses after 32
weeks, so there’s some growth retardation in twins if they remain in utero
at 38 to 40 weeks.
87. Answer: (A) conjoined twins. The type of placenta that develops in
monozygotic twins depends on the time at which cleavage of the ovum
occurs. Cleavage in conjoined twins occurs more than 13 days after
fertilization. Cleavage that occurs less than 3 day after fertilization results
in diamniotic dicchorionic twins. Cleavage that occurs between days 3
and 8 results in diamniotic monochorionic twins. Cleavage that occurs
between days 8 to 13 result in monoamniotic monochorionic twins.
88. Answer: (D) Ultrasound. Once the mother and the fetus are stabilized,
ultrasound evaluation of the placenta should be done to determine the
cause of the bleeding. Amniocentesis is contraindicated in placenta
previa. A digital or speculum examination shouldn’t be done as this may
lead to severe bleeding or hemorrhage. External fetal monitoring won’t
detect a placenta previa, although it will detect fetal distress, which may
result from blood loss or placenta separation.
89. Answer: (A) Increased tidal volume. A pregnant client breathes deeper,
which increases the tidal volume of gas moved in and out of the
respiratory tract with each breath. The expiratory volume and residual
volume decrease as the pregnancy progresses. The inspiratory capacity
increases during pregnancy. The increased oxygen consumption in the
pregnant client is 15% to 20% greater than in the nonpregnant state.
90. Answer: (A) Diet. Clients with gestational diabetes are usually managed
by diet alone to control their glucose intolerance. Oral hypoglycemic drugs
are contraindicated in pregnancy. Long-acting insulin usually isn’t needed
for blood glucose control in the client with gestational diabetes.
91. Answer: (D) Seizure. The anticonvulsant mechanism of magnesium is
believes to depress seizure foci in the brain and peripheral neuromuscular
blockade. Hypomagnesemia isn’t a complication of preeclampsia.
Antihypertensive drug other than magnesium are preferred for sustained
hypertension. Magnesium doesn’t help prevent hemorrhage in
preeclamptic clients.
92. Answer: (C) I.V. fluids. A sickle cell crisis during pregnancy is usually
managed by exchange transfusion oxygen, and L.V. Fluids. The client
usually needs a stronger analgesic than acetaminophen to control the
pain of a crisis. Antihypertensive drugs usually aren’t necessary. Diuretic
wouldn’t be used unless fluid overload resulted.
93. Answer: (A) Calcium gluconate (Kalcinate). Calcium gluconate is the
antidote for magnesium toxicity. Ten milliliters of 10% calcium gluconate
is given L.V. push over 3 to 5 minutes. Hydralazine is given for sustained
elevated blood pressure in preeclamptic clients. Rho (D) immune globulin
is given to women with Rh-negative blood to prevent antibody formation
from RH-positive conceptions. Naloxone is used to correct narcotic
toxicity.
94. Answer: (B) An indurated wheal over 10 mm in diameter appears in 48 to 72
hours. A positive PPD result would be an indurated wheal over 10 mm in
diameter that appears in 48 to 72 hours. The area must be a raised wheal,
not a flat circumcised area to be considered positive.
95. Answer: (C) Pyelonephritis. The symptoms indicate acute pyelonephritis,
a serious condition in a pregnant client. UTI symptoms include dysuria,
urgency, frequency, and suprapubic tenderness. Asymptomatic bacteriuria
doesn’t cause symptoms. Bacterial vaginosis causes milky white vaginal
discharge but no systemic symptoms.
96. Answer: (B) Rh-positive fetal blood crosses into maternal blood, stimulating
maternal antibodies. Rh isoimmunization occurs when Rh-positive fetal
blood cells cross into the maternal circulation and stimulate maternal
antibody production. In subsequent pregnancies with Rh-positive fetuses,
maternal antibodies may cross back into the fetal circulation and destroy
the fetal blood cells.
97. Answer: (C) Supine position. The supine position causes compression of
the client’s aorta and inferior vena cava by the fetus. This, in turn, inhibits
maternal circulation, leading to maternal hypotension and, ultimately, fetal
hypoxia. The other positions promote comfort and aid labor progress. For
instance, the lateral, or side-lying, position improves maternal and fetal
circulation, enhances comfort, increases maternal relaxation, reduces
muscle tension, and eliminates pressure points. The squatting position
promotes comfort by taking advantage of gravity. The standing position
also takes advantage of gravity and aligns the fetus with the pelvic angle.
98. Answer: (B) Irritability and poor sucking. Neonates of heroin-addicted
mothers are physically dependent on the drug and experience withdrawal
when the drug is no longer supplied. Signs of heroin withdrawal include
irritability, poor sucking, and restlessness. Lethargy isn’t associated with
neonatal heroin addiction. A flattened nose, small eyes, and thin lips are
seen in infants with fetal alcohol syndrome. Heroin use during pregnancy
hasn’t been linked to specific congenital anomalies.
99. Answer: (A) 7th to 9th day postpartum. The normal involutional process
returns the uterus to the pelvic cavity in 7 to 9 days. A significant
involutional complication is the failure of the uterus to return to the pelvic
cavity within the prescribed time period. This is known as subinvolution.
100. Answer: (B) Uterine atony. Multiple fetuses, extended labor stimulation
with oxytocin, and traumatic delivery commonly are associated with
uterine atony, which may lead to postpartum hemorrhage. Uterine
inversion may precede or follow delivery and commonly results from
apparent excessive traction on the umbilical cord and attempts to deliver
the placenta manually. Uterine involution and some uterine discomfort are
normal after delivery.
PNLE III for Care of Clients with
Physiologic and Psychosocial
Alterations (Part 1)
1. Nurse Michelle should know that the drainage is normal 4 days after
a sigmoid colostomy when the stool is:
A. Green liquid
B. Solid formed
C. Loose, bloody
D. Semiformed
2. Where would nurse Kristine place the call light for a male client with a right-
sided brain attack and left homonymous hemianopsia?
A. On the client’s right side
B. On the client’s left side
C. Directly in front of the client
D. Where the client like
3. A male client is admitted to the emergency department following
an accident. What are the first nursing actions of the nurse?
A. Check respiration, circulation, neurological response.
B. Align the spine, check pupils, and check for hemorrhage.
C. Check respirations, stabilize spine, and check circulation.
D. Assess level of consciousness and circulation.
4. In evaluating the effect of nitroglycerin, Nurse Arthur should know that
it reduces preload and relieves angina by:
A. Increasing contractility and slowing heart rate.
B. Increasing AV conduction and heart rate.
C. Decreasing contractility and oxygen consumption.
D. Decreasing venous return through vasodilation.
5. Nurse Patricia finds a female client who is post-myocardial infarction
(MI) slumped on the side rails of the bed and unresponsive to shaking
or shouting. Which is the nurse next action?
A. Call for help and note the time.
B. Clear the airway
C. Give two sharp thumps to the precordium, and check the pulse.
D. Administer two quick blows.
6. Nurse Monett is caring for a client recovering from gastro-
intestinal bleeding. The nurse should:
A. Plan care so the client can receive 8 hours of uninterrupted sleep each
night.
B. Monitor vital signs every 2 hours.
C. Make sure that the client takes food and medications at
prescribed intervals.
D. Provide milk every 2 to 3 hours.
7. A male client was on warfarin (Coumadin) before admission, and has been
receiving heparin I.V. for 2 days. The partial thromboplastin time (PTT) is 68
seconds. What should Nurse Carla do?
A. Stop the I.V. infusion of heparin and notify the physician.
B. Continue treatment as ordered.
C. Expect the warfarin to increase the PTT.
D. Increase the dosage, because the level is lower than normal.
8. A client undergone ileostomy, when should the drainage appliance
be applied to the stoma?
A. 24 hours later, when edema has subsided.
B. In the operating room.
C. After the ileostomy begin to function.
D. When the client is able to begin self-care procedures.
9. A client undergone spinal anesthetic, it will be important that the
nurse immediately position the client in:
A. On the side, to prevent obstruction of airway by tongue.
B. Flat on back.
C. On the back, with knees flexed 15 degrees.
D. Flat on the stomach, with the head turned to the side.
10.While monitoring a male client several hours after a motor
vehicle accident, which assessment data suggest increasing
intracranial pressure?
A. Blood pressure is decreased from 160/90 to 110/70.
B. Pulse is increased from 87 to 95, with an occasional skipped beat.
C. The client is oriented when aroused from sleep, and goes back to sleep
immediately.
D. The client refuses dinner because of anorexia.
11.Mrs. Cruz, 80 years old is diagnosed with pneumonia. Which of
the following symptoms may appear first?
A. Altered mental status and dehydration
B. Fever and chills
C. Hemoptysis and Dyspnea
D. Pleuritic chest pain and cough
12. A male client has active tuberculosis (TB). Which of the following
symptoms will be exhibit?
A. Chest and lower back pain
B. Chills, fever, night sweats, and hemoptysis
C. Fever of more than 104°F (40°C) and nausea
D. Headache and photophobia
13. Mark, a 7-year-old client is brought to the emergency department.
He’s tachypneic and afebrile and has a respiratory rate of 36 breaths/minute
and has a nonproductive cough. He recently had a cold. Form this history; the
client may have which of the following conditions?
A. Acute asthma
B. Bronchial pneumonia
C. Chronic obstructive pulmonary disease (COPD)
D. Emphysema
14. Marichu was given morphine sulfate for pain. She is sleeping and
her respiratory rate is 4 breaths/minute. If action isn’t taken quickly, she might
have which of the following reactions?
A. Asthma attack
B. Respiratory arrest
C. Seizure
D. Wake up on his own
15. A 77-year-old male client is admitted for elective knee surgery.
Physical examination reveals shallow respirations but no sign of respiratory
distress. Which of the following is a normal physiologic change related to
aging?
A. Increased elastic recoil of the lungs
B. Increased number of functional capillaries in the alveoli
C. Decreased residual volume
D. Decreased vital capacity
16. Nurse John is caring for a male client receiving lidocaine I.V. Which factor
is the most relevant to administration of this medication?
A. Decrease in arterial oxygen saturation (SaO2) when measured with
a pulse oximeter.
B. Increase in systemic blood pressure.
C. Presence of premature ventricular contractions (PVCs) on a
cardiac monitor.
D. Increase in intracranial pressure (ICP).
17. Nurse Ron is caring for a male client taking an anticoagulant. The
nurse should teach the client to:
A. Report incidents of diarrhea.
B. Avoid foods high in vitamin K
C. Use a straight razor when shaving.
D. Take aspirin to pain relief.
18. Nurse Lhynnette is preparing a site for the insertion of an I.V. catheter.
The nurse should treat excess hair at the site by:
A. Leaving the hair intact
B. Shaving the area
C. Clipping the hair in the area
D. Removing the hair with a depilatory.
19. Nurse Michelle is caring for an elderly female with osteoporosis.
When teaching the client, the nurse should include information about which
major complication:
A. Bone fracture
B. Loss of estrogen
C. Negative calcium balance
D. Dowager’s hump
20. Nurse Len is teaching a group of women to perform BSE. The nurse
should explain that the purpose of performing the examination is to discover:
A. Cancerous lumps
B. Areas of thickness or fullness
C. Changes from previous examinations.
D. Fibrocystic masses
21. When caring for a female client who is being treated for hyperthyroidism, it
is important to:
A. Provide extra blankets and clothing to keep the client warm.
B. Monitor the client for signs of restlessness, sweating, and
excessive weight loss during thyroid replacement therapy.
C. Balance the client’s periods of activity and rest.
D. Encourage the client to be active to prevent constipation.
22. Nurse Kris is teaching a client with history of atherosclerosis. To
decrease the risk of atherosclerosis, the nurse should encourage the client to:
A. Avoid focusing on his weight.
B. Increase his activity level.
C. Follow a regular diet.
D. Continue leading a high-stress lifestyle.
23. Nurse Greta is working on a surgical floor. Nurse Greta must logroll a
client following a:
A. Laminectomy
B. Thoracotomy
C. Hemorrhoidectomy
D. Cystectomy.
24. A 55-year old client underwent cataract removal with intraocular lens
implant. Nurse Oliver is giving the client discharge instructions. These
instructions should include which of the following?
A. Avoid lifting objects weighing more than 5 lb (2.25 kg).
B. Lie on your abdomen when in bed
C. Keep rooms brightly lit.
D. Avoiding straining during bowel movement or bending at the waist.
25. George should be taught about testicular examinations during:
A. when sexual activity starts
B. After age 69
C. After age 40
D. Before age 20.
26. A male client undergone a colon resection. While turning him,
wound dehiscence with evisceration occurs. Nurse Trish first response is to:
A. Call the physician
B. Place a saline-soaked sterile dressing on the wound.
C. Take a blood pressure and pulse.
D. Pull the dehiscence closed.
27. Nurse Audrey is caring for a client who has suffered a
severe cerebrovascular accident. During routine assessment, the nurse
notices Cheyne- Strokes respirations. Cheyne-strokes respirations are:
A. A progressively deeper breaths followed by shallower breaths
with apneic periods.
B. Rapid, deep breathing with abrupt pauses between each breath.
C. Rapid, deep breathing and irregular breathing without pauses.
D. Shallow breathing with an increased respiratory rate.
28. Nurse Bea is assessing a male client with heart failure. The breath
sounds commonly auscultated in clients with heart failure are:
A. Tracheal
B. Fine crackles
C. Coarse crackles
D. Friction rubs
29. The nurse is caring for Kenneth experiencing an acute asthma attack.
The client stops wheezing and breath sounds aren’t audible. The reason for
this change is that:
A. The attack is over.
B. The airways are so swollen that no air cannot get through.
C. The swelling has decreased.
D. Crackles have replaced wheezes.
30. Mike with epilepsy is having a seizure. During the active seizure phase,
the nurse should:
A. Place the client on his back remove dangerous objects, and insert a bite
block.
B. Place the client on his side, remove dangerous objects, and insert a bite
block.
C. Place the client o his back, remove dangerous objects, and hold
down his arms.
D. Place the client on his side, remove dangerous objects, and protect
his head.
31. After insertion of a cheat tube for a pneumothorax, a client
becomes hypotensive with neck vein distention, tracheal shift, absent breath
sounds, and diaphoresis. Nurse Amanda suspects a tension pneumothorax
has occurred. What cause of tension pneumothorax should the nurse check
for?
A. Infection of the lung.
B. Kinked or obstructed chest tube
C. Excessive water in the water-seal chamber
D. Excessive chest tube drainage
32. Nurse Maureen is talking to a male client, the client begins choking on
his lunch. He’s coughing forcefully. The nurse should:
A. Stand him up and perform the abdominal thrust maneuver from behind.
B. Lay him down, straddle him, and perform the abdominal
thrust maneuver.
C. Leave him to get assistance
D. Stay with him but not intervene at this time.
33. Nurse Ron is taking a health history of an 84 year old client.
Which information will be most useful to the nurse for planning care?
A. General health for the last 10 years.
B. Current health promotion activities.
C. Family history of diseases.
D. Marital status.
34. When performing oral care on a comatose client, Nurse Krina should:
A. Apply lemon glycerin to the client’s lips at least every 2 hours.
B. Brush the teeth with client lying supine.
C. Place the client in a side lying position, with the head of the
bed lowered.
D. Clean the client’s mouth with hydrogen peroxide.
35. A 77-year-old male client is admitted with a diagnosis of dehydration
and change in mental status. He’s being hydrated with L.V. fluids. When the
nurse takes his vital signs, she notes he has a fever of 103°F (39.4°C) a
cough producing yellow sputum and pleuritic chest pain. The nurse suspects
this client may have which of the following conditions?
A. Adult respiratory distress syndrome (ARDS)
B. Myocardial infarction (MI)
C. Pneumonia
D. Tuberculosis
36. Nurse Oliver is working in a out patient clinic. He has been alerted that
there is an outbreak of tuberculosis (TB). Which of the following clients
entering the clinic today most likely to have TB?
A. A 16-year-old female high school student
B. A 33-year-old day-care worker
C. A 43-yesr-old homeless man with a history of alcoholism
D. A 54-year-old businessman
37. Virgie with a positive Mantoux test result will be sent for a chest X-ray.
The nurse is aware that which of the following reasons this is done?
A. To confirm the diagnosis
B. To determine if a repeat skin test is needed
C. To determine the extent of lesions
D. To determine if this is a primary or secondary infection
38. Kennedy with acute asthma showing inspiratory and expiratory wheezes
and a decreased forced expiratory volume should be treated with which of
the following classes of medication right away?
A. Beta-adrenergic blockers
B. Bronchodilators
C. Inhaled steroids
D. Oral steroids
39. Mr. Vasquez 56-year-old client with a 40-year history of smoking one to
two packs of cigarettes per day has a chronic cough producing thick sputum,
peripheral edema and cyanotic nail beds. Based on this information, he
most likely has which of the following conditions?
A. Adult respiratory distress syndrome (ARDS)
B. Asthma
C. Chronic obstructive bronchitis
D. Emphysema
Situation: Francis, age 46 is admitted to the hospital with diagnosis of
Chronic Lymphocytic Leukemia.
40. The treatment for patients with leukemia is bone marrow
transplantation. Which statement about bone marrow transplantation is not
correct?
A. The patient is under local anesthesia during the procedure
B. The aspirated bone marrow is mixed with heparin.
C. The aspiration site is the posterior or anterior iliac crest.
D. The recipient receives cyclophosphamide (Cytoxan) for 4
consecutive days before the procedure.
41. After several days of admission, Francis becomes disoriented and
complains of frequent headaches. The nurse in-charge first action would be:
A. Call the physician
B. Document the patient’s status in his charts.
C. Prepare oxygen treatment
D. Raise the side rails
42. During routine care, Francis asks the nurse, “How can I be anemic if
this disease causes increased my white blood cell production?” The nurse in-
charge best response would be that the increased number of white blood cells
(WBC) is:
A. Crowd red blood cells
B. Are not responsible for the anemia.
C. Uses nutrients from other cells
D. Have an abnormally short life span of cells.
43. Diagnostic assessment of Francis would probably not reveal:
A. Predominance of lymhoblasts
B. Leukocytosis
C. Abnormal blast cells in the bone marrow
D. Elevated thrombocyte counts
44. Robert, a 57-year-old client with acute arterial occlusion of the left
leg undergoes an emergency embolectomy. Six hours later, the nurse isn’t
able to obtain pulses in his left foot using Doppler ultrasound. The nurse
immediately notifies the physician, and asks her to prepare the client for
surgery. As the nurse enters the client’s room to prepare him, he states that he
won’t have any more surgery. Which of the following is the best initial
response by the nurse?
A. Explain the risks of not having the surgery
B. Notifying the physician immediately
C. Notifying the nursing supervisor
D. Recording the client’s refusal in the nurses’ notes
45. During the endorsement, which of the following clients should the on-
duty nurse assess first?
A. The 58-year-old client who was admitted 2 days ago with heart
failure, blood pressure of 126/76 mm Hg, and a respiratory rate of 22
breaths/minute.
B. The 89-year-old client with end-stage right-sided heart failure,
blood pressure of 78/50 mm Hg, and a “do not resuscitate” order
C. The 62-year-old client who was admitted 1 day ago
with thrombophlebitis and is receiving L.V. heparin
D. The 75-year-old client who was admitted 1 hour ago with new-
onset atrial fibrillation and is receiving L.V. dilitiazem (Cardizem)
46. Honey, a 23-year old client complains of substernal chest pain and
states that her heart feels like “it’s racing out of the chest”. She reports no
history of cardiac disorders. The nurse attaches her to a cardiac monitor and
notes sinus tachycardia with a rate of 136beats/minutes. Breath sounds are
clear and the respiratory rate is 26 breaths/minutes. Which of the following
drugs should the nurse question the client about using?
A. Barbiturates
B. Opioids
C. Cocaine
D. Benzodiazepines
47. A 51-year-old female client tells the nurse in-charge that she has found
a painless lump in her right breast during her monthly self-examination.
Which assessment finding would strongly suggest that this client’s lump is
cancerous?
A. Eversion of the right nipple and mobile mass
B. Nonmobile mass with irregular edges
C. Mobile mass that is soft and easily delineated
D. Nonpalpable right axillary lymph nodes
48. A 35-year-old client with vaginal cancer asks the nurse, “What is the
usual treatment for this type of cancer?” Which treatment should the nurse
name?
A. Surgery
B. Chemotherapy
C. Radiation
D. Immunotherapy
49. Cristina undergoes a biopsy of a suspicious lesion. The biopsy
report classifies the lesion according to the TNM staging system as follows:
TIS, N0, M0. What does this classification mean?
A. No evidence of primary tumor, no abnormal regional lymph nodes,
and no evidence of distant metastasis
B. Carcinoma in situ, no abnormal regional lymph nodes, and no evidence
of distant metastasis
C. Can’t assess tumor or regional lymph nodes and no evidence
of metastasis
D. Carcinoma in situ, no demonstrable metastasis of the regional
lymph nodes, and ascending degrees of distant metastasis
50. Lydia undergoes a laryngectomy to treat laryngeal cancer. When
teaching the client how to care for the neck stoma, the nurse should include
which instruction?
A. “Keep the stoma uncovered.”
B. “Keep the stoma dry.”
C. “Have a family member perform stoma care initially until you get used to
the procedure.”
D. “Keep the stoma moist.”
51. A 37-year-old client with uterine cancer asks the nurse, “Which is the
most common type of cancer in women?” The nurse replies that it’s breast
cancer. Which type of cancer causes the most deaths in women?
A. Breast cancer
B. Lung cancer
C. Brain cancer
D. Colon and rectal cancer
52. Antonio with lung cancer develops Horner’s syndrome when the
tumor invades the ribs and affects the sympathetic nerve ganglia. When
assessing for signs and symptoms of this syndrome, the nurse should note:
A. miosis, partial eyelid ptosis, and anhidrosis on the affected side of
the face.
B. chest pain, dyspnea, cough, weight loss, and fever.
C. arm and shoulder pain and atrophy of arm and hand muscles, both
on the affected side.
D. hoarseness and dysphagia.
53. Vic asks the nurse what PSA is. The nurse should reply that it stands for:
A. prostate-specific antigen, which is used to screen for prostate cancer.
B. protein serum antigen, which is used to determine protein levels.
C. pneumococcal strep antigen, which is a bacteria that
causes pneumonia.
D. Papanicolaou-specific antigen, which is used to screen for
cervical cancer.
54. What is the most important postoperative instruction that nurse Kate
must give a client who has just returned from the operating room after
receiving a subarachnoid block?
A. “Avoid drinking liquids until the gag reflex returns.”
B. “Avoid eating milk products for 24 hours.”
C. “Notify a nurse if you experience blood in your urine.”
D. “Remain supine for the time specified by the physician.”
55. A male client suspected of having colorectal cancer will require
which diagnostic study to confirm the diagnosis?
A. Stool Hematest
B. Carcinoembryonic antigen (CEA)
C. Sigmoidoscopy
D. Abdominal computed tomography (CT) scan
56. During a breast examination, which finding most strongly suggests that
the Luz has breast cancer?
A. Slight asymmetry of the breasts.
B. A fixed nodular mass with dimpling of the overlying skin
C. Bloody discharge from the nipple
D. Multiple firm, round, freely movable masses that change with
the menstrual cycle
57. A female client with cancer is being evaluated for possible metastasis.
Which of the following is one of the most common metastasis sites for
cancer cells?
A. Liver
B. Colon
C. Reproductive tract
D. White blood cells (WBCs)
58. Nurse Mandy is preparing a client for magnetic resonance imaging (MRI)
to confirm or rule out a spinal cord lesion. During the MRI scan, which of the
following would pose a threat to the client?
A. The client lies still.
B. The client asks questions.
C. The client hears thumping sounds.
D. The client wears a watch and wedding band.
59. Nurse Cecile is teaching a female client about preventing
osteoporosis. Which of the following teaching points is correct?
A. Obtaining an X-ray of the bones every 3 years is recommended to detect
bone loss.
B. To avoid fractures, the client should avoid strenuous exercise.
C. The recommended daily allowance of calcium may be found in a
wide variety of foods.
D. Obtaining the recommended daily allowance of calcium requires
taking a calcium supplement.
60. Before Jacob undergoes arthroscopy, the nurse reviews the
assessment findings for contraindications for this procedure. Which finding is
a contraindication?
A. Joint pain
B. Joint deformity
C. Joint flexion of less than 50%
D. Joint stiffness
61. Mr. Rodriguez is admitted with severe pain in the knees. Which form
of arthritis is characterized by urate deposits and joint pain, usually in the feet
and legs, and occurs primarily in men over age 30?
A. Septic arthritis
B. Traumatic arthritis
C. Intermittent arthritis
D. Gouty arthritis
62. A heparin infusion at 1,500 unit/hour is ordered for a 64-year-old client
with stroke in evolution. The infusion contains 25,000 units of heparin in 500
ml of saline solution. How many milliliters per hour should be given?
A. 15 ml/hour
B. 30 ml/hour
C. 45 ml/hour
D. 50 ml/hour
63. A 76-year-old male client had a thromboembolic right stroke; his left arm
is swollen. Which of the following conditions may cause swelling after a
stroke?
A. Elbow contracture secondary to spasticity
B. Loss of muscle contraction decreasing venous return
C. Deep vein thrombosis (DVT) due to immobility of the ipsilateral side
D. Hypoalbuminemia due to protein escaping from an inflamed glomerulus
64. Heberden’s nodes are a common sign of osteoarthritis. Which of the
following statement is correct about this deformity?
A. It appears only in men
B. It appears on the distal interphalangeal joint
C. It appears on the proximal interphalangeal joint
D. It appears on the dorsolateral aspect of the interphalangeal joint.
65. Which of the following statements explains the main difference
between rheumatoid arthritis and osteoarthritis?
A. Osteoarthritis is gender-specific, rheumatoid arthritis isn’t
B. Osteoarthritis is a localized disease rheumatoid arthritis is systemic
C. Osteoarthritis is a systemic disease, rheumatoid arthritis is localized
D. Osteoarthritis has dislocations and subluxations, rheumatoid
arthritis doesn’t
66. Mrs. Cruz uses a cane for assistance in walking. Which of the
following statements is true about a cane or other assistive devices?
A. A walker is a better choice than a cane.
B. The cane should be used on the affected side
C. The cane should be used on the unaffected side
D. A client with osteoarthritis should be encouraged to ambulate
without the cane
67. A male client with type 1 diabetes is scheduled to receive 30 U of
70/30 insulin. There is no 70/30 insulin available. As a substitution, the nurse
may give the client:
A. 9 U regular insulin and 21 U neutral protamine Hagedorn (NPH).
B. 21 U regular insulin and 9 U NPH.
C. 10 U regular insulin and 20 U NPH.
D. 20 U regular insulin and 10 U NPH.
68. Nurse Len should expect to administer which medication to a client
with gout?
A. aspirin
B. furosemide (Lasix)
C. colchicines
D. calcium gluconate (Kalcinate)
69. Mr. Domingo with a history of hypertension is diagnosed with
primary hyperaldosteronism. This diagnosis indicates that the client’s
hypertension is caused by excessive hormone secretion from which of the
following glands?
A. Adrenal cortex
B. Pancreas
C. Adrenal medulla
D. Parathyroid
70. For a diabetic male client with a foot ulcer, the doctor orders bed rest, a
wetto- dry dressing change every shift, and blood glucose monitoring before
meals and bedtime. Why are wet-to-dry dressings used for this client?
A. They contain exudate and provide a moist wound environment.
B. They protect the wound from mechanical trauma and promote healing.
C. They debride the wound and promote healing by secondary intention.
D. They prevent the entrance of microorganisms and minimize
wound discomfort.
71. Nurse Zeny is caring for a client in acute addisonian crisis. Which
laboratory data would the nurse expect to find?
A. Hyperkalemia
B. Reduced blood urea nitrogen (BUN)
C. Hypernatremia
D. Hyperglycemia
72. A client is admitted for treatment of the syndrome of inappropriate
antidiuretic hormone (SIADH). Which nursing intervention is appropriate?
A. Infusing I.V. fluids rapidly as ordered
B. Encouraging increased oral intake
C. Restricting fluids
D. Administering glucose-containing I.V. fluids as ordered
73. A female client tells nurse Nikki that she has been working hard for the
last 3 months to control her type 2 diabetes mellitus with diet and exercise.
To determine the effectiveness of the client’s efforts, the nurse should check:
A. urine glucose level.
B. fasting blood glucose level.
C. serum fructosamine level.
D. glycosylated hemoglobin level.
74. Nurse Trinity administered neutral protamine Hagedorn (NPH) insulin to
a diabetic client at 7 a.m. At what time would the nurse expect the client to be
most at risk for a hypoglycemic reaction?
A. 10:00 am
B. Noon
C. 4:00 pm
D. 10:00 pm
75. The adrenal cortex is responsible for producing which substances?
A. Glucocorticoids and androgens
B. Catecholamines and epinephrine
C. Mineralocorticoids and catecholamines
D. Norepinephrine and epinephrine
76. On the third day after a partial thyroidectomy, Proserfina exhibits
muscle twitching and hyperirritability of the nervous system. When
questioned, the client reports numbness and tingling of the mouth and
fingertips. Suspecting a lifethreatening electrolyte disturbance, the nurse
notifies the surgeon immediately. Which electrolyte disturbance most
commonly follows thyroid surgery?
A. Hypocalcemia
B. Hyponatremia
C. Hyperkalemia
D. Hypermagnesemia
77. Which laboratory test value is elevated in clients who smoke and can’t
be used as a general indicator of cancer?
A. Acid phosphatase level
B. Serum calcitonin level
C. Alkaline phosphatase level
D. Carcinoembryonic antigen level
78. Francis with anemia has been admitted to the medical-surgical unit.
Which assessment findings are characteristic of iron-deficiency anemia?
A. Nights sweats, weight loss, and diarrhea
B. Dyspnea, tachycardia, and pallor
C. Nausea, vomiting, and anorexia
D. Itching, rash, and jaundice
79. In teaching a female client who is HIV-positive about pregnancy, the
nurse would know more teaching is necessary when the client says:
A. The baby can get the virus from my placenta.”
B. “I’m planning on starting on birth control pills.”
C. “Not everyone who has the virus gives birth to a baby who has
the virus.”
D. “I’ll need to have a C-section if I become pregnant and have a baby.”
80. When preparing Judy with acquired immunodeficiency syndrome (AIDS)
for discharge to the home, the nurse should be sure to include which
instruction?
A. “Put on disposable gloves before bathing.”
B. “Sterilize all plates and utensils in boiling water.”
C. “Avoid sharing such articles as toothbrushes and razors.”
D. “Avoid eating foods from serving dishes shared by other
family members.”
81. Nurse Marie is caring for a 32-year-old client admitted with
pernicious anemia. Which set of findings should the nurse expect when
assessing the
client?
A. Pallor, bradycardia, and reduced pulse pressure
B. Pallor, tachycardia, and a sore tongue
C. Sore tongue, dyspnea, and weight gain
D. Angina, double vision, and anorexia
82. After receiving a dose of penicillin, a client develops dyspnea
and hypotension. Nurse Celestina suspects the client is experiencing
anaphylactic shock. What should the nurse do first?
A. Page an anesthesiologist immediately and prepare to intubate
the client.
B. Administer epinephrine, as prescribed, and prepare to intubate the client
if necessary.
C. Administer the antidote for penicillin, as prescribed, and continue
to monitor the client’s vital signs.
D. Insert an indwelling urinary catheter and begin to infuse I.V. fluids
as ordered.
83. Mr. Marquez with rheumatoid arthritis is about to begin aspirin therapy
to reduce inflammation. When teaching the client about aspirin, the nurse
discusses adverse reactions to prolonged aspirin therapy. These include:
A. weight gain.
B. fine motor tremors.
C. respiratory acidosis.
D. bilateral hearing loss.
84. A 23-year-old client is diagnosed with human immunodeficiency virus
(HIV). After recovering from the initial shock of the diagnosis, the client
expresses a desire to learn as much as possible about HIV and acquired
immunodeficiency syndrome (AIDS). When teaching the client about the
immune system, the nurse states that adaptive immunity is provided by which
type of white blood cell?
A. Neutrophil
B. Basophil
C. Monocyte
D. Lymphocyte
85. In an individual with Sjögren’s syndrome, nursing care should focus on:
A. moisture replacement.
B. electrolyte balance.
C. nutritional supplementation.
D. arrhythmia management.
86. During chemotherapy for lymphocytic leukemia, Mathew develops
abdominal pain, fever, and “horse barn” smelling diarrhea. It would be most
important for the nurse to advise the physician to order:
A. enzyme-linked immunosuppressant assay (ELISA) test.
B. electrolyte panel and hemogram.
C. stool for Clostridium difficile test.
D. flat plate X-ray of the abdomen.
87. A male client seeks medical evaluation for fatigue, night sweats, and a 20-
lb weight loss in 6 weeks. To confirm that the client has been infected with the
human immunodeficiency virus (HIV), the nurse expects the physician to
order:
A. E-rosette immunofluorescence.
B. quantification of T-lymphocytes.
C. enzyme-linked immunosorbent assay (ELISA).
D. Western blot test with ELISA.
88. A complete blood count is commonly performed before a Joe goes
into surgery. What does this test seek to identify?
A. Potential hepatic dysfunction indicated by decreased blood
urea nitrogen (BUN) and creatinine levels
B. Low levels of urine constituents normally excreted in the urine
C. Abnormally low hematocrit (HCT) and hemoglobin (Hb) levels
D. Electrolyte imbalance that could affect the blood’s ability to
coagulate properly
89. While monitoring a client for the development of disseminated
intravascular coagulation (DIC), the nurse should take note of what
assessment parameters?
A. Platelet count, prothrombin time, and partial thromboplastin time
B. Platelet count, blood glucose levels, and white blood cell (WBC) count
C. Thrombin time, calcium levels, and potassium levels
D. Fibrinogen level, WBC, and platelet count
90. When taking a dietary history from a newly admitted female client, Nurse
Len should remember that which of the following foods is a common
allergen?
A. Bread
B. Carrots
C. Orange
D. Strawberries
91. Nurse John is caring for clients in the outpatient clinic. Which of the
following phone calls should the nurse return first?
A. A client with hepatitis A who states, “My arms and legs are itching.”
B. A client with cast on the right leg who states, “I have a funny feeling
in my right leg.”
C. A client with osteomyelitis of the spine who states, “I am so
nauseous that I can’t eat.”
D. A client with rheumatoid arthritis who states, “I am having
trouble sleeping.”
92. Nurse Sarah is caring for clients on the surgical floor and has just
received report from the previous shift. Which of the following clients should
the nurse see first?
A. A 35-year-old admitted three hours ago with a gunshot wound; 1.5
cm area of dark drainage noted on the dressing.
B. A 43-year-old who had a mastectomy two days ago; 23 ml
of serosanguinous fluid noted in the Jackson-Pratt drain.
C. A 59-year-old with a collapsed lung due to an accident; no
drainage noted in the previous eight hours.
D. A 62-year-old who had an abdominal-perineal resection three days ago;
client complaints of chills.
93. Nurse Eve is caring for a client who had a thyroidectomy 12 hours ago
for treatment of Grave’s disease. The nurse would be most concerned if which
of the following was observed?
A. Blood pressure 138/82, respirations 16, oral temperature 99
degrees Fahrenheit.
B. The client supports his head and neck when turning his head to
the right.
C. The client spontaneously flexes his wrist when the blood pressure
is obtained.
D. The client is drowsy and complains of sore throat.
94. Julius is admitted with complaints of severe pain in the lower right
quadrant of the abdomen. To assist with pain relief, the nurse should take
which of the following actions?
A. Encourage the client to change positions frequently in bed.
B. Administer Demerol 50 mg IM q 4 hours and PRN.
C. Apply warmth to the abdomen with a heating pad.
D. Use comfort measures and pillows to position the client.
95. Nurse Tina prepares a client for peritoneal dialysis. Which of the
following actions should the nurse take first?
A. Assess for a bruit and a thrill.
B. Warm the dialysate solution.
C. Position the client on the left side.
D. Insert a Foley catheter
96. Nurse Jannah teaches an elderly client with right-sided weakness how to
use cane. Which of the following behaviors, if demonstrated by the client to
the nurse, indicates that the teaching was effective?
A. The client holds the cane with his right hand, moves the can
forward followed by the right leg, and then moves the left leg.
B. The client holds the cane with his right hand, moves the cane
forward followed by his left leg, and then moves the right leg.
C. The client holds the cane with his left hand, moves the cane
forward followed by the right leg, and then moves the left leg.
D. The client holds the cane with his left hand, moves the cane
forward followed by his left leg, and then moves the right leg.
97. An elderly client is admitted to the nursing home setting. The client
is occasionally confused and her gait is often unsteady. Which of the
following actions, if taken by the nurse, is most appropriate?
A. Ask the woman’s family to provide personal items such as photos
or mementos.
B. Select a room with a bed by the door so the woman can look down
the hall.
C. Suggest the woman eat her meals in the room with her roommate.
D. Encourage the woman to ambulate in the halls twice a day.
98. Nurse Evangeline teaches an elderly client how to use a standard
aluminum walker. Which of the following behaviors, if demonstrated by the
client, indicates that the nurse’s teaching was effective?
A. The client slowly pushes the walker forward 12 inches, then takes small
steps forward while leaning on the walker.
B. The client lifts the walker, moves it forward 10 inches, and then
takes several small steps forward.
C. The client supports his weight on the walker while advancing it
forward, then takes small steps while balancing on the walker.
D. The client slides the walker 18 inches forward, then takes small
steps while holding onto the walker for balance.
99. Nurse Deric is supervising a group of elderly clients in a residential
home setting. The nurse knows that the elderly are at greater risk of
developing sensory deprivation for what reason?
A. Increased sensitivity to the side effects of medications.
B. Decreased visual, auditory, and gustatory abilities.
C. Isolation from their families and familiar surroundings.
D. Decrease musculoskeletal function and mobility.
100. A male client with emphysema becomes restless and confused. What
step should nurse Jasmine take next?
A. Encourage the client to perform pursed lip breathing.
B. Check the client’s temperature.
C. Assess the client’s potassium level.
D. Increase the client’s oxygen flow rate.
Answers and Rationales
1. Answer: (C) Loose, bloody. Normal bowel function and soft-formed stool
usually do not occur until around the seventh day following surgery. The
stool consistency is related to how much water is being absorbed.
2. Answer: (A) On the client’s right side. The client has left visual field
blindness. The client will see only from the right side.
3. Answer: (C) Check respirations, stabilize spine, and check
circulation. Checking the airway would be priority, and a neck injury
should be suspected.
4. Answer: (D) Decreasing venous return through vasodilation. The significant
effect of nitroglycerin is vasodilation and decreased venous return, so the
heart does not have to work hard.
5. Answer: (A) Call for help and note the time. Having established, by
stimulating the client, that the client is unconscious rather than sleep, the
nurse should immediately call for help. This may be done by dialing the
operator from the client’s phone and giving the hospital code for cardiac
arrest and the client’s room number to the operator, of if the phone is not
available, by pulling the emergency call button. Noting the time is
important baseline information for cardiac arrest procedure.
6. Answer: (C) Make sure that the client takes food and medications at prescribed
intervals. Food and drug therapy will prevent the accumulation
of hydrochloric acid, or will neutralize and buffer the acid that
does accumulate.
7. Answer: (B) Continue treatment as ordered. The effects of heparin are
monitored by the PTT is normally 30 to 45 seconds; the therapeutic level
is 1.5 to 2 times the normal level.
8. Answer: (B) In the operating room. The stoma drainage bag is applied in
the operating room. Drainage from the ileostomy contains secretions that
are rich in digestive enzymes and highly irritating to the skin. Protection of
the skin from the effects of these enzymes is begun at once. Skin exposed
to these enzymes even for a short time becomes reddened, painful,
and excoriated.
9. Answer: (B) Flat on back. To avoid the complication of a painful spinal
headache that can last for several days, the client is kept in flat in a supine
position for approximately 4 to 12 hours postoperatively. Headaches are
believed to be causes by the seepage of cerebral spinal fluid from the
puncture site. By keeping the client flat, cerebral spinal fluid pressures are
equalized, which avoids trauma to the neurons.
10. Answer: (C) The client is oriented when aroused from sleep, and goes back to
sleep immediately. This finding suggest that the level of consciousness
is decreasing.
11. Answer: (A) Altered mental status and dehydration. Fever, chills, hemortysis,
dyspnea, cough, and pleuritic chest pain are the common symptoms of
pneumonia, but elderly clients may first appear with only an altered lentil
status and dehydration due to a blunted immune response.
12. Answer: (B) Chills, fever, night sweats, and hemoptysis. Typical signs and
symptoms are chills, fever, night sweats, and hemoptysis. Chest pain may
be present from coughing, but isn’t usual. Clients with TB typically have
low-grade fevers, not higher than 102°F (38.9°C). Nausea, headache, and
photophobia aren’t usual TB symptoms.
13. Answer:(A) Acute asthma. Based on the client’s history and symptoms,
acute asthma is the most likely diagnosis. He’s unlikely to have bronchial
pneumonia without a productive cough and fever and he’s too young to
have developed (COPD) and emphysema.
14. Answer: (B) Respiratory arrest. Narcotics can cause respiratory arrest if
given in large quantities. It’s unlikely the client will have asthma attack or a
seizure or wake up on his own.
15. Answer: (D) Decreased vital capacity.  Reduction in vital capacity is a
normal physiologic changes include decreased elastic recoil of the lungs,
fewer functional capillaries in the alveoli, and an increased in residual
volume.
16. Answer: (C) Presence of premature ventricular contractions (PVCs) on
a cardiac monitor. Lidocaine drips are commonly used to treat clients
whose arrhythmias haven’t been controlled with oral medication and who
are having PVCs that are visible on the cardiac monitor. SaO2,
blood pressure, and ICP are important factors but aren’t as significant as
PVCs in the situation.
17. Answer: (B) Avoid foods high in vitamin K. The client should avoid
consuming large amounts of vitamin K because vitamin K can interfere
with anticoagulation. The client may need to report diarrhea, but isn’t
effect of taking an anticoagulant. An electric razor-not a straight razor-
should be used to prevent cuts that cause bleeding. Aspirin may increase
the risk of bleeding; acetaminophen should be used to pain relief.
18. Answer: (C) Clipping the hair in the area. Hair can be a source of infection
and should be removed by clipping. Shaving the area can cause skin
abrasions and depilatories can irritate the skin.
19. Answer: (A) Bone fracture. Bone fracture is a major complication of
osteoporosis that results when loss of calcium and phosphate increased
the fragility of bones. Estrogen deficiencies result from menopause-not
osteoporosis. Calcium and vitamin D supplements may be used to
support normal bone metabolism, But a negative calcium balance isn’t a
complication of osteoporosis. Dowager’s hump results from bone
fractures. It develops when repeated vertebral fractures increase spinal
curvature.
20. Answer: (C) Changes from previous examinations. Women are instructed to
examine themselves to discover changes that have occurred in the breast.
Only a physician can diagnose lumps that are cancerous, areas of
thickness or fullness that signal the presence of a malignancy, or masses
that are fibrocystic as opposed to malignant.
21. Answer: (C) Balance the client’s periods of activity and rest. A client with
hyperthyroidism needs to be encouraged to balance periods of activity
and rest. Many clients with hyperthyroidism are hyperactive and complain
of feeling very warm.
22. Answer: (B) Increase his activity level. The client should be encouraged to
increase his activity level. Maintaining an ideal weight; following a low-
cholesterol, low sodium diet; and avoiding stress are all important factors
in decreasing the risk of atherosclerosis.
23. Answer: (A) Laminectomy. The client who has had spinal surgery, such as
laminectomy, must be log rolled to keep the spinal column straight when
turning. Thoracotomy and cystectomy may turn themselves or may be
assisted into a comfortable position. Under normal
circumstances, hemorrhoidectomy is an outpatient procedure, and the
client may resume normal activities immediately after surgery.
24. Answer: (D) Avoiding straining during bowel movement or bending at
the waist. The client should avoid straining, lifting heavy objects,
and coughing harshly because these activities increase intraocular
pressure. Typically, the client is instructed to avoid lifting objects weighing
more than 15 lb (7kg) – not 5lb. instruct the client when lying in bed to lie
on either the side or back. The client should avoid bright light by
wearing sunglasses.
25. Answer: (D) Before age 20. Testicular cancer commonly occurs in men
between ages 20 and 30. A male client should be taught how to perform
testicular selfexamination before age 20, preferably when he enters his
teens.
26. Answer: (B) Place a saline-soaked sterile dressing on the wound. The nurse
should first place saline-soaked sterile dressings on the open wound to
prevent tissue drying and possible infection. Then the nurse should call
the physician and take the client’s vital signs. The dehiscence needs to be
surgically closed, so the nurse should never try to close it.
27. Answer: (A) A progressively deeper breaths followed by shallower breaths
with apneic periods. Cheyne-Strokes respirations are breaths that
become progressively deeper fallowed by shallower respirations with
apneas periods. Biot’s respirations are rapid, deep breathing with abrupt
pauses between each breath, and equal depth between each breath.
Kussmaul’s respirations are rapid, deep breathing without pauses.
Tachypnea is shallow breathing with increased respiratory rate.
28. Answer: (B) Fine crackles. Fine crackles are caused by fluid in the alveoli
and commonly occur in clients with heart failure. Tracheal breath sounds
are auscultated over the trachea. Coarse crackles are caused by secretion
accumulation in the airways. Friction rubs occur with pleural
inflammation.
29. Answer: (B) The airways are so swollen that no air cannot get through. During
an acute attack, wheezing may stop and breath sounds become inaudible
because the airways are so swollen that air can’t get through. If the attack
is over and swelling has decreased, there would be no more wheezing and
less emergent concern. Crackles do not replace wheezes during an acute
asthma attack.
30. Answer: (D) Place the client on his side, remove dangerous objects, and protect
his head. During the active seizure phase, initiate precautions by
placing the client on his side, removing dangerous objects, and protecting
his head from injury. A bite block should never be inserted during the
active seizure phase. Insertion can break the teeth and lead to aspiration.
31. Answer: (B) Kinked or obstructed chest tube. Kinking and blockage of the
chest tube is a common cause of a tension pneumothorax. Infection and
excessive drainage won’t cause a tension pneumothorax. Excessive water
won’t affect the chest tube drainage.
32. Answer: (D) Stay with him but not intervene at this time. If the client is
coughing, he should be able to dislodge the object or cause a complete
obstruction. If complete obstruction occurs, the nurse should perform the
abdominal thrust maneuver with the client standing. If the client is
unconscious, she should lay him down. A nurse should never leave a
choking client alone.
33. Answer: (B) Current health promotion activities. Recognizing an individual’s
positive health measures is very useful. General health in the previous 10
years is important, however, the current activities of an 84 year old client
are most significant in planning care. Family history of disease for a client
in later years is of minor significance. Marital status information may be
important for discharge planning but is not as significant for addressing
the immediate medical problem.
34. Answer: (C) Place the client in a side lying position, with the head of the bed
lowered.  The client should be positioned in a side-lying position with
the head of the bed lowered to prevent aspiration. A small amount
of toothpaste should be used and the mouth swabbed or suctioned
to remove pooled secretions. Lemon glycerin can be drying if used
for extended periods. Brushing the teeth with the client lying supine may
lead to aspiration. Hydrogen peroxide is caustic to tissues and should not
be used.
35. Answer: (C) Pneumonia. Fever productive cough and pleuritic chest pain
are common signs and symptoms of pneumonia. The client with ARDS
has dyspnea and hypoxia with worsening hypoxia over time, if not treated
aggressively. Pleuritic chest pain varies with respiration, unlike the
constant chest pain during an MI; so this client most likely isn’t having an
MI. the client with TB typically has a cough producing blood-tinged
sputum. A sputum culture should be obtained to confirm the nurse’s
suspicions.
36. Answer: (C) A 43-yesr-old homeless man with a history of alcoholism. Clients
who are economically disadvantaged, malnourished, and have reduced
immunity, such as a client with a history of alcoholism,  are at extremely
high risk for developing TB. A high school student, daycare worker, and
businessman probably have a much low risk of contracting TB.
37. Answer: (C ) To determine the extent of lesions. If the lesions are large
enough, the chest X-ray will show their presence in the lungs. Sputum
culture confirms the diagnosis. There can be false-positive and false-
negative skin test results. A chest X-ray can’t determine if this is a primary
or secondary infection.
38. Answer: (B) Bronchodilators. Bronchodilators are the first line of
treatment for asthma because broncho-constriction is the cause of
reduced airflow. Beta adrenergic blockers aren’t used to treat asthma and
can cause bronchoconstriction. Inhaled oral steroids may be given to
reduce the inflammation but aren’t used for emergency relief.
39. Answer: (C) Chronic obstructive bronchitis. Because of this extensive
smoking history and symptoms the client most likely has chronic
obstructive bronchitis. Client with ARDS have acute symptoms of hypoxia
and typically need large amounts of oxygen. Clients with asthma and
emphysema tend not to have chronic cough or peripheral edema.
40. Answer: (A) The patient is under local anesthesia during the procedure. Before
the procedure, the patient is administered with drugs that would help to
prevent infection and rejection of the transplanted cells such as
antibiotics, cytotoxic, and corticosteroids. During the transplant, the
patient is placed under general anesthesia.
41. Answer: (D) Raise the side rails. A patient who is disoriented is at risk of
falling out of bed. The initial action of the nurse should be raising the side
rails to ensure patients safety.
42. Answer: (A) Crowd red blood cells. The excessive production of white
blood cells crowd out red blood cells production which causes anemia to
occur.
43. Answer: (B) Leukocytosis. Chronic Lymphocytic leukemia (CLL) is
characterized by increased production of leukocytes and lymphocytes
resulting in leukocytosis, and proliferation of these cells within the bone
marrow, spleen and liver.
44. Answer: (A) Explain the risks of not having the surgery. The best initial
response is to explain the risks of not having the surgery. If the client
understands the risks but still refuses the nurse should notify the
physician and the nurse supervisor and then record the client’s refusal in
the nurses’ notes.
45. Answer: (D) The 75-year-old client who was admitted 1 hour ago with new-
onset atrial fibrillation and is receiving L.V. dilitiazem (Cardizem). The client
with atrial fibrillation has the greatest potential to become unstable and is
on L.V. medication that requires close monitoring. After assessing this
client, the nurse should assess the client with thrombophlebitis who is
receiving a heparin infusion, and then the 58- year-old client admitted 2
days ago with heart failure (his signs and symptoms are resolving and
don’t require immediate attention). The lowest priority is the 89-year-old
with end stage right-sided heart failure, who requires time-consuming
supportive measures.
46. Answer: (C) Cocaine. Because of the client’s age and negative medical
history, the nurse should question her about cocaine use. Cocaine
increases myocardial oxygen consumption and can cause coronary artery
spasm, leading to tachycardia, ventricular fibrillation, myocardial ischemia,
and myocardial infarction. Barbiturate overdose may trigger
respiratory depression and slow pulse. Opioids can cause marked
respiratory depression, while benzodiazepines can cause drowsiness and
confusion.
47. Answer: (B) Nonmobile mass with irregular edges. Breast cancer tumors are
fixed, hard, and poorly delineated with irregular edges. A mobile mass that
is soft and easily delineated is most often a fluid-filled benign cyst. Axillary
lymph nodes may or may not be palpable on initial detection of a
cancerous mass. Nipple retraction — not eversion — may be a sign of
cancer.
48. Answer: (C) Radiation. The usual treatment for vaginal cancer is external
or intravaginal radiation therapy. Less often, surgery is
performed. Chemotherapy typically is prescribed only if vaginal cancer is
diagnosed in an early stage, which is rare. Immunotherapy isn’t used to
treat vaginal cancer.
49. Answer: (B) Carcinoma in situ, no abnormal regional lymph nodes, and no
evidence of distant metastasis. TIS, N0, M0 denotes carcinoma in situ, no
abnormal regional lymph nodes, and no evidence of distant metastasis.
No evidence of primary tumor, no abnormal regional lymph nodes, and no
evidence of distant metastasis is classified as T0, N0, M0. If the tumor
and regional lymph nodes can’t be assessed and no evidence of
metastasis exists, the lesion is classified as TX, NX, M0. A progressive
increase in tumor size, no demonstrable metastasis of the regional lymph
nodes, and ascending degrees of distant metastasis is classified as T1,
T2, T3, or T4; N0; and M1, M2, or M3.
50. Answer: (D) “Keep the stoma moist.” The nurse should instruct the client
to keep the stoma moist, such as by applying a thin layer of petroleum jelly
around the edges, because a dry stoma may become irritated. The nurse
should recommend placing a stoma bib over the stoma to filter and warm
air before it enters the stoma. The client should begin performing stoma
care without assistance as soon as possible to gain independence in self-
care activities.
51. Answer: (B) Lung cancer.  Lung cancer is the most deadly type of cancer
in both women and men. Breast cancer ranks second in women, followed
(in descending order) by colon and rectal cancer, pancreatic cancer,
ovarian cancer, uterine cancer, lymphoma, leukemia, liver cancer, brain
cancer, stomach cancer, and multiple myeloma.
52. Answer: (A) miosis, partial eyelid ptosis, and anhidrosis on the affected side of
the face. Horner’s syndrome, which occurs when a lung tumor invades the
ribs and affects the sympathetic nerve ganglia, is characterized by miosis,
partial eyelid ptosis, and anhidrosis on the affected side of the face. Chest
pain, dyspnea, cough, weight loss, and fever are associated with pleural
tumors. Arm and shoulder pain and atrophy of the arm and hand muscles
on the affected side suggest Pancoast’s tumor, a lung tumor involving the
first thoracic and eighth cervical nerves within the brachial plexus.
Hoarseness in a client with lung cancer suggests that the tumor has
extended to the recurrent laryngeal nerve; dysphagia suggests that the
lung tumor is compressing the esophagus.
53. Answer: (A) prostate-specific antigen, which is used to screen for
prostate cancer. PSA stands for prostate-specific antigen, which is used
to screen for prostate cancer. The other answers are incorrect.
54. Answer: (D) “Remain supine for the time specified by the physician.” The
nurse should instruct the client to remain supine for the time specified by
the physician. Local anesthetics used in a subarachnoid block don’t alter
the gag reflex. No interactions between local anesthetics and food occur.
Local anesthetics don’t cause hematuria.
55. Answer: (C) Sigmoidoscopy. Used to visualize the lower GI tract,
sigmoidoscopy and proctoscopy aid in the detection of two-thirds of all
colorectal cancers. Stool Hematest detects blood, which is a sign of
colorectal cancer; however, the test doesn’t confirm the diagnosis. CEA
may be elevated in colorectal cancer but isn’t considered a confirming
test. An abdominal CT scan is used to stage the presence of colorectal
cancer.
56. Answer: (B) A fixed nodular mass with dimpling of the overlying skin. A fixed
nodular mass with dimpling of the overlying skin is common during late
stages of breast cancer. Many women have slightly asymmetrical breasts.
Bloody nipple discharge is a sign of intraductal papilloma, a benign
condition. Multiple firm, round, freely movable masses that change with
the menstrual cycle indicate fibrocystic breasts, a benign condition.
57. Answer: (A) Liver. The liver is one of the five most common cancer
metastasis sites. The others are the lymph nodes, lung, bone, and brain.
The colon, reproductive tract, and WBCs are occasional metastasis sites.
58. Answer: (D) The client wears a watch and wedding band. During an MRI, the
client should wear no metal objects, such as jewelry, because the strong
magnetic field can pull on them, causing injury to the client and (if they fly
off) to others. The client must lie still during the MRI but can talk to those
performing the test by way of the microphone inside the scanner tunnel.
The client should hear thumping sounds, which are caused by the sound
waves thumping on the magnetic field.
59. Answer: (C) The recommended daily allowance of calcium may be found in a
wide variety of foods. Premenopausal women require 1,000 mg of calcium
per day. Postmenopausal women require 1,500 mg per day. It’s often,
though not always, possible to get the recommended daily requirement in
the foods we eat. Supplements are available but not always
necessary. Osteoporosis doesn’t show up on ordinary X-rays until 30% of
the bone loss has occurred. Bone densitometry can detect bone loss of
3% or less. This test is sometimes recommended routinely for women
over 35 who are at risk. Strenuous exercise won’t cause fractures.
60. Answer: (C) Joint flexion of less than 50%. Arthroscopy is contraindicated
in clients with joint flexion of less than 50% because of technical problems
in inserting the instrument into the joint to see it clearly. Other
contraindications for this procedure include skin and wound infections.
Joint pain may be an indication, not a contraindication, for arthroscopy.
Joint deformity and joint stiffness aren’t contraindications for this
procedure.
61. Answer: (D) Gouty arthritis. Gouty arthritis, a metabolic disease, is
characterized by urate deposits and pain in the joints, especially those in
the feet and legs. Urate deposits don’t occur in septic or traumatic
arthritis. Septic arthritis results from bacterial invasion of a joint and leads
to inflammation of the synovial lining. Traumatic arthritis results from
blunt trauma to a joint or ligament. Intermittent arthritis is a rare, benign
condition marked by regular, recurrent joint effusions, especially in the
knees.
62. Answer: (B) 30 ml/hour. An infusion prepared with 25,000 units of heparin
in 500 ml of saline solution yields 50 units of heparin per milliliter of
solution. The equation is set up as 50 units times X (the unknown
quantity) equals 1,500 units/hour, X equals 30 ml/hour.
63. Answer: (B) Loss of muscle contraction decreasing venous return. In clients
with hemiplegia or hemiparesis loss of muscle contraction decreases
venous return and may cause swelling of the affected extremity.
Contractures, or bony calcifications may occur with a stroke, but don’t
appear with swelling. DVT may develop in clients with a stroke but is more
likely to occur in the lower extremities. A stroke isn’t linked to protein loss.
64. Answer: (B) It appears on the distal interphalangeal joint. Heberden’s nodes
appear on the distal interphalageal joint on both men and women.
Bouchard’s node appears on the dorsolateral aspect of the proximal
interphalangeal joint.
65. Answer: (B) Osteoarthritis is a localized disease rheumatoid arthritis
is systemic.  Osteoarthritis is a localized disease, rheumatoid arthritis
is systemic. Osteoarthritis isn’t gender-specific, but rheumatoid arthritis
is. Clients have dislocations and subluxations in both disorders.
66. Answer: (C) The cane should be used on the unaffected side. A cane should
be used on the unaffected side. A client with osteoarthritis should be
encouraged to ambulate with a cane, walker, or other assistive device as
needed; their use takes weight and stress off joints.
67. Answer: (A) 9 U regular insulin and 21 U neutral protamine
Hagedorn (NPH).  A 70/30 insulin preparation is 70% NPH and 30%
regular insulin. Therefore, a correct substitution requires mixing 21 U of
NPH and 9 U of regular insulin. The other choices are incorrect dosages
for the prescribed insulin.
68. Answer: (C) colchicines. A disease characterized by joint inflammation
(especially in the great toe), gout is caused by urate crystal deposits in the
joints. The physician prescribes colchicine to reduce these deposits and
thus ease joint inflammation. Although aspirin is used to reduce joint
inflammation and pain in clients with osteoarthritis and rheumatoid
arthritis, it isn’t indicated for gout because it has no effect on urate crystal
formation. Furosemide, a diuretic, doesn’t relieve gout. Calcium gluconate
is used to reverse a negative calcium balance and relieve muscle cramps,
not to treat gout.
69. Answer: (A) Adrenal cortex. Excessive secretion of aldosterone in the
adrenal cortex is responsible for the client’s hypertension. This hormone
acts on the renal tubule, where it promotes reabsorption of sodium and
excretion of potassium and hydrogen ions. The pancreas mainly secretes
hormones involved in fuel metabolism. The adrenal medulla secretes
the catecholamines — epinephrine and norepinephrine. The
parathyroids secrete parathyroid hormone.
70. Answer: (C) They debride the wound and promote healing by
secondary intention. For this client, wet-to-dry dressings are most
appropriate because they clean the foot ulcer by debriding exudate and
necrotic tissue, thus promoting healing by secondary intention. Moist,
transparent dressings contain exudate and provide a moist wound
environment. Hydrocolloid dressings prevent the entrance of
microorganisms and minimize wound discomfort. Dry sterile dressings
protect the wound from mechanical trauma and promote healing.
71. Answer: (A) Hyperkalemia. In adrenal insufficiency, the client has
hyperkalemia due to reduced aldosterone secretion. BUN increases as the
glomerular filtration rate is reduced. Hyponatremia is caused by reduced
aldosterone secretion. Reduced cortisol secretion leads to impaired
glyconeogenesis and a reduction of glycogen in the liver and muscle,
causing hypoglycemia.
72. Answer: (C) Restricting fluids. To reduce water retention in a client with
the SIADH, the nurse should restrict fluids. Administering fluids by any
route would further increase the client’s already heightened fluid load.
73. Answer: (D) glycosylated hemoglobin level. Because some of the glucose
in the bloodstream attaches to some of the hemoglobin and stays
attached during the 120-day life span of red blood cells, glycosylated
hemoglobin levels provide information about blood glucose levels during
the previous 3 months. Fasting blood glucose and urine glucose levels
only give information about glucose levels at the point in time when they
were obtained. Serum fructosamine levels provide information about
blood glucose control over the past 2 to 3 weeks.
74. Answer: (C) 4:00 pm. NPH is an intermediate-acting insulin that peaks 8
to 12 hours after administration. Because the nurse administered NPH
insulin at 7 a.m., the client is at greatest risk for hypoglycemia from 3 p.m.
to 7 p.m.
75. Answer: (A) Glucocorticoids and androgens. The adrenal glands have two
divisions, the cortex and medulla. The cortex produces three types of
hormones: glucocorticoids, mineralocorticoids, and androgens. The
medulla produces catecholamines — epinephrine and norepinephrine.
76. Answer: (A) Hypocalcemia. Hypocalcemia may follow thyroid surgery if
the parathyroid glands were removed accidentally. Signs and symptoms
of hypocalcemia may be delayed for up to 7 days after surgery. Thyroid
surgery doesn’t directly cause serum sodium, potassium, or magnesium
abnormalities. Hyponatremia may occur if the client inadvertently received
too much fluid; however, this can happen to any surgical client receiving
I.V. fluid therapy, not just one recovering from thyroid surgery.
Hyperkalemia and hypermagnesemia usually are associated with reduced
renal excretion of potassium and magnesium, not thyroid surgery.
77. Answer: (D) Carcinoembryonic antigen level. In clients who smoke, the
level of carcinoembryonic antigen is elevated. Therefore, it can’t be used
as a general indicator of cancer. However, it is helpful in monitoring
cancer treatment because the level usually falls to normal within 1 month
if treatment is successful. An elevated acid phosphatase level may
indicate prostate cancer. An elevated alkaline phosphatase level may
reflect bone metastasis. An elevated serum calcitonin level usually signals
thyroid cancer.
78. Answer: (B) Dyspnea, tachycardia, and pallor. Signs of iron-deficiency
anemia include dyspnea, tachycardia, and pallor as well as fatigue,
listlessness, irritability, and headache. Night sweats, weight loss, and
diarrhea may signal acquired immunodeficiency syndrome (AIDS).
Nausea, vomiting, and anorexia may be signs of hepatitis B. Itching, rash,
and jaundice may result from an allergic or hemolytic reaction.
79. Answer: (D) “I’ll need to have a C-section if I become pregnant and have a
baby.” The human immunodeficiency virus (HIV) is transmitted
from mother to child via the transplacental route, but a Cesarean
section delivery isn’t necessary when the mother is HIV-positive. The use
of birth control will prevent the conception of a child who might have HIV.
It’s true that a mother who’s HIV positive can give birth to a baby who’s
HIV negative.
80. Answer: (C) “Avoid sharing such articles as toothbrushes and razors.” The
human immunodeficiency virus (HIV), which causes AIDS, is most
concentrated in the blood. For this reason, the client shouldn’t share
personal articles that may be blood-contaminated, such as toothbrushes
and razors, with other family members. HIV isn’t transmitted by bathing or
by eating from plates, utensils, or serving dishes used by a person with
AIDS.
81. Answer: (B) Pallor, tachycardia, and a sore tongue. Pallor, tachycardia, and a
sore tongue are all characteristic findings in pernicious anemia. Other
clinical manifestations include anorexia; weight loss; a smooth, beefy red
tongue; a wide pulse pressure; palpitations; angina; weakness; fatigue; and
paresthesia of the hands and feet. Bradycardia, reduced pulse pressure,
weight gain, and double vision aren’t characteristic findings in pernicious
anemia.
82. Answer: (B) Administer epinephrine, as prescribed, and prepare to intubate the
client if necessary. To reverse anaphylactic shock, the nurse first
should administer epinephrine, a potent bronchodilator as prescribed.
The physician is likely to order additional medications, such as
antihistamines and corticosteroids; if these medications don’t relieve the
respiratory compromise associated with anaphylaxis, the nurse should
prepare to intubate the client. No antidote for penicillin exists; however,
the nurse should continue to monitor the client’s vital signs. A client who
remains hypotensive may need fluid resuscitation and fluid intake and
output monitoring; however, administering epinephrine is the first priority.
83. Answer: (D) bilateral hearing loss. Prolonged use of aspirin and other
salicylates sometimes causes bilateral hearing loss of 30 to 40 decibels.
Usually, this adverse effect resolves within 2 weeks after the therapy is
discontinued. Aspirin doesn’t lead to weight gain or fine motor tremors.
Large or toxic salicylate doses may cause respiratory alkalosis, not
respiratory acidosis.
84. Answer: (D) Lymphocyte. The lymphocyte provides adaptive immunity —
recognition of a foreign antigen and formation of memory cells against
the antigen. Adaptive immunity is mediated by B and T lymphocytes and
can be acquired actively or passively. The neutrophil is crucial to
phagocytosis. The basophil plays an important role in the release of
inflammatory mediators. The monocyte functions in phagocytosis and
monokine production.
85. Answer: (A) moisture replacement. Sjogren’s syndrome is an autoimmune
disorder leading to progressive loss of lubrication of the skin, GI tract,
ears, nose, and vagina. Moisture replacement is the mainstay of therapy.
Though malnutrition and electrolyte imbalance may occur as a result of
Sjogren’s syndrome’s effect on the GI tract, it isn’t the predominant
problem. Arrhythmias aren’t a problem associated with Sjogren’s
syndrome.
86. Answer: (C) stool for Clostridium difficile test. Immunosuppressed clients
— for example, clients receiving chemotherapy, — are at risk for infection
with C. difficile, which causes “horse barn” smelling diarrhea. Successful
treatment begins with an accurate diagnosis, which includes a stool test.
The ELISA test is diagnostic for human immunodeficiency virus (HIV) and
isn’t indicated in this case. An electrolyte panel and hemogram may be
useful in the overall evaluation of a client but aren’t diagnostic for specific
causes of diarrhea. A flat plate of the abdomen may provide useful
information about bowel function but isn’t indicated in the case of “horse
barn” smelling diarrhea.
87. Answer: (D) Western blot test with ELISA. HIV infection is detected by
analyzing blood for antibodies to HIV, which form approximately 2 to 12
weeks after exposure to HIV and denote infection. The Western blot test —
electrophoresis of antibody proteins — is more than 98% accurate in
detecting HIV antibodies when used in conjunction with the ELISA. It isn’t
specific when used alone. Erosette immunofluorescence is used to detect
viruses in general; it doesn’t confirm HIV infection. Quantification of T-
lymphocytes is a useful monitoring test but isn’t diagnostic for HIV. The
ELISA test detects HIV antibody particles but may yield inaccurate results;
a positive ELISA result must be confirmed by the Western blot test.
88. Answer: (C) Abnormally low hematocrit (HCT) and hemoglobin
(Hb) levels. Low preoperative HCT and Hb levels indicate the client
may require a blood transfusion before surgery. If the HCT and Hb
levels decrease during surgery because of blood loss, the potential need
for a transfusion increases. Possible renal failure is indicated by elevated
BUN or creatinine levels. Urine constituents aren’t found in the
blood. Coagulation is determined by the presence of appropriate clotting
factors, not electrolytes.
89. Answer: (A) Platelet count, prothrombin time, and partial
thromboplastin time. The diagnosis of DIC is based on the results of
laboratory studies of prothrombin time, platelet count, thrombin time,
partial thromboplastin time, and fibrinogen level as well as client history
and other assessment factors. Blood glucose levels, WBC count, calcium
levels, and potassium levels aren’t used to confirm a diagnosis of DIC.
90. Answer: (D) Strawberries. Common food allergens include berries,
peanuts, Brazil nuts, cashews, shellfish, and eggs. Bread, carrots, and
oranges rarely cause allergic reactions.
91. Answer: (B) A client with cast on the right leg who states, “I have a
funny feeling in my right leg.” It may indicate neurovascular compromise,
requires immediate assessment.
92. Answer: (D) A 62-year-old who had an abdominal-perineal resection three days
ago; client complaints of chills. The client is at risk for peritonitis; should be
assessed for further symptoms and infection.
93. Answer: (C) The client spontaneously flexes his wrist when the blood pressure
is obtained. Carpal spasms indicate hypocalcemia.
94. Answer: (D) Use comfort measures and pillows to position the client.Using
comfort measures and pillows to position the client is a non-
pharmacological methods of pain relief.
95. Answer: (B) Warm the dialysate solution. Cold dialysate increases
discomfort. The solution should be warmed to body temperature in
warmer or heating pad; don’t use microwave oven.
96. Answer: (C) The client holds the cane with his left hand, moves the
cane forward followed by the right leg, and then moves the left leg. The cane
acts as a support and aids in weight bearing for the weaker right leg.
97. Answer: (A) Ask the woman’s family to provide personal items such as photos
or mementos.Photos and mementos provide visual stimulation to
reduce sensory deprivation.
98. Answer: (B) The client lifts the walker, moves it forward 10 inches, and then
takes several small steps forward. A walker needs to be picked up, placed
down on all legs.
99. Answer: (C) Isolation from their families and familiar surroundings. Gradual
loss of sight, hearing, and taste interferes with normal functioning.
100. Answer: (A) Encourage the client to perform pursed lip breathing. Purse lip
breathing prevents the collapse of lung unit and helps client control rate
and depth of breathing.
PNLE IV for Care of Clients with
Physiologic and Psychosocial
Alterations (Part 2)
1. Randy has undergone kidney transplant, what assessment would prompt
Nurse Katrina to suspect organ rejection?
A. Sudden weight loss
B. Polyuria
C. Hypertension
D. Shock
2. The immediate objective of nursing care for an overweight, mildly
hypertensive male client with ureteral colic and hematuria is to decrease:
A. Pain
B. Weight
C. Hematuria
D. Hypertension
3. Matilda, with hyperthyroidism is to receive Lugol’s iodine solution before a
subtotal thyroidectomy is performed. The nurse is aware that this medication
is given to:
A. Decrease the total basal metabolic rate.
B. Maintain the function of the parathyroid glands.
C. Block the formation of thyroxine by the thyroid gland.
D. Decrease the size and vascularity of the thyroid gland.
4. Ricardo, was diagnosed with type I diabetes. The nurse is aware that acute
hypoglycemia also can develop in the client who is diagnosed with:
A. Liver disease
B. Hypertension
C. Type 2 diabetes
D. Hyperthyroidism
5. Tracy is receiving combination chemotherapy for treatment of metastatic
carcinoma. Nurse Ruby should monitor the client for the systemic side effect
of:
A. Ascites
B. Nystagmus
C. Leukopenia
D. Polycythemia
6. Norma, with recent colostomy expresses concern about the inability to
control the passage of gas. Nurse Oliver should suggest that the client plan to:
A. Eliminate foods high in cellulose.
B. Decrease fluid intake at meal times.
C. Avoid foods that in the past caused flatus.
D. Adhere to a bland diet prior to social events.
7. Nurse Ron begins to teach a male client how to perform colostomy
irrigations. The nurse would evaluate that the instructions were understood
when the client states, “I should:
A. Lie on my left side while instilling the irrigating solution.”
B. Keep the irrigating container less than 18 inches above the stoma.”
C. Instill a minimum of 1200 ml of irrigating solution to
stimulate evacuation of the bowel.”
D. Insert the irrigating catheter deeper into the stoma if cramping occurs
during the procedure.”
8. Patrick is in the oliguric phase of acute tubular necrosis and is experiencing
fluid and electrolyte imbalances. The client is somewhat confused and
complains of nausea and muscle weakness. As part of the prescribed therapy
to correct this electrolyte imbalance, the nurse would expect to:
A. Administer Kayexalate
B. Restrict foods high in protein
C. Increase oral intake of cheese and milk.
D. Administer large amounts of normal saline via I.V.
9. Mario has burn injury. After Forty48 hours, the physician orders for Mario 2
liters of IV fluid to be administered q12 h. The drop factor of the tubing is 10
gtt/ml. The nurse should set the flow to provide:
A. 18 gtt/min
B. 28 gtt/min
C. 32 gtt/min
D. 36 gtt/min
10.Terence suffered form burn injury. Using the rule of nines, which has the
largest percent of burns?
A. Face and neck
B. Right upper arm and penis
C. Right thigh and penis
D. Upper trunk
11. Herbert, a 45 year old construction engineer is brought to the hospital
unconscious after falling from a 2-story building. When assessing the client,
the nurse would be most concerned if the assessment revealed:
A. Reactive pupils
B. A depressed fontanel
C. Bleeding from ears
D. An elevated temperature
12. Nurse Sherry is teaching male client regarding his permanent artificial
pacemaker. Which information given by the nurse shows her knowledge
deficit about the artificial cardiac pacemaker?
A. take the pulse rate once a day, in the morning upon awakening
B. May be allowed to use electrical appliances
C. Have regular follow up care
D. May engage in contact sports
13.The nurse is ware that the most relevant knowledge about oxygen
administration to a male client with COPD is
A. Oxygen at 1-2L/min is given to maintain the hypoxic stimulus for
breathing.
B. Hypoxia stimulates the central chemoreceptors in the medulla that
makes the client breath.
C. Oxygen is administered best using a non-rebreathing mask
D. Blood gases are monitored using a pulse oximeter.
14.Tonny has undergoes a left thoracotomy and a partial pneumonectomy.
Chest tubes are inserted, and one-bottle water-seal drainage is instituted in
the operating room. In the postanesthesia care unit Tonny is placed in
Fowler’s position on either his right side or on his back. The nurse is aware
that this position:
A. Reduce incisional pain.
B. Facilitate ventilation of the left lung.
C. Equalize pressure in the pleural space.
D. Increase venous return
15.Kristine is scheduled for a bronchoscopy. When teaching Kristine what to
expect afterward, the nurse’s highest priority of information would be:
A. Food and fluids will be withheld for at least 2 hours.
B. Warm saline gargles will be done q 2h.
C. Coughing and deep-breathing exercises will be done q2h.
D. Only ice chips and cold liquids will be allowed initially.
16.Nurse Tristan is caring for a male client in acute renal failure. The nurse
should expect hypertonic glucose, insulin infusions, and sodium bicarbonate
to be used to treat:
A. hypernatremia.
B. hypokalemia.
C. hyperkalemia.
D. hypercalcemia.
17.Ms. X has just been diagnosed with condylomata acuminata (genital
warts). What information is appropriate to tell this client?
A. This condition puts her at a higher risk for cervical cancer; therefore, she
should have a Papanicolaou (Pap) smear annually.
B. The most common treatment is metronidazole (Flagyl), which should
eradicate the problem within 7 to 10 days.
C. The potential for transmission to her sexual partner will be eliminated if
condoms are used every time they have sexual intercourse.
D. The human papillomavirus (HPV), which causes condylomata
acuminata, can’t be transmitted during oral sex.
18.Maritess was recently diagnosed with a genitourinary problem and is being
examined in the emergency department. When palpating the her kidneys, the
nurse should keep which anatomical fact in mind?
A. The left kidney usually is slightly higher than the right one.
B. The kidneys are situated just above the adrenal glands.
C. The average kidney is approximately 5 cm (2″) long and 2 to 3 cm (¾” to
1-1/8″) wide.
D. The kidneys lie between the 10th and 12th thoracic vertebrae.
19.Jestoni with chronic renal failure (CRF) is admitted to the urology unit. The
nurse is aware that the diagnostic test are consistent with CRF if the result is:
A. Increased pH with decreased hydrogen ions.
B. Increased serum levels of potassium, magnesium, and calcium.
C. Blood urea nitrogen (BUN) 100 mg/dl and serum creatinine 6.5 mg/ dl.
D. Uric acid analysis 3.5 mg/dl and phenolsulfonphthalein (PSP) excretion
75%.
20. Katrina has an abnormal result on a Papanicolaou test. After admitting
that she read her chart while the nurse was out of the room, Katrina asks what
dysplasia means. Which definition should the nurse provide?
A. Presence of completely undifferentiated tumor cells that don’t resemble
cells of the tissues of their origin.
B. Increase in the number of normal cells in a normal arrangement in a
tissue or an organ.
C. Replacement of one type of fully differentiated cell by another in tissues
where the second type normally isn’t found.
D. Alteration in the size, shape, and organization of differentiated cells.
21. During a routine checkup, Nurse Mariane assesses a male client with
acquired immunodeficiency syndrome (AIDS) for signs and symptoms of
cancer. What is the most common AIDS-related cancer?
A. Squamous cell carcinoma
B. Multiple myeloma
C. Leukemia
D. Kaposi’s sarcoma
22.Ricardo is scheduled for a prostatectomy, and the anesthesiologist plans
to use a spinal (subarachnoid) block during surgery. In the operating room, the
nurse positions the client according to the anesthesiologist’s instructions.
Why does the client require special positioning for this type of anesthesia?
A. To prevent confusion
B. To prevent seizures
C. To prevent cerebrospinal fluid (CSF) leakage
D. To prevent cardiac arrhythmias
23.A male client had a nephrectomy 2 days ago and is now complaining of
abdominal pressure and nausea. The first nursing action should be to:
A. Auscultate bowel sounds.
B. Palpate the abdomen.
C. Change the client’s position.
D. Insert a rectal tube.
24.Wilfredo with a recent history of rectal bleeding is being prepared for a
colonoscopy. How should the nurse Patricia position the client for this test
initially?
A. Lying on the right side with legs straight
B. Lying on the left side with knees bent
C. Prone with the torso elevated
D. Bent over with hands touching the floor
25.A male client with inflammatory bowel disease undergoes an ileostomy. On
the first day after surgery, Nurse Oliver notes that the client’s stoma appears
dusky. How should the nurse interpret this finding?
A. Blood supply to the stoma has been interrupted.
B. This is a normal finding 1 day after surgery.
C. The ostomy bag should be adjusted.
D. An intestinal obstruction has occurred.
26.Anthony suffers burns on the legs, which nursing intervention helps
prevent contractures?
A. Applying knee splints
B. Elevating the foot of the bed
C. Hyperextending the client’s palms
D. Performing shoulder range-of-motion exercises
27.Nurse Ron is assessing a client admitted with second- and third-degree
burns on the face, arms, and chest. Which finding indicates a potential
problem?
A. Partial pressure of arterial oxygen (PaO2) value of 80 mm Hg.
B. Urine output of 20 ml/hour.
C. White pulmonary secretions.
D. Rectal temperature of 100.6° F (38° C).
28. Mr. Mendoza who has suffered a cerebrovascular accident (CVA) is too
weak to move on his own. To help the client avoid pressure ulcers, Nurse Celia
should:
A. Turn him frequently.
B. Perform passive range-of-motion (ROM) exercises.
C. Reduce the client’s fluid intake.
D. Encourage the client to use a footboard.
29.Nurse Maria plans to administer dexamethasone cream to a female client
who has dermatitis over the anterior chest. How should the nurse apply this
topical agent?
A. With a circular motion, to enhance absorption.
B. With an upward motion, to increase blood supply to the affected area
C. In long, even, outward, and downward strokes in the direction of hair
growth
D. In long, even, outward, and upward strokes in the direction opposite hair
growth
30.Nurse Kate is aware that one of the following classes of medication
protect the ischemic myocardium by blocking catecholamines
and sympathetic nerve stimulation is:
A. Beta -adrenergic blockers
B. Calcium channel blocker
C. Narcotics
D. Nitrates
31.A male client has jugular distention. On what position should the nurse
place the head of the bed to obtain the most accurate reading of jugular vein
distention?
A. High Fowler’s
B. Raised 10 degrees
C. Raised 30 degrees
D. Supine position
32.The nurse is aware that one of the following classes of medications
maximizes cardiac performance in clients with heart failure by increasing
ventricular contractility?
A. Beta-adrenergic blockers
B. Calcium channel blocker
C. Diuretics
D. Inotropic agents
33.A male client has a reduced serum high-density lipoprotein (HDL) level and
an elevated low-density lipoprotein (LDL) level. Which of the following dietary
modifications is not appropriate for this client?
A. Fiber intake of 25 to 30 g daily
B. Less than 30% of calories form fat
C. Cholesterol intake of less than 300 mg daily
D. Less than 10% of calories from saturated fat
34. A 37-year-old male client was admitted to the coronary care unit (CCU) 2
days ago with an acute myocardial infarction. Which of the following actions
would breach the client confidentiality?
A. The CCU nurse gives a verbal report to the nurse on the telemetry unit
before transferring the client to that unit
B. The CCU nurse notifies the on-call physician about a change in the
client’s condition
C. The emergency department nurse calls up the latest electrocardiogram
results to check the client’s progress.
D. At the client’s request, the CCU nurse updates the client’s wife on his
condition
35. A male client arriving in the emergency department is receiving
cardiopulmonary resuscitation from paramedics who are giving ventilations
through an endotracheal (ET) tube that they placed in the client’s home.
During a pause in compressions, the cardiac monitor shows narrow QRS
complexes and a heart rate of beats/minute with a palpable pulse. Which of
the following actions
should the nurse take first?
A. Start an L.V. line and administer amiodarone (Cardarone), 300 mg L.V.
over 10 minutes.
B. Check endotracheal tube placement.
C. Obtain an arterial blood gas (ABG) sample.
D. Administer atropine, 1 mg L.V.
36. After cardiac surgery, a client’s blood pressure measures 126/80 mm Hg.
Nurse Katrina determines that mean arterial pressure (MAP) is which of the
following?
A. 46 mm Hg
B. 80 mm Hg
C. 95 mm Hg
D. 90 mm Hg
37. A female client arrives at the emergency department with chest and
stomach pain and a report of black tarry stool for several months. Which of
the following order should the nurse Oliver anticipate?
A. Cardiac monitor, oxygen, creatine kinase and lactate dehydrogenase
levels
B. Prothrombin time, partial thromboplastin time, fibrinogen and fibrin split
product values.
C. Electrocardiogram, complete blood count, testing for occult blood,
comprehensive serum metabolic panel.
D. Electroencephalogram, alkaline phosphatase and aspartate
aminotransferase levels, basic serum metabolic panel
38. Macario had coronary artery bypass graft (CABG) surgery 3 days ago.
Which of the following conditions is suspected by the nurse when a decrease
in platelet count from 230,000 ul to 5,000 ul is noted?
A. Pancytopenia
B. Idiopathic thrombocytopemic purpura (ITP)
C. Disseminated intravascular coagulation (DIC)
D. Heparin-associated thrombosis and thrombocytopenia (HATT)
39. Which of the following drugs would be ordered by the physician to improve
the platelet count in a male client with idiopathic thrombocytopenic purpura
(ITP)?
A. Acetylsalicylic acid (ASA)
B. Corticosteroids
C. Methotrezate
D. Vitamin K
40. A female client is scheduled to receive a heart valve replacement with a
porcine valve. Which of the following types of transplant is this?
A. Allogeneic
B. Autologous
C. Syngeneic
D. Xenogeneic
41. Marco falls off his bicycle and injuries his ankle. Which of the following
actions shows the initial response to the injury in the extrinsic pathway?
A. Release of Calcium
B. Release of tissue thromboplastin
C. Conversion of factors XII to factor XIIa
D. Conversion of factor VIII to factor VIIIa
42. Instructions for a client with systemic lupus erythematosus (SLE) would
include information about which of the following blood dyscrasias?
A. Dressler’s syndrome
B. Polycythemia
C. Essential thrombocytopenia
D. Von Willebrand’s disease
43. The nurse is aware that the following symptoms is most commonly an
early indication of stage 1 Hodgkin’s disease?
A. Pericarditis
B. Night sweat
C. Splenomegaly
D. Persistent hypothermia
44. Francis with leukemia has neutropenia. Which of the following functions
must frequently assessed?
A. Blood pressure
B. Bowel sounds
C. Heart sounds
D. Breath sounds
45. The nurse knows that neurologic complications of multiple myeloma
(MM) usually involve which of the following body system?
A. Brain
B. Muscle spasm
C. Renal dysfunction
D. Myocardial irritability
46. Nurse Patricia is aware that the average length of time from human
immunodeficiency virus (HIV) infection to the development of acquired
immunodeficiency syndrome (AIDS)?
A. Less than 5 years
B. 5 to 7 years
C. 10 years
D. More than 10 years
47. An 18-year-old male client admitted with heat stroke begins to show signs
of disseminated intravascular coagulation (DIC). Which of the following
laboratory findings is most consistent with DIC?
A. Low platelet count
B. Elevated fibrinogen levels
C. Low levels of fibrin degradation products
D. Reduced prothrombin time
48. Mario comes to the clinic complaining of fever, drenching night sweats,
and unexplained weight loss over the past 3 months. Physical examination
reveals a single enlarged supraclavicular lymph node. Which of the following
is the most probable diagnosis?
A. Influenza
B. Sickle cell anemia
C. Leukemia
D. Hodgkin’s disease
49. A male client with a gunshot wound requires an emergency blood
transfusion. His blood type is AB negative. Which blood type would be the
safest for him to receive?
A. AB Rh-positive
B. A Rh-positive
C. A Rh-negative
D. O Rh-positive
Situation: Stacy is diagnosed with acute lymphoid leukemia (ALL) and
beginning chemotherapy.
50. Stacy is discharged from the hospital following her chemotherapy
treatments. Which statement of Stacy’s mother indicated that she
understands when she will contact the physician?
A. “I should contact the physician if Stacy has difficulty in sleeping”.
B. “I will call my doctor if Stacy has persistent vomiting and diarrhea”.
C. “My physician should be called if Stacy is irritable and unhappy”.
D. “Should Stacy have continued hair loss, I need to call the doctor”.
51. Stacy’s mother states to the nurse that it is hard to see Stacy with no hair.
The best response for the nurse is:
A. “Stacy looks very nice wearing a hat”.
B. “You should not worry about her hair, just be glad that she is alive”.
C. “Yes it is upsetting. But try to cover up your feelings when you are with
her or else she may be upset”.
D. “This is only temporary; Stacy will re-grow new hair in 3-6 months, but
may be different in texture”.
52. Stacy has beginning stomatitis. To promote oral hygiene and comfort, the
nurse in-charge should:
A. Provide frequent mouthwash with normal saline.
B. Apply viscous Lidocaine to oral ulcers as needed.
C. Use lemon glycerine swabs every 2 hours.
D. Rinse mouth with Hydrogen Peroxide.
53. During the administration of chemotherapy agents, Nurse Oliver observed
that the IV site is red and swollen, when the IV is touched Stacy shouts in pain.
The first nursing action to take is:
A. Notify the physician
B. Flush the IV line with saline solution
C. Immediately discontinue the infusion
D. Apply an ice pack to the site, followed by warm compress.
54. The term “blue bloater” refers to a male client which of the following
conditions?
A. Adult respiratory distress syndrome (ARDS)
B. Asthma
C. Chronic obstructive bronchitis
D. Emphysema
55. The term “pink puffer” refers to the female client with which of the
following conditions?
A. Adult respiratory distress syndrome (ARDS)
B. Asthma
C. Chronic obstructive bronchitis
D. Emphysema
56. Jose is in danger of respiratory arrest following the administration of a
narcotic analgesic. An arterial blood gas value is obtained. Nurse Oliver would
expect the paco2 to be which of the following values?
A. 15 mm Hg
B. 30 mm Hg
C. 40 mm Hg
D. 80 mm Hg
57. Timothy’s arterial blood gas (ABG) results are as follows; pH 7.16; Paco2
80 mm Hg; Pao2 46 mm Hg; HCO3- 24mEq/L; Sao2 81%. This ABG result
represents which of the following conditions?
A. Metabolic acidosis
B. Metabolic alkalosis
C. Respiratory acidosis
D. Respirator y alkalosis
58. Norma has started a new drug for hypertension. Thirty minutes after she
takes the drug, she develops chest tightness and becomes short of breath and
tachypneic. She has a decreased level of consciousness. These signs indicate
which of the following conditions?
A. Asthma attack
B. Pulmonary embolism
C. Respiratory failure
D. Rheumatoid arthritis
Situation: Mr. Gonzales was admitted to the hospital with ascites and
jaundice. To rule out cirrhosis of the liver:
59. Which laboratory test indicates liver cirrhosis?
A. Decreased red blood cell count
B. Decreased serum acid phosphate level
C. Elevated white blood cell count
D. Elevated serum aminotransferase
60.The biopsy of Mr. Gonzales confirms the diagnosis of cirrhosis. Mr.
Gonzales is at increased risk for excessive bleeding primarily because of:
A. Impaired clotting mechanism
B. Varix formation
C. Inadequate nutrition
D. Trauma of invasive procedure
61. Mr. Gonzales develops hepatic encephalopathy. Which clinical
manifestation is most common with this condition?
A. Increased urine output
B. Altered level of consciousness
C. Decreased tendon reflex
D. Hypotension
62. When Mr. Gonzales regained consciousness, the physician orders 50 ml of
Lactose p.o. every 2 hours. Mr. Gozales develops diarrhea. The nurse best
action would be:
A. “I’ll see if your physician is in the hospital”.
B. “Maybe your reacting to the drug; I will withhold the next dose”.
C. “I’ll lower the dosage as ordered so the drug causes only 2 to 4 stools a
day”.
D. “Frequently, bowel movements are needed to reduce sodium level”.
63. Which of the following groups of symptoms indicates a ruptured
abdominal aortic aneurysm?
A. Lower back pain, increased blood pressure, decreased re blood cell
(RBC) count, increased white blood (WBC) count.
B. Severe lower back pain, decreased blood pressure, decreased RBC
count, increased WBC count.
C. Severe lower back pain, decreased blood pressure, decreased RBC
count, decreased RBC count, decreased WBC count.
D. Intermitted lower back pain, decreased blood pressure, decreased RBC
count, increased WBC count.
64. After undergoing a cardiac catheterization, Tracy has a large puddle of
blood under his buttocks. Which of the following steps should the nurse take
first?
A. Call for help.
B. Obtain vital signs
C. Ask the client to “lift up”
D. Apply gloves and assess the groin site
65. Which of the following treatment is a suitable surgical intervention for a
client with unstable angina?
A. Cardiac catheterization
B. Echocardiogram
C. Nitroglycerin
D. Percutaneous transluminal coronary angioplasty (PTCA)
66. The nurse is aware that the following terms used to describe reduced
cardiac output and perfusion impairment due to ineffective pumping of the
heart is:
A. Anaphylactic shock
B. Cardiogenic shock
C. Distributive shock
D. Myocardial infarction (MI)
67. A client with hypertension ask the nurse which factors can cause blood
pressure to drop to normal levels?
A. Kidneys’ excretion to sodium only.
B. Kidneys’ retention of sodium and water
C. Kidneys’ excretion of sodium and water
D. Kidneys’ retention of sodium and excretion of water
68. Nurse Rose is aware that the statement that best explains why furosemide
(Lasix) is administered to treat hypertension is:
A. It dilates peripheral blood vessels.
B. It decreases sympathetic cardioacceleration.
C. It inhibits the angiotensin-coverting enzymes
D. It inhibits reabsorption of sodium and water in the loop of Henle.
69. Nurse Nikki knows that laboratory results supports the diagnosis of
systemic lupus erythematosus (SLE) is:
A. Elavated serum complement level
B. Thrombocytosis, elevated sedimentation rate
C. Pancytopenia, elevated antinuclear antibody (ANA) titer
D. Leukocysis, elevated blood urea nitrogen (BUN) and creatinine levels
70. Arnold, a 19-year-old client with a mild concussion is discharged from the
emergency department. Before discharge, he complains of a headache. When
offered acetaminophen, his mother tells the nurse the headache is severe and
she would like her son to have something stronger. Which of the following
responses by the nurse is appropriate?
A. “Your son had a mild concussion, acetaminophen is strong enough.”
B. “Aspirin is avoided because of the danger of Reye’s syndrome in
children or young adults.”
C. “Narcotics are avoided after a head injury because they may hide a
worsening condition.”
D. Stronger medications may lead to vomiting, which increases the
intracarnial pressure (ICP).”
71. When evaluating an arterial blood gas from a male client with a subdural
hematoma, the nurse notes the Paco2 is 30 mm Hg. Which of the following
responses best describes the result?
A. Appropriate; lowering carbon dioxide (CO2) reduces intracranial
pressure (ICP)
B. Emergent; the client is poorly oxygenated
C. Normal
D. Significant; the client has alveolar hypoventilation
72. When prioritizing care, which of the following clients should the nurse
Olivia assess first?
A. A 17-year-old clients 24-hours postappendectomy
B. A 33-year-old client with a recent diagnosis of Guillain-Barre syndrome
C. A 50-year-old client 3 days postmyocardial infarction
D. A 50-year-old client with diverticulitis
73. JP has been diagnosed with gout and wants to know why colchicine is
used in the treatment of gout. Which of the following actions of colchicines
explains why it’s effective for gout?
A. Replaces estrogen
B. Decreases infection
C. Decreases inflammation
D. Decreases bone demineralization
74. Norma asks for information about osteoarthritis. Which of the following
statements about osteoarthritis is correct?
A. Osteoarthritis is rarely debilitating
B. Osteoarthritis is a rare form of arthritis
C. Osteoarthritis is the most common form of arthritis
D. Osteoarthritis afflicts people over 60
75. Ruby is receiving thyroid replacement therapy develops the flu and forgets
to take her thyroid replacement medicine. The nurse understands that
skipping this medication will put the client at risk for developing which of the
following lifethreatening complications?
A. Exophthalmos
B. Thyroid storm
C. Myxedema coma
D. Tibial myxedema
76. Nurse Sugar is assessing a client with Cushing’s syndrome. Which
observation should the nurse report to the physician immediately?
A. Pitting edema of the legs
B. An irregular apical pulse
C. Dry mucous membranes
D. Frequent urination
77. Cyrill with severe head trauma sustained in a car accident is admitted to
the intensive care unit. Thirty-six hours later, the client’s urine output suddenly
rises above 200 ml/hour, leading the nurse to suspect diabetes insipidus.
Which laboratory findings support the nurse’s suspicion of diabetes insipidus?
A. Above-normal urine and serum osmolality levels
B. Below-normal urine and serum osmolality levels
C. Above-normal urine osmolality level, below-normal serum osmolality
level
D. Below-normal urine osmolality level, above-normal serum osmolality
level
78. Jomari is diagnosed with hyperosmolar hyperglycemic nonketotic
syndrome (HHNS) is stabilized and prepared for discharge. When preparing
the client for discharge and home management, which of the following
statements indicates that the client understands her condition and how to
control it?
A. “I can avoid getting sick by not becoming dehydrated and by paying
attention to my need to urinate, drink, or eat more than usual.”
B. “If I experience trembling, weakness, and headache, I should drink a
glass of soda that contains sugar.”
C. “I will have to monitor my blood glucose level closely and notify the
physician if it’s constantly elevated.”
D. “If I begin to feel especially hungry and thirsty, I’ll eat a snack high in
carbohydrates.”
79. A 66-year-old client has been complaining of sleeping more, increased
urination, anorexia, weakness, irritability, depression, and bone pain that
interferes with her going outdoors. Based on these assessment findings, the
nurse would suspect which of the following disorders?
A. Diabetes mellitus
B. Diabetes insipidus
C. Hypoparathyroidism
D. Hyperparathyroidism
80. Nurse Lourdes is teaching a client recovering from addisonian crisis about
the need to take fludrocortisone acetate and hydrocortisone at home. Which
statement by the client indicates an understanding of the instructions?
A. “I’ll take my hydrocortisone in the late afternoon, before dinner.”
B. “I’ll take all of my hydrocortisone in the morning, right after I wake up.”
C. “I’ll take two-thirds of the dose when I wake up and one-third in the late
afternoon.”
D. “I’ll take the entire dose at bedtime.”
81. Which of the following laboratory test results would suggest to the nurse
Len that a client has a corticotropin-secreting pituitary adenoma?
A. High corticotropin and low cortisol levels
B. Low corticotropin and high cortisol levels
C. High corticotropin and high cortisol levels
D. Low corticotropin and low cortisol levels
82. A male client is scheduled for a transsphenoidal hypophysectomy to
remove a pituitary tumor. Preoperatively, the nurse should assess  for
potential complications by doing which of the following?
A. Testing for ketones in the urine
B. Testing urine specific gravity
C. Checking temperature every 4 hours
D. Performing capillary glucose testing every 4 hours
83. Capillary glucose monitoring is being performed every 4 hours for a client
diagnosed with diabetic ketoacidosis. Insulin is administered using a scale of
regular insulin according to glucose results. At 2 p.m., the client has a
capillary glucose level of 250 mg/dl for which he receives 8 U of regular
insulin. Nurse Mariner should expect the dose’s:
A. onset to be at 2 p.m. and its peak to be at 3 p.m.
B. onset to be at 2:15 p.m. and its peak to be at 3 p.m.
C. onset to be at 2:30 p.m. and its peak to be at 4 p.m.
D. onset to be at 4 p.m. and its peak to be at 6 p.m.
84. The physician orders laboratory tests to confirm hyperthyroidism in a
female client with classic signs and symptoms of this disorder. Which test
result would confirm the diagnosis?
A. No increase in the thyroid-stimulating hormone (TSH) level after 30
minutes during the TSH stimulation test
B. A decreased TSH level
C. An increase in the TSH level after 30 minutes during the TSH stimulation
test
D. Below-normal levels of serum triiodothyronine (T3) and serum thyroxine
(T4) as detected by radioimmunoassay
85. Rico with diabetes mellitus must learn how to self-administer insulin. The
physician has prescribed 10 U of U-100 regular insulin and 35 U of U-100
isophane insulin suspension (NPH) to be taken before breakfast. When
teaching the client how to select and rotate insulin injection sites, the nurse
should provide which instruction?
A. “Inject insulin into healthy tissue with large blood vessels and nerves.”
B. “Rotate injection sites within the same anatomic region, not among
different regions.”
C. “Administer insulin into areas of scar tissue or hypotrophy whenever
possible.”
D. “Administer insulin into sites above muscles that you plan to exercise
heavily later that day.”
86. Nurse Sarah expects to note an elevated serum glucose level in a client
with hyperosmolar hyperglycemic nonketotic syndrome (HHNS). Which other
laboratory finding should the nurse anticipate?
A. Elevated serum acetone level
B. Serum ketone bodies
C. Serum alkalosis
D. Below-normal serum potassium level
87. For a client with Graves’ disease, which nursing intervention promotes
comfort?
A. Restricting intake of oral fluids
B. Placing extra blankets on the client’s bed
C. Limiting intake of high-carbohydrate foods
D. Maintaining room temperature in the low-normal range
88. Patrick is treated in the emergency department for a Colles’ fracture
sustained during a fall. What is a Colles’ fracture?
A. Fracture of the distal radius
B. Fracture of the olecranon
C. Fracture of the humerus
D. Fracture of the carpal scaphoid
89. Cleo is diagnosed with osteoporosis. Which electrolytes are involved in the
development of this disorder?
A. Calcium and sodium
B. Calcium and phosphorous
C. Phosphorous and potassium
D. Potassium and sodium
90. Johnny a firefighter was involved in extinguishing a house fire and is being
treated to smoke inhalation. He develops severe hypoxia 48 hours after the
incident, requiring intubation and mechanical ventilation. He most likely has
developed which of the following conditions?
A. Adult respiratory distress syndrome (ARDS)
B. Atelectasis
C. Bronchitis
D. Pneumonia
91. A 67-year-old client develops acute shortness of breath and progressive
hypoxia requiring right femur. The hypoxia was probably caused by which of
the following conditions?
A. Asthma attack
B. Atelectasis
C. Bronchitis
D. Fat embolism
92. A client with shortness of breath has decreased to absent breath sounds
on the right side, from the apex to the base. Which of the following conditions
would best explain this?
A. Acute asthma
B. Chronic bronchitis
C. Pneumonia
D. Spontaneous pneumothorax
93. A 62-year-old male client was in a motor vehicle accident as an
unrestrained driver. He’s now in the emergency department complaining of
difficulty of breathing and chest pain. On auscultation of his lung field, no
breath sounds are present in the upper lobe. This client may have which of the
following conditions?
A. Bronchitis
B. Pneumonia
C. Pneumothorax
D. Tuberculosis (TB)
94. If a client requires a pneumonectomy, what fills the area of the thoracic
cavity?
A. The space remains filled with air only
B. The surgeon fills the space with a gel
C. Serous fluids fills the space and consolidates the region
D. The tissue from the other lung grows over to the other side
95. Hemoptysis may be present in the client with a pulmonary embolism
because of which of the following reasons?
A. Alveolar damage in the infracted area
B. Involvement of major blood vessels in the occluded area
C. Loss of lung parenchyma
D. Loss of lung tissue
96. Aldo with a massive pulmonary embolism will have an arterial blood gas
analysis performed to determine the extent of hypoxia. The acid-base disorder
that may be present is?
A. Metabolic acidosis
B. Metabolic alkalosis
C. Respiratory acidosis
D. Respiratory alkalosis
97. After a motor vehicle accident, Armand an 22-year-old client is admitted
with a pneumothorax. The surgeon inserts a chest tube and attaches it to a
chest drainage system. Bubbling soon appears in the water seal chamber.
Which of the following is the most likely cause of the bubbling?
A. Air leak
B. Adequate suction
C. Inadequate suction
D. Kinked chest tube
98. Nurse Michelle calculates the IV flow rate for a postoperative client. The
client receives 3,000 ml of Ringer’s lactate solution IV to run over 24 hours.
The IV infusion set has a drop factor of 10 drops per milliliter. The nurse
should regulate the client’s IV to deliver how many drops per minute?
A. 18
B. 21
C. 35
D. 40
99. Mickey, a 6-year-old child with a congenital heart disorder is admitted with
congestive heart failure. Digoxin (lanoxin) 0.12 mg is ordered for the child. The
bottle of Lanoxin contains .05 mg of Lanoxin in 1 ml of solution. What amount
should the nurse administer to the child?
A. 1.2 ml
B. 2.4 ml
C. 3.5 ml
D. 4.2 ml
100. Nurse Alexandra teaches a client about elastic stockings. Which of the
following statements, if made by the client, indicates to the   nurse that the
teaching was successful?
A. “I will wear the stockings until the physician tells me to remove them.”
B. “I should wear the stockings even when I am sleep.”
C. “Every four hours I should remove the stockings for a half hour.”
D. “I should put on the stockings before getting out of bed in the morning.”
Answers and Rationales
1. Answer: (C) Hypertension. Hypertension, along with fever, and tenderness
over the grafted kidney, reflects acute rejection.
2. Answer: (A) Pain. Sharp, severe pain (renal colic) radiating toward the
genitalia and thigh is caused by uretheral distention and smooth muscle
spasm; relief form pain is the priority.
3. Answer: (D) Decrease the size and vascularity of the thyroid gland. Lugol’s
solution provides iodine, which aids in decreasing the vascularity of the
thyroid gland, which limits the risk of hemorrhage when surgery is
performed.
4. Answer: (A) Liver Disease. The client with liver disease has a decreased
ability to metabolize carbohydrates because of a decreased ability to form
glycogen (glycogenesis) and to form glucose from glycogen.
5. Answer: (C) Leukopenia. Leukopenia, a reduction in WBCs, is a systemic
effect of chemotherapy as a result of myelosuppression.
6. Answer: (C) Avoid foods that in the past caused flatus. Foods that bothered
a person preoperatively will continue to do so after a colostomy.
7. Answer: (B) Keep the irrigating container less than 18 inches above
the stoma.”  This height permits the solution to flow slowly with little
force so that excessive peristalsis is not immediately precipitated.
8. Answer: (A) Administer Kayexalate. Kayexalate,a potassium exchange
resin, permits sodium to be exchanged for potassium in the intestine,
reducing the serum potassium level.
9. Answer:(B) 28 gtt/min.  This is the correct flow rate; multiply the amount
to be infused (2000 ml) by the drop factor (10) and divide the result by the
amount of time in minutes (12 hours x 60 minutes)
10. Answer: (D) Upper trunk.  The percentage designated for each burned
part of the body using the rule of nines: Head and neck 9%; Right upper
extremity 9%; Left upper extremity 9%; Anterior trunk 18%; Posterior trunk
18%; Right lower extremity 18%; Left lower extremity 18%; Perineum 1%.
11. Answer: (C) Bleeding from ears.  The nurse needs to perform a thorough
assessment that could indicate alterations in cerebral function, increased
intracranial pressures, fractures and bleeding. Bleeding from the ears
occurs only with basal skull fractures that can easily contribute to
increased intracranial pressure and brain herniation.
12. Answer: (D) may engage in contact sports. The client should be advised by
the nurse to avoid contact sports. This will prevent trauma to the area of
the pacemaker generator.
13. Answer: (A) Oxygen at 1-2L/min is given to maintain the hypoxic stimulus for
breathing. COPD causes a chronic CO2 retention that renders the medulla
insensitive to the CO2 stimulation for breathing. The hypoxic state of the
client then becomes the stimulus for breathing. Giving the client oxygen in
low concentrations will maintain the client’s hypoxic drive.
14. Answer: (B) Facilitate ventilation of the left lung. Since only a partial
pneumonectomy is done, there is a need to promote expansion of this
remaining Left lung by positioning the client on the opposite unoperated
side.
15. Answer: (A) Food and fluids will be withheld for at least 2 hours.  Prior to
bronchoscopy, the doctors sprays the back of the throat with anesthetic to
minimize the gag reflex and thus facilitate the insertion of the
bronchoscope. Giving the client food and drink after the procedure without
checking on the return of the gag reflex can cause the client to aspirate.
The gag reflex usually returns after two hours.
16. Answer: (C) hyperkalemia. Hyperkalemia is a common complication of
acute renal failure. It’s life-threatening if immediate action isn’t taken to
reverse it. The administration of glucose and regular insulin, with sodium
bicarbonate if necessary, can temporarily prevent cardiac arrest by
moving potassium into the cells and temporarily reducing serum
potassium levels. Hypernatremia, hypokalemia, and hypercalcemia don’t
usually occur with acute renal failure and aren’t treated with glucose,
insulin, or sodium bicarbonate.
17. Answer: (A) This condition puts her at a higher risk for cervical
cancer; therefore, she should have a Papanicolaou (Pap) smear annually. Women
with condylomata acuminata are at risk for cancer of the cervix and vulva.
Yearly Pap smears are very important for early detection. Because
condylomata acuminata is a virus, there is no permanent cure. Because
condylomata acuminata can occur on the vulva, a condom won’t protect
sexual partners. HPV can be transmitted to other parts of the body, such
as the mouth, oropharynx, and larynx.
18. Answer: (A) The left kidney usually is slightly higher than the right one. The
left kidney usually is slightly higher than the right one. An adrenal gland
lies atop each kidney. The average kidney measures approximately 11 cm
(4-3/8″) long, 5 to 5.8 cm (2″ to 2¼”) wide, and 2.5 cm (1″) thick. The
kidneys are located retroperitoneally, in the posterior aspect of the
abdomen, on either side of the vertebral column. They lie between the
12th thoracic and 3rd lumbar vertebrae.
19. Answer: (C) Blood urea nitrogen (BUN) 100 mg/dl and serum creatinine 6.5
mg/dl. The normal BUN level ranges 8 to 23 mg/dl; the normal
serum creatinine level ranges from 0.7 to 1.5 mg/dl. The test results in
option C are abnormally elevated, reflecting CRF and the kidneys’
decreased ability to remove nonprotein nitrogen waste from the blood.
CRF causes decreased pH and increased hydrogen ions — not vice versa.
CRF also increases serum levels of potassium, magnesium, and
phosphorous, and decreases serum levels of calcium. A uric acid analysis
of 3.5 mg/dl falls within the normal range of 2.7 to 7.7 mg/dl; PSP
excretion of 75% also falls with the normal range of 60% to 75%.
20. Answer: (D) Alteration in the size, shape, and organization of differentiated
cells. Dysplasia refers to an alteration in the size, shape, and organization
of differentiated cells. The presence of completely undifferentiated tumor
cells that don’t resemble cells of the tissues of their origin is called
anaplasia. An increase in the number of normal cells in a normal
arrangement in a tissue or an organ is called hyperplasia. Replacement of
one type of fully differentiated cell by another in tissues where the second
type normally isn’t found is called metaplasia.
21. Answer: (D) Kaposi’s sarcoma. Kaposi’s sarcoma is the most common
cancer associated with AIDS. Squamous cell carcinoma, multiple
myeloma, and leukemia may occur in anyone and aren’t associated
specifically with AIDS.
22. Answer: (C) To prevent cerebrospinal fluid (CSF) leakage. The client
receiving a subarachnoid block requires special positioning to prevent CSF
leakage and headache and to ensure proper anesthetic distribution.
Proper positioning doesn’t help prevent confusion, seizures, or cardiac
arrhythmias.
23. Answer: (A) Auscultate bowel sounds. If abdominal distention is
accompanied by nausea, the nurse must first auscultate bowel sounds. If
bowel sounds are absent, the nurse should suspect gastric or small
intestine dilation and these findings must be reported to the physician.
Palpation should be avoided postoperatively with abdominal distention. If
peristalsis is absent, changing positions and inserting a rectal tube won’t
relieve the client’s discomfort.
24. Answer: (B) Lying on the left side with knees bent. For a colonoscopy, the
nurse initially should position the client on the left side with knees bent.
Placing the client on the right side with legs straight, prone with the torso
elevated, or bent over with hands touching the floor wouldn’t allow proper
visualization of the large intestine.
25. Answer: (A) Blood supply to the stoma has been interrupted. An ileostomy
stoma forms as the ileum is brought through the abdominal wall to the
surface skin, creating an artificial opening for waste elimination. The
stoma should appear cherry red, indicating adequate arterial perfusion. A
dusky stoma suggests decreased perfusion, which may result from
interruption of the stoma’s blood supply and may lead to tissue damage or
necrosis. A dusky stoma isn’t a normal finding. Adjusting the ostomy bag
wouldn’t affect stoma color, which depends on blood supply to the area.
An intestinal obstruction also wouldn’t change stoma color.
26. Answer: (A) Applying knee splints. Applying knee splints prevents leg
contractures by holding the joints in a position of function. Elevating the
foot of the bed can’t prevent contractures because this action doesn’t hold
the joints in a position of function. Hyperextending a body part for an
extended time is inappropriate because it can cause contractures.
Performing shoulder range-of-motion exercises can prevent contractures
in the shoulders, but not in the legs.
27. Answer: (B) Urine output of 20 ml/hour. A urine output of less than 40
ml/hour in a client with burns indicates a fluid volume deficit. This client’s
PaO2 value falls within the normal range (80 to 100 mm Hg). White
pulmonary secretions also are normal. The client’s rectal temperature isn’t
significantly elevated and probably results from the fluid volume deficit.
28. Answer: (A) Turn him frequently. The most important intervention to
prevent pressure ulcers is frequent position changes, which relieve
pressure on the skin and underlying tissues. If pressure isn’t relieved,
capillaries become occluded, reducing circulation and oxygenation of the
tissues and resulting in cell death and ulcer formation. During passive
ROM exercises, the nurse moves each joint through its range of
movement, which improves joint mobility and circulation to the affected
area but doesn’t prevent pressure ulcers. Adequate hydration is necessary
to maintain healthy skin and ensure tissue repair. A footboard prevents
plantar flexion and footdrop by maintaining the foot in a dorsiflexed
position.
29. Answer: (C) In long, even, outward, and downward strokes in the direction of
hair growth. When applying a topical agent, the nurse should begin at
the midline and use long, even, outward, and downward strokes in
the direction of hair growth. This application pattern reduces the risk of
follicle irritation and skin inflammation.
30. Answer: (A) Beta -adrenergic blockers. Beta-adrenergic blockers work by
blocking beta receptors in the myocardium, reducing the response to
catecholamines and sympathetic nerve stimulation. They protect the
myocardium, helping to reduce the risk of another infraction by decreasing
myocardial oxygen demand. Calcium channel blockers reduce the
workload of the heart by decreasing the heart rate. Narcotics reduce
myocardial oxygen demand, promote vasodilation, and decrease anxiety.
Nitrates reduce myocardial oxygen consumption bt decreasing left
ventricular end diastolic pressure (preload) and systemic vascular
resistance (afterload).
31. Answer: (C) Raised 30 degrees. Jugular venous pressure is measured with
a centimeter ruler to obtain the vertical distance between the sternal angle
and the point of highest pulsation with the head of the bed inclined
between 15 to 30 degrees. Increased pressure can’t be seen when the
client is supine or when the head of the bed is raised 10 degrees because
the point that marks the pressure level is above the jaw (therefore, not
visible). In high Fowler’s position, the veins would be barely discernible
above the clavicle.
32. Answer: (D) Inotropic agents. Inotropic agents are administered to
increase the force of the heart’s contractions, thereby increasing
ventricular contractility and ultimately increasing cardiac output. Beta-
adrenergic blockers and calcium channel blockers decrease the heart rate
and ultimately decreased the workload of the heart. Diuretics are
administered to decrease the overall vascular volume, also decreasing the
workload of the heart.
33. Answer: (B) Less than 30% of calories form fat. A client with low serum
HDL and high serum LDL levels should get less than 30% of daily calories
from fat. The other modifications are appropriate for this client.
34. Answer: (C) The emergency department nurse calls up the
latest electrocardiogram results to check the client’s progress. The emergency
department nurse is no longer directly involved with the client’s care and
thus has no legal right to information about his present condition. Anyone
directly involved in his care (such as the telemetry nurse and the on-call
physician) has the right to information about his condition. Because the
client requested that the nurse update his wife on his condition, doing so
doesn’t breach confidentiality.
35. Answer: (B) Check endotracheal tube placement. ET tube placement should
be confirmed as soon as the client arrives in the emergency department.
Once the airways is secured, oxygenation and ventilation should be
confirmed using an end-tidal carbon dioxide monitor and pulse oximetry.
Next, the nurse should make sure L.V. access is established. If the client
experiences symptomatic bradycardia, atropine is administered as
ordered 0.5 to 1 mg every 3 to 5 minutes to a total of 3 mg. Then the nurse
should try to find the cause of the client’s arrest by obtaining an ABG
sample. Amiodarone is indicated for ventricular tachycardia, ventricular
fibrillation and atrial flutter – not symptomatic bradycardia.
36. Answer: (C) 95 mm Hg. Use the following formula to calculate MAP
 MAP = systolic + 2 (diastolic) /3
 MAP=[126 mm Hg + 2 (80 mm Hg) ]/3
 MAP=286 mm HG/ 3
 MAP=95 mm Hg
37. Answer: (C) Electrocardiogram, complete blood count, testing for occult blood,
comprehensive serum metabolic panel. An electrocardiogram evaluates the
complaints of chest pain, laboratory tests determines anemia, and the
stool test for occult blood determines blood in the stool. Cardiac
monitoring, oxygen, and creatine kinase and lactate dehydrogenase levels
are appropriate for a cardiac primary problem. A basic metabolic panel
and alkaline phosphatase and aspartate aminotransferase levels assess
liver function. Prothrombin time, partial thromboplastin time, fibrinogen
and fibrin split products are measured to verify bleeding dyscrasias, An
electroencephalogram evaluates brain electrical activity.
38. Answer: (D) Heparin-associated thrombosis and
thrombocytopenia (HATT). HATT may occur after CABG surgery due to
heparin use during surgery. Although DIC and ITP cause platelet
aggregation and bleeding, neither is common in a client after
revascularization surgery. Pancytopenia is a reduction in all blood cells.
39. Answer: (B) Corticosteroids. Corticosteroid therapy can decrease antibody
production and phagocytosis of the antibody-coated platelets, retaining
more functioning platelets. Methotrexate can cause thrombocytopenia.
Vitamin K is used to treat an excessive anticoagulate state from warfarin
overload, and ASA decreases platelet aggregation.
40. Answer: (D) Xenogeneic. An xenogeneic transplant is between is between
human and another species. A syngeneic transplant is between identical
twins, allogeneic transplant is between two humans, and autologous is
a transplant from the same individual.
41. Answer: (B). Tissue thromboplastin is released when damaged
tissue comes in contact with clotting factors. Calcium is released to assist
the conversion of factors X to Xa. Conversion of factors XII to XIIa and VIII
to VIII a are part of the intrinsic pathway.
42. Answer: (C) Essential thrombocytopenia. Essential thrombocytopenia is
linked to immunologic disorders, such as SLE and human
immunodeficiency vitus. The disorder known as von Willebrand’s disease
is a type of hemophilia and isn’t linked to SLE. Moderate to severe anemia
is associated with SLE, not polycythermia. Dressler’s syndrome is
pericarditis that occurs after a myocardial infarction and isn’t linked to
SLE.
43. Answer: (B) Night sweat. In stage 1, symptoms include a single enlarged
lymph node (usually), unexplained fever, night sweats, malaise, and
generalized pruritis. Although splenomegaly may be present in some
clients, night sweats are generally more prevalent. Pericarditis isn’t
associated with Hodgkin’s disease, nor is hypothermia. Moreover,
splenomegaly and pericarditis aren’t symptoms. Persistent hypothermia is
associated with Hodgkin’s but isn’t an early sign of the disease.
44. Answer: (D) Breath sounds.  Pneumonia, both viral and fungal, is a
common cause of death in clients with neutropenia, so frequent
assessment of respiratory rate and breath sounds is required. Although
assessing blood pressure, bowel sounds, and heart sounds is important, it
won’t help detect pneumonia.
45. Answer: (B) Muscle spasm. Back pain or paresthesia in the lower
extremities may indicate impending spinal cord compression from a
spinal tumor. This should be recognized and treated promptly as
progression of the tumor may result in paraplegia. The other options,
which reflect parts of the nervous system, aren’t usually affected by MM.
46. Answer: (C)10 years. Epidermiologic studies show the average time from
initial contact with HIV to the development of AIDS is 10 years.
47. Answer: (A) Low platelet count. In DIC, platelets and clotting factors are
consumed, resulting in microthrombi and excessive bleeding. As clots
form, fibrinogen levels decrease and the prothrombin time increases.
Fibrin degeneration products increase as fibrinolysis takes places.
48. Answer: (D) Hodgkin’s disease. Hodgkin’s disease typically causes fever
night sweats, weight loss, and lymph mode enlargement. Influenza doesn’t
last for months. Clients with sickle cell anemia manifest signs and
symptoms of chronic anemia with pallor of the mucous membrane,
fatigue, and decreased tolerance for exercise; they don’t show fever, night
sweats, weight loss or lymph node enlargement. Leukemia doesn’t cause
lymph node enlargement.
49. Answer: (C) A Rh-negative. Human blood can sometimes contain an
inherited D antigen. Persons with the D antigen have Rh-positive blood
type; those lacking the antigen have Rh-negative blood. It’s important that
a person with Rhnegative blood receives Rh-negative blood. If Rh-positive
blood is administered to an Rh-negative person, the recipient develops
anti-Rh agglutinins, and sub sequent transfusions with Rh-positive blood
may cause serious reactions with clumping and hemolysis of red blood
cells.
50. Answer: (B) “I will call my doctor if Stacy has persistent vomiting
and diarrhea”.  Persistent (more than 24 hours) vomiting, anorexia,
and diarrhea are signs of toxicity and the patient should stop the
medication and notify the health care provider. The other manifestations
are expected side effects of chemotherapy.
51. Answer: (D) “This is only temporary; Stacy will re-grow new hair in 3-
6 months, but may be different in texture”. This is the appropriate response.
The nurse should help the mother how to cope with her own feelings
regarding the child’s disease so as not to affect the child negatively. When
the hair grows back, it is still of the same color and texture.
52. Answer: (B) Apply viscous Lidocaine to oral ulcers as needed. Stomatitis can
cause pain and this can be relieved by applying topical anesthetics such
as lidocaine before mouth care. When the patient is already comfortable,
the nurse can proceed with providing the patient with oral rinses of saline
solution mixed with equal part of water or hydrogen peroxide mixed water
in 1:3 concentrations to promote oral hygiene. Every 2-4 hours.
53. Answer: (C) Immediately discontinue the infusion. Edema or swelling at the
IV site is a sign that the needle has been dislodged and the IV solution is
leaking into the tissues causing the edema. The patient feels pain as the
nerves are irritated by pressure and the IV solution. The first action of the
nurse would be to discontinue the infusion right away to prevent further
edema and other complication.
54. Answer: (C) Chronic obstructive bronchitis. Clients with chronic obstructive
bronchitis appear bloated; they have large barrel chest and peripheral
edema, cyanotic nail beds, and at times, circumoral cyanosis. Clients with
ARDS are acutely short of breath and frequently need intubation for
mechanical ventilation and large amount of oxygen. Clients with asthma
don’t exhibit characteristics of chronic disease, and clients with
emphysema appear pink and cachectic.
55. Answer: (D) Emphysema. Because of the large amount of energy it takes
to breathe, clients with emphysema are usually cachectic. They’re pink
and usually breathe through pursed lips, hence the term “puffer.” Clients
with ARDS are usually acutely short of breath. Clients with asthma don’t
have any particular characteristics, and clients with chronic obstructive
bronchitis are bloated and cyanotic in appearance.
56. Answer: D 80 mm Hg. A client about to go into respiratory arrest will have
inefficient ventilation and will be retaining carbon dioxide. The value
expected would be around 80 mm Hg. All other values are lower than
expected.
57. Answer: (C) Respiratory acidosis. Because Paco2 is high at 80 mm Hg and
the metabolic measure, HCO3- is normal, the client has respiratory
acidosis. The pH is less than 7.35, academic, which eliminates metabolic
and respiratory alkalosis as possibilities. If the HCO3- was below 22
mEq/L the client would have metabolic acidosis.
58. Answer: (C) Respiratory failure. The client was reacting to the drug with
respiratory signs of impending anaphylaxis, which could lead to eventually
respiratory failure. Although the signs are also related to an asthma attack
or a pulmonary embolism, consider the new drug first. Rheumatoid
arthritis doesn’t manifest these signs.
59. Answer: (D) Elevated serum aminotransferase. Hepatic cell death causes
release of liver enzymes alanine aminotransferase (ALT), aspartate
aminotransferase (AST) and lactate dehydrogenase (LDH) into the
circulation. Liver cirrhosis is a chronic and irreversible disease of the liver
characterized by generalized inflammation and fibrosis of the liver tissues.
60. Answer: (A) Impaired clotting mechanism. Cirrhosis of the liver results in
decreased Vitamin K absorption and formation of clotting factors
resulting in impaired clotting mechanism.
61. Answer: (B) Altered level of consciousness. Changes in behavior and level
of consciousness are the first sins of hepatic encephalopathy. Hepatic
encephalopathy is caused by liver failure and develops when the liver is
unable to convert protein metabolic product ammonia to urea. This results
in accumulation of ammonia and other toxic in the blood that damages
the cells.
62. Answer: (C) “I’ll lower the dosage as ordered so the drug causes only 2 to 4
stools a day”. Lactulose is given to a patients with hepatic encephalopathy
to reduce absorption of ammonia in the intestines by binding with
ammonia and promoting more frequent bowel movements. If the patient
experience diarrhea, it indicates over dosage and the nurse must reduce
the amount of medication given to the patient. The stool will be mashy or
soft. Lactulose is also very sweet and may cause cramping and bloating.
63. Answer: (B) Severe lower back pain, decreased blood pressure, decreased RBC
count, increased WBC count.Severe lower back pain indicates an aneurysm
rupture, secondary to pressure being applied within the abdominal cavity.
When ruptured occurs, the pain is constant because it can’t be alleviated
until the aneurysm is repaired. Blood pressure decreases due to the loss
of blood. After the aneurysm ruptures, the vasculature is interrupted
and blood volume is lost, so blood pressure wouldn’t increase. For the
same reason, the RBC count is decreased – not increased. The WBC
count increases as cell migrate to the site of injury.
64. Answer: (D) Apply gloves and assess the groin site. Observing standard
precautions is the first priority when dealing with any blood fluid.
Assessment of the groin site is the second priority. This establishes where
the blood is coming from and determineshow much blood has been lost.
The goal in this situation is to stop the bleeding. The nurse would call for
help if it were warranted after the assessment of the situation. After
determining the extent of the bleeding, vital signs assessment is
important. The nurse should never move the client, in case a clot has
formed. Moving can disturb the clot and cause rebleeding.
65. Answer: (D) Percutaneous transluminal coronary angioplasty (PTCA). PTCA
can alleviate the blockage and restore blood flow and oxygenation. An
echocardiogram is a noninvasive diagnosis test. Nitroglycerin is an oral
sublingual medication. Cardiac catheterization is a diagnostic tool – not a
treatment.
66. Answer: (B) Cardiogenic shock. Cardiogenic shock is shock related to
ineffective pumping of the heart. Anaphylactic shock results from an
allergic reaction. Distributive shock results from changes in the
intravascular volume distribution and is usually associated with increased
cardiac output. MI isn’t a shock state, though a severe MI can lead to
shock.
67. Answer: (C) Kidneys’ excretion of sodium and water. The kidneys respond
to rise in blood pressure by excreting sodium and excess water. This
response ultimately affects sysmolic blood pressure by regulating blood
volume. Sodium or water retention would only further increase blood
pressure. Sodium and water travel together across the membrane in the
kidneys; one can’t travel without the other.
68. Answer: (D) It inhibits reabsorption of sodium and water in the loop
of Henle. Furosemide is a loop diuretic that inhibits sodium and
water reabsorption in the loop Henle, thereby causing a decrease in
blood pressure. Vasodilators cause dilation of peripheral blood vessels,
directly relaxing vascular smooth muscle and decreasing blood
pressure. Adrenergic blockers decrease sympathetic cardioacceleration
and decrease blood pressure. Angiotensin-converting enzyme
inhibitors decrease blood pressure due to their action on angiotensin.
69. Answer: (C) Pancytopenia, elevated antinuclear antibody (ANA)
titer. Laboratory findings for clients with SLE usually show pancytopenia,
elevated ANA titer, and decreased serum complement levels. Clients may
have elevated BUN and creatinine levels from nephritis, but the increase
does not indicate SLE.
70. Answer: (C) Narcotics are avoided after a head injury because they may hide a
worsening condition. Narcotics may mask changes in the level of
consciousness that indicate increased ICP and shouldn’t acetaminophen
is strong enough ignores the mother’s question and therefore isn’t
appropriate. Aspirin is contraindicated in conditions that may have
bleeding, such as trauma, and for children or young adults with viral
illnesses due to the danger of Reye’s syndrome. Stronger medications
may not necessarily lead to vomiting but will sedate the client, thereby
masking changes in his level of consciousness.
71. Answer: (A) Appropriate; lowering carbon dioxide (CO2) reduces intracranial
pressure (ICP). A normal Paco2 value is 35 to 45 mm Hg CO2
has vasodilating properties; therefore, lowering Paco2 through
hyperventilation will lower ICP caused by dilated cerebral vessels.
Oxygenation is evaluated through Pao2 and oxygen saturation. Alveolar
hypoventilation would be reflected in an increased Paco2.
72. Answer: (B) A 33-year-old client with a recent diagnosis of Guillain-
Barre syndrome . Guillain-Barre syndrome is characterized by
ascending paralysis and potential respiratory failure. The order of client
assessment should follow client priorities, with disorder of airways,
breathing, and then circulation. There’s no information to suggest the
postmyocardial infarction client has an arrhythmia or other complication.
There’s no evidence to suggest hemorrhage or perforation for the
remaining clients as a priority of care.
73. Answer: (C) Decreases inflammation. Then action of colchicines is to
decrease inflammation by reducing the migration of leukocytes to synovial
fluid. Colchicine doesn’t replace estrogen, decrease infection, or decrease
bone demineralization.
74. Answer: (C) Osteoarthritis is the most common form of
arthritis. Osteoarthritis is the most common form of arthritis and can
be extremely debilitating. It can afflict people of any age, although most
are elderly.
75. Answer: (C) Myxedema coma. Myxedema coma, severe hypothyroidism, is
a life-threatening condition that may develop if thyroid replacement
medication isn’t taken. Exophthalmos, protrusion of the eyeballs, is seen
with hyperthyroidism. Thyroid storm is life-threatening but is caused by
severe hyperthyroidism. Tibial myxedema, peripheral mucinous edema
involving the lower leg, is associated with hypothyroidism but isn’t life-
threatening.
76. Answer: (B) An irregular apical pulse. Because Cushing’s syndrome
causes aldosterone overproduction, which increases urinary potassium
loss, the disorder may lead to hypokalemia. Therefore, the nurse should
immediately report signs and symptoms of hypokalemia, such as an
irregular apical pulse, to the physician. Edema is an expected finding
because aldosterone overproduction causes sodium and fluid retention.
Dry mucous membranes and frequent urination signal dehydration, which
isn’t associated with Cushing’s syndrome.
77. Answer: (D) Below-normal urine osmolality level, above-normal
serum osmolality level. In diabetes insipidus, excessive polyuria causes
dilute urine, resulting in a below-normal urine osmolality level. At the same
time, polyuria depletes the body of water, causing dehydration that leads
to an above-normal serum osmolality level. For the same reasons,
diabetes insipidus doesn’t cause above-normal urine osmolality or below-
normal serum osmolality levels.
78. Answer: (A) “I can avoid getting sick by not becoming dehydrated and
by paying attention to my need to urinate, drink, or eat more than
usual.” Inadequate fluid intake during hyperglycemic episodes often leads
to HHNS. By recognizing the signs of hyperglycemia (polyuria, polydipsia,
and polyphagia) and increasing fluid intake, the client may prevent HHNS.
Drinking a glass of nondiet soda would be appropriate for hypoglycemia. A
client whose diabetes is controlled with oral antidiabetic agents usually
doesn’t need to monitor blood glucose levels. A highcarbohydrate diet
would exacerbate the client’s condition, particularly if fluid intake is low.
79. Answer: (D) Hyperparathyroidism. Hyperparathyroidism is most common
in older women and is characterized by bone pain and weakness from
excess parathyroid hormone (PTH). Clients also exhibit hypercaliuria-
causing polyuria. While clients with diabetes mellitus and diabetes
insipidus also have polyuria, they don’t have bone pain and increased
sleeping. Hypoparathyroidism is characterized by urinary frequency rather
than polyuria.
80. Answer: (C) “I’ll take two-thirds of the dose when I wake up and one-third in
the late afternoon.” Hydrocortisone, a glucocorticoid, should be
administered according to a schedule that closely reflects the body’s own
secretion of this hormone; therefore, two-thirds of the dose of
hydrocortisone should be taken in the morning and one-third in the late
afternoon. This dosage schedule reduces adverse effects.
81. Answer: (C) High corticotropin and high cortisol levels. A corticotropin-
secreting pituitary tumor would cause high corticotropin and high cortisol
levels. A high corticotropin level with a low cortisol level and a low
corticotropin level with a low cortisol level would be associated with
hypocortisolism. Low corticotropin and high cortisol levels would be seen
if there was a primary defect in the adrenal glands.
82. Answer: (D) Performing capillary glucose testing every 4 hours. The nurse
should perform capillary glucose testing every 4 hours because excess
cortisol may cause insulin resistance, placing the client at risk for
hyperglycemia. Urine ketone testing isn’t indicated because the client
does secrete insulin and, therefore, isn’t at risk for ketosis. Urine specific
gravity isn’t indicated because although fluid balance can be
compromised, it usually isn’t dangerously imbalanced. Temperature
regulation may be affected by excess cortisol and isn’t an accurate
indicator of infection.
83. Answer: (C) onset to be at 2:30 p.m. and its peak to be at 4 p.m.. Regular
insulin, which is a short-acting insulin, has an onset of 15 to 30 minutes
and a peak of 2 to 4 hours. Because the nurse gave the insulin at 2 p.m.,
the expected onset would be from 2:15 p.m. to 2:30 p.m. and the peak
from 4 p.m. to 6 p.m.
84. Answer: (A) No increase in the thyroid-stimulating hormone (TSH) level after
30 minutes during the TSH stimulation test. In the TSH test, failure of the TSH
level to rise after 30 minutes confirms hyperthyroidism. A decreased TSH
level indicates a pituitary deficiency of this hormone. Below-normal levels
of T3 and T4, as detected by radioimmunoassay, signal hypothyroidism. A
below-normal T4 level also occurs in malnutrition and liver disease and
may result from administration of phenytoin and certain other drugs.
85. Answer: (B) “Rotate injection sites within the same anatomic region,
not among different regions.”  The nurse should instruct the client to rotate
injection sites within the same anatomic region. Rotating sites among
different regions may cause excessive day-to-day variations in the blood
glucose level; also, insulin absorption differs from one region to the next.
Insulin should be injected only into healthy tissue lacking large blood
vessels, nerves, or scar tissue or other deviations. Injecting insulin into
areas of hypertrophy may delay absorption. The client shouldn’t inject
insulin into areas of lipodystrophy (such as hypertrophy or atrophy); to
prevent lipodystrophy, the client should rotate injection sites
systematically. Exercise speeds drug absorption, so the client shouldn’t
inject insulin into sites above muscles that will be exercised heavily.
86. Answer: (D) Below-normal serum potassium level. A client with HHNS has
an overall body deficit of potassium resulting from diuresis, which occurs
secondary to the hyperosmolar, hyperglycemic state caused by the
relative insulin deficiency. An elevated serum acetone level and serum
ketone bodies are characteristic of diabetic ketoacidosis. Metabolic
acidosis, not serum alkalosis, may occur in HHNS.
87. Answer: (D) Maintaining room temperature in the low-normal range. Graves’
disease causes signs and symptoms of hypermetabolism, such as heat
intolerance, diaphoresis, excessive thirst and appetite, and weight loss. To
reduce heat intolerance and diaphoresis, the nurse should keep the
client’s room temperature in the low-normal range. To replace fluids lost
via diaphoresis, the nurse should encourage, not restrict, intake of oral
fluids. Placing extra blankets on the bed of a client with heat intolerance
would cause discomfort. To provide needed energy and calories, the nurse
should encourage the client to eat high-carbohydrate foods.
88. Answer: (A) Fracture of the distal radius. Colles’ fracture is a fracture of the
distal radius, such as from a fall on an outstretched hand. It’s most
common in women. Colles’ fracture doesn’t refer to a fracture of the
olecranon, humerus, or carpal scaphoid.
89. Answer: (B) Calcium and phosphorous. In osteoporosis, bones lose
calcium and phosphate salts, becoming porous, brittle, and abnormally
vulnerable to fracture. Sodium and potassium aren’t involved in the
development of osteoporosis.
90. Answer: (A) Adult respiratory distress syndrome (ARDS). Severe hypoxia
after smoke inhalation is typically related to ARDS. The other conditions
listed aren’t typically associated with smoke inhalation and severe
hypoxia.
91. Answer: (D) Fat embolism. Long bone fractures are correlated with fat
emboli, whichcause shortness of breath and hypoxia. It’s unlikely the
client has developed asthma or bronchitis without a previous history. He
could develop atelectasis but it typically doesn’t produce progressive
hypoxia.
92. Answer: (D) Spontaneous pneumothorax. A spontaneous pneumothorax
occurs when the client’s lung collapses, causing an acute decreased in the
amount of functional lung used in oxygenation. The sudden collapse was
the cause of his chest pain and shortness of breath. An asthma attack
would show wheezing breath sounds, and bronchitis would have rhonchi.
Pneumonia would have bronchial breath sounds over the area of
consolidation.
93. Answer: (C) Pneumothorax. From the trauma the client experienced, it’s
unlikely he has bronchitis, pneumonia, or TB; rhonchi with bronchitis,
bronchial breath sounds with TB would be heard.
94. Answer: (C) Serous fluids fills the space and consolidates the region. Serous
fluid fills the space and eventually consolidates, preventing extensive
mediastinal shift of the heart and remaining lung. Air can’t be left in the
space. There’s no gel that can be placed in the pleural space. The tissue
from the other lung can’t cross the mediastinum, although a temporary
mediastinal shift exits until the space is filled.
95. Answer: (A) Alveolar damage in the infracted area. The infracted area
produces alveolar damage that can lead to the production of bloody
sputum, sometimes in massive amounts. Clot formation usually occurs in
the legs. There’s a loss of lung parenchyma and subsequent scar tissue
formation.
96. Answer: (D) Respiratory alkalosis. A client with massive pulmonary
embolism will have a large region and blow off large amount of carbon
dioxide, which crosses the unaffected alveolar-capillary membrane more
readily than does oxygen and results in respiratory alkalosis.
97. Answer: (A) Air leak. Bubbling in the water seal chamber of a chest
drainage system stems from an air leak. In pneumothorax an air leak can
occur as air is pulled from the pleural space. Bubbling doesn’t normally
occur with either adequate or inadequate suction or any preexisting
bubbling in the water seal chamber.
98. Answer: (B) 21. 3000 x 10 divided by 24 x 60.
99. Answer: (B) 2.4 ml. .05 mg/ 1 ml = .12mg/ x ml, .05x = .12, x = 2.4 ml.
100. Answer: (D) “I should put on the stockings before getting out of bed in the
morning. Promote venous return by applying external pressure on veins.

PNLE V for Care of Clients with


Physiologic and Psychosocial
Alterations (Part 3)
1. Mr. Marquez reports of losing his job, not being able to sleep at night, and
feeling upset with his wife. Nurse John responds to the client, “You may want
to talk about your employment situation in group today.” The Nurse is using
which therapeutic technique?

A. Observations
B. Restating
C. Exploring
D. Focusing
2. Tony refuses his evening dose of Haloperidol (Haldol), then becomes
extremely agitated in the dayroom while other clients are watching television.
He begins cursing and throwing furniture. Nurse Oliver first action is to:

A. Check the client’s medical record for an order for an as-needed I.M.
dose of medication for agitation.
B. Place the client in full leather restraints.
C. Call the attending physician and report the behavior.
D. Remove all other clients from the dayroom.
3. Tina who is manic, but not yet on medication, comes to the drug treatment
center. The nurse would not let this client join the group session because:

A. The client is disruptive.


B. The client is harmful to self.
C. The client is harmful to others.
D. The client needs to be on medication first.
4. Dervid, an adolescent boy was admitted for substance abuse and
hallucinations. The client’s mother asks Nurse Armando to talk with his
husband when he arrives at the hospital. The mother says that she is afraid of
what the father might say to the boy. The most appropriate nursing
intervention would be to:

A. Inform the mother that she and the father can work through this
problem themselves.
B. Refer the mother to the hospital social worker.
C. Agree to talk with the mother and the father together.
D. Suggest that the father and son work things out.
5. What is Nurse John likely to note in a male client being admitted for alcohol
withdrawal?

A. Perceptual disorders.
B. Impending coma.
C. Recent alcohol intake.
D. Depression with mutism.
6. Aira has taken amitriptyline HCL (Elavil) for 3 days, but now complains that
it “doesn’t help” and refuses to take it. What should the nurse say or do?

A. Withhold the drug.


B. Record the client’s response.
C. Encourage the client to tell the doctor.
D. Suggest that it takes awhile before seeing the results.
7. Dervid, an adolescent has a history of truancy from school, running away
from home and “barrowing” other people’s things without their permission.
The adolescent denies stealing, rationalizing instead that as long as no one
was using the items, it was all right to borrow them. It is important for the
nurse to understand the psychodynamically, this behavior may be largely
attributed to a developmental defect related to the:

A. Id
B. Ego
C. Superego
D. Oedipal complex
8. In preparing a female client for electroconvulsive therapy (ECT), Nurse
Michelle knows that succinylcoline (Anectine) will be administered for which
therapeutic effect?

A. Short-acting anesthesia
B. Decreased oral and respiratory secretions.
C. Skeletal muscle paralysis.
D. Analgesia.
9. Nurse Gina is aware that the dietary implications for a client in manic phase
of bipolar disorder is:

A. Serve the client a bowl of soup, buttered French bread, and apple slices.
B. Increase calories, decrease fat, and decrease protein.
C. Give the client pieces of cut-up steak, carrots, and an apple.
D. Increase calories, carbohydrates, and protein.
10.What parental behavior toward a child during an admission procedure
should cause Nurse Ron to suspect child abuse?

A. Flat affect
B. Expressing guilt
C. Acting overly solicitous toward the child.
D. Ignoring the child.
11.Nurse Lynnette notices that a female client with obsessive-compulsive
disorder washes her hands for long periods each day. How should the nurse
respond to this compulsive behavior?

A. By designating times during which the client can focus on the behavior.


B. By urging the client to reduce the frequency of the behavior as rapidly as
possible.
C. By calling attention to or attempting to prevent the behavior.
D. By discouraging the client from verbalizing anxieties.
12.After seeking help at an outpatient mental health clinic, Ruby who
was raped while walking her dog is diagnosed with posttraumatic
stress disorder (PTSD). Three months later, Ruby returns to the
clinic, complaining of fear, loss of control, and helpless feelings. Which
nursing intervention is most appropriate for Ruby?

A. Recommending a high-protein, low-fat diet.


B. Giving sleep medication, as prescribed, to restore a normal
sleepwake cycle.
C. Allowing the client time to heal.
D. Exploring the meaning of the traumatic event with the client.
13.Meryl, age 19, is highly dependent on her parents and fears leaving
home to go away to college. Shortly before the semester starts, she
complains that her legs are paralyzed and is rushed to the emergency
department. When physical examination rules out a physical cause for her
paralysis, the physician admits her to the psychiatric unit where she is
diagnosed with conversion disorder. Meryl asks the nurse, “Why has this
happened to me?” What is the nurse’s best response?

A. “You’ve developed this paralysis so you can stay with your parents. You
must deal with this conflict if you want to walk again.”
B. “It must be awful not to be able to move your legs. You may feel better if
you realize the problem is psychological, not physical.”
C. “Your problem is real but there is no physical basis for it. We’ll work on
what is going on in your life to find out why it’s happened.”
D. “It isn’t uncommon for someone with your personality to develop
a conversion disorder during times of stress.”
14.Nurse Krina knows that the following drugs have been known to
be effective in treating obsessive-compulsive disorder (OCD):

A. benztropine (Cogentin) and diphenhydramine (Benadryl).


B. chlordiazepoxide (Librium) and diazepam (Valium)
C. fluvoxamine (Luvox) and clomipramine (Anafranil)
D. divalproex (Depakote) and lithium (Lithobid)
15.Alfred was newly diagnosed with anxiety disorder. The
physician prescribed buspirone (BuSpar). The nurse is aware that the
teaching instructions for newly prescribed buspirone should include which of
the following?

A. A warning about the drugs delayed therapeutic effect, which is from 14


to 30 days.
B. A warning about the incidence of neuroleptic malignant
syndrome (NMS).
C. A reminder of the need to schedule blood work in 1 week to check blood
levels of the drug.
D. A warning that immediate sedation can occur with a resultant drop in
pulse.
16.Richard with agoraphobia has been symptom-free for 4 months.
Classic signs and symptoms of phobias include:

A. Insomnia and an inability to concentrate.


B. Severe anxiety and fear.
C. Depression and weight loss.
D. Withdrawal and failure to distinguish reality from fantasy.
17.Which medications have been found to help reduce or eliminate
panic attacks?

A. Antidepressants
B. Anticholinergics
C. Antipsychotics
D. Mood stabilizers
18.A client seeks care because she feels depressed and has gained
weight. To treat her atypical depression, the physician prescribes
tranylcypromine sulfate (Parnate), 10 mg by mouth twice per day. When this
drug is used to treat atypical depression, what is its onset of action?

A. 1 to 2 days
B. 3 to 5 days
C. 6 to 8 days
D. 10 to 14 days
19. A 65 years old client is in the first stage of Alzheimer’s disease.
Nurse Patricia should plan to focus this client’s care on:

A. Offering nourishing finger foods to help maintain the client’s nutritional


status.
B. Providing emotional support and individual counseling.
C. Monitoring the client to prevent minor illnesses from turning into major
problems.
D. Suggesting new activities for the client and family to do together.
20.The nurse is assessing a client who has just been admitted to
the emergency department. Which signs would suggest an overdose of
an antianxiety agent?

A. Combativeness, sweating, and confusion


B. Agitation, hyperactivity, and grandiose ideation
C. Emotional lability, euphoria, and impaired memory
D. Suspiciousness, dilated pupils, and increased blood pressure
21.The nurse is caring for a client diagnosed with antisocial
personality disorder. The client has a history of fighting, cruelty to animals,
and stealing. Which of the following traits would the nurse be most likely
to uncover during assessment?

A. History of gainful employment


B. Frequent expression of guilt regarding antisocial behavior
C. Demonstrated ability to maintain close, stable relationships
D. d. A low tolerance for frustration
22.Nurse Amy is providing care for a male client undergoing
opiate withdrawal. Opiate withdrawal causes severe physical discomfort and
can be life-threatening. To minimize these effects, opiate users are
commonly detoxified with:

A. Barbiturates
B. Amphetamines
C. Methadone
D. Benzodiazepines
23.Nurse Cristina is caring for a client who experiences false
sensory perceptions with no basis in reality. These perceptions are known as:
A. Delusions
B. Hallucinations
C. Loose associations
D. Neologisms
24. Nurse Marco is developing a plan of care for a client with
anorexia nervosa. Which action should the nurse include in the plan?

A. Restricts visits with the family and friends until the client begins to eat.
B. Provide privacy during meals.
C. Set up a strict eating plan for the client.
D. Encourage the client to exercise, which will reduce her anxiety.
25.Tim is admitted with a diagnosis of delusions of grandeur. The nurse
is aware that this diagnosis reflects a belief that one is:

A. Highly important or famous.


B. Being persecuted
C. Connected to events unrelated to oneself
D. Responsible for the evil in the world.
26.Nurse Jen is caring for a male client with manic depression. The plan
of care for a client in a manic state would include:

A. Offering a high-calorie meals and strongly encouraging the client


to finish all food.
B. Insisting that the client remain active through the day so that he’ll sleep
at night.
C. Allowing the client to exhibit hyperactive, demanding,
manipulative behavior without setting limits.
D. Listening attentively with a neutral attitude and avoiding
power struggles.
27.Ramon is admitted for detoxification after a cocaine overdose. The
client tells the nurse that he frequently uses cocaine but that he can control
his use if he chooses. Which coping mechanism is he using?

A. Withdrawal
B. Logical thinking
C. Repression
D. Denial
28.Richard is admitted with a diagnosis of schizotypal personality
disorder. Which signs would this client exhibit during social situations?

A. Aggressive behavior
B. Paranoid thoughts
C. Emotional affect
D. Independence needs
29. Nurse Mickey is caring for a client diagnosed with bulimia. The
most appropriate initial goal for a client diagnosed with bulimia is to:

A. Avoid shopping for large amounts of food.


B. Control eating impulses.
C. Identify anxiety-causing situations
D. Eat only three meals per day.
30.Rudolf is admitted for an overdose of amphetamines. When assessing
the client, the nurse should expect to see:

A. Tension and irritability


B. Slow pulse
C. Hypotension
D. Constipation
31.Nicolas is experiencing hallucinations tells the nurse, “The voices
are telling me I’m no good.” The client asks if the nurse hears the voices.
The most appropriate response by the nurse would be:

A. “It is the voice of your conscience, which only you can control.”
B. “No, I do not hear your voices, but I believe you can hear them”.
C. “The voices are coming from within you and only you can hear them.”
D. “Oh, the voices are a symptom of your illness; don’t pay any attention to
them.”
32.The nurse is aware that the side effect of electroconvulsive therapy that
a client may experience:

A. Loss of appetite
B. Postural hypotension
C. Confusion for a time after treatment
D. Complete loss of memory for a time
33.A dying male client gradually moves toward resolution of
feelings regarding impending death. Basing care on the theory of Kubler-
Ross, Nurse Trish plans to use nonverbal interventions when
assessment reveals that the client is in the:

A. Anger stage
B. Denial stage
C. Bargaining stage
D. Acceptance stage
34.The outcome that is unrelated to a crisis state is:

A. Learning more constructive coping skills


B. Decompensation to a lower level of functioning.
C. Adaptation and a return to a prior level of functioning.
D. A higher level of anxiety continuing for more than 3 months.
35.Miranda a psychiatric client is to be discharged with orders for
haloperidol (haldol) therapy. When developing a teaching plan for discharge,
the nurse should include cautioning the client against:

A. Driving at night
B. Staying in the sun
C. Ingesting wines and cheeses
D. Taking medications containing aspirin
36.Jen a nursing student is anxious about the upcoming board
examination but is able to study intently and does not become distracted by
a roommate’s talking and loud music. The student’s ability to
ignore distractions and to focus on studying demonstrates:

A. Mild-level anxiety
B. Panic-level anxiety
C. Severe-level anxiety
D. Moderate-level anxiety
37.When assessing a premorbid personality characteristics of a client with
a major depression, it would be unusual for the nurse to find that this
client demonstrated:

A. Rigidity
B. Stubbornness
C. Diverse interest
D. Over meticulousness
38.Nurse Krina recognizes that the suicidal risk for depressed client
is greatest:

A. As their depression begins to improve


B. When their depression is most severe
C. Before nay type of treatment is started
D. As they lose interest in the environment
39.Nurse Kate would expect that a client with vascular dementis
would experience:

A. Loss of remote memory related to anoxia


B. Loss of abstract thinking related to emotional state
C. Inability to concentrate related to decreased stimuli
D. Disturbance in recalling recent events related to cerebral hypoxia.
40.Josefina is to be discharged on a regimen of lithium carbonate. In
the teaching plan for discharge the nurse should include:

A. Advising the client to watch the diet carefully


B. Suggesting that the client take the pills with milk
C. Reminding the client that a CBC must be done once a month.
D. Encouraging the client to have blood levels checked as ordered.
41.The psychiatrist orders lithium carbonate 600 mg p.o t.i.d for a
female client. Nurse Katrina would be aware that the teaching about the
side effects of this drug were understood when the client state, “I will call
my doctor immediately if I notice any:

A. Sensitivity to bright light or sun


B. Fine hand tremors or slurred speech
C. Sexual dysfunction or breast enlargement
D. d. Inability to urinate or difficulty when urinating
42.Nurse Mylene recognizes that the most important factor necessary for
the establishment of trust in a critical care area is:

A. Privacy
B. Respect
C. Empathy
D. Presence
43.When establishing an initial nurse-client relationship, Nurse Hazel
should explore with the client the:

A. Client’s perception of the presenting problem.


B. Occurrence of fantasies the client may experience.
C. Details of any ritualistic acts carried out by the client
D. Client’s feelings when external; controls are instituted.
44.Tranylcypromine sulfate (Parnate) is prescribed for a depressed client
who has not responded to the tricyclic antidepressants. After teaching the
client about the medication, Nurse Marian evaluates that learning has
occurred when the client states, “I will avoid:

A. Citrus fruit, tuna, and yellow vegetables.”


B. Chocolate milk, aged cheese, and yogurt’”
C. Green leafy vegetables, chicken, and milk.”
D. Whole grains, red meats, and carbonated soda.”
45.Nurse John is a aware that most crisis situations should resolve in about:

A. 1 to 2 weeks
B. 4 to 6 weeks
C. 4 to 6 months
D. 6 to 12 months
46. Nurse Judy knows that statistics show that in adolescent
suicide behavior:

A. Females use more dramatic methods than males


B. Males account for more attempts than do females
C. Females talk more about suicide before attempting it
D. Males are more likely to use lethal methods than are females
47. Dervid with paranoid schizophrenia repeatedly uses profanity during
an activity therapy session. Which response by the nurse would be
most appropriate?

A. “Your behavior won’t be tolerated. Go to your room immediately.”


B. “You’re just doing this to get back at me for making you come
to therapy.”
C. “Your cursing is interrupting the activity. Take time out in your room for
10 minutes.”
D. “I’m disappointed in you. You can’t control yourself even for a
few minutes.”
48.Nurse Maureen knows that the nonantipsychotic medication used to
treat some clients with schizoaffective disorder is:

A. phenelzine (Nardil)
B. chlordiazepoxide (Librium)
C. lithium carbonate (Lithane)
D. imipramine (Tofranil)
49.Which information is most important for the nurse Trinity to include in
a teaching plan for a male schizophrenic client taking clozapine (Clozaril)?

A. Monthly blood tests will be necessary.


B. Report a sore throat or fever to the physician immediately.
C. Blood pressure must be monitored for hypertension.
D. Stop the medication when symptoms subside.
50.Ricky with chronic schizophrenia takes neuroleptic medication is
admitted to the psychiatric unit. Nursing assessment reveals rigidity,
fever, hypertension, and diaphoresis. These findings suggest which
lifethreatening reaction:

A. Tardive dyskinesia.
B. Dystonia.
C. Neuroleptic malignant syndrome.
D. Akathisia.
51.Which nursing intervention would be most appropriate if a male
client develop orthostatic hypotension while taking amitriptyline (Elavil)?

A. Consulting with the physician about substituting a different type


of antidepressant.
B. Advising the client to sit up for 1 minute before getting out of bed.
C. Instructing the client to double the dosage until the problem resolves.
D. Informing the client that this adverse reaction should disappear within 1
week.
52.Mr. Cruz visits the physician’s office to seek treatment for
depression, feelings of hopelessness, poor appetite, insomnia, fatigue, low
selfesteem, poor concentration, and difficulty making decisions. The
client states that these symptoms began at least 2 years ago. Based on
this report, the nurse Tyfany suspects:

A. Cyclothymic disorder.
B. Atypical affective disorder.
C. Major depression.
D. Dysthymic disorder.
53. After taking an overdose of phenobarbital (Barbita), Mario is admitted
to the emergency department. Dr. Trinidad prescribes activated
charcoal (Charcocaps) to be administered by mouth immediately.
Before administering the dose, the nurse verifies the dosage ordered. What is
the usual minimum dose of activated charcoal?

A. 5 g mixed in 250 ml of water


B. 15 g mixed in 500 ml of water
C. 30 g mixed in 250 ml of water
D. 60 g mixed in 500 ml of water
54.What herbal medication for depression, widely used in Europe, is
now being prescribed in the United States?

A. Ginkgo biloba
B. Echinacea
C. St. John’s wort
D. Ephedra
55.Cely with manic episodes is taking lithium. Which electrolyte level
should the nurse check before administering this medication?

A. Calcium
B. Sodium
C. Chloride
D. Potassium
56.Nurse Josefina is caring for a client who has been diagnosed with delirium.
Which statement about delirium is true?

A. It’s characterized by an acute onset and lasts about 1 month.


B. It’s characterized by a slowly evolving onset and lasts about 1 week.
C. It’s characterized by a slowly evolving onset and lasts about 1 month.
D. It’s characterized by an acute onset and lasts hours to a number
of days.
57.Edward, a 66 year old client with slight memory impairment and
poor concentration is diagnosed with primary degenerative dementia of
the Alzheimer’s type. Early signs of this dementia include subtle
personality changes and withdrawal from social interactions. To assess
for progression to the middle stage of Alzheimer’s disease, the nurse
should observe the client for:

A. Occasional irritable outbursts.


B. Impaired communication.
C. Lack of spontaneity.
D. Inability to perform self-care activities.
58.Isabel with a diagnosis of depression is started on imipramine
(Tofranil), 75 mg by mouth at bedtime. The nurse should tell the client that:

A. This medication may be habit forming and will be discontinued as soon


as the client feels better.
B. This medication has no serious adverse effects.
C. The client should avoid eating such foods as aged cheeses, yogurt, and
chicken livers while taking the medication.
D. This medication may initially cause tiredness, which should
become less bothersome over time.
59.Kathleen is admitted to the psychiatric clinic for treatment of
anorexia nervosa. To promote the client’s physical health, the nurse should
plan to:

A. Severely restrict the client’s physical activities.


B. Weigh the client daily, after the evening meal.
C. Monitor vital signs, serum electrolyte levels, and acid-base balance.
D. Instruct the client to keep an accurate record of food and fluid intake.
60.Celia with a history of polysubstance abuse is admitted to the facility.
She complains of nausea and vomiting 24 hours after admission. The
nurse assesses the client and notes piloerection, pupillary dilation,
and lacrimation. The nurse suspects that the client is going through which
of the following withdrawals?

A. Alcohol withdrawal
B. Cannibis withdrawal
C. Cocaine withdrawal
D. Opioid withdrawal
61.Mr. Garcia, an attorney who throws books and furniture around the
office after losing a case is referred to the psychiatric nurse in the law
firm’s employee assistance program. Nurse Beatriz knows that the
client’s behavior most likely represents the use of which defense mechanism?

A. Regression
B. Projection
C. Reaction-formation
D. Intellectualization
62.Nurse Anne is caring for a client who has been treated long term
with antipsychotic medication. During the assessment, Nurse Anne checks
the client for tardive dyskinesia. If tardive dyskinesia is present, Nurse
Anne would most likely observe:

A. Abnormal movements and involuntary movements of the


mouth, tongue, and face.
B. Abnormal breathing through the nostrils accompanied by a “thrill.”
C. Severe headache, flushing, tremors, and ataxia.
D. Severe hypertension, migraine headache,
63.Dennis has a lithium level of 2.4 mEq/L. The nurse immediately
would assess the client for which of the following signs or symptoms?

A. Weakness
B. Diarrhea
C. Blurred vision
D. Fecal incontinence
64.Nurse Jannah is monitoring a male client who has been placed
inrestraints because of violent behavior. Nurse determines that it will be safe
to remove the restraints when:

A. The client verbalizes the reasons for the violent behavior.


B. The client apologizes and tells the nurse that it will never happen again.
C. No acts of aggression have been observed within 1 hour after
the release of two of the extremity restraints.
D. The administered medication has taken effect.
65.Nurse Irish is aware that Ritalin is the drug of choice for a child with ADHD.
The side effects of the following may be noted by the nurse:

A. Increased attention span and concentration


B. Increase in appetite
C. Sleepiness and lethargy
D. Bradycardia and diarrhea
66.Kitty, a 9 year old child has very limited vocabulary and interaction
skills. She has an I.Q. of 45. She is diagnosed to have Mental retardation of
this classification:

A. Profound
B. Mild
C. Moderate
D. Severe
67.The therapeutic approach in the care of Armand an autistic child
include the following EXCEPT:

A. Engage in diversionary activities when acting -out


B. Provide an atmosphere of acceptance
C. Provide safety measures
D. Rearrange the environment to activate the child
68.Jeremy is brought to the emergency room by friends who state that
he took something an hour ago. He is actively hallucinating, agitated,
with irritated nasal septum.

A. Heroin
B. Cocaine
C. LSD
D. Marijuana
69.Nurse Pauline is aware that Dementia unlike delirium is characterized by:

A. Slurred speech
B. Insidious onset
C. Clouding of consciousness
D. Sensory perceptual change
70.A 35 year old female has intense fear of riding an elevator. She claims “ As
if I will die inside.” The client is suffering from:

A. Agoraphobia
B. Social phobia
C. Claustrophobia
D. Xenophobia
71.Nurse Myrna develops a counter-transference reaction. This is
evidenced by:

A. Revealing personal information to the client


B. Focusing on the feelings of the client.
C. Confronting the client about discrepancies in verbal or non-
verbal behavior
D. The client feels angry towards the nurse who resembles his mother.
72.Tristan is on Lithium has suffered from diarrhea and vomiting. What should
the nurse in-charge do first:

A. Recognize this as a drug interaction


B. Give the client Cogentin
C. Reassure the client that these are common side effects of
lithium therapy
D. Hold the next dose and obtain an order for a stat serum lithium level
73.Nurse Sarah ensures a therapeutic environment for all the client. Which
of the following best describes a therapeutic milieu?

A. A therapy that rewards adaptive behavior


B. A cognitive approach to change behavior
C. A living, learning or working environment.
D. A permissive and congenial environment
74.Anthony is very hostile toward one of the staff for no apparent reason.
He is manifesting:

A. Splitting
B. Transference
C. Countertransference
D. Resistance
75.Marielle, 17 years old was sexually attacked while on her way home
from school. She is brought to the hospital by her mother. Rape is an
example of which type of crisis:

A. Situational
B. Adventitious
C. Developmental
D. Internal
76. Nurse Greta is aware that the following is classified as an Axis I
disorder by the Diagnosis and Statistical Manual of Mental Disorders,
Text Revision (DSM-IV-TR) is:

A. Obesity
B. Borderline personality disorder
C. Major depression
D. Hypertension
77.Katrina, a newly admitted is extremely hostile toward a staff member
she has just met, without apparent reason. According to Freudian theory,
the nurse should suspect that the client is experiencing which of the
following phenomena?

A. Intellectualization
B. Transference
C. Triangulation
D. Splitting
78.An 83year-old male client is in extended care facility is anxious most of
the time and frequently complains of a number of vague symptoms
that interfere with his ability to eat. These symptoms indicate which of
the following disorders?
A. Conversion disorder
B. Hypochondriasis
C. Severe anxiety
D. Sublimation
79. Charina, a college student who frequently visited the health center during
the past year with multiple vague complaints of GI symptoms before
course examinations. Although physical causes have been eliminated, the
student continues to express her belief that she has a serious illness. These
symptoms are typically of which of the following disorders?

A. Conversion disorder
B. Depersonalization
C. Hypochondriasis
D. Somatization disorder
80. Nurse Daisy is aware that the following pharmacologic agents
are sedative hypnotic medication is used to induce sleep for a client
experiencing a sleep disorder is:

A. Triazolam (Halcion)
B. Paroxetine (Paxil)\
C. Fluoxetine (Prozac)
D. Risperidone (Risperdal)
81. Aldo, with a somatoform pain disorder may obtain secondary gain. Which
of the following statement refers to a secondary gain?

A. It brings some stability to the family


B. It decreases the preoccupation with the physical illness
C. It enables the client to avoid some unpleasant activity
D. It promotes emotional support or attention for the client
82. Dervid is diagnosed with panic disorder with agoraphobia is talking with
the nurse in-charge about the progress made in treatment. Which of the
following statements indicates a positive client response?

A. “I went to the mall with my friends last Saturday”


B. “I’m hyperventilating only when I have a panic attack”
C. “Today I decided that I can stop taking my medication”
D. “Last night I decided to eat more than a bowl of cereal”
83. The effectiveness of monoamine oxidase (MAO) inhibitor drug therapy in
client with posttraumatic stress disorder can be demonstrated by which of
the following client self –reports?

A. “I’m sleeping better and don’t have nightmares”


B. “I’m not losing my temper as much”
C. “I’ve lost my craving for alcohol”
D. “I’ve lost my phobia for water”
84. Mark, with a diagnosis of generalized anxiety disorder wants to stop
taking his lorazepam (Ativan). Which of the following important facts should
nurse Betty discuss with the client about discontinuing the medication?

A. Stopping the drug may cause depression


B. Stopping the drug increases cognitive abilities
C. Stopping the drug decreases sleeping difficulties
D. Stopping the drug can cause withdrawal symptoms
85. Jennifer, an adolescent who is depressed and reported by his parents
as having difficulty in school is brought to the community mental health
center to be evaluated. Which of the following other health problems would
the nurse suspect?

A. Anxiety disorder
B. Behavioral difficulties
C. Cognitive impairment
D. Labile moods
86. Ricardo, an outpatient in psychiatric facility is diagnosed with
dysthymic disorder. Which of the following statement about dysthymic
disorder is true?

A. It involves a mood range from moderate depression to hypomania


B. It involves a single manic depression
C. It’s a form of depression that occurs in the fall and winter
D. It’s a mood disorder similar to major depression but of mild to moderate
severity
87. The nurse is aware that the following ways in vascular dementia
different from Alzheimer’s disease is:
A. Vascular dementia has more abrupt onset
B. The duration of vascular dementia is usually brief
C. Personality change is common in vascular dementia
D. The inability to perform motor activities occurs in vascular dementia
88. Loretta, a newly admitted client was diagnosed with delirium and has
history of hypertension and anxiety. She had been taking digoxin, furosemide
(Lasix), and diazepam (Valium) for anxiety. This client’s impairment may be
related to which of the following conditions?

A. Infection
B. Metabolic acidosis
C. Drug intoxication
D. Hepatic encephalopathy
89. Nurse Ron enters a client’s room, the client says, “They’re crawling on
my sheets! Get them off my bed!” Which of the following assessment is the
most accurate?

A. The client is experiencing aphasia


B. The client is experiencing dysarthria
C. The client is experiencing a flight of ideas
D. The client is experiencing visual hallucination
90. Which of the following descriptions of a client’s experience and behavior
can be assessed as an illusion?

A. The client tries to hit the nurse when vital signs must be taken
B. The client says, “I keep hearing a voice telling me to run away”
C. The client becomes anxious whenever the nurse leaves the bedside
D. The client looks at the shadow on a wall and tells the nurse she sees
frightening faces on the wall.
91. During conversation of Nurse John with a client, he observes that the
client shift from one topic to the next on a regular basis. Which of the
following terms describes this disorder?

A. Flight of ideas
B. Concrete thinking
C. Ideas of reference
D. Loose association
92. Francis tells the nurse that her coworkers are sabotaging the
computer. When the nurse asks questions, the client becomes argumentative.
This behavior shows personality traits associated with which of the
following personality disorder?

A. Antisocial
B. Histrionic
C. Paranoid
D. Schizotypal
93. Which of the following interventions is important for a Cely experiencing
with paranoid personality disorder taking olanzapine (Zyprexa)?

A. Explain effects of serotonin syndrome


B. Teach the client to watch for extrapyramidal adverse reaction
C. Explain that the drug is less affective if the client smokes
D. Discuss the need to report paradoxical effects such as euphoria
94. Nurse Alexandra notices other clients on the unit avoiding a client
diagnosed with antisocial personality disorder. When discussing appropriate
behavior in group therapy, which of the following comments is expected about
this client by his peers?

A. Lack of honesty
B. Belief in superstition
C. Show of temper tantrums
D. Constant need for attention
95. Tommy, with dependent personality disorder is working to increase his
selfesteem. Which of the following statements by the Tommy shows teaching
was successful?

A. “I’m not going to look just at the negative things about myself”
B. “I’m most concerned about my level of competence and progress”
C. “I’m not as envious of the things other people have as I used to be”
D. “I find I can’t stop myself from taking over things other should be doing”
96. Norma, a 42-year-old client with a diagnosis of chronic
undifferentiated schizophrenia lives in a rooming house that has a weekly
nursing clinic. She scratches while she tells the nurse she feels creatures
eating away at her skin. Which of the following interventions should be done
first?

A. Talk about his hallucinations and fears


B. Refer him for anticholinergic adverse reactions
C. Assess for possible physical problems such as rash
D. Call his physician to get his medication increased to control
his psychosis
97. Ivy, who is on the psychiatric unit is copying and imitating the movements
of her primary nurse. During recovery, she says, “I thought the nurse was
my mirror. I felt connected only when I saw my nurse.” This behavior is known
by which of the following terms?

A. Modeling
B. Echopraxia
C. Ego-syntonicity
D. Ritualism
98. Jun approaches the nurse and tells that he hears a voice telling him that
he’s evil and deserves to die. Which of the following terms describes the
client’s perception?

A. Delusion
B. Disorganized speech
C. Hallucination
D. Idea of reference
99. Mike is admitted to a psychiatric unit with a diagnosis of
undifferentiated schizophrenia. Which of the following defense mechanisms
is probably used by mike?

A. Projection
B. Rationalization
C. Regression
D. Repression
100. Rocky has started taking haloperidol (Haldol). Which of the
following instructions is most appropriate for Ricky before taking haloperidol?

A. Should report feelings of restlessness or agitation at once


B. Use a sunscreen outdoors on a year-round basis
C. Be aware you’ll feel increased energy taking this drug
D. This drug will indirectly control essential hypertension
Answers and Rationales
1. Answer: (D) Focusing. The nurse is using focusing by suggesting that the
client discuss a specific issue. The nurse didn’t restate the question,
make observation, or ask further question (exploring).
2. Answer: (D) Remove all other clients from the dayroom. The nurse’s first
priority is to consider the safety of the clients in the therapeutic setting.
The other actions are appropriate responses after ensuring the safety of
other clients.
3. Answer: (A) The client is disruptive. Group activity provides too much
stimulation, which the client will not be able to handle (harmful to self) and
as a result will be disruptive to others.
4. Answer: (C) Agree to talk with the mother and the father together. By agreeing
to talk with both parents, the nurse can provide emotional support and
further assess and validate the family’s needs.
5. Answer: (A) Perceptual disorders. Frightening visual hallucinations are
especially common in clients experiencing alcohol withdrawal.
6. Answer: (D) Suggest that it takes awhile before seeing the results. The client
needs a specific response; that it takes 2 to 3 weeks (a delayed effect)
until the therapeutic blood level is reached.
7. Answer: (C) Superego.  This behavior shows a weak sense of moral
consciousness. According to Freudian theory, personality disorders stem
from a weak superego.
8. Answer: (C) Skeletal muscle paralysis. Anectine is a depolarizing muscle
relaxant causing paralysis. It is used to reduce the intensity of muscle
contractions during the convulsive stage, thereby reducing the risk of bone
fractures or dislocation.
9. Answer: (D) Increase calories, carbohydrates, and protein.This client
increased protein for tissue building and increased calories to replace
what is burned up (usually via carbohydrates).
10. Answer: (C) Acting overly solicitous toward the child. This behavior is an
example of reaction formation, a coping mechanism.
11. Answer: (A) By designating times during which the client can focus on
the behavior. The nurse should designate times during which the client
can focus on the compulsive behavior or obsessive thoughts. The
nurse should urge the client to reduce the frequency of the compulsive
behavior gradually, not rapidly. She shouldn’t call attention to or try to
prevent the behavior. Trying to prevent the behavior may cause pain and
terror in the client. The nurse should encourage the client to verbalize
anxieties to help distract attention from the compulsive behavior.
12. Answer: (D) Exploring the meaning of the traumatic event with the client. The
client with PTSD needs encouragement to examine and understand the
meaning of the traumatic event and consequent losses. Otherwise,
symptoms may worsen and the client may become depressed or engage
in self-destructive behavior such as substance abuse. The client must
explore the meaning of the event and won’t heal without this, no matter
how much time passes. Behavioral techniques, such as relaxation therapy,
may help decrease the client’s anxiety and induce sleep. The physician
may prescribe antianxiety agents or antidepressants cautiously to avoid
dependence; sleep medication is rarely appropriate. A special diet isn’t
indicated unless the client also has an eating disorder or a nutritional
problem.
13. Answer: (C) “Your problem is real but there is no physical basis for it. We’ll
work on what is going on in your life to find out why it’s happened.” The nurse
must be honest with the client by telling her that the paralysis has no
physiologic cause while also conveying empathy and acknowledging that
her symptoms are real. The client will benefit from psychiatric treatment,
which will help her understand the underlying cause of her symptoms.
After the psychological conflict is resolved, her symptoms will disappear.
Saying that it must be awful not to be able to move her legs wouldn’t
answer the client’s question; knowing that the cause is psychological
wouldn’t necessarily make her feel better. Telling her that she has
developed paralysis to avoid leaving her parents or that her personality
caused her disorder wouldn’t help her understand and resolve the
underlying conflict.
14. Answer: (C) fluvoxamine (Luvox) and clomipramine (Anafranil). The
antidepressants fluvoxamine and clomipramine have been effective in the
treatment of OCD. Librium and Valium may be helpful in treating anxiety
related to OCD but aren’t drugs of choice to treat the illness. The other
medications mentioned aren’t effective in the treatment of OCD.
15. Answer: (A) A warning about the drugs delayed therapeutic effect, which is
from 14 to 30 days. The client should be informed that the drug’s
therapeutic effect might not be reached for 14 to 30 days. The client must
be instructed to continue taking the drug as directed. Blood level checks
aren’t necessary. NMS hasn’t been reported with this drug, but tachycardia
is frequently reported.
16. Answer: (B) Severe anxiety and fear. Phobias cause severe anxiety (such
as a panic attack) that is out of proportion to the threat of the feared
object or situation. Physical signs and symptoms of phobias include
profuse sweating, poor motor control, tachycardia, and elevated blood
pressure. Insomnia, an inability to concentrate, and weight loss are
common in depression. Withdrawal and failure to distinguish reality from
fantasy occur in schizophrenia.
17. Answer: (A) Antidepressants. Tricyclic and monoamine oxidase (MAO)
inhibitor antidepressants have been found to be effective in treating
clients with panic attacks. Why these drugs help control panic attacks isn’t
clearly understood. Anticholinergic agents, which are smooth-muscle
relaxants, relieve physical symptoms of anxiety but don’t relieve the
anxiety itself. Antipsychotic drugs are inappropriate because clients who
experience panic attacks aren’t psychotic. Mood stabilizers aren’t
indicated because panic attacks are rarely associated with mood
changes.
18. Answer: (B) 3 to 5 days.  Monoamine oxidase inhibitors, such as
tranylcypromine, have an onset of action of approximately 3 to 5 days. A
full clinical response may be delayed for 3 to 4 weeks. The therapeutic
effects may continue for 1 to 2 weeks after discontinuation.
19. Answer: (B) Providing emotional support and individual counseling. Clients in
the first stage of Alzheimer’s disease are aware that something is
happening to them and may become overwhelmed and frightened.
Therefore, nursing care typically focuses on providing emotional support
and individual counseling. The other options are appropriate during the
second stage of Alzheimer’s disease, when the client needs continuous
monitoring to prevent minor illnesses from progressing into major
problems and when maintaining adequate nutrition may become a
challenge. During this stage, offering nourishing finger foods helps clients
to feed themselves and maintain adequate nutrition.
20. Answer: (C) Emotional lability, euphoria, and impaired memory. Signs of
antianxiety agent overdose include emotional lability, euphoria, and
impaired memory. Phencyclidine overdose can cause combativeness,
sweating, and confusion. Amphetamine overdose can result in agitation,
hyperactivity, and grandiose ideation. Hallucinogen overdose can produce
suspiciousness, dilated pupils, and increased blood pressure.
21. Answer: (D) A low tolerance for frustration. Clients with an antisocial
personality disorder exhibit a low tolerance for frustration, emotional
immaturity, and a lack of impulse control. They commonly have a history
of unemployment, miss work repeatedly, and quit work without other plans
for employment. They don’t feel guilt about their behavior and commonly
perceive themselves as victims. They also display a lack of responsibility
for the outcome of their actions. Because of a lack of trust in others,
clients with antisocial personality disorder commonly have difficulty
developing stable, close relationships.
22. Answer: (C) Methadone. Methadone is used to detoxify opiate users
because it binds with opioid receptors at many sites in the central nervous
system but doesn’t have the same deterious effects as other opiates, such
as cocaine, heroin, and morphine. Barbiturates, amphetamines,
and benzodiazepines are highly addictive and would require
detoxification treatment.
23. Answer: (B) Hallucinations. Hallucinations are visual, auditory, gustatory,
tactile, or olfactory perceptions that have no basis in reality. Delusions are
false beliefs, rather than perceptions, that the client accepts as real.
Loose associations are rapid shifts among unrelated ideas. Neologisms
are bizarre words that have meaning only to the client.
24. Answer: (C) Set up a strict eating plan for the client. Establishing a
consistent eating plan and monitoring the client’s weight are very
important in this disorder. The family and friends should be included in the
client’s care. The client should be monitored during meals-not given
privacy. Exercise must be limited and supervised.
25. Answer: (A) Highly important or famous. A delusion of grandeur is a false
belief that one is highly important or famous. A delusion of persecution is
a false belief that one is being persecuted. A delusion of reference is a
false belief that one is connected to events unrelated to oneself or a belief
that one is responsible for the evil in the world.
26. Answer: (D) Listening attentively with a neutral attitude and avoiding power
struggles. The nurse should listen to the client’s requests,
express willingness to seriously consider the request, and respond later.
The nurse should encourage the client to take short daytime naps
because he expends so much energy. The nurse shouldn’t try to restrain
the client when he feels the need to move around as long as his activity
isn’t harmful. High calorie finger foods should be offered to supplement
the client’s diet, if he can’t remain seated long enough to eat a complete
meal. The nurse shouldn’t be forced to stay seated at the table to finish
a meal. The nurse should set limits in a calm, clear, and self-confident
tone of voice.
27. Answer: (D) Denial. Denial is unconscious defense mechanism in which
emotional conflict and anxiety is avoided by refusing to acknowledge
feelings, desires, impulses, or external facts that are consciously
intolerable. Withdrawal is a common response to stress, characterized by
apathy. Logical thinking is the ability to think rationally and make
responsible decisions, which would lead the client admitting the problem
and seeking help. Repression is suppressing past events from the
consciousness because of guilty association.
28. Answer: (B) Paranoid thoughts. Clients with schizotypal personality
disorder experience excessive social anxiety that can lead to paranoid
thoughts. Aggressive behavior is uncommon, although these clients may
experience agitation with anxiety. Their behavior is emotionally cold with a
flattened affect, regardless of the situation. These clients demonstrate a
reduced capacity for close or dependent relationships.
29. Answer: (C) Identify anxiety-causing situations. Bulimic behavior is
generally a maladaptive coping response to stress and underlying issues.
The client must identify anxiety-causing situations that stimulate the
bulimic behavior and then learn new ways of coping with the anxiety.
30. Answer: (A) Tension and irritability. An amphetamine is a nervous system
stimulant that is subject to abuse because of its ability to produce
wakefulness and euphoria. An overdose increases tension and irritability.
Options B and C are incorrect because amphetamines stimulate
norepinephrine, which increase the heart rate and blood flow. Diarrhea is a
common adverse effect so option D in is incorrect.
31. Answer: (B) “No, I do not hear your voices, but I believe you can
hear them”. The nurse, demonstrating knowledge and
understanding, accepts the client’s perceptions even though they are
hallucinatory.
32. Answer: (C) Confusion for a time after treatment. The electrical energy
passing through the cerebral cortex during ECT results in a temporary
state of confusion after treatment.
33. Answer: (D) Acceptance stage. Communication and intervention during
this stage are mainly nonverbal, as when the client gestures to hold the
nurse’s hand.
34. Answer: (D) A higher level of anxiety continuing for more than 3 months. This
is not an expected outcome of a crisis because by definition a crisis would
be resolved in 6 weeks.
35. Answer: (B) Staying in the sun. Haldol causes photosensitivity. Severe
sunburn can occur on exposure to the sun.
36. Answer: (D) Moderate-level anxiety. A moderately anxious person can
ignore peripheral events and focuses on central concerns.
37. Answer: (C) Diverse interest. Before onset of depression, these clients
usually have very narrow, limited interest.
38. Answer: (A) As their depression begins to improve. At this point the client
may have enough energy to plan and execute an attempt.
39. Answer: (D) Disturbance in recalling recent events related to
cerebral hypoxia. Cell damage seems to interfere with registering input
stimuli, which affects the ability to register and recall recent events;
vascular dementia is related to multiple vascular lesions of the cerebral
cortex and subcortical structure.
40. Answer: (D) Encouraging the client to have blood levels checked
as ordered. Blood levels must be checked monthly or bimonthly when
the client is on maintenance therapy because there is only a small
range between therapeutic and toxic levels.
41. Answer: (B) Fine hand tremors or slurred speech. These are common side
effects of lithium carbonate.
42. Answer: (D) Presence. The constant presence of a nurse provides
emotional support because the client knows that someone is attentive
and available in case of an emergency.
43. Answer: (A) Client’s perception of the presenting problem. The nurse can be
most therapeutic by starting where the client is, because it is the client’s
concept of the problem that serves as the starting point of the
relationship.
44. Answer: (B) Chocolate milk, aged cheese, and yogurt’. These high-tyramine
foods, when ingested in the presence of an MAO inhibitor, cause a severe
hypertensive response.
45. Answer: (B) 4 to 6 weeks. Crisis is self-limiting and lasts from 4 to 6
weeks.
46. Answer: (D) Males are more likely to use lethal methods than are
females.  This finding is supported by research; females account for 90% of
suicide attempts but males are three times more successful because
of methods used.
47. Answer: (C) “Your cursing is interrupting the activity. Take time out in
your room for 10 minutes.”  The nurse should set limits on client behavior to
ensure a comfortable environment for all clients. The nurse should accept
hostile or quarrelsome client outbursts within limits without becoming
personally offended, as in option A. Option B is incorrect because it
implies that the client’s actions reflect feelings toward the staff instead of
the client’s own misery. Judgmental remarks, such as option D, may
decrease the client’s self-esteem.
48. Answer: (C) lithium carbonate (Lithane). Lithium carbonate, an antimania
drug, is used to treat clients with cyclical schizoaffective disorder, a
psychotic disorder once classified under schizophrenia that causes
affective symptoms, including maniclike activity. Lithium helps control the
affective component of this disorder. Phenelzine is a monoamine oxidase
inhibitor prescribed for clients who don’t respond to other antidepressant
drugs such as imipramine. Chlordiazepoxide, an antianxiety agent,
generally is contraindicated in psychotic clients. Imipramine, primarily
considered an antidepressant agent, is also used to treat clients with
agoraphobia and that undergoing cocaine detoxification.
49. Answer: (B) Report a sore throat or fever to the physician immediately. A sore
throat and fever are indications of an infection caused by agranulocytosis,
a potentially life-threatening complication of clozapine. Because of the
risk of agranulocytosis, white blood cell (WBC) counts are necessary
weekly, not monthly. If the WBC count drops below 3,000/μl, the
medication must be stopped. Hypotension may occur in clients taking this
medication. Warn the client to stand up slowly to avoid dizziness
from orthostatic hypotension. The medication should be continued, even
when symptoms have been controlled. If the medication must be stopped,
it should be slowly tapered over 1 to 2 weeks and only under
the supervision of a physician.
50. Answer: (C) Neuroleptic malignant syndrome. The client’s signs and
symptoms suggest neuroleptic malignant syndrome, a life-threatening
reaction to neuroleptic medication that requires immediate treatment.
Tardive dyskinesia causes involuntary movements of the tongue, mouth,
facial muscles, and arm and leg muscles. Dystonia is characterized by
cramps and rigidity of the tongue, face, neck, and back muscles. Akathisia
causes restlessness, anxiety, and jitteriness.
51. Answer: (B) Advising the client to sit up for 1 minute before getting out
of bed. To minimize the effects of amitriptyline-induced
orthostatic hypotension, the nurse should advise the client to sit up for 1
minute before getting out of bed. Orthostatic hypotension commonly
occurs with tricyclic antidepressant therapy. In these cases, the dosage
may be reduced or the physician may prescribe nortriptyline, another
tricyclic antidepressant. Orthostatic hypotension disappears only when
the drug is discontinued.
52. Answer: (D) Dysthymic disorder. Dysthymic disorder is marked by feelings
of depression lasting at least 2 years, accompanied by at least two of the
following symptoms: sleep disturbance, appetite disturbance, low energy
or fatigue, low selfesteem, poor concentration, difficulty making decisions,
and hopelessness. These symptoms may be relatively continuous
or separated by intervening periods of normal mood that last a few days
to a few weeks. Cyclothymic disorder is a chronic mood disturbance of at
least 2 years’ duration marked by numerous periods of depression
and hypomania. Atypical affective disorder is characterized by manic
signs and symptoms. Major depression is a recurring, persistent sadness
or loss of interest or pleasure in almost all activities, with signs and
symptoms recurring for at least 2 weeks.
53. Answer: (C) 30 g mixed in 250 ml of water.  The usual adult dosage of
activated charcoal is 5 to 10 times the estimated weight of the drug or
chemical ingested, or a minimum dose of 30 g, mixed in 250 ml of water.
Doses less than this will be ineffective; doses greater than this can
increase the risk of adverse reactions, although toxicity doesn’t occur with
activated charcoal, even at the maximum dose.
54. Answer: (C) St. John’s wort. St. John’s wort has been found to have
serotonin-elevating properties, similar to prescription antidepressants.
Ginkgo biloba is prescribed to enhance mental acuity. Echinacea has
immune-stimulating properties. Ephedra is a naturally occurring stimulant
that is similar to ephedrine.
55. Answer: (B) Sodium. Lithium is chemically similar to sodium. If sodium
levels are reduced, such as from sweating or diuresis, lithium will be
reabsorbed by the kidneys, increasing the risk of toxicity. Clients taking
lithium shouldn’t restrict their intake of sodium and should drink adequate
amounts of fluid each day. The other electrolytes are important for normal
body functions but sodium is most important to the absorption of lithium.
56. Answer: (D) It’s characterized by an acute onset and lasts hours to a number of
days. Delirium has an acute onset and typically can last from several hours
to several days.
57. Answer: (B) Impaired communication. Initially, memory impairment may be
the only cognitive deficit in a client with Alzheimer’s disease. During the
early stage of this disease, subtle personality changes may also be
present. However, other than occasional irritable outbursts and lack of
spontaneity, the client is usually cooperative and exhibits socially
appropriate behavior. Signs of advancement to the middle stage of
Alzheimer’s disease include exacerbated cognitive impairment with
obvious personality changes and impaired communication, such as
inappropriate conversation, actions, and responses. During the late stage,
the client can’t perform self-care activities and may become mute.
58. Answer: (D) This medication may initially cause tiredness, which
should become less bothersome over time. Sedation is a common early
adverse effect of imipramine, a tricyclic antidepressant, and usually
decreases as tolerance develops. Antidepressants aren’t habit forming
and don’t cause physical or psychological dependence. However, after a
long course of high-dose therapy, the dosage should be decreased
gradually to avoid mild withdrawal symptoms. Serious adverse effects,
although rare, include myocardial infarction, heart failure, and tachycardia.
Dietary restrictions, such as avoiding aged cheeses, yogurt, and chicken
livers, are necessary for a client taking a monoamine oxidase inhibitor, not
a tricyclic antidepressant.
59. Answer: (C) Monitor vital signs, serum electrolyte levels, and acid-
base balance. An anorexic client who requires hospitalization is in
poor physical condition from starvation and may die as a result of
arrhythmias, hypothermia, malnutrition, infection, or cardiac abnormalities
secondary to electrolyte imbalances. Therefore, monitoring the client’s
vital signs, serum electrolyte level, and acid base balance is crucial. Option
A may worsen anxiety. Option B is incorrect because a weight obtained
after breakfast is more accurate than one obtained after the evening meal.
Option D would reward the client with attention for not eating and
reinforce the control issues that are central to the underlying
psychological problem; also, the client may record food and fluid intake
inaccurately.
60. Answer: (D) Opioid withdrawal. The symptoms listed are specific to
opioid withdrawal. Alcohol withdrawal would show elevated vital signs.
There is no real withdrawal from cannibis. Symptoms of cocaine
withdrawal include depression, anxiety, and agitation.
61. Answer: (A) Regression. An adult who throws temper tantrums, such as
this one, is displaying regressive behavior, or behavior that is appropriate
at a younger age. In projection, the client blames someone or something
other than the source. In reaction formation, the client acts in opposition
to his feelings. In intellectualization, the client overuses rational
explanations orabstract thinking to decrease the significance of a feeling
or event.
62. Answer: (A) Abnormal movements and involuntary movements of the mouth,
tongue, and face. Tardive dyskinesia is a severe reaction associated with
long term use of antipsychotic medication. The clinical manifestations
include abnormal movements (dyskinesia) and involuntary movements of
the mouth, tongue (fly catcher tongue), and face.
63. Answer: (C) Blurred vision. At lithium levels of 2 to 2.5 mEq/L the client
will experienced blurred vision, muscle twitching, severe hypotension, and
persistent nausea and vomiting. With levels between 1.5 and 2 mEq/L the
client experiencing vomiting, diarrhea, muscle weakness, ataxia,
dizziness, slurred speech, and confusion. At lithium levels of 2.5 to 3
mEq/L or higher, urinary and fecal incontinence occurs, as well as
seizures, cardiac dysrythmias, peripheral vascular collapse, and death.
64. Answer: (C) No acts of aggression have been observed within 1 hour after the
release of two of the extremity restraints. The best indicator that the behavior
is controlled, if the client exhibits no signs of aggression after partial
release of restraints. Options A, B, and D do not ensure that the client has
controlled the behavior.
65. Answer: (A) increased attention span and concentration. The medication has
a paradoxic effect that decrease hyperactivity and impulsivity among
children with ADHD. B, C, D. Side effects of Ritalin include anorexia,
insomnia, diarrhea and irritability.
66. Answer: (C) Moderate. The child with moderate mental retardation has an
I.Q. of 35- 50 Profound Mental retardation has an I.Q. of below 20; Mild
mental retardation 50-70 and Severe mental retardation has an I.Q. of 20-
35.
67. Answer: (D) Rearrange the environment to activate the child. The child with
autistic disorder does not want change. Maintaining a consistent
environment is therapeutic. A. Angry outburst can be re-channeling
through safe activities. B. Acceptance enhances a trusting relationship. C.
Ensure safety from self-destructive behaviors like head banging and hair
pulling.
68. Answer: (B) cocaine. The manifestations indicate intoxication with
cocaine, a CNS stimulant. A. Intoxication with heroine is manifested by
euphoria then impairment in judgment, attention and the presence of
papillary constriction. C. Intoxication with hallucinogen like LSD is
manifested by grandiosity, hallucinations, synesthesia and increase in vital
signs D. Intoxication with Marijuana, a cannabinoid is manifested by
sensation of slowed time, conjunctival redness, social withdrawal,
impaired judgment and hallucinations.
69. Answer: (B) insidious onset. Dementia has a gradual onset and
progressive deterioration. It causes pronounced memory and cognitive
disturbances. A,C and D are all characteristics of delirium.
70. Answer: (C) Claustrophobia. Claustrophobia is fear of closed space. A.
Agoraphobia is fear of open space or being a situation where escape is
difficult. B. Social phobia is fear of performing in the presence of others in
a way that will be humiliating or embarrassing. D. Xenophobia is fear of
strangers.
71. Answer: (A) Revealing personal information to the client. Counter-
transference is an emotional reaction of the nurse on the client based on
her unconscious needs and conflicts. B and C. These are therapeutic
approaches. D. This is transference reaction where a client has an
emotional reaction towards the nurse based on her past.
72. Answer: (D) Hold the next dose and obtain an order for a stat serum lithium
level. Diarrhea and vomiting are manifestations of Lithium toxicity. The
next dose of lithium should be withheld and test is done to validate the
observation. A. The manifestations are not due to drug interaction.
B. Cogentin is used to manage the extra pyramidal symptom side effects
of antipsychotics. C. The common side effects of Lithium are fine
hand tremors, nausea, polyuria and polydipsia.
73. Answer: (C) A living, learning or working environment. A therapeutic milieu
refers to a broad conceptual approach in which all aspects of the
environment are channeled to provide a therapeutic environment for the
client. The six environmental elements include structure, safety, norms;
limit setting, balance and unit modification. A. Behavioral approach in
psychiatric care is based on the premise that behavior can be learned or
unlearned through the use of reward and punishment. B. Cognitive
approach to change behavior is done by correcting distorted perceptions
and irrational beliefs to correct maladaptive behaviors. D. This is not
congruent with therapeutic milieu.
74. Answer: (B) Transference. Transference is a positive or negative feeling
associated with a significant person in the client’s past that are
unconsciously assigned to another A. Splitting is a defense mechanism
commonly seen in a client with personality disorder in which the world is
perceived as all good or all bad C. Countert-transference is a phenomenon
where the nurse shifts feelings assigned to someone in her past to the
patient D. Resistance is the client’s refusal to submit himself to the care of
the nurse
75. Answer: (B) Adventitious. Adventitious crisis is a crisis involving a
traumatic event. It is not part of everyday life. A. Situational crisis is from
an external source that upset ones psychological equilibrium C and D. Are
the same. They are transitional or developmental periods in life
76. Answer: (C) Major depression. The DSM-IV-TR classifies major depression
as an Axis I disorder. Borderline personality disorder as an Axis II; obesity
and hypertension, Axis III.
77. Answer: (B) Transference. Transference is the unconscious assignment
of negative or positive feelings evoked by a significant person in the
client’s past to another person. Intellectualization is a defense mechanism
in which the client avoids dealing with emotions by focusing on facts.
Triangulation refers to conflicts involving three family members. Splitting
is a defense mechanism commonly seen in clients with personality
disorder in which the world is perceived as all good or all bad.
78. Answer: (B) Hypochondriasis. Complains of vague physical symptoms
that have no apparent medical causes are characteristic of clients with
hypochondriasis. In many cases, the GI system is affected. Conversion
disorders are characterized by one or more neurologic symptoms. The
client’s symptoms don’t suggest severe anxiety. A client experiencing
sublimation channels maladaptive feelings or impulses into socially
acceptable behavior
79. Answer: (C) Hypochondriasis. Hypochodriasis in this case is shown by the
client’s belief that she has a serious illness, although pathologic causes
have been eliminated. The disturbance usually lasts at lease 6 with
identifiable life stressor such as, in this case, course examinations.
Conversion disorders are characterized by one or more neurologic
symptoms. Depersonalization refers to persistent recurrent episodes of
feeling detached from one’s self or body. Somatoform disorders generally
have a chronic course with few remissions.
80. Answer: (A) Triazolam (Halcion). Triazolam is one of a group of sedative
hypnotic medication that can be used for a limited time because of the
risk of dependence. Paroxetine is a scrotonin-specific reutake inhibitor
used for treatment of depression panic disorder, and obsessive-
compulsive disorder. Fluoxetine is a scrotonin-specific reuptake inhibitor
used for depressive disorders and obsessive-compulsive disorders.
Risperidome is indicated for psychotic disorders.
81. Answer: (D) It promotes emotional support or attention for the
client. Secondary gain refers to the benefits of the illness that allow the
client to receive emotional support or attention. Primary gain enables the
client to avoid some unpleasant activity. A dysfunctional family
may disregard the real issue, although some conflict is relieved.
Somatoform pain disorder is a preoccupation with pain in the absence of
physical disease.
82. Answer: (A) “I went to the mall with my friends last Saturday”. Clients with
panic disorder tent to be socially withdrawn. Going to the mall is a sign of
working on avoidance behaviors. Hyperventilating is a key symptom of
panic disorder. Teaching breathing control is a major intervention for
clients with panic disorder. The client taking medications for panic
disorder; such as tricylic antidepressants and benzodiazepines, must be
weaned off these drugs. Most clients with panic disorder
with agoraphobia don’t have nutritional problems.
83. Answer: (A) “I’m sleeping better and don’t have nightmares” MAO inhibitors
are used to treat sleep problems, nightmares, and intrusive daytime
thoughts in individual with posttraumatic stress disorder. MAO inhibitors
aren’t used to help control flashbacks or phobias or to decrease the
craving for alcohol.
84. Answer: (D) Stopping the drug can cause withdrawal symptoms. Stopping
antianxiety drugs such as benzodiazepines can cause the client to have
withdrawal symptoms. Stopping a benzodiazepine doesn’t tend to cause
depression, increase cognitive abilities, or decrease sleeping difficulties.
85. Answer: (B) Behavioral difficulties. Adolescents tend to demonstrate
severe irritability and behavioral problems rather than simply a depressed
mood. Anxiety disorder is more commonly associated with small children
rather than with adolescents. Cognitive impairment is typically associated
with delirium or dementia. Labile mood is more characteristic of a client
with cognitive impairment or bipolar disorder.
86. Answer: (D) It’s a mood disorder similar to major depression but of mild
to moderate severity. Dysthymic disorder is a mood disorder similar to
major depression but it remains mild to moderate in severity.
Cyclothymic disorder is a mood disorder characterized by a mood range
from moderate depression to hypomania. Bipolar I disorder is
characterized by a single manic episode with no past major depressive
episodes. Seasonalaffective disorder is a form of depression occurring in
the fall and winter.
87. Answer: (A) Vascular dementia has more abrupt onset.  Vascular dementia
differs from Alzheimer’s disease in that it has a more abrupt onset and
runs a highly variable course. Personally change is common in Alzheimer’s
disease. The duration of delirium is usually brief. The inability to carry out
motor activities is common in Alzheimer’s disease.
88. Answer: (C) Drug intoxication. This client was taking several medications
that have a propensity for producing delirium; digoxin (a digitalis
glycoxide), furosemide (a thiazide diuretic), and diazepam (a
benzodiazepine). Sufficient supporting data don’t exist to suspect the
other options as causes.
89. Answer: (D) The client is experiencing visual hallucination. The presence of
a sensory stimulus correlates with the definition of a hallucination, which
is a false sensory perception. Aphasia refers to a communication
problem. Dysarthria is difficulty in speech production. Flight of ideas is
rapid shifting from one topic to another.
90. Answer: (D) The client looks at the shadow on a wall and tells the nurse she
sees frightening faces on the wall. Minor memory problems are distinguished
from dementia by their minor severity and their lack of significant
interference with the client’s social or occupational lifestyle. Other options
would be included in the history data but don’t directly correlate with the
client’s lifestyle.
91. Answer: (D) Loose association. Loose associations are conversations that
constantly shift in topic. Concrete thinking implies highly definitive thought
processes. Flight of ideas is characterized by conversation that’s
disorganized from the onset. Loose associations don’t necessarily start in
a cogently, then becomes loose.
92. Answer: (C) Paranoid. Because of their suspiciousness, paranoid
personalities ascribe malevolent activities to others and tent to be
defensive, becoming quarrelsome and argumentative. Clients with
antisocial personality disorder can also be antagonistic and
argumentative but are less suspicious than paranoid personalities. Clients
with histrionic personality disorder are dramatic, not suspicious and
argumentative. Clients with schizoid personality disorder are usually
detached from other and tend to have eccentric behavior.
93. Answer: (C) Explain that the drug is less affective if the client
smokes. Olanzapine (Zyprexa) is less effective for clients who
smoke cigarettes. Serotonin syndrome occurs with clients who take
a combination of antidepressant medications. Olanzapine doesn’t
cause euphoria, and extrapyramidal adverse reactions aren’t a
problem. However, the client should be aware of adverse effects such as
tardive dyskinesia.
94. Answer: (A) Lack of honesty. Clients with antisocial personality disorder
tent to engage in acts of dishonesty, shown by lying. Clients with
schizotypal personality disorder tend to be superstitious. Clients with
histrionic personality disorders tend to overreact to frustrations and
disappointments, have temper tantrums, and seek attention.
95. Answer: (A) “I’m not going to look just at the negative things about
myself”. As the clients makes progress on improving self-esteem,
selfblame and negative self evaluation will decrease. Clients with
dependent personality disorder tend to feel fragile and inadequate and
would be extremely unlikely to discuss their level of competence and
progress. These clients focus on self and aren’t envious or jealous.
Individuals with dependent personality disorders don’t take over situations
because they see themselves as inept and inadequate.
96. Answer: (C) Assess for possible physical problems such as rash. Clients with
schizophrenia generally have poor visceral recognition because they live
so fully in their fantasy world. They need to have as in-depth assessment
of physical complaints that may spill over into their delusional symptoms.
Talking with the client won’t provide as assessment of his itching, and
itching isn’t as adverse reaction of antipsychotic drugs, calling the
physician to get the client’s medication increased doesn’t address his
physical complaints.
97. Answer: (B) Echopraxia. Echopraxia is the copying of another’s behaviors
and is the result of the loss of ego boundaries. Modeling is the conscious
copying of someone’s behaviors. Ego-syntonicity refers to behaviors that
correspond with the individual’s sense of self. Ritualism behaviors are
repetitive and compulsive.
98. Answer: (C) Hallucination. Hallucinations are sensory experiences that
are misrepresentations of reality or have no basis in reality. Delusions
are beliefs not based in reality. Disorganized speech is characterized
by jumping from one topic to the next or using unrelated words. An idea
of reference is a belief that an unrelated situation holds special meaning
for the client.
99. Answer: (C) Regression. Regression, a return to earlier behavior to reduce
anxiety, is the basic defense mechanism in schizophrenia. Projection is a
defense mechanism in which one blames others and attempts to justify
actions; it’s used primarily by people with paranoid schizophrenia and
delusional disorder. Rationalization is a defense mechanism used to
justify one’s action. Repression is the basic defense mechanism in the
neuroses; it’s an involuntary exclusion of painful thoughts, feelings, or
experiences from awareness.
100. Answer: (A) Should report feelings of restlessness or agitation at
once. Agitation and restlessness are adverse effect of haloperidol and can
be treated with antocholinergic drugs. Haloperidol isn’t likely to cause
photosensitivity or control essential hypertension. Although the client may
experience increased concentration and activity, these effects are due to a
decreased in symptoms, not the drug itself.
PNLE I Nursing Practice
Scope of this Nursing Test I is parallel to the NP1 NLE Coverage:
 Foundation of Nursing
 Nursing Research
 Professional Adjustment
 Leadership and Management
1. The registered nurse is planning to delegate tasks to unlicensed assistive
personnel (UAP). Which of the following task could the registered nurse safely
assigned to a UAP?

A. Monitor the I&O of a comatose toddler client with salicylate poisoning


B. Perform a complete bed bath on a 2-year-old with multiple injuries from
a serious fall
C. Check the IV of a preschooler with Kawasaki disease
D. Give an outmeal bath to an infant with eczema
2. A nurse manager assigned a registered nurse from telemetry unit to the
pediatrics unit. There were three patients assigned to the RN. Which of the
following patients should not be assigned to the floated nurse?

A. A 9-year-old child diagnosed with rheumatic fever


B. A young infant after pyloromyotomy
C. A 4-year-old with VSD following cardiac catheterization
D. A 5-month-old with Kawasaki disease
3. A nurse in charge in the pediatric unit is absent. The nurse manager
decided to assign the nurse in the obstetrics unit to the pediatrics unit. Which
of the following patients could the nurse manager safely assign to the float
nurse?

A. A child who had multiple injuries from a serious vehicle accident


B. A child diagnosed with Kawasaki disease and with cardiac
complications
C. A child who has had a nephrectomy for Wilm’s tumor
D. A child receiving an IV chelating therapy for lead poisoning
4. The registered nurse is planning to delegate task to a certified nursing
assistant. Which of the following clients should not be assigned to a CAN?
A. A client diagnosed with diabetes and who has an infected toe
B. A client who had a CVA in the past two months
C. A client with Chronic renal failure
D. A client with chronic venous insufficiency
5. The nurse in the medication unit passes the medications for all the clients
on the nursing unit. The head nurse is making rounds with the physician and
coordinates clients’ activities with other departments. The nurse assistant
changes the bed lines and answers call lights. A second nurse is assigned for
changing wound dressings; a licensed practitioner nurse takes vital signs and
bathes theclients. This illustrates of what method of nursing care?

A. Case management method


B. Primary nursing method
C. Team method
D. Functional method
6. A registered nurse has been assigned to six clients on the 12-hour shift. The
RN is responsible for every aspect of care such as formulating the care of
plan, intervention and evaluating the care during her shift. At the end of her
shift, the RN will pass this same task to the next RN in charge. This nursing
care illustrates of what kind of method?

A. primary nursing method


B. case method
C. team method
D. functional method
7. A newly hired nurse on an adult medicine unit with 3 months experience
was asked to float to pediatrics. The nurse hesitates to perform pediatric
skills and receive an interesting assignment that feels overwhelming. The
nurse should:

A. resign on the spot from the nursing position and apply for a position
that does not require floating
B. Inform the nursing supervisor and the charge nurse on the pediatric
floor about the nurse’s lack of skill and feelings of hesitations and request
assistance
C. Ask several other nurses how they feel about pediatrics and find
someone else who is willing to accept the assignment
D. Refuse the assignment and leave the unit requesting a vacation a day
8. An experienced nurse who voluntarily trained a less experienced nurse with
the intention of enhancing the skills and knowledge and promoting
professional advancement to the nurse is called a:

A. mentor
B. team leader
C. case manager
D. change agent
9. The pediatrics unit is understaffed and the nurse manager informs the
nurses in the obstetrics unit that she is going to assign one nurse to float in
the pediatric units. Which statement by the designated float nurse may put her
job at risk?

A. “I do not get along with one of the nurses on the pediatrics unit”
B. “I have a vacation day coming and would like to take that now”
C. “I do not feel competent to go and work on that area”
D. “ I am afraid I will get the most serious clients in the unit”
10. The newly hired staff nurse has been working on a medical unit for 3
weeks. The nurse manager has posted the team leader assignments for the
following week. The new staff knows that a major responsibility of the team
leader is to:

A. Provide care to the most acutely ill client on the team


B. Know the condition and needs of all the patients on the team
C. Document the assessments completed by the team members
D. Supervise direct care by nursing assistants
11. A 15-year-old girl just gave birth to a baby boy who needs emergency
surgery. The nurse prepared the consent form and it should be signed by:

A. The Physician
B. The Registered Nurse caring for the client
C. The 15-year-old mother of the baby boy
D. The mother of the girl
12. A nurse caring to a client with Alzheimer’s disease overheard a family
member say to the client, “if you pee one more time, I won’t give you any more
food and drinks”. What initial action is best for the nurse to take?

A. Take no action because it is the family member saying that to the client
B. Talk to the family member and explain that what she/he has said is not
appropriate for the client
C. Give the family member the number for an Elder Abuse Hot line
D. Document what the family member has said
13. Which is true about informed consent?

A. A nurse may accept responsibility signing a consent form if the client is


unable
B. Obtaining consent is not the responsibility of the physician
C. A physician will not subject himself to liability if he withholds any facts
that are necessary to form the basis of an intelligent consent
D. If the nurse witnesses a consent for surgery, the nurse is, in effect,
indicating that the signature is that of the purported person and that the
person’s condition is as indicated at the time of signing
14. A mother in labor told the nurse that she was expecting that her baby has
no chance to survive and expects that the baby will be born dead. The mother
accepts the fate of the baby and informs the nurse that when the baby is born
and requires resuscitation, the mother refuses any treatment to her baby and
expresses hostility toward the nurse while the pediatric team is taking care of
the baby. The nurse is legally obligated to:

A. Notify the pediatric team that the mother has refused resuscitation and
any treatment for the baby and take the baby to the mother
B. Get a court order making the baby a ward of the court
C. Record the statement of the mother, notify the pediatric team, and
observe carefully for signs of impaired bonding and neglect as a
reasonable suspicion of child abuse
D. Do nothing except record the mother’s statement in the medical record
15. The hospitalized client with a chronic cough is scheduled for
bronchoscopy. The nurse is tasks to bring the informed consent document
into the client’s room for a signature. The client asks the nurse for details of
the procedure and demands an explanation why the process of informed
consent is necessary. The nurse responds that informed consent means:

A. The patient releases the physician from all responsibility for the
procedure.
B. The immediate family may make decision against the patient’s will.
C. The physician must give the client or surrogates enough information to
make health care judgments consistent with their values and goals.
D. The patient agrees to a procedure ordered by the physician even if the
client does not understand what the outcome will be.
16. A hospitalized client with severe necrotizing ulcer of the lower leg is
schedule for an amputation. The client tells the nurse that he will not sign the
consent form and he does not want any surgery or treatment because of
religious beliefs about reincarnation. What is the role of the RN?

A. call a family meeting


B. discuss the religious beliefs with the physician
C. encourage the client to have the surgery
D. inform the client of other options
17. While in the hospital lobby, the RN overhears the three staff discussing the
health condition of her client. What would be the appropriate nursing action
for the RN to take?

A. Tell them it is not appropriate to discuss the condition of the client


B. Ignore them, because it is their right to discuss anything they want to
C. Join in the conversation, giving them supportive input about the case of
the client
D. Report this incident to the nursing supervisor
18. A staff nurse has had a serious issue with her colleague. In this situation,
it is best to:

A. Discuss this with the supervisor


B. Not discuss the issue with anyone. It will probably resolve itself
C. Try to discuss with the colleague about the issue and resolve it when
both are calmer
D. Tell other members of the network what the team member did
19. The nurse is caring to a client who just gave birth to a healthy baby boy.
The nurse may not disclose confidential information when:

A. The nurse discusses the condition of the client in a clinical conference


with other nurses
B. The client asks the nurse to discuss the her condition with the family
C. The father of a woman who just delivered a baby is on the phone to find
out the sex of the baby
D. A researcher from an institutionally approved research study reviews
the medical record of a patient
20. A 17-year-old married client is scheduled for surgery. The nurse taking
care of the client realizes that consent has not been signed after preoperative
medications were given. What should the nurse do?

A. Call the surgeon


B. Ask the spouse to sign the consent
C. Obtain a consent from the client as soon as possible
D. Get a verbal consent from the parents of the client
21. A 12-year-old client is admitted to the hospital. The physician ordered
Dilantin to the client. In administering IV phenytoin (Dilantin) to a child, the
nurse would be most correct in mixing it with:

A. Normal Saline
B. Heparinized normal saline
C. 5% dextrose in water
D. Lactated Ringer’s solution
22. The nurse is caring to a client who is hypotensive. Following a large
hematemesis, how should the nurse position the client?

A. Feet and legs elevated 20 degrees, trunk horizontal, head on small


pillow
B. Low Fowler’s with knees gatched at 30 degrees
C. Supine with the head turned to the left
D. Bed sloped at a 45 degree angle with the head lowest and the legs
highest
23. The client is brought to the emergency department after a serious
accident. What would be the initial nursing action of the nurse to the client?
A. assess the level of consciousness and circulation
B. check respirations, circulation, neurological response
C. align the spine, check pupils, check for hemorrhage
D. check respiration, stabilize spine, check circulation
24. A nurse is assigned to care to a client with Parkinson’s disease. What
interventions are important if the nurse wants to improve nutrition and
promote effective swallowing of the client?

A. Eat solid food


B. Give liquids with meals
C. Feed the client
D. Sit in an upright position to eat
25. During tracheal suctioning, the nurse should implement safety measures.
Which of the following should the nurse implements?

A. limit suction pressure to 150-180 mmHg


B. suction for 15-20 seconds
C. wear eye goggles
D. remove the inner cannula
26. The nurse is conducting a discharge instructions to a client diagnosed
with diabetes. What sign of hypoglycemia should be taught to a client?

A. warm, flushed skin


B. hunger and thirst
C. increase urinary output
D. palpitation and weakness
27. A client admitted to the hospital and diagnosed with Addison’s disease.
What would be the appropriate nursing action to the client?

A. administering insulin-replacement therapy


B. providing a low-sodium diet
C. restricting fluids to 1500 ml/day
D. reducing physical and emotional stress
28. The nurse is to perform tracheal suctioning. During tracheal suctioning,
which nursing action is essential to prevent hypoxemia?
A. aucultating the lungs to determine the baseline data to assess the
effectiveness of suctioning
B. removing oral and nasal secretions
C. encouraging the patient to deep breathe and cough to facilitate removal
of upper-airway secretions
D. administering 100% oxygen to reduce the effects of airway obstruction
during suctioning.
29. An infant is admitted and diagnosed with pneumonia and suspicious-
looking red marks on the swollen face resembling a handprint. The nurse does
further assessment to the client. How would the nurse document the finding?

A. Facial edema with ecchymosis and handprint mark: crackles and


wheezes
B. Facial edema, with red marks; crackles in the lung
C. Facial edema with ecchymosis that looks like a handprint
D. Red bruise mark and ecchymosis on face
30. On the evening shift, the triage nurse evaluates several clients who were
brought to the emergency department. Which in the following clients should
receive highest priority?

A. an elderly woman complaining of a loss of appetite and fatigue for the


past week
B. A football player limping and complaining of pain and swelling in the
right ankle
C. A 50-year-old man, diaphoretic and complaining of severe chest pain
radiating to his jaw
D. A mother with a 5-year-old boy who says her son has been complaining
of nausea and vomited once since noon
31. A 80-year-old female client is brought to the emergency department by her
caregiver, on the nurse’s assessment; the following are the manifestations of
the client: anorexia, cachexia and multiple bruises. What would be the best
nursing intervention?

A. check the laboratory data for serum albumin, hematocrit, and


hemoglobin
B. talk to the client about the caregiver and support system
C. complete a police report on elder abuse
D. complete a gastrointestinal and neurological assessment
32. The night shift nurse is making rounds. When the nurse enters a client’s
room, the client is on the floor next to the bed. What would be the initial action
of the nurse?

A. chart that the patient fell


B. call the physician
C. chart that the client was found on the floor next to the bed
D. fill out an incident report
33. The nurse on the night shift is about to administer medication to a
preschooler client and notes that the child has no ID bracelet. The best way
for the nurse to identify the client is to ask:

A. The adult visiting, “The child’s name is ____________________?”


B. The child, “Is your name____________?”
C. Another staff nurse to identify this child
D. The other children in the room what the child’s name is
34. The nurse caring to a client has completed the assessment. Which of the
following will be considered to be the most accurate charting of a lump felt in
the right breast?

A. “abnormally felt area in the right breast, drainage noted”


B. “hard nodular mass in right breast nipple”
C. “firm mass at five ‘ clock, outer quadrant, 1cm from right nipple’
D. “mass in the right breast 4cmx1cm
35. The physician instructed the nurse that intravenous pyelogram will be
done to the client. The client asks the nurse what is the purpose of the
procedure. The appropriate nursing response is to:

A. outline the kidney vasculature


B. determine the size, shape, and placement of the kidneys
C. test renal tubular function and the patency of the urinary tract
D. measure renal blood flow
36. A client visits the clinic for screening of scoliosis. The nurse should ask
the client to:
A. bend all the way over and touch the toes
B. stand up as straight and tall as possible
C. bend over at a 90-degree angle from the waist
D. bend over at a 45-degree angle from the waist
37. A client with tuberculosis is admitted in the hospital for 2 weeks. When a
client’s family members come to visit, they would be adhering to respiratory
isolation precautions when they:

A. wash their hands when leaving


B. put on gowns, gloves and masks
C. avoid contact with the client’s roommate
D. keep the client’s room door open
38. An infant is brought to the emergency department and diagnosed with
pyloric stenosis. The parents of the client ask the nurse, “Why does my baby
continue to vomit?” Which of the following would be the best nursing
response of the nurse?

A. “Your baby eats too rapidly and overfills the stomach, which causes
vomiting
B. “Your baby can’t empty the formula that is in the stomach into the
bowel”
C. “The vomiting is due to the nausea that accompanies pyloric stenosis”
D. “Your baby needs to be burped more thoroughly after feeding”
39. A 70-year-old client with suspected tuberculosis is brought to the geriatric
care facilities. An intradermal tuberculosis test is schedule to be done. The
client asks the nurse what is the purpose of the test. Which of the following
would be the best rationale for this?

A. reactivation of an old tuberculosis infection


B. increased incidence of new cases of tuberculosis in persons over 65
years old
C. greater exposure to diverse health care workers
D. respiratory problems are characteristic in this population
40. The nurse is making a health teaching to the parents of the client. In
teaching parents how to measure the area of induration in response to a PPD
test, the nurse would be most accurate in advising the parents to measure:
A. both the areas that look red and feel raised
B. The entire area that feels itchy to the child
C. Only the area that looks reddened
D. Only the area that feels raised
41. A community health nurse is schedule to do home visit. She visits to an
elderly person living alone. Which of the following observation would be a
concern?

A. Picture windows
B. Unwashed dishes in the sink
C. Clear and shiny floors
D. Brightly lit rooms
42. After a birth, the physician cut the cord of the baby, and before the baby is
given to the mother, what would be the initial nursing action of the nurse?

A. examine the infant for any observable abnormalities


B. confirm identification of the infant and apply bracelet to mother and
infant
C. instill prophylactic medication in the infant’s eyes
D. wrap the infant in a prewarmed blanket and cover the head
43. A 2-year-old client is admitted to the hospital with severe eczema lesions
on the scalp, face, neck and arms. The client is scratching the affected areas.
What would be the best nursing intervention to prevent the client from
scratching the affected areas?

A. elbow restraints to the arms


B. Mittens to the hands
C. Clove-hitch restraints to the hands
D. A posey jacket to the torso
44. The parents of the hospitalized client ask the nurse how their baby might
have gotten pyloric stenosis. The appropriate nursing response would be:

A. There is no way to determine this preoperatively


B. Their baby was born with this condition
C. Their baby developed this condition during the first few weeks of life
D. Their baby acquired it due to a formula allergy
45. A male client comes to the clinic for check-up. In doing a physical
assessment, the nurse should report to the physician the most common
symptom of gonorrhea, which is:

A. pruritus
B. pus in the urine
C. WBC in the urine
D. Dysuria
46. Which of the following would be the most important goal in the nursing
care of an infant client with eczema?

A. preventing infection
B. maintaining the comfort level
C. providing for adequate nutrition
D. decreasing the itching
47. The nurse is making a discharge instruction to a client receiving
chemotherapy. The client is at risk for bone marrow depression. The nurse
gives instructions to the client about how to prevent infection at home. Which
of the following health teaching would be included?

A. “Get a weekly WBC count”


B. “Do not share a bathroom with children or pregnant woman”
C. “Avoid contact with others while receiving chemotherapy”
D. “Do frequent hand washing and maintain good hygiene”
48. The nurse is assigned to care the client with infectious disease. The best
antimicrobial agent for the nurse to use in handwashing is:

A. Isopropyl alcohol
B. Hexachlorophene (Phisohex)
C. Soap and water
D. Chlorhexidine gluconate (CHG) (Hibiclens)
49. The mother of the client tells the nurse, “ I’m not going to have my baby
get any immunization”. What would be the best nursing response to the
mother?

A. “You and I need to review your rationale for this decision”


B. “Your baby will not be able to attend day care without immunizations”
C. “Your decision can be viewed as a form of child abuse and neglect”
D. “You are needlessly placing other people at risk for communicable
diseases”
50. The nurse is teaching the client about breast self-examination. Which
observation should the client be taught to recognize when doing the
examination for detection of breast cancer?

A. tender, movable lump


B. pain on breast self-examination
C. round, well-defined lump
D. dimpling of the breast tissue
Answers and Rationales
1. D. Bathing an infant with eczema can be safely delegated to an aide;
this task is basic and can competently performed by an aid.
2. B. The RN floated from the telemetry unit would be least prepared to
care for a young infant who has just had GI surgery and requires a specific
feeding regimen.
3. C. RN floated from the obstetrics unit should be able to care for a client
with major abdominal surgery, because this nurse has experienced caring
for clients with cesarean births.
4. A. The patient is experiencing a potentially serious complication related
to diabetes and needs ongoing assessment by an RN
5. D. It describes functional nursing. Staff is assigned to specific task
rather than specific clients.
6. B. Case management. The nurse assumes total responsibility for
meeting the needs of the client during her entire duty.
7. B. The nurse is ethically obligated to inform the person responsible for
the assignment and the person responsible for the unit about the nurse’s
skill level. The nurse therefore avoids a situation of abandoningclients and
exposing them to greater risks
8. A. This describes a mentor
9. B. This action demonstrates a lack of responsibility and the nurse
should attempt negotiation with the nurse manager.
10. B. The team leader is responsible for the overall management of all
clients and staff on the team, and this information is essential in order to
accomplish this
11. C. Even though the mother is a minor, she is legally able to sign consent
for her own child.
12. B.  This response is the most direct and immediate. This is a case of
potential need for advocacy and patient’s rights.
13. D.  The nurse who witness a consent for treatment or surgery is
witnessing only that the client signed the form and that the client’s
condition is as indicated at the time of signing. The nurse is not
witnessing that the client is “informed”.
14. C. Although the statements by the mother may not create a suspicion of
neglect, when they are coupled with observations about impaired bonding
and maternal attachment, they may impose the obligation to report child
neglect. The nurse is further obligated to notify caregivers of refusal to
consent to treatment
15. C. It best explains what informed consent is and provides for legal rights
of the patient
16. B. The physician may not be aware of the role that religious beliefs play
in making a decision about surgery.
17. A. The behavior should be stopped. The first step is to remind the staff
that confidentiality may be violated
18. C. Waiting for emotions to dissipate and sitting down with the colleague
is the first rule of conflict resolution.
19. C. The nurse has no idea who the person is on the phone and therefore
may not share the information even if the patient gives permission
20. A. The priority is to let the surgeon know, who in turn may ask the
husband to sign the consent.
21. A. Phenytoin (Dilantin) can cause venous irritation due to its alkalinity,
therefore it should be mixed with normal saline.
22. A. This position increases venous return, improves cardiac volume, and
promotes adequate ventilation and cerebral perfusion
23. D. Checking the airway would be a priority, and a neck injury should be
suspected
24. D. Client with Parkinson’s disease are at a high risk for aspiration and
undernutrition. Sitting upright promotes more effective swallowing.
25. C.  It is important to protect the RN’s eyes from the possible
contamination of coughed-up secretions
26. D. There has been too little food or too much insulin. Glucose levels can
be markedly decreased (less than 50 mg/dl). Severe hypoglycemia may be
fatal if not detected
27. D. Because the client’s ability to react to stress is decreased,
maintaining a quiet environment becomes a nursing priority. Dehydration
is a common problem in Addison’s disease, so close observation of the
client’s hydration level is crucial.
28. D.  Presuctioning and postsuctioning ventilation with 100% oxygen is
important in reducing hypoxemia which occurs when the flow of gases in
the airway is obstructed by the suctioning catheter.
29. B. This is an example of objective data of both pulmonary status and
direct observation on the skin by the nurse.
30. C.  These are likely signs of an acute myocardial infarction (MI). An
acute MI is a cardiovascular emergency requiring immediate attention.
Acute MI is potentially fatal if not treated immediately.
31. D. Assessment and more data collection are needed. The client may
have gastrointestinal or neurological problems that account for the
symptoms. The anorexia could result from medications, poor dentition, or
indigestion, and the bruises may be attributed to ataxia, frequent falls,
vertigo or medication.
32. B. This is closest to suggesting action-assessment, rather than
paperwork- and is therefore the best of the four.
33. C. The only acceptable way to identify a preschooler client is to have a
parent or another staff member identify the client.
34. C. It describes the mass in the greatest detail.
35. C. Intravenous pyelogram tests both the function and patency of the
kidneys. After the intravenous injection of a radiopaque contrast medium,
the size, location, and patency of the kidneys can be observed by
roentgenogram, as well as the patency of the urethra and bladder as the
kidneys function to excrete the contrast medium.
36. C. This is the recommended position for screening for scoliosis. It
allows the nurse to inspect the alignment of the spine, as well as to
compare both shoulders and both hips.
37. A.  Handwashing is the best method for reducing cross-contamination.
Gowns and gloves are not always required when entering a client’s room.
38. B. Pyloric stenosis is an anomaly of the upper gastrointestinal tract. The
condition involves a thickening, or hypertrophy, of the pyloric sphincter
located at the distal end of the stomach. This causes a mechanical
intestinal obstruction, which leads to vomiting after feeding the infant. The
vomiting associated with pyloric stenosis is described as being projectile
in nature. This is due to the increasing amounts of formula the infant
begins to consume coupled with the increasing thickening of the pyloric
sphincter.
39. B. Increased incidence of TB has been seen in the general population
with a high incidence reported in hospitalized elderly clients.
Immunosuppression and lack of classic manifestations because of the
aging process are just two of the contributing factors of tuberculosis in
the elderly.
40. D. Parents should be taught to feel the area that is raised and measure
only that.
41. C. It is a safety hazard to have shiny floors because they can cause
falls.
42. D. The first priority, beside maintaining a newborn’s patent airway, is
body temperature.
43. B.  The purpose of restraints for this child is to keep the child from
scratching the affected areas. Mittens restraint would prevent scratching,
while allowing the most movement permissible.
44. C. Pyloric stenosis is not a congenital anatomical defect, but the precise
etiology is unknown. It develops during the first few weeks of life.
45. B. Pus is usually the first symptom, because the bacteria reproduce in
the bladder.
46. A. Preventing infection in the infant with eczema is the nurse’s most
important goal. The infant with eczema is at high risk for infection due to
numerous breaks in the skin’s integrity. Intact skin is always the infant’s
first line of defense against infection.
47. D. Frequent hand washing and good hygiene are the best means of
preventing infection.
48. D. CHG is a highly effective antimicrobial ingredient, especially when it
is used consistently over time.
49. A.  The mother may have many reasons for such a decision. It is the
nurse’s responsibility to review this decision with the mother and clarify
any misconceptions regarding immunizations that may exist.
50. D. The tumor infiltrates nearby tissue, it can cause retraction of the
overlying skin and create a dimpling appearance.

PNLE II Nursing Practice


The scope of  this Nursing Test II is parallel to the NP2 NLE Coverage:
 Maternal and Child Health
 Community Health Nursing
 Communicable Diseases
 Integrated Management of Childhood Illness
1. The student nurse is assigned to take the vital signs of the clients in the
pediatric ward. The student nurse reports to the staff nurse that the parent of
a toddler who is 2 days postoperative after a cleft palate repair has given the
toddler a pacifier. What would be the best immediate action of the nurse?

A. Notify the pediatrician of this finding


B. Reassure the student that this is an acceptable action on the parent’s
part
C. Discuss this action with the parents
D. Ask the student nurse to remove the pacifier from the toddler’s mouth
2. The nurse is providing a health teaching to the mother of an 8-year-old child
with cystic fibrosis. Which of the following statement if made by the mother
would indicate to the nurse the need for further teaching about the medication
regimen of the child?

A. “My child might need an extra capsule if the meal is high in fat”
B. “I’ll give the enzyme capsule before every snack”
C. “I’ll give the enzyme capsule before every meal”
D. “My child hates to take pills, so I’ll mix the capsule into a cup of hot
chocolate
3. The mother brought her child to the clinic for follow-up check up. The
mother tells the nurse that 14 days after starting an oral iron supplement, her
child’s stools are black. Which of the following is the best nursing response to
the mother?

A. “I will notify the physician, who will probably decrease the dosage
slightly”
B. “This is a normal side effect and means the medication is working”
C. “You sound quite concerned. Would you like to talk about this further?”
D. “I will need a specimen to check the stool for possible bleeding”
4. An 8-year-old boy with asthma is brought to the clinic for check up. The
mother asks the nurse if the treatment given to her son is effective. What
would be the appropriate response of the nurse?

A. I will review first the child’s height on a growth chart to know if the
treatment is working
B. I will review first the child’s weight on a growth chart to know if the
treatment is working
C. I will review first the number of prescriptions refills the child has
required over the last 6 months to give you an accurate answer
D. I will review first the number of times the child has seen the pediatrician
during the last 6 months to give you an accurate answer
5. The nurse is caring to a child client who is receiving tetracycline. The nurse
is aware that in taking this medication, it is very important to:

A. Administer the drug between meals


B. Monitor the child’s hearing
C. Give the drug through a straw
D. Keep the child out of the sunlight
6. A 14 day-old infant with a cyanotic heart defects and mild congestive heart
failure is brought to the emergency department. During assessment, the nurse
checks the apical pulse rate of the infant. The apical pulse rate is 130 beats
per minute. Which of the following is the appropriate nursing action?

A. Retake the apical pulse in 15 minutes


B. Retake the apical pulse in 30 minutes
C. Notify the pediatrician immediately
D. Administer the medication as scheduled
7. The physician prescribed gentamicin (Garamycin) to a child who is also
receiving chemotherapy. Before administering the drug, the nurse should
check the results of the child’s:

A. CBC and platelet count


B. Auditory tests
C. Renal Function tests
D. Abdominal and chest x-rays
8. Which of the following is the suited size of the needle would the nurse
select to administer the IM injection to a preschool child?

A. 18 G, 1-1/2 inch
B. 25 G, 5/8 inch
C. 21 G, 1 inch
D. 18 G, 1inch
9. A 9-year-old boy is admitted to the hospital. The boy is being treated with
salicylates for the migratory polyarthritis accompanying the diagnosis of
rheumatic fever. Which of the following activities performed by the child
would give a best sign that the medication is effective?

A. Listening to story of his mother


B. Listening to the music in the radio
C. Playing mini piano
D. Watching movie in the dvd mini player
10. The physician decided to schedule the 4-year-old client for repair of left
undescended testicle. The Injection of a hormone, HCG finds it less
successful for treatment. To administer a pentobarbital sodium (Nembutal)
suppository preoperatively to this client, in which position should the nurse
place him?

A. Supine with foot of bed elevated


B. Prone with legs abducted
C. Sitting with foot of bed elevated
D. Side-lying with upper leg flexed
11. The nurse is caring to a 24-month-old child diagnosed with congenital
heart defect. The physician prescribed digoxin (Lanoxin) to the client. Before
the administration of the drug, the nurse checks the apical pulse rate to be
110 beats per minute and regular. What would be the next nursing action?

A. Check the other vital signs and level of consciousness


B. Withhold the digoxin and notify the physician
C. Give the digoxin as prescribed
D. Check the apical and radial simultaneously, and if they are the same,
give the digoxin.
12. An 8-year-old client with cystic fibrosis is admitted to the hospital and will
undergo a chest physiotherapy treatment. The therapy should be properly
coordinated by the nurse with the respiratory therapy department so that
treatments occur during:

A. After meals
B. Between meals
C. After medication
D. Around the child’s play schedule
13. The nurse is providing health teaching about the breastfeeding and family
planning to the client who gave birth to a healthy baby girl. Which of the
following statement would alert the nurse that the client needs further
teaching?

A. “I understand that the hormones for breastfeeding may affect when my


periods come”
B. “Breastfeeding causes my womb to tighten and bleed less after birth”
C. “I may not have periods while I am breastfeeding, so I don’t need family
planning”
D. “I can get pregnant as early as one month after my baby was born”
14. A toddler is brought to the hospital because of severe diarrhea and
vomiting. The nurse assigned to the client enters the client’s room and finds
out that the client is using a soiled blanket brought in from home. The nurse
attempts to remove the blanket and replace it with a new and clean blanket.
The toddler refuses to give the soiled blanket. The nurse realizes that the best
explanation for the toddler’s behavior is:

A. The toddler did not bond well with the maternal figure
B. The blanket is an important transitional object
C. The toddler is anxious about the hospital experience
D. The toddler is resistive to nursing interventions
15. The nurse has knowledge about the developmental task of the child. In
caring a 3-year-old-client, the nurse knows that the suited developmental task
of this child is to:

A. Learn to play with other children


B. Able to trust others
C. Express all needs through speaking
D. Explore and manipulate the environment
16. A mother who gave birth to her second daughter is so concerned about
her 2-year old daughter. She tells the nurse, “I am afraid that my 2-year-old
daughter may not accept her newly born sister”. It is appropriate to the nurse
to response that:

A. The older daughter be given more responsibility and assure her “that
she is a big girl now, and doesn’t need Mommy as much”
B. The older daughter not have interaction with the baby at the hospital,
because she may harm her new sibling
C. The older daughter stay with her grandmother for a few days until the
parents and new baby are settled at home
D. The mother spend time alone with her older daughter when the baby is
sleeping
17. A 2-year-old client with cystic fibrosis is confined to bed and is not allowed
to go to the playroom. Which of the following is an appropriate toy would the
nurse select for the child:

A. Puzzle
B. Musical automobile
C. Arranging stickers in the album
D. Pounding board and hammer
18. Which of the following clients is at high risk for developmental problem?

A. A toddler with acute Glomerulonephritis on antihypertensive and


antibiotics
B. A 5-year-old with asthma on cromolyn sodium
C. A preschooler with tonsillitis
D. A 2 1/2 –year old boy with cystic fibrosis
19. Which of the following would be the best divesionary activity for the nurse
to select for a 2 weeks hospitalized 3-year-old girl?

A. Crayons and coloring books


B. doll
C. xylophone toy
D. puzzles
20. A nurse is providing safety instructions to the parents of the 11-month-old
child. Which of the following will the nurse includes in the instructions?

A. Plugging all electrical outlets in the house


B. Installing a gate at the top and bottom of any stairs in the home
C. Purchasing an infant car seat as soon as possible
D. Begin to teach the child not to place small objects in the mouth
21. An 8-year-old girl is in second grade and the parents decided to enroll her
to a new school. While the child is focusing on adjusting to new environment
and peers, her grades suffer. The child’s father severely punishes the child and
forces her daughter to study after school. The father does not allow also her
daughter to play with other children. These data indicate to the nurse that this
child is deprived of forming which normal phase of development?

A. Heterosexual relationships
B. A love relationship with the father
C. A dependency relationship with the father
D. Close relationship with peers
22. A 5-year-old boy client is scheduled for hernia surgery. The nurse is
preparing to do preoperative teaching with the child. The nurse should knows
that the 5-year-old would:

A. Expect a simple yet logical explanation regarding the surgery


B. Asks many questions regarding the condition and the procedure
C. Worry over the impending surgery
D. Be uninterested in the upcoming surgery
23. The nine-year-old client is admitted in the hospital for almost 1 week and
is on bed rest. The child complains of being bored and it seems tiresome to
stay on bed and doing nothing. What activity selected by the nurse would the
child most likely find stimulating?

A. Watching a video
B. Putting together a puzzle
C. Assembling handouts with the nurse for an upcoming staff
development meeting
D. Listening to a compact disc
24. The parent of a 16-year-old boy tells the nurse that his son is driving a
motorbike very fast and with one hand. “It is making me crazy!” What would
be the best explanation of the nurse to the behavior of the boy?

A. The adolescent might have an unconscious death wish


B. The adolescent feels indestructible
C. The adolescent lacks life experience to realize how dangerous the
behavior is
D. The adolescent has found a way to act out hostility toward the parent
25. An 8-month-old infant is admitted to the hospital due to diarrhea. The
nurse caring for the client tells the mother to stay beside the infant while
making assessment. Which of the following developmental milestones the
infant has reached?

A. Has a three-word vocabulary


B. Interacts with other infants
C. Stands alone
D. Recognizes but is fearful of strangers
26. The community nurse is conducting a health teaching in the group of
married women. When teaching a woman about fertility awareness, the nurse
should emphasize that the basal body temperature:

A. Should be recorded each morning before any activity


B. Is the average temperature taken each morning
C. Can be done with a mercury thermometer but not a digital one
D. Has a lower degree of accuracy in predicting ovulation than the cervical
mucus test
27. The community nurse is providing an instruction to the clients in the health
center about the use of diaphragm for family planning. To evaluate the
understanding of the woman, the nurse asks her to demonstrate the use of
the diaphragm. Which of following statement indicates a need for further
health teaching?

A. “I should check the diaphragm carefully for holes every time I use it.”
B. “The diaphragm must be left in place for at least 6 hours after
intercourse.”
C. “I really need to use the diaphragm and jelly most during the middle of
my menstrual cycle
D. “I may need a different size diaphragm if I gain or lose more than 20
pounds”
28. The client visits the clinic for prenatal check-up. While waiting for the
physician, the nurse decided to conduct health teaching to the client. The
nurse informed the client that primigravida mother should go to the hospital
when which patter is evident?

A. Contractions are 2-3 minutes apart, lasting 90 seconds, and membranes


have ruptured
B. Contractions are 5-10 minutes apart, lasting 30 seconds, and are felt as
strong menstrual cramps
C. Contractions are 3-5 minutes apart, accompanied by rectal pressure and
bloody show
D. Contractions are 5 minutes apart, lasting 60 seconds, and increasing in
intensity
29. A nurse is planning a home visit program to a new mother who is 2 weeks
postpartum and breastfeeding, the nurse includes in her health teaching about
the resumption of fertility, contraception and sexual activity. Which of the
following statement indicates that the mother has understood the teaching?

A. “Because breastfeeding speeds the healing process after birth, I can


have sex right away and not worry about infection”
B. “Because I am breastfeeding and my hormones are decreased, I may
need to use a vaginal lubricant when I have sex”
C. “After birth, you have to have a period before you can get pregnant
again’
D. “Breastfeeding protects me from pregnancy because it keeps my
hormones down, so I don’t need any contraception until I stop
breastfeeding”
30. A community nurse enters the home of the client for follow-up visit. Which
of the following is the most appropriate area to place the nursing bag of the
nurse when conducting a home visit?

A. cushioned footstool
B. bedside wood table
C. kitchen countertop
D. living room sofa
31. The nurse in the health center is making an assessment to the infant
client. The nurse notes some rashes and small fluid-filled bumps in the skin.
The nurse suspects that the infant has eczema. Which of the following is the
most important nursing goal:

A. Preventing infection
B. Providing for adequate nutrition
C. Decreasing the itching
D. Maintaining the comfort level
32. The nurse in the health center is providing immunization to the children.
The nurse is carefully assessing the condition of the children before giving the
vaccines. Which of the following would the nurse note to withhold the infant’s
scheduled immunizations?

A. a dry cough
B. a skin rash
C. a low-grade fever
D. a runny nose
33. A mother brought her child in the health center for hepatitis B vaccination
in a series. The mother informs the nurse that the child missed an
appointment last month to have the third hepatitis B vaccination. Which of the
following statements is the appropriate nursing response to the mother?

A. “I will examine the child for symptoms of hepatitis B”


B. “Your child will start the series again”
C. “Your child will get the next dose as soon as possible”
D. “Your child will have a hepatitis titer done to determine if immunization
has taken place.”
34. The community health nurse implemented a new program about effective
breast cancer screening technique for the female personnel of the health
department of Valenzuela. Which of the following technique should the nurse
consider to be of the lowest priority?

A. Yearly breast exam by a trained professional


B. Detailed health history to identify women at risk
C. Screening mammogram every year for women over age 50
D. Screening mammogram every 1-2 years for women over age of 40.
35. Which of the following technique is considered an aseptic practice during
the home visit of the community health nurse?

A. Wrapping used dressing in a plastic bag before placing them in the


nursing bag
B. Washing hands before removing equipment from the nursing bag
C. Using the client’s soap and cloth towel for hand washing
D. Placing the contaminated needles and syringes in a labeled container
inside the nursing bag
36. The nurse is planning to conduct a home visit in a small community.
Which of the following is the most important factor when planning the best
time for a home care visit?

A. Purpose of the home visit


B. Preference of the patient’s family
C. Location of the patient’s home
D. Length of time of the visit will take
37. The nurse assigned in the health center is counseling a 30-year-old client
requesting oral contraceptives. The client tells the nurse that she has an
active yeast infection that has recurred several times in the past year. Which
statement by the nurse is inaccurate concerning health promotion actions to
prevent recurring yeast infection?

A. “During treatment for yeast, avoid vaginal intercourse for one week”
B. “Wear loose-fitting cotton underwear”
C. “Avoid eating large amounts of sugar or sugar-bingeing”
D. “Douche once a day with a mild vinegar and water solution”
38. During immunization week in the health center, the parent of a 6-month-
old infant asks the health nurse, “Why is our baby going to receive so many
immunizations over a long time period?” The best nursing response would be:

A. “The number of immunizations your baby will receive shows how many
pediatric communicable and infectious diseases can now be prevented.”
B. “You need to ask the physician”
C. “The number of immunizations your baby will receive is determined by
your baby’s health history and age”
D. “It is easier on your baby to receive several immunizations rather than
one at a time”
39. The community health nurse is conducting a health teaching about
nutrition to a group of pregnant women who are anemic and are lactose
intolerant. Which of the following foods should the nurse especially
encourage during the third trimester?

A. Cheese, yogurt, and fish for protein and calcium needs plus prenatal
vitamins and iron supplements
B. Prenatal iron and calcium supplements plus a regular adult diet
C. Red beans, green leafy vegetables, and fish for iron and calcium needs
plus prenatal vitamins and iron supplements
D. Red meat, milk and eggs for iron and calcium needs plus prenatal
vitamins and iron supplements
40. A woman with active tuberculosis (TB) and has visited the health center
for regular therapy for five months wants to become pregnant. The nurse
knows that further information is necessary when the woman states:

A. “Spontaneous abortion may occur in one out of five women who are
infected”
B. “Pulmonary TB may jeopardize my pregnancy”
C. “I know that I may not be able to have close contact with my baby until
contagious is no longer a problem
D. “I can get pregnant after I have been free of TB for 6 months”
41. The Department of Health is alarmed that almost 33 million people suffer
from food poisoning every year. Salmonella enteritis is responsible for almost
4 million cases of food poisoning. One of the major goals is to promote proper
food preparation. The community health nurse is tasks to conduct health
teaching about the prevention of food poisoning to a group of mother
everyday. The nurse can help identify signs and symptoms of specific
organisms to help patients get appropriate treatment. Typical symptoms of
salmonella include:

A. Nausea, vomiting and paralysis


B. Bloody diarrhea
C. Diarrhea and abdominal cramps
D. Nausea, vomiting and headache
42. A community health nurse makes a home visit to an elderly person living
alone in a small house. Which of the following observation would be a great
concern?

A. Big mirror in a wall


B. Scattered and unwashed dishes in the sink
C. Shiny floors with scattered rugs
D. Brightly lit rooms
43. The health nurse is conducting health teaching about “safe” sex to a group
of high school students. Which of the following statement about the use of
condoms should the nurse avoid making?

A. “Condoms should be used because they can prevent infection and


because they may prevent pregnancy”
B. “Condoms should be used even if you have recently tested negative for
HIV”
C. “Condoms should be used every time you have sex because condoms
prevent all forms of sexually transmitted diseases”
D. “Condoms should be used every time you have sex even if you are
taking the pill because condoms can prevent the spread of HIV and
gonorrhea”
44. The department of health is promoting the breastfeeding program to all
newly mothers. The nurse is formulating a plan of care to a woman who gave
birth to a baby girl. The nursing care plan for a breast-feeding mother takes
into account that breast-feeding is contraindicated when the woman:

A. Is pregnant
B. Has genital herpes infection
C. Develops mastitis
D. Has inverted nipples
45. The City health department conducted a medical mission in Barangay
Marulas. Majority of the children in the Barangay Marulas were diagnosed
with pinworms. The community health nurse should anticipate that the
children’s chief complaint would be:

A. Lack of appetite
B. Severe itching of the scalp
C. Perianal itching
D. Severe abdominal pain
46. The mother brought her daughter to the health center. The child has head
lice. The nurse anticipates that the nursing diagnosis most closely correlated
with this is:

A. Fluid volume deficit related to vomiting


B. Altered body image related to alopecia
C. Altered comfort related to itching
D. Diversional activity deficit related to hospitalization
47. The mother brings a child to the health care clinic because of severe
headache and vomiting. During the assessment of the health care nurse, the
temperature of the child is 40 degree Celsius, and the nurse notes the
presence of nuchal rigidity. The nurse is suspecting that the child might be
suffering from bacterial meningitis. The nurse continues to assess the child
for the presence of Kernig’s sign. Which finding would indicate the presence
of this sign?

A. Flexion of the hips when the neck is flexed from a lying position
B. Calf pain when the foot is dorsiflexed
C. Inability of the child to extend the legs fully when lying supine
D. Pain when the chin is pulled down to the chest
48. A community health nurse makes a home visit to a child with an infectious
and communicable disease. In planning care for the child, the nurse must
determine that the primary goal is that the:

A. Child will experience mild discomfort


B. Child will experience only minor complications
C. Child will not spread the infection to others
D. Public health department will be notified
49. The mother brings her daughter to the health care clinic. The child was
diagnosed with conjunctivitis. The nurse provides health teaching to the
mother about the proper care of her daughter while at home. Which statement
by the mother indicates a need for additional information?

A. “I do not need to be concerned about the spreading of this infection to


others in my family”
B. “I should apply warm compresses before instilling antibiotic drops if
purulent discharge is present in my daughter’s eye”
C. “I can use an ophthalmic analgesic ointment at nighttime if I have eye
discomfort”
D. “I should perform a saline eye irrigation before instilling, the antibiotic
drops into my daughter’s eye if purulent discharge is present”
50. A community health nurse is caring for a group of flood victims in Marikina
area. In planning for the potential needs of this group, which is the most
immediate concern?

A. Finding affordable housing for the group


B. Peer support through structured groups
C. Setting up a 24-hour crisis center and hotline
D. Meeting the basic needs to ensure that adequate food, shelter and
clothing are available
Answers and Rationales
1. C. Nothing must be placed in the mouth of a toddler who just undergone
a cleft palate repair until the suture line has completely healed. It is the
nurse’s responsibility to inform the parent of the client. Spoon, forks,
straws, and tongue blades are other unacceptable items to place in the
mouth of a toddler who just undergone cleft palate repair. The general
principle of care is that nothing should enter the mouth until the suture line
has completely healed.
2. D. The pancreatic capsules contain pancreatic enzyme that should be
administered in a cold, not a hot, medium (example: chilled applesauce
versus hot chocolate) to maintain the medication’s integrity.
3. B. When oral iron preparations are given correctly, the stools normally
turn dark green or black. Parents of children receiving this medication
should be advised that this side effect indicates the medication is being
absorbed and is working well.
4. C. Reviewing the number of prescription refills the child has required
over the last 6 months would be the best indicator of how well controlled
and thus how effective the child’s asthma treatment is. Breakthrough
wheezing, shortness of breath, and upper respiratory infections would
require that the child take additional medication. This would be reflected
in the number of prescription refills.
5. D. Tetracycline may cause a phototoxic reaction.
6. D. The normal heart rate of an infant is 120-160 beats per minute.
7. C. Both gentamicin and chemotherapeutic agents can cause renal
impairment and acute renal failure; thus baseline renal function must be
evaluated before initiating either medication.
8. C. In selecting the correct needle to administer an IM injection to a
preschooler, the nurse should always look at the child and use judgment in
evaluating muscle mass and amount of subcutaneous fat. In this case, in
the absence of further data, the nurse would be most correct in selecting a
needle gauge and length appropriate for the “average’ preschool child. A
medium-gauge needle (21G) that is 1 inch long would be most
appropriate.
9. C. The purpose of the salicylate therapy is to relieve the pain associated
with the migratory polyarthritis accompanying the rheumatic fever. Playing
mini piano would require movement of the child’s joints and would provide
the nurse with a means of evaluating the child’s level of pain.
10. D. The recommended position to administer rectal medications to
children is side-lying with the upper leg flexed. This position allows the
nurse to safely and effectively administer the medication while promoting
comfort for the child.
11. C. For a 12month-old child, 110 apical pulse rate is normal and therefore
it is safe to give the digoxin. A toddler’s normal pulse rate is slightly lower
than an infant’s (120).
12. B. Chest physiotherapy treatments are scheduled between meals to
prevent aspiration of stomach contents, because the child is placed in a
variety of positions during the treatment process.
13. C. It is common misconception that breastfeeding may prevent
pregnancy.
14. B. The “security blanket” is an important transitional object for the
toddler. It provides a feeling of comfort and safety when the maternal
figure is not present or when in a new situation for which the toddler was
not prepared. Virtually any object (stuffed animal, doll, book etc) can
become a security blanket for the toddler.
15. D. Toddlers need to meet the developmental milestone of autonomy
versus shame and doubt. In order to accomplish this, the toddler must be
able to explore and manipulate the environment.
16. D. The introduction of a baby into a family with one or more children
challenges parent to promote acceptance of the baby by siblings. The
parent’s attitudes toward the arrival of the baby can set the stage for the
other children’s reaction. Spending time with the older siblings alone will
also reassure them of their place in the family, even though the older
children will have to eventually assume new positions within the family
hierarchy.
17. D. The autonomous toddler would be frustrated by being confined to be.
The pounding board and hammer is developmentally appropriate and an
excellent way for the toddler to release frustration.
18. D.  It is the developmental task of an 18-month-old toddler to explore
and learn about the environment. The respiratory complications
associated with cystic fibrosis (which are present in almost all children
with cystic fibrosis) could prevent this development task from occurring.
19. C. The best diversion for a hospitalized child aged 2-3 years old would
be anything that makes noise or makes a mess; xylophone which certainly
makes noise or music would be the best choice.
20. B. An 11-month-old child stands alone and can walk holding onto people
or objects. Therefore the installation of a gate at the top and bottom of
any stairs in the house is crucial for the child’s safety.
21. D. In second grade a child needs to form a close relationships with
peers.
22. B.  A 5-year-old is highly concerned with body integrity. The preschool-
age child normally asks many questions and in a situation such as this,
could be expected to ask even more.
23. C. A 9-year-old enjoys working and feeling a sense of accomplishment.
The school-age child also enjoys “showing off,” and doing something with
the nurse on the pediatric unit would allow this. This activity also provides
the school-age child a needed opportunity to interact with others in the
absence of school and personal friends.
24. B. Adolescents do feel indestructible, and this is reflected in many risk-
taking behaviors.
25. D. An 8-month-old infant both recognizes and is fearful of strangers.
This developmental milestone is known as “stranger anxiety”.
26. A. The basal body temperature (BBT) is the lowest body temperature of
a healthy person that is taken immediately after waking and before getting
out of bed. The BBT usually varies from 36.2 – 36.3 degree Celsius during
menses and for about 5-7 days afterward. About the time of ovulation, a
slight drop approximately 0.05 degree Celsius in temperature may be
seen; after ovulation, in concert with the increasing progesterone levels of
the early luteal phase, the BBT rises 0.2-0.4 degree Celsius. This elevation
remains until 2-3 days before menstruation, or if pregnancy has occurred.
27. C.  The woman must understand that, although the “fertile” period is
approximately midcycle, hormonal variations do occur and can result in
early or late ovulations. To be effective, the diaphragm should be inserted
before every intercourse.
28. D. Although instructions vary among birth centers, primigravidas should
seek care when regular contractions are felt about 5 minutes apart,
becoming longer and stronger.
29. B.  Prolactin suppresses estrogen, which is needed to stimulate vaginal
lubrication during arousal.
30. B. A wood surface provides the least chance for organisms to be
present.
31. A. Preventing infection in the infant with eczema is the nurse’s most
important goal. The infant with eczema is at high risk for infection due to
numerous breaks in the skin’s integrity. Intact skin is always the infant’s
first line of defense against infection.
32. B.  A skin rash could indicate a concurrent infectious disease process in
the infant. The scheduled immunizations should be withheld until the
status of the infant’s health can be determined. Fevers above 38.5
degrees Celsius, alteration in skin integrity, and infectious-appearing
secretions are indications to withhold immunizations.
33. C. Continuity is essential to promote active immunity and give hepatitis
B lifelong prophylaxis. Optimally, the third vaccination is given 6 months
after the first.
34. B. Because of the high incidence of breast cancer, all women are
considered to be at risk regardless of health history.
35. B. Handwashing is the best way to prevent the spread of infection.
36. A. The purpose of the visit takes priority.
37. D. Frequent douching interferes with the natural protective barriers in
the vagina that resist yeast infection and should be avoided.
38. A. Completion for the recommended schedule of infant immunizations
does not require a large number of immunizations, but it also provides
protection against multiple pediatric communicable and infectious
diseases.
39. C. This is appropriate foods that are high in iron and calcium but would
not affect lactose intolerance.
40. D. Intervention is needed when the woman thinks that she needs to wait
only 6 months after being free of TB before she can get pregnant. She
needs to wait 1.5-2years after she is declared to be free of TB before she
should attempt pregnancy.
41. C. Salmonella organisms cause lower GI symptoms
42. C. It is a safety hazard to have shiny floors and scattered rugs because
they can cause falls and rugs should be removed.
43. C. Condoms do not prevent ALL forms of sexually transmitted diseases.
44. A.  Pregnancy is one contraindication to breast-feeding. Milk secretion
is inhibited and the baby’s sucking may stimulate uterine contractions.
45. C.  Perianal itching is the child’s chief complaint associated with the
diagnosis of pinworms. The itching, in this instance, is often described as
being “intense” in nature. Pinworms infestation usually occurs because
the child is in the anus-to-mouth stage of development (child uses the
toilet, does not wash hands, places hands and pinworm eggs in mouth).
Teaching the child hand washing before eating and after using the toilet
can assist in breaking the cycle.
46. C.  Severe itching of the scalp is the classic sign and symptom of head
lice in a child. In turn, this would lead to the nursing diagnosis of “altered
comfort”.
47. C. Kernig’s sign is the inability of the child to extend the legs fully when
lying supine. This sign is frequently present in bacterial meningitis. Nuchal
rigidity is also present in bacterial meningitis and occurs when pain
prevents the child from touching the chin to the chest.
48. C. The primary goal is to prevent the spread of the disease to others.
The child should experience no complication. Although the health
department may need to be notified at some point, it is no the primary
goal. It is also important to prevent discomfort as much as possible.
49. A. Conjunctivitis is highly contagious. Antibiotic drops are usually
administered four times a day. When purulent discharge is present, saline
eye irrigations or eye applications of warm compresses may be necessary
before instilling the medication. Ophthalmic analgesic ointment or drops
may be instilled, especially at bedtime, because discomfort becomes
more noticeable when the eyelids are closed.
50. D.  The question asks about the immediate concern. The ABCs of
community health care are always attending to people’s basic needs of
food, shelter, and clothing
PNLE III Nursing Practice
The scope of this Nursing Test III is parallel to the NP3 NLE Coverage:
 Medical Surgical Nursing
1. The nurse is going to replace the Pleur-O-Vac attached to the client with a
small, persistent left upper lobe pneumothorax with a Heimlich Flutter Valve.
Which of the following is the best rationale for this?

A. Promote air and pleural drainage


B. Prevent kinking of the tube
C. Eliminate the need for a dressing
D. Eliminate the need for a water-seal drainage
2. The client with acute pancreatitis and fluid volume deficit is transferred
from the ward to the ICU. Which of the following will alert the nurse?

A. Decreased pain in the fetal position


B. Urine output of 35mL/hr
C. CVP of 12 mmHg
D. Cardiac output of 5L/min
3. The nurse in the morning shift is making rounds in the ward. The nurse
enters the client’s room and found the client in discomfort condition. The
client complains of stiffness in the joints. To reduce the early morning
stiffness of the joints of the client,the nurse can encourage the client to:

A. Sleep with a hot pad


B. Take to aspirins before arising, and wait 15 minutes before attempting
locomotion
C. Take a hot tub bath or shower in the morning
D. Put joints through passive ROM before trying to move them actively
4. The nurse is planning of care to a client with peptic ulcer disease. To avoid
the worsening condition of the client, the nurse should carefully plan the diet
of the client. Which of the following will be included in the diet regime of the
client?

A. Eating mainly bland food and milk or dairy products


B. Reducing intake of high-fiber foods
C. Eating small, frequent meals and a bedtime snack
D. Eliminating intake of alcohol and coffee
5. The physician has given instruction to the nurse that the client can be
ambulated on crutches, with no weight bearing on the affected limb. The
nurse is aware that the appropriate crutch gait for the nurse to teach the client
would be:

A. Tripod gait
B. Two-point gait
C. Four-point gait
D. Three-point gait
6. The client is transferred to the nursing care unit from the operating room
after a transurethral resection of the prostate. The client is complaining of
pain in the abdomen area. The nurse suspects of bladder spasms, which of
the following is the best nursing action to minimize the pain felt by the client?

A. Advising the client not to urinate around catheter


B. Intermittent catheter irrigation with saline
C. Giving prescribed narcotics every 4 hour
D. Repositioning catheter to relieve pressure
7. A client is diagnosed with peptic ulcer. The nurse caring for the client
expects the physician to order which diet?

A. NPO
B. Small feedings of bland food
C. A regular diet given frequently in small amounts
D. Frequent feedings of clear liquids
8. The nurse is going to insert a Miller-Abbott tube to the client. Before
insertion of the tube, the balloon is tested for patency and capacity and then
deflated. Which of the following nursing measure will ease the insertion to the
tube?

A. Positioning the client in Semi-Fowler’s position


B. Administering a sedative to reduce anxiety
C. Chilling the tube before insertion
D. Warming the tube before insertion
9. The physician ordered a low-sodium diet to the client. Which of the
following food will the nurse avoid to give to the client?

A. Orange juice.
B. Whole milk.
C. Ginger ale.
D. Black coffee.
10. Mr. Bean, a 70-year-old client is admitted in the hospital for almost one
month. The nurse understands that prolonged immobilization could lead to
decubitus ulcers. Which of the following would be the least appropriate
nursing intervention in the prevention of decubitus?

A. Giving backrubs with alcohol


B. Use of a bed cradle
C. Frequent assessment of the skin
D. Encouraging a high-protein diet
11. The physician prescribed digoxin 0.125 mg PO qd to a client and
instructed the nurse that the client is on high-potassium diet. High potassium
foods are recommended in the diet of a client taking digitalis preparations
because a low serum potassium has which of the following effects?

A. Potentiates the action of digoxin


B. Promotes calcium retention
C. Promotes sodium excretion
D. Puts the client at risk for digitalis toxicity
12. The nurse is caring for a client who is transferred from the operating room
for pneumonectomy. The nurse knows that immediately following
pneumonectomy; the client should be in what position?

A. Supine on the unaffected side


B. Low-Fowler’s on the back
C. Semi-Fowler’s on the affected side
D. Semi-Fowler’s on the unaffected side
13. A client is placed on digoxin, high potassium foods are recommended in
the diet of the client. Which of the following foods willthe nurse give to the
client?
A. Whole grain cereal, orange juice, and apricots
B. Turkey, green bean, and Italian bread
C. Cottage cheese, cooked broccoli, and roast beef
D. Fish, green beans and cherry pie
14. The nurse is assigned to care to a client who undergone thyroidectomy.
What nursing intervention is important during the immediate postoperative
period following a thyroidectomy?

A. Assess extremities for weakness and flaccidity


B. Support the head and neck during position changes
C. Position the client in high Fowler’s
D. Medicate for restlessness and anxiety
15. What would be the recommended diet the nurse will implement to a client
with burns of the head, face, neck and anterior chest?

A. Serve a high-protein, high-carbohydrate diet


B. Encourage full liquid diet
C. Serve a high-fat diet, high-fiber diet
D. Monitor intake to prevent weight gain
16. A client with multiple fractures of both lower extremities is admitted for 3
days ago and is on skeletal traction. The client is complaining of having
difficulty in bowel movement. Which of the following would be the most
appropriate nursing intervention?

A. Administer an enema
B. Perform range-of-motion exercise to all extremities
C. Ensure maximum fluid intake (3000ml/day)
D. Put the client on the bedpan every 2 hours
17. John is diagnosed with Addison’s disease and admitted in the hospital.
What would be the appropriate nursing care for John?

A. Reducing physical and emotional stress


B. Providing a low-sodium diet
C. Restricting fluids to 1500ml/day
D. Administering insulin-replacement therapy
18. Mr. Smith is scheduled for an above-the-knee amputation. After the
surgery he was transferred to the nursing care unit. The nurse assigned to him
knows that 72 hours after the procedure the client should be positioned
properly to prevent contractures. Which of the following is the best position to
the client?

A. Side-lying, alternating left and right sides


B. Sitting in a reclining chair twice a day
C. Lying on abdomen several times daily
D. Supine with stump elevated at least 30 degrees
19. A client is scheduled to have an inguinal herniorraphy in the outpatient
surgical department. The nurse is providing health teaching about post
surgical care to the client. Which of the following statement if made by the
client would reflect the need for more teaching?

A. “I should call the physician if I have a cough or cold before surgery”


B. “I will be able to drive soon after surgery”
C. “I will not be able to do any heavy lifting for 3-6 weeks after surgery”
D. “I should support my incision if I have to cough or turn”
20. Ms Jones is brought to the emergency room and is complaining of muscle
spasms, numbness, tremors and weakness in the arms and legs. The client
was diagnosed with multiple sclerosis. The nurse assigned to Ms. Jones is
aware that she has to prevent fatigue to the client to alleviate the discomfort.
Which of the following teaching is necessary to prevent fatigue?

A. Avoid extremes in temperature


B. Install safety devices in the home
C. Attend support group meetings
D. Avoid physical exercise
21. Mr. Stewart is in sickle cell crisis and complaining pain in the joints and
difficulty of breathing. On the assessment of the nurse, his temperature is
38.1 ºC. The physician ordered Morphine sulfate via patient-controlled
analgesia (PCA), and oxygen at 4L/min. A priority nursing diagnosis to Mr.
Stewart is risk for infection. A nursing intervention to assist in preventing
infection is:

A. Using standard precautions and medical asepsis


B. Enforcing a “no visitors” rule
C. Using moist heat on painful joints
D. Monitoring a vital signs every 2 hour
22. Mrs. Maupin is a professor in a prestigious university for 30 years. After
lecture, she experience blurring of vision and tiredness. Mrs. Maupin is
brought to the emergency department. On assessment, the nurse notes that
the blood pressure of the client is 139/90. Mrs. Maupin has been diagnosed
with essential hypertension and placed on medication to control her BP.
Which potential nursing diagnosis will be a priority for discharge teaching?

A. Sleep Pattern disturbance


B. Impaired physical mobility
C. Noncompliance
D. Fluid volume excess
23. Following a needle biopsy of the kidney, which assessment is an
indication that the client is bleeding?

A. Slow, irregular pulse


B. Dull, abdominal discomfort
C. Urinary frequency
D. Throbbing headache
24. A client with acute bronchitis is admitted in the hospital. The nurse
assigned to the client is making a plan of care regarding expectoration of
thick sputum. Which nursing action is most effective?

A. Place the client in a lateral position every 2 hour


B. Splint the patient’s chest with pillows when coughing
C. Use humified oxygen
D. Offer fluids at regular intervals
25. The nurse is going to assess the bowel sound of the client. For accurate
assessment of the bowel sound, the nurse should listen for at least:

A. 5 minutes
B. 60 seconds
C. 30 seconds
D. 2 minutes
26. The nurse encourages the client to wear compression stockings. What is
the rationale behind in using compression stockings?
A. Compression stockings promote venous return
B. Compression stockings divert blood to major vessels
C. Compression stockings decreases workload on the heart
D. Compression stockings improve arterial circulation
27. Mr. Whitman is a stroke client and is having difficulty in swallowing. Which
is the best nursing intervention is most likely to assist the client?

A. Placing food in the unaffected side of the mouth


B. Increasing fiber in the diet
C. Asking the patient to speak slowly
D. Increasing fluid intake
28. Following nephrectomy, the nurse closely monitors the urinary output of
the client. Which assessment finding is an early indicator of fluid retention in
the postoperative period?

A. Periorbital edema
B. Increased specific gravity of urine
C. A urinary output of 50mL/hr
D. Daily weight gain of 2 lb or more
29. A nurse is completing an assessment to a client with cirrhosis. Which of
the following nursing assessment is important to notify the physician?

A. Expanding ecchymosis
B. Ascites and serum albumin of 3.2 g/dl
C. Slurred speech
D. Hematocrit of 37% and hemoglobin of 12g/dl
30. Mr. Park is 32-year-old, a badminton player and has a type 1 diabetes
mellitus. After the game, the client complains of becoming diaphoretic and
light-headedness. The client asks the nurse how to avoid this reaction. The
nurse will recommend to:

A. Allow plenty of time after the insulin injection and before beginning the
match
B. Eat a carbohydrate snack before and during the badminton match
C. Drink plenty of fluids before, during, and after bed time
D. Take insulin just before starting the badminton match
31. A client is rushed to the emergency room due to serious vehicle accident.
The nurse is suspecting of head injury. Which of the following assessment
findings would the nurse report to the physician?

A. CVP of 5mmHa
B. Glasgow Coma Scale score of 13
C. Polyuria and dilute urinary output
D. Insomnia
32. Mrs. Moore, 62-year-old, with diabetes is in the emergency department.
She stepped on a sharp sea shells while walking barefoot along the beach.
Mrs. Moore did not notice that the object pierced the skin until later that
evening. What problem does the client most probably have?

A. Nephropathy
B. Macroangiopathy
C. Carpal tunnel syndrome
D. Peripheral neuropathy
33. A client with gangrenous foot has undergone a below-knee amputation.
The nurse in the nursing care unit knows that the priority nursing intervention
in the immediate post operative care of this client is:

A. Elevate the stump on a pillow for the first 24 hours


B. Encourage use of trapeze
C. Position the client prone periodically
D. Apply a cone-shaped dressing
34. A client with a diagnosis of gastric ulcer is complaining of syncope and
vertigo. What would be the initial nursing intervention by the nurse?

A. Monitor the client’s vital signs


B. Keep the client on bed rest
C. Keep the patient on bed rest
D. Give a stat dose of Sucralfate (Carafate)
35. After a right lower lobectomy on a 55-year-old client, which action should
the nurse initiate when the client is transferred from the post anesthesia care
unit?

A. Notify the family to report the client’s condition


B. Immediately administer the narcotic as ordered
C. Keep client on right side supported by pillows
D. Encourage coughing and deep breathing every 2 hours
36. The nurse is providing a discharge instruction about the prevention of
urinary stasis to a client with frequent bladder infection. Which of the
following will the nurse include in the instruction?

A. Drink 3-4 quarts of fluid every day


B. Empty the bladder every 2-4 hours while awake
C. Encourage the use of coffee, tea, and colas for their diuretic effect
D. Teach Kegel exercises to control bladder flow
37. A male client visits the clinic for check-up. The client tells the nurse that
there is a yellow discharge from his penis. He also experiences a burning
sensation when urinating. The nurse is suspecting of gonorrhea. What
teaching is necessary for this client?

A. Sex partner of 3 months ago must be treated


B. Women with gonorrhea are symptomatic
C. Use a condom for sexual activity
D. Sex partner needs to be evaluated
38. A client with AIDS is admitted in the hospital. He is receiving intravenous
therapy. While the nurse is assessing the IV site, the client becomes confused
and restless and the intravenous catheter becomes disconnected and
minimal amount of the client’s blood spills onto the floor. Which action will the
nurse take to remove the blood spill?

A. Promptly clean with a 1:10 solution of household bleach and water


B. Promptly clean up the blood spill with full-strength antimicrobial
cleaning solution
C. Immediately mop the floor with boiling water
D. Allow the blood to dry before cleaning to decrease the possibility of
cross-contamination
39. Before surgery, the physician ordered pentobarbital sodium (Nembutal) for
the client to sleep. The night before the scheduled surgery, the nurse gave the
pre-medication. One hour later the client is still unable to sleep. The nurse
review the client’s chart and note the physician’s prescription with an order to
repeat. What should the nurse do next?

A. Rub the client’s back until relaxed


B. Prepare a glass of warm milk
C. Give the second dose of pentobarbital sodium
D. Explore the client’s feelings about surgery
40. The nurse on the night shift is making rounds in the nursing care unit. The
nurse is about to enter to the client’s room when a ventilator alarm sounds,
what is the first action the nurse should do?

A. Assess the lung sounds


B. Suction the client right away
C. Look at the client
D. Turn and position the client
41. What effective precautions should the nurse use to control the
transmission of methicillin-resistant Staphylococcus aureus (MRSA)?

A. Use gloves and handwashing before and after client contact


B. Do nasal cultures on healthcare providers
C. Place the client on total isolation
D. Use mask and gown during care of the MRSA client
42. The postoperative gastrectomy client is scheduled for discharge. The
client asks the nurse, “When I will be allowed to eat three meals a day like the
rest of my family?”. The appropriate nursing response is:

A. “You will probably have to eat six meals a day for the rest of your life.”
B. “Eating six meals a day can be a bother, can’t it?”
C. “Some clients can tolerate three meals a day by the time they leave the
hospital. Maybe it will be a little longer for you.”
D. “ It varies from client to client, but generally in 6-12 months most clients
can return to their previous meal patterns”
43. A male client with cirrhosis is complaining of belly pain, itchiness and his
breasts are getting larger and also the abdomen. The client is so upset
because of the discomfort and asks the nurse why his breast and abdomen
are getting larger. Which of the following is the appropriate nursing response?
A. “How much of a difference have you noticed”
B. “It’s part of the swelling your body is experiencing”
C. “It’s probably because you have been less physically active”
D. “Your liver is not destroying estrogen hormones that all men produce”
44. A client is diagnosed with detached retina and scheduled for surgery.
Preoperative teaching of the nurse to the client includes:

A. No eye pain is expected postoperatively


B. Semi-fowler’s position will be used to reduce pressure in the eye.
C. Eye patches may be used postoperatively
D. Return of normal vision is expected following surgery
45. A 70-year-old client is brought to the emergency department with a
caregiver. The client has manifestations of anorexia, wasting of muscles and
multiple bruises. What nursing interventions would the nurse implement?

A. Talk to the client about the caregiver and support system


B. Complete a gastrointestinal and neurological assessment
C. Check the lab data for serum albumin, hematocrit and hemoglobin
D. Complete a police report on elder abuse
46. A nurse is providing a discharge instruction to the client about the self-
catheterization at home. Which of the following instructions would the nurse
include?

A. Wash the catheter with soap and water after each use
B. Lubricate the catheter with Vaseline
C. Perform the Valsalva maneuver to promote insertion
D. Replace the catheter with a new one every 24 hour
47. The nurse in the nursing care unit is assigned to care to a client who is
Immunocompromised. The client tells the nurse that his chest is painful and
the blisters are itchy. What would be the nursing intervention to this client?

A. Call the physician


B. Give a prn pain medication
C. Clarify if the client is on a new medication
D. Use gown and gloves while assessing the lesions
48. A client is admitted and has been diagnosed with bacterial
(meningococcal) meningitis. The infection control registered nurse visits the
staff nurse caring to the client. What statement made by the nurse reflects an
understanding of the management of this client?

A. speech pattern may be altered


B. Respiratory isolation is necessary for 24 hours after antibiotics are
started
C. Perform skin culture on the macular popular rash
D. Expect abnormal general muscle contractions
49. A 18-year-old male client had sustained a head injury from a motorbike
accident. It is uncertain whether the client may have minimal but permanent
disability. The family is concerned regarding the client’s difficulty accepting
the possibility of long term effects. Which nursing diagnosis is best for this
situation?

A. Nutrition, less than body requirements


B. Injury, potential for sensory-perceptual alterations
C. Impaired mobility, related to muscle weakness
D. Anticipatory grieving, due to the loss of independence
50. A client with AIDS is scheduled for discharge. The client tells the nurse
that one of his hobbies at home is gardening. What will be the discharge
instruction of the nurse to the client knowing that the client is prone to
toxoplasmosis?

A. Wash all vegetables before cooking


B. Wear gloves when gardening
C. Wear a mask when travelling to foreign countries
D. Avoid contact with cats and birds
Answers and Rationales
1. D. The Heimlich flutter valve has a one-way valve that allows air and
fluid to drain. Underwater seal drainage is not necessary. This can be
connected to a drainage bag for the patient’s mobility. The absence of a
long drainage tubing and the presence of a one-way valve promote
effective therapy
2. C. C = the normal CVP is 0-8 mmHg. This value reflects hypervolemia.
The right ventricular function of this client reflects fluid volume overload,
and the physician should be notified.
3. C. A hot tub bath or shower in the morning helps many patients limber
up and reduces the symptoms of early morning stiffness. Cold and ice
packs are used to a lesser degree, though some clients state that cold
decreases localized pain, particularly during acute attacks.
4. D. These substances stimulate the production of hydrochloric acid,
which is detrimental in peptic ulcer disease.
5. D. The three-point gait is appropriate when weight bearing is not
allowed on the affected limb. The swing-to and swing-through crutch gaits
may also be used when only one leg can be used for weight bearing
6. A. The client needs to be told before surgery that the catheter causes
the urge to void. Attempts to void around the catheter cause the bladder
muscles to contract and result in painful spasms.
7. B. Bland feedings should be given in small amounts on a frequent basis
to neutralize the hydrochloric acid and to prevent overload
8. C. Chilling the tube before insertion assists in relieving some of the
nasal discomfort. Water-soluble lubricants along with viscous lidocaine
(Xylocaine) may also be used. It is usually only lightly lubricated before
insertion
9. B. Whole milk should be avoided to include in the client’s diet because it
has 120 mg of sodium in 8 0z of milk.
10. A. Alcohol is extremely drying and contributes to skin break down. An
emollient lotion should be used.
11. D.  Potassium influences the excitability of nerves and muscles. When
potassium is low and the client is on digoxin, the risk of digoxin toxicity is
increased.
12. C. This position allows maximum expansion, ventilation, and perfusion
of the remaining lung.
13. A. These foods are high in potassium
14. B. Stress on the suture line should be avoided. Prevent flexion or
hyperextension of the neck, and provide a small pillow under thehead and
neck. Neck muscles have been affected during a thyroidectomy, support
essential for comfort and incisional support.
15. A. A positive nitrogen balance is important for meeting metabolic
needs, tissue repair, and resistance to infection. Caloric goals may be as
high as 5000 calories per day.
16. C. The best early intervention would be to increase fluid intake, because
constipation is common when activity is decreased or usual routines have
been interrupted.
17. A. Because the client’s ability is to react to stress is decreased,
maintaining a quiet environment becomes A nursing priority. Dehydration
is a common problem in Addison’s disease, so close observation of the
client’s hydration level is crucial. To promote optimal hydration and
sodium intake, fluid intake is increased, particularly fluid containing
electrolytes, such as broths, carbonated beverages, and juices.
18. C. At about 48-72 hours, the client must be turned onto the abdomen to
prevent flexion contractures.
19. B. The client should not drive for 2 weeks after surgery to avoid stress
on the incision. This reflects a need for additional teaching.
20. A. Extremes in heat and cold will exacerbate symptoms. Heat delays
transmission of impulses and increases fatigue.
21. A. Vigilant implementation of standard precautions and medical
asepsis is an effective means of preventing infection
22. C. Noncompliance is a major problem in the management of chronic
disease. In hypertension, the client often does not feel ill and thus does
not see a need to follow a treatment regimen.
23. B. An accumulation of blood from the kidney into the abdomen would
manifest itself with these symptoms
24. D. Fluids liquefy secretions and therefore make it easier to expectorate
25. D. Physical assessment guidelines recommend listening for atleast 2
minutes in each quadrant (and up to 5 minutes, not at least 5 minutes).
26. A. Compression stockings promote venous return and prevent
peripheral pooling.
27. A. Placing food in the unaffected side of the mouth assists in the
swallowing process because the client has sensation on that side and will
have more control over the swallowing process.
28. D. Daily weights are taken following nephrectomy. Daily increases of 2
lb or more are indicative of fluid retention and should be reported to the
physician. Intake and output records may also reflect this imbalance.
29. A. Clients with cirrhosis have already coagulation due to
thrombocytopenia and vitamin K deficiency. This could be a sign of
bleeding
30. B. Exercise enhances glucose uptake, and the client is at risk for an
insulin reaction. Snacks with carbohydrates will help.
31. C. These are symptoms of diabetes insipidus. The patient can become
hypovolemic and vasopressin may reverse the Polyuria.
32. D. Peripheral neuropathy refers to nerve damage of the hands and feet.
The client did not notice that the object pierced the skin.
33. A. The elevation of the stump on a pillow for the first 24 hours
decreases edema and increases venous return.
34. B. The priority is to maintain client’s safety. With syncope and vertigo,
the client is at high risk for falling.
35. D. Coughing and deep breathing are essential for re-expansion of the
lung
36. B. Avoiding stasis of urine by emptying the bladder every 2-4 hours will
prevent overdistention of the bladder and future urinary tract infections.
37. D. If infected, the sex partner must be evaluated and treated
38. A. A 1:10 solution of household bleach and water is recommended by
the Centers for Disease Control and Prevention to kill the human
immunodeficiency virus (HIV).
39. D. Given the data, presurgical anxiety is suspected. The client needs an
opportunity to talk about concerns related to surgery before further
actions (which may mask the anxiety).
40. C. A quick look at the client can help identify the type and cause of the
ventilator alarm. Disconnection of the tube from the ventilator,
bronchospasm, and anxiety are some of the obvious reasons that could
trigger an alarm.
41. A. Contact isolation has been advised by the Centers for Disease
Control and Prevention (CDC) to control transmission of MRSA, which
includes gloves and handwashing.
42. D. In response to the question of the client, the nurse needs to provide
brief, accurate information. Some clients who have had gastrectomies are
able to tolerate three meals a day before discharge from the hospital.
However, for the majority of clients, it takes 6-12 months before their
surgically reduced stomach has stretched enough to accommodate a
larger meal.
43. A. This allows the client to elaborate his concern and provides the nurse
a baseline of assessment
44. C. Use of eye patches may be continued postoperatively, depending on
surgeon preference. This is done to achieve >90% success rate of the
surgery.
45. B. Assessment and more data collection are needed. The client may
have gastrointestinal or neurological problems that account for the
symptoms. The anorexia could result from medications, poor dentition, or
indigestion, the bruises may be attributed to ataxia, frequent falls, vertigo,
or medication.
46. A. The catheter should be washed with soap and water after withdrawal
and placed in a clean container. It can be reused until it is too hard or too
soft for insertion. Self-care, prevention of complications, and cost-
effectiveness are important in home management.
47. D. The client may have herpes zoster (shingles), a viral infection. The
nurse should use standard precautions in assessing the lesions.
Immunocompromised clients are at risk for infection.
48. B. After a minimum of 24 hours of IV antibiotics, the client is no longer
considered communicable. Evaluation of the nurse’s knowledge is needed
for safe care and continuity of care.
49. D. Stem of the question supports this choice by stating that the client
has difficulty accepting the potential disability.
50. B. Toxoplasmosis is an opportunistic infection and a parasite of birds
and mammals. The oocysts remain infectious in moist soil for about 1
year.
PNLE IV Nursing Practice
The scope of this Nursing Test IV is parallel to the NP4 NLE Coverage:
 Medical Surgical Nursing
1. Following spinal injury, the nurse should encourage the client to drink fluids
to avoid:

A. Urinary tract infection.


B. Fluid and electrolyte imbalance.
C. Dehydration.
D. Skin breakdown.
2. The client is transferred from the operating room to recovery room after an
open-heart surgery. The nurse assigned is taking the vital signs of the client.
The nurse notified the physician when the temperature of the client rises to
38.8 ºC or 102 ºF because elevated temperatures:

A. May be a forerunner of hemorrhage.


B. Are related to diaphoresis and possible chilling.
C. May indicate cerebral edema.
D. Increase the cardiac output.
3. After radiation therapy for cancer of the prostate, the client experienced
irritation in the bladder. Which of the following sign of bladder irritability is
correct?
A. Hematuria
B. Dysuria
C. Polyuria
D. Dribbling
4. A client is diagnosed with a brain tumor in the occipital lobe. Which of the
following will the client most likely experience?

A. Visual hallucinations.
B. Receptive aphasia.
C. Hemiparesis.
D. Personality changes.
5. A client with Addison’s disease has a blood pressure of 65/60. The nurse
understands that decreased blood pressure of the client with Addison’s
disease involves a disturbance in the production of:

A. Androgens
B. Glucocorticoids
C. Mineralocorticoids
D. Estrogen
6. The nurse is planning to teach the client about a spontaneous
pneumothorax. The nurse would base the teaching on the understanding that:

A. Inspired air will move from the lung into the pleural space.
B. There is greater negative pressure within the chest cavity.
C. The heart and great vessels shift to the affected side.
D. The other lung will collapse if not treated immediately.
7. During an assessment, the nurse recognizes that the client has an
increased risk for developing cancer of the tongue. Which of the following
health history will be a concern?

A. Heavy consumption of alcohol.


B. Frequent gum chewing.
C. Nail biting.
D. Poor dental habits.
8. The client in the orthopedic unit asks the nurse the reason behind why
compact bone is stronger than cancellous bone. Which of the following is the
correct response of the nurse?
A. Compact bone is stronger than cancellous bone because of its greater
size.
B. Compact bone is stronger than cancellous bone because of its greater
weight.
C. Compact bone is stronger than cancellous bone because of its greater
volume.
D. Compact bone is stronger than cancellous bone because of its greater
density.
9. The nurse is reviewing the laboratory results of the client. In reviewing the
results of the RBC count, the nurse understands that the higher the red blood
cell count, the :

A. Greater the blood viscosity.


B. Higher the blood pH.
C. Less it contributes to immunity.
D. Lower the hematocrit.
10. The physician advised the client with Hemiparesis to use a cane. The
client asks the nurse why cane will be needed. The nurse explains to the client
that cane is advised specifically to:

A. Aid in controlling involuntary muscle movements.


B. Relieve pressure on weight-bearing joints.
C. Maintain balance and improve stability.
D. Prevent further injury to weakened muscles.
11. The nurse is conducting a discharge teaching regarding the prevention of
further problems to a client who undergone surgery for carpal tunnel
syndrome of the right hand. Which of the following instruction will the nurse
includes?

A. Learn to type using your left hand only.


B. Avoid typing in a long period of time.
C. Avoid carrying heavy things using the right hand.
D. Do manual stretching exercise during breaks.
12. A female client is admitted because of recurrent urinary tract infections.
The client asks the nurse why she is prone to this disease. The nurse states
that the client is most susceptible because of:
A. Continuity of the mucous membrane.
B. Inadequate fluid intake.
C. The length of the urethra.
D. Poor hygienic practices.
13. A 55-year-old client is admitted with chest pain that radiates to the neck,
jaw and shoulders that occurs at rest, with high body temperature, weak with
generalized sweating and with decreased blood pressure. A myocardial
infarction is diagnosed. The nurse knows that the most accurate explanation
for one of these presenting adaptations is:

A. Catecholamines released at the site of the infarction causes


intermittent localized pain.
B. Parasympathetic reflexes from the infarcted myocardium causes
diaphoresis.
C. Constriction of central and peripheral blood vessels causes a decrease
in blood pressure.
D. Inflammation in the myocardium causes a rise in the systemic body
temperature.
14. Following an amputation of a lower limb to a male client, the nurse
provides an instruction on how to prevent a hip flexion contracture. The nurse
should instruct the client to:.

A. Perform quadriceps muscle setting exercises twice a day.


B. Sit in a chair for 30 minutes three times a day.
C. Lie on the abdomen 30 minutes every four hours.
D. Turn from side to side every 2 hours.
15. The physician scheduled the client with rheumatoid arthritis for the
injection of hydrocortisone into the knee joint. The client asks the nurse why
there is a need for this injection. The nurse explains that the most important
reason for doing this is to:

A. Lubricate the joint.


B. Prevent ankylosis of the joint.
C. Reduce inflammation.
D. Provide physiotherapy.
16. The nurse is assigned to care for a 57-year-old female client who had a
cataract surgery an hour ago. The nurse should:

A. Advise the client to refrain from vigorous brushing of teeth and hair.
B. Instruct the client to avoid driving for 2 weeks.
C. Encourage eye exercises to strengthen the ocular musculature.
D. Teach the client coughing and deep-breathing techniques.
17. A client with AIDS develops bacterial pneumonia is admitted in the
emergency department. The client’s arterial blood gases is drawn and the
result is PaO2 80mmHg. then arterial blood gases are drawn again and the
level is reduced from 80 mmHg to 65 mmHg. The nurse should;

A. Have arterial blood gases performed again to check for accuracy.


B. Increase the oxygen flow rate.
C. Notify the physician.
D. Decrease the tension of oxygen in the plasma.
18. An 18-year-old college student is brought to the emergency department
due to serious motor vehicle accident. Right above-knee-amputation is done.
Upon awakening from surgery the client tells the nurse, “What happened to
me? I cannot remember anything?” Which of the following would be the
appropriate initial nursing response?

A. “You sound concerned; You’ll probably remember more as you wake


up.”
B. “Tell me what you think happened.”
C. “You were in a car accident this morning.”
D. “An amputation of your right leg was necessary because of an
accident.”
19. A 38-year-old client with severe hypertension is hospitalized. The
physician prescribed a Captopril (Capoten) and Alprazolam (Xanax) for
treatment. The client tells the nurse that there is something wrong with the
medication and nursing care. The nurse recognizes this behavior is probably a
manifestation of the client’s:

A. Reaction to hypertensive medications.


B. Denial of illness.
C. Response to cerebral anoxia.
D. Fear of the health problem.
20. Before discharge, the nurse scheduled the client who had a colostomy for
colorectal cancer for discharge instruction about resuming activities. The
nurse should plan to help the client understands that:

A. After surgery, changes in activities must be made to accommodate for


the physiologic changes caused by the operation.
B. Most sports activities, except for swimming, can be resumed based on
the client’s overall physical condition.
C. With counseling and medical guidance, a near normal lifestyle, including
complete sexual function is possible.
D. Activities of daily living should be resumed as quickly as possible to
avoid depression and further dependency.
21. A client is scheduled for bariatric surgery. Preoperative teaching is done.
Which of the following statement would alert the nurse that further teaching to
the client is necessary?

A. “I will be limiting my intake to 600 to 800 calories a day once I start


eating again.”
B. “I’m going to have a figure like a model in about a year.”
C. “I need to eat more high-protein foods.”
D. “I will be going to be out of bed and sitting in a chair the first day after
surgery.”.
22. The client who had transverse colostomy asks the nurse about the
possible effect of the surgery on future sexual relationship. What would be the
best nursing response?

A. The surgery will temporarily decrease the client’s sexual impulses.


B. Sexual relationships must be curtailed for several weeks.
C. The partner should be told about the surgery before any sexual activity.
D. The client will be able to resume normal sexual relationships.
23. A 75-year-old male client tells the nurse that his wife has osteoporosis and
asks what chances he had of getting also osteoporosis like his wife. Which of
the following is the correct response of the nurse?

A. “This is only a problem for women.”


B. “You are not at risk because of your small frame.”
C. “You might think about having a bone density test,”
D. “Exercise is a good way to prevent this problem.”
24. An older adult client with acute pain is admitted in the hospital. The nurse
understands that in managing acute pain of the client during the first 24 hours,
the nurse should ensure that:

A. Ordered PRN analgesics are administered on a scheduled basis.


B. Patient controlled analgesia is avoided in this population.
C. Pain medication is ordered via the intramuscular route.
D. An order for meperidine (Demerol) is secured for pain relief.
25. A nurse is caring to an older adult with presbycusis. In formulating nursing
care plan for this client, the nurse should expect that hearing loss of the client
that is caused by aging to have:

A. Overgrowth of the epithelial auditory lining.


B. Copious, moist cerumen.
C. Difficulty hearing women’s voices.
D. Tears in the tympanic membrane.
26. The nurse is reviewing the client’s chart about the ordered medication. The
nurse must observe for signs of hyperkalemia when administering:

A. Furosemide (Lasix)
B. Hydrochlorothiazide (HydroDIURIL)
C. Metolazone (Zaroxolyn)
D. Spironolactone (Aldactone)
27. The physician prescribed Albuterol (Proventil) to the client with severe
asthma. After the administration of the medication the nurse should monitor
the client for:

A. Palpitation
B. Visual disturbance
C. Decreased pulse rate
D. Lethargy
28. A client is receiving diltiazem (Cardizem). What should the nurse include in
a teaching plan aimed at reducing the side effects of this medication?

A. Take the drug with an antacid.


B. Lie down after meals.
C. Avoid dairy products in diet.
D. Change positions slowly.
29. A client is receiving simvastatin (Zocor). The nurse is aware that this
medication is effective when there is decrease in:

A. The triglycerides
B. The INR
C. Chest pain
D. Blood pressure
30. A client is taking nitroglycerine tablets, the nurse should teach the client
the importance of:

A. Increasing the number of tablets if dizziness or hypertension occurs.


B. Limiting the number of tablets to 4 per day.
C. Making certain the medication is stored in a dark container.
D. Discontinuing the medication if a headache develops.
31. The physician prescribes Ibuprofen (Motrin) and hydroxychloroquine
sulfate (Plaquenil) for a 58-year-old male client with arthritis. The nurse
provides information about toxicity of the hydroxychloroquine. The nurse can
determine if the information is clearly understood if the client states:

A. “I will contact the physician immediately if I develop blurred vision.”


B. “I will contact the physician immediately if I develop urinary retention.”
C. “I will contact the physician immediately if I develop swallowing
difficulty.”
D. “I will contact the physician immediately if I develop feelings of
irritability.”
32. The client with an acute myocardial infarction is hospitalized for almost
one week. The client experiences nausea and loss of appetite. The nurse
caring for the client recognizes that these symptoms may indicate the:

A. Adverse effects of spironolactone (Aldactone)


B. Adverse effects of digoxin (Lanoxin)
C. Therapeutic effects of propranolol (Indiral)
D. Therapeutic effects of furosemide (Lasix)
33. A client with a partial occlusion of the left common carotid artery is
scheduled for discharge. The client is still receiving Coumadin. The nurse
provided a discharge instruction to the client regarding adverse effects of
Coumadin. The nurse should tell the client to consult with the physician if:

A. Swelling of the ankles increases.


B. Blood appears in the urine.
C. Increased transient Ischemic attacks occur.
D. The ability to concentrate diminishes.
34. Levodopa is ordered for a client with Parkinson’s disease. Before starting
the medication, the nurse should know that:

A. Levodopa is inadequately absorbed if given with meals.


B. Levodopa may cause the side effects of orthostatic hypotension.
C. Levodopa must be monitored by weekly laboratory tests.
D. Levodopa causes an initial euphoria followed by depression.
35. In making a diagnosis of myasthenia gravis Edrophonium HCI (Tensilon) is
used. The nurse knows that this drug will cause a temporary increase in:

A. Muscle strength
B. Symptoms
C. Blood pressure
D. Consciousness
36. The nurse can determine the effectiveness of carbamazepine (Tegretol) in
the management of trigeminal neuralgia by monitoring the client’s:

A. Seizure activity
B. Liver function
C. Cardiac output
D. Pain relief
37. Administration of potassium iodide solution is ordered to the client who
will undergo a subtotal thyroidectomy. The nurse understands that this
medication is given to:

A. Ablate the cells of the thyroid gland that produce T4.


B. Decrease the total basal metabolic rate.
C. Decrease the size and vascularity of the thyroid.
D. Maintain function of the parathyroid gland.
38. A client with Addison’s disease is scheduled for discharge. Before the
discharge, the physician prescribes hydrocortisone and fludrocortisone. The
nurse expects the hydrocortisone to:

A. Increase amounts of angiotensin II to raise the client’s blood pressure.


B. Control excessive loss of potassium salts.
C. Prevent hypoglycemia and permit the client to respond to stress.
D. Decrease cardiac dysrhythmias and dyspnea.
39. A client with diabetes insipidus is taking Desmopressin acetate (DDAVP).
To determine if the drug is effective, the nurse should monitor the client’s:

A. Arterial blood pH
B. Pulse rate
C. Serum glucose
D. Intake and output
40. A client with recurrent urinary tract infections is to be discharged. The
client will be taking nitrofurantoin (Macrobid) 50 mg po every evening at
home. The nurse provides discharge instructions to the client. Which of the
following instructions will be correct?

A. Strain urine for crystals and stones


B. Increase fluid intake.
C. Stop the drug if the urinary output increases
D. Maintain the exact time schedule for drug taking.
41. A client with cancer of the lung is receiving chemotherapy. The physician
orders antibiotic therapy for the client. The nurse understands that
chemotherapy destroys rapidly growing leukocytes in the:

A. Bone marrow
B. Liver
C. Lymph nodes
D. Blood
42. The physician reduced the client’s Dexamethasone (Decadron) dosage
gradually and to continue a lower maintenance dosage. The client asks the
nurse about the change of dosage. The nurse explains to the client that the
purpose of gradual dosage reduction is to allow:
A. Return of cortisone production by the adrenal glands.
B. Production of antibodies by the immune system
C. Building of glycogen and protein stores in liver and muscle
D. Time to observe for return of increases intracranial pressure
43. The nurse is assigned to care for a client with diarrhea. Excessive fluid
loss is expected. The nurse is aware that fluid deficit can most accurately be
assessed by:

A. The presence of dry skin


B. A change in body weight
C. An altered general appearance
D. A decrease in blood pressure
44. Which of the following is the most important electrolyte of intracellular
fluid?

A. Potassium
B. Sodium
C. Chloride
D. Calcium
45. Which of the following client has a high risk for developing hyperkalemia?

A. Crohn’s disease
B. End-Stage renal disease
C. Cushing’s syndrome
D. Chronic heart failure
46. The nurse is reviewing the laboratory result of the client. The client’s
serum potassium level is 5.8 mEq/L. Which of the following is the initial
nursing action?

A. Call the cardiac arrest team to alert them


B. Call the laboratory and repeat the test
C. Take the client’s vital signs and notify the physician
D. Obtain an ECG strip and have lidocaine available
47. Potassium chloride, 20 mEq, is ordered and to be added in the IV solution
of a client in a diabetic ketoacidosis. The primary reason for administering
this drug is:
A. Replacement of excessive losses
B. Treatment of hyperpnea
C. Prevention of flaccid paralysis
D. Treatment of cardiac dysrhythmias
48. A female client is brought to the emergency unit. The client is complaining
of abdominal cramps. On assessment, client is experiencing anorexia and
weight is reduced. The physician’s diagnosis is colitis. Which of the following
symptoms of fluid and electrolyte imbalance should the nurse report
immediately?

A. Skin rash, diarrhea, and diplopia


B. Development of tetaniy with muscles spasms
C. Extreme muscle weakness and tachycardia
D. Nausea, vomiting, and leg and stomach cramps.
49. The client is to receive an IV piggyback medication. When preparing the
medication the nurse should be aware that it is very important to:

A. Use strict sterile technique


B. Use exactly 100mL of fluid to mix the medication
C. Change the needle just before adding the medication
D. Rotate the bag after adding the medication
50. The nurse is reviewing the laboratory result of the client. An arterial blood
gas report indicates the client’s pH is 7.20, PCO2 35 mmHg and HCO3 is 19
mEq/L. The results are consistent with:

A. Metabolic acidosis
B. Metabolic alkalosis
C. Respiratory acidosis
D. Respiratory alkalosis
Answers and Rationales
1. A. Clients in the early stage of spinal cord damage experience an atonic
bladder, which is characterized by the absence of muscle tone, an
enlarged capacity, no feeling of discomfort with distention, and overflow
with a large residual. This leads to urinary stasis and infection. High fluid
intake limits urinary stasis and infection by diluting the urine and
increasing urinary output.
2. D. The temperature of 102 ºF (38.8ºC) or greater lead to an increased
metabolism and cardiac workload.
3. B. Dysuria, nocturia, and urgency are all signs an irritable bladder after
radiation therapy.
4. A. The occipital lobe is involve with visual interpretation.
5. C. Mineralocorticoids such as aldosterone cause the kidneys to retain
sodium ions. With sodium, water is also retained, elevating blood
pressure. Absence of this hormone thus causes hypotension.
6. B. As a person with a tear in the lung inhales, air moves through that
opening into the intrapleural and causes partial or complete collapse of
the lungs.
7. A. Heavy alcohol ingestion predisposes an individual to the
development of oral cancer.
8. D. The greater the density of compact bone makes it stronger than the
cancellous bone. Compact bone forms from cancellous bone by the
addition of concentric rings of bones substances to the marrow spaces of
cancellous bone. The large marrow spaces are reduced to haversian
canals.
9. A. Viscosity, a measure of a fluid’s internal resistance to flow, is
increased as the number of red cells suspended in plasma.
10. C. Hemiparesis creates instability. Using a cane provides a wider base
of support and, therefore greater stability.
11. D. Manual stretching exercises will assist in keeping the muscles and
tendons supple and pliable, reducing the traumatic consequences of
repetitive activity.
12. C. The length of the urethra is shorter in females than in males;
therefore microorganisms have a shorter distance to travel to reach the
bladder. The proximity of the meatus to the anus in females also
increases this incidence.
13. D. Temperature may increase within the first 24 hours and persist as
long as a week.
14. C. The hips are in extension when the client is prone; this keeps the hips
from flexing.
15. C. Steroids have an anti-inflammatory effect that can reduce arthritic
pannus formation.
16. A. Activities such as rigorous brushing of hair and teeth cause
increased intraocular pressure and may lead to hemorrhage in the anterior
chamber.
17. C. This decrease in PaO2 indicates respiratory failure; it warrants
immediate medical evaluation.
18. C. This is truthful and provides basic information that may prompt
recollection of what happened; it is a starting point.
19. D. Clients adapting to illness frequently feel afraid and helpless and
strike out at health team members as a way of maintaining control or
denying their fear.
20. C. There are few physical restraints on activity postoperatively, but the
client may have emotional problems resulting from the body image
changes.
21. B. Clients need to be prepared emotionally for the body image changes
that occur after bariatric surgery. Clients generally experience excessive
abdominal skin folds after weight stabilizes, which may require a
panniculectomy. Body image disturbance often occurs in response to
incorrectly estimating one’s size; it is not uncommon for the client to still
feel fat no matter how much weight is lost.
22. D. Surgery on the bowel has no direct anatomic or physiologic effect on
sexual performance. However, the nurse should encourage verbalization.
23. C. Osteoporosis is not restricted to women; it is a potential major health
problem of all older adults; estimates indicate that half of all women have
at least one osteoporitic fracture and the risk in men is estimated between
13% and 25%; a bone mineral density measurement assesses the mass of
bone per unit volume or how tightly the bone is packed.
24. A. Around-the-clock administration of analgesics is recommended for
acute pain in the older adult population; this help to maintain a therapeutic
blood level of pain medication.
25. C. Generally, female voices have a higher pitch than male voices; older
adults with presbycusis (hearing loss caused by the aging process) have
more difficulty hearing higher-pitched sounds.
26. D. Aldactone is a potassium-sparing diuretic; hyperkalemia is an
adverse effect.
27. A. Albuterol’s sympathomimetic effect causes cardiac stimulation that
may cause tachycardia and palpitation.
28. D. Changing positions slowly will help prevent the side effect of
orthostatic hypotension.
29. A. Therapeutic effects of simvastatin include decreased serum
triglyceries, LDL and cholesterol.
30. C. Nitroglycerine is sensitive to light and moisture ad must be stored in
a dark, airtight container.
31. A. Visual disturbance are a sign of toxicity because retinopathy can
occur with this drug.
32. B. Toxic levels of Lanoxin stimulate the medullary chemoreceptor
trigger zone, resulting in nausea and subsequent anorexia.
33. B. Warfarin derivatives cause an increase in the prothrombin time and
INR, leading to an increased risk for bleeding. Any abnormal or excessive
bleeding must be reported, because it may indicate toxic levels of the
drug.
34. B. Levodopa is the metabolic precursor of dopamine. It reduces
sympathetic outflow by limiting vasoconstriction, which may result in
orthostatic hypotension.
35. A. Tensilon, an anticholinesterase drug, causes temporary relief of
symptoms of myasthenia gravis in client who have the disease and is
therefore an effective diagnostic aid.
36. D. Carbamazepine ( Tegretol) is administered to control pain by
reducing the transmission of nerve impulses in clients with trigeminal
neuralgia.
37. C. Potassium iodide, which aids in decreasing the vascularity of the
thyroid gland, decreases the risk for hemorrhage.
38. C. Hydrocortisone is a glucocorticoid that has anti-inflammatory action
and aids in metabolism of carbohydrate, fat, and protein, causing elevation
of blood glucose. Thus it enables the body to adapt to stress.
39. D. DDAVP replaces the ADH, facilitating reabsorption of water and
consequent return of normal urine output and thirst.
40. B. To prevent crystal formation, the client should have sufficient intake
to produce 1000 to 1500 mL of urine daily while taking this drug.
41. A. Prolonged chemotherapy may slow the production of leukocytes in
bone marrow, thus suppressing the activity of the immune system.
Antibiotics may be required to help counter infections that the body can
no longer handle easily.
42. A. Any hormone normally produced by the body must be withdrawn
slowly to allow the appropriate organ to adjust and resume production.
43. B. Dehydration is most readily and accurately measured by serial
assessment of body weight; 1 L of fluid weighs 2.2 pounds.
44. A. The concentration of potassium is greater inside the cell and is
important in establishing a membrane potential, a critical factor in the
cell’s ability to function.
45. B. The kidneys normally eliminate potassium from the body;
hyperkalemia may necessitate dialysis.
46. C. Vital signs monitor cardiorespiratory status; hyperkalemia causes
serious cardiac dysrhythmias.
47. A. Once treatment with insulin for diabetic ketoacidosis is begun,
potassium ions reenter the cell, causing hypokalemia; therefore
potassium, along with the replacement fluid, is generally supplied.
48. C. Potassium, the major intracellular cation, functions with sodium and
calcium to regulate neuromuscular activity and contraction of muscle
fibers, particularly the heart muscle. In hypokalemia these symptoms
develop.
49. A. Because IV solutions enter the body’s internal environment, all
solutions and medications utilizing this route must be sterile to prevent
the introduction of microbes.
50. A. A low pH and bicarbonate level are consistent with metabolic
acidosis.

PNLE V Nursing Practice


The scope of this Nursing Test V is parallel to the NP5 NLE Coverage:
 Psychiatric Nursing
1. A 17-year-old client has a record of being absent in the class without
permission, and “borrowing” other people’s things without asking permission.
The client denies stealing; rationalizing instead that as long as no one was
using the items, there is no problem to use it by other people. It is important
for the nurse to understand that psychodynamically, the behavior of the client
may be largely attributed to a development defect related to the:

A. Oedipal complex
B. Superego
C. Id
D. Ego
2. A client tells the nurse, “Yesterday, I was planning to kill myself.” What is the
best nursing response to this cient?

A. “What are you going to do this time?”


B. Say nothing. Wait for the client’s next comment
C. “You seem upset. I am going to be here with you; perhaps you will want
to talk about it”
D. “Have you felt this way before?”
3. In crisis intervention therapy, which of the following principle that the nurse
will use to plan her/his goals?

A. Crises are related to deep, underlying problems


B. Crises seldom occur in normal people’s lives
C. Crises may go on indefinitely.
D. Crises usually resolved in 4-6 weeks.
4. The nurse enters the room of the male client and found out that the client
urinates on the floor. The client hides when the nurse is about to talk to him.
Which of the following is the best nursing intervention?

A. Place restriction on the client’s activities when his behavior occurs.


B. Ask the client to clean the soiled floor.
C. Take the client to the bathroom at regular intervals.
D. Limit fluid intake.
5. A young lady with a diagnosis of schizophrenic reaction is admitted to the
psychiatric unit. In the past two months, the client has poor appetite,
experienced difficulty in sleeping, was mute for long periods of time, just
stayed in her room, grinning and pointing at things. What would be the initial
nursing action on admitting the client to the unit?

A. Assure the client that “ You will be well cared for.”


B. Introduce the client to some of the other clients.
C. Ask “Do you know where you are?”
D. Take the client to the assigned room.
6. A 16-year-old girl was diagnosed with anorexia. What would be the first
assessment of the nurse?

A. What food she likes.


B. Her desired weight.
C. Her body image.
D. What causes her behavior.
7. On an adolescent unit, a nurse caring to a client was informed that her
client’s closest roommate dies at night. What would be the most appropriate
nursing action?

A. Do not bring it up unless the client asks.


B. Tell the client that her roommate went home.
C. Tell the client, if asked, “You should ask the doctor.”
D. Tell the client that her closest roommate died.
8. A woman gave birth to an unhealthy infant, and with some body defects.
The nurse should expect the woman’s initial reactions to include:

A. Depression
B. Withdrawal
C. Apathy
D. Anger
9. A client in the psychiatric unit is shouting out loud and tells the nurse,
“Please, help me. They are coming to get me.” What would be the appropriate
nursing response?

A. “ I won’t let anyone get you.”


B. “Who are they?”
C. “I don’t see anyone coming.”
D. “You look frightened.”
10. A client who is severely obese tells the nurse, “My therapist told me that I
eat a lot because I didn’t get any attention and love from my mother. What
does the therapist mean?” What is the best nursing response?

A. “What do you think is the connection between your not getting enough
love and overeating?”
B. “Tell me what you think the therapist means.”
C. “You need to ask your therapist.”
D. “ We are here to deal with your diet, not with your psychological
problems.”
11. After the discussion about the procedure the physician scheduled the
client for mastectomy. The client tells the nurse, “If my breasts will be
removed, I’m afraid my husband will not love me anymore and maybe he will
never touch me.” What should the nurse’s response?

A. “I doubt that he feels that way.”


B. “What makes you feel that way?”
C. “Have you discussed your feelings with your husband?”
D. Ask the husband, in front of the wife, how he feels about this.
12. The child is brought to the hospital by the parents. During assessment of
the nurse, what parental behavior toward a child should alert the nurse to
suspect child abuse?

A. Ignoring the child.


B. Flat affect.
C. Expressions of guilt.
D. Acting overly solicitous toward the child
13. A nurse is caring to a client with manic disorder in the psychiatric ward. On
the morning shift, the nurse is talking with the client who is now exhibiting a
manic episode with flight of ideas. The nurse primarily needs to:

A. Focus on the feelings conveyed rather than the thoughts expressed.


B. Speak loudly and rapidly to keep the client’s attention, because the
client is easily distracted.
C. Allow the client to talk freely.
D. Encourage the client to complete one thought at a time.
14. The nurse is caring to an autistic child. Which of the following play
behavior would the nurse expect to see in a child?

A. competitive play
B. nonverbal play
C. cooperative play
D. solitary play
15. The client is telling the nurse in the psychiatric ward, “I hate them.” Which
of the following is the most appropriate nursing response to the client?

A. “Tell me about your hate.”


B. “I will stay with you as long as you feel this way.”
C. “For whom do you have these feelings?”
D. “I understand how you can feel this way.”
16. The mother visits her son with major depression in the psychiatric unit.
After the conversation of the client and the mother, the nurse asks the mother
how it is talking to her son. The mother tells the nurse that it was a stressful
time. During an interview with the client, the client says, “we had a marvelous
visit.” Which of the following coping mechanism can be described to
thestatement of the client?

A. Identification.
B. Rationalization.
C. Denial.
D. Compensation.
17. A male client is quiet when the physician told him that he has stage IV
cancer and has 4 months to live. The nurse determines that this reaction may
be an example of:

A. Indifference
B. Denial
C. Resignation
D. Anger
18. A nurse is caring to a female client with five young children. The family
member told the client that her ex-husband has died 2 days ago. The reaction
of the client is stunned silence, followed by anger that the ex-husband left no
insurance money for their young children. The nurse should understand that:
A. The children and the injustice done to them by their father’s death are
the woman’s main concern.
B. To explain the woman’s reaction, the nurse needs more information
about the relationship and breakup.
C. The woman is not reacting normally to the news.
D. The woman is experiencing a normal bereavement reaction.
19. A client who is manic comes to the outpatient department. The nurse is
assigning an activity for the client. What activity is best for the nurse to
encourage for a client in a manic phase?

A. Solitary activity, such as walking with the nurse, to decrease stimulation.


B. Competitive activity, such as bingo, to increase the client’s self-esteem.
C. Group activity, such as basketball, to decrease isolation.
D. Intellectual activity, such as scrabble, to increase concentration.
20. The nurse is about to administer Imipramine HCI (Tofranil) to the client,
the client says, “Why should I take this?” The doctor started me on this 10days
ago; it didn’t help me at all.” Which of the following is the best nursing
response:

A. “What were you expecting to happen?”


B. “It usually takes 2-3 weeks to be effective.”
C. “Do you want to refuse this medication? You have the right.”
D. “That’s a long time wait when you feel so depressed.”
21. Which of the following drugs the nurse should choose to administer to a
client to prevent pseudoparkinsonism?

A. Isocarboxazid (Marplan)
B. Chlorpromazine HCI (Thorazine)
C. Trihexyphenidyl HCI (Artane)
D. Trifluoperazine HCI (Stelazine)
22. The nurse is caring to an 80-year-old client with dementia? What is the
most important psychosocial need for this client?

A. Focus on the there-and-then rather the here-and-now.


B. Limit in the number of visitors, to minimize confusion.
C. Variety in their daily life, to decrease depression.
D. A structured environment, to minimize regressive behaviors.
23. A client tells the nurse, “I don’t want to eat any meals offered in this
hospital because the food is poisoned.” The nurse is aware that the client is
expressing an example of:

A. Delusion.
B. Hallucination.
C. Negativism.
D. Illusion.
24. A client is admitted in the hospital. On assessment, the nurse found out
that the client had several suicidal attempts. Which of the following is the
most important nursing action?

A. Ignore the client as long as he or she is talking about suicide, because


suicide attempt is unlikely.
B. Administer medication.
C. Relax vigilance when the client seems to be recovering from depression.
D. Maintain constant awareness of the client’s whereabouts.
25. The nurse suspects that the client is suffering from depression. During
assessment, what are the most characteristic signs and symptoms of
depression the nurse would note?

A. Constipation, increased appetite.


B. Anorexia, insomnia.
C. Diarrhea, anger.
D. Verbosity, increased social interaction.
26. The client in the psychiatric unit states that, “The goodas are coming! I
must be ready.” In response to this neologism, the nurse’s initial response is
to:

A. Acknowledge that the word has some special meaning for the client.
B. Try to interpret what the client means.
C. Divert the client’s attention to an aspect of reality.
D. State that what the client is saying has not been understood and then
divert attention to something that is really bound.
27. A male client diagnosed with depression tells the nurse, “I don’t want to
look weak and I don’t even cry because my wife and my kids can’t bear it.” The
nurse understands that this is an example of:
A. Repression.
B. Suppression.
C. Undoing.
D. Rationalization.
28. A female client tells the nurse that she is afraid to go out from her room
because she thinks that the other client might kill her. The nurse is aware that
this behavior is related to:

A. Hallucination.
B. Ideas of reference.
C. Delusion of persecution.
D. Illusion.
29. A female client is taking Imipramine HCI (Tofranil) for almost 1 week and
shows less awareness of the physical body. What problem would the nurse be
most concerned?

A. Nausea.
B. Gait disturbances.
C. Bowel movements.
D. Voiding.
30. A 6-year-old client dies in the nursing unit. The parents want to see the
child. What is the most appropriate nursing action?

A. Give the parents time alone with the body.


B. Ask the physician for permission.
C. Complete the postmortem care and quietly accompany the family to the
child’s room.
D. Suggest the parents to wait until the funeral service to say “good-bye.”
31. A 20-year-old female client is diagnosed with anxiety disorder. The
physician prescribed Flouxetine (Prozac). What is the most important side
effects should a nurse be concerned?

A. Tremor, drowsiness.
B. Seizures, suicidal tendencies.
C. Visual disturbance, headache.
D. Excessive diaphoresis, diarrhea.
32. A nurse is assigned to activate a client who is withdrawn, hears voices and
negativistic. What would be the best nursing approach?

A. Mention that the “voices” would want the client to participate.


B. Demand that the client must join a group activity.
C. Give the client a long explanation of the benefits of activity.
D. Tell the client that the nurse needs a partner for an activity.
33. A nurse is going to give a rectal suppository as a preoperative medication
to a 4-year-old boy. The boy is very anxious and frightened. Which of the
following statement by the nurse would be most appropriate to gain the
child’s cooperation?

A. “Be a big kid! Everyone’s waiting for you.”


B. “Lie still now and I’ll let you have one of your presents before you even
have your operation.”
C. “Take a nice, big, deep breath and then let me hear you count to five.”
D. “You look so scared. Want to know a secret? This won’t hurt a bit!”
34. A depressed client is on an MAO inhibitor? What should the nurse watch
out for?

A. Hypertensive crisis.
B. Diet restrictions.
C. Taking medication with meals.
D. Exposure to sunlight.
35. A 16-year-old girl is admitted for treatment of a fracture. The client shares
to the nurse caring to her that her step-father has made sexual advances to
her. She got the chance to tell it to her mother but refuses to believe. What is
the most therapeutic action of the nurse would be:

A. Tell the client to work it out with her father.


B. Tell the client to discuss it with her mother.
C. Ask the father about it.
D. Ask the mother what she thinks.
36. A client with a diagnosis of paranoid disorder is admitted in the psychiatric
hospital. The client tells the nurse, “the FBI is following me. These people are
plotting against me.” With this statement the nurse will need to:
A. Acknowledge that this is the client’s belief but not the nurse’s belief.
B. Ask how that makes the client feel.
C. Show the client that no one is behind.
D. Use logic to help the client doubt this belief.
37. A nurse is completing the routine physical examination to a healthy 16-
year-old male client. The client shares to the nurse that he feels like killing his
girlfriend because he found out that her girlfriend had another boyfriend. He
then laughs, and asks the nurse to keep this a secret just between the two of
them. The nurse reviews his chart and notes that there is no previously history
of violence or psychiatric illness. Which of the following would be the best
action of the nurse to take at this time?

A. Suggest the teen meet with a counselor to discuss his feelings about
his girlfriend.
B. Tell the teen that his feelings are normal, and recommend that he find
another girlfriend to take his mind off the problem.
C. Recall the teenage boys often say things they really do not mean and
ignore the comment.
D. Regard the comment seriously and notify the teen’s primary health care
provider and parents
38. Which of the following person will be at highest risk for suicide?

A. A student at exam time


B. A married woman, age 40, with 6 children.
C. A person who is an alcoholic.
D. A person who made a previous suicide attempt.
39. A male client is repetitively doing the handwashing every time he touches
things. It is important for a nurse to understand that the client’s behavior is
probably an attempt to:

A. Seek attention from the staff.


B. Control unacceptable impulses or feelings.
C. Do what the voices the patient hears tell him or her to do.
D. Punish himself or herself for guilt feeling.
40. In a mental health settings, the basic goal of nursing is to:
A. Advance the science of psychiatry by initiating research and gathering
data for current statistics on emotional illness.
B. Plan activity programs for clients.
C. Understand various types of family therapy and psychological tests and
how to interpret them.
D. Maintain a therapeutic environment.
41. A 3-year-old boy is brought to the emergency department. After an hour,
the boy dies of respiratory failure. The mother of the boy becomes upset,
shouting and abusive, saying to the nurse, “If it had been your son, they would
have done more to save it. “What should the nurse say or do?

A. Touch her and tell her exactly what was done for her baby.
B. Allow the mother to continue her present behavior while sitting quietly
with her.
C. “No, all clients are given the same good care.”
D. “Yes, you’re probably right. Your son did not get better care.”
42. The nurse is interacting to a client with an antisocial personality disorder.
What would be the most therapeutic approach of the nurse to an antisocial
behavior?

A. Gratify the client’s inner needs.


B. Give the client opportunities to test reality.
C. Provide external controls.
D. Reinforce the client’s self-concept.
43. A 55-year-old male client tells the nurse that he needs his glasses and
hearing aid with him in the recovery room after the surgery, or he will be upset
for not granting his request. What is the appropriate nursing response?

A. “Do you get upset and confused often?”


B. “You won’t need your glasses or hearing aid. The nurses will take care of
you.”
C. “I understand. You will be able to cooperate best if you know what is
going on, so I will find out how I can arrange to have your glasses and
hearing aid available to you in the recovery room.”
D. I understand you might be more cooperative if you have your aid and
glasses, but that is just not possible. Rules, you know.”
44. The male client had fight with his roommates in the psychiatric unit. The
client agitated client is placed in isolation for seclusion. The nurse knows it is
essential that:

A. A staff member has frequent contacts with the client.


B. Restraints are applied.
C. The client is allowed to come out after 4 hours.
D. All the furniture is removed form the isolation room.
45. A medical representative comes to the hospital unit for the promotion of a
new product. A female client, admitted for hysterical behavior, is found
embracing him. What should the nurse say?

A. “Have you considered birth control?”


B. “This isn’t the purpose of either of you being here.”
C. “I see you’ve made a new friend.”
D. “Think about what you are doing.”
46. A client with dementia is for discharge. The nurse is providing a discharge
instruction to the family member regarding safety measures at home. What
suggestion can the nurse make to the family members?

A. Avoid stairs without banisters.


B. Use restraints while the client is in bed to keep him or her from
wandering off during the night.
C. Use restraints while the client is sitting in a chair to keep him or her from
wandering off during the day.
D. Provide a night-light and a big clock.
47. A 30-year-old married woman comes to the hospital for treatment of
fractures. The woman tells the nurse that she was physically abused by her
husband. The woman receives a call from her husband telling her to get home
and things will be different. He felt sorry of what he did. What can the nurse
advise her?

A. “Do you think so?”


B. “It’s not likely.”
C. “What will be different?”
D. “I hope so, for your sake.”
48. A female client was diagnosed with breast cancer. It is found to be stage
IV, and a modified mastectomy is performed. After the procedure, what
behaviors could the nurse expects the client to display?

A. Denial of the possibility of carcinoma.


B. Signs of grief reaction.
C. Relief that the operation is over.
D. Signs of deep depression.
49. A client is withdrawn and does not want to interact to anybody even to the
nurse. What is the best initial nursing approach to encourage communication
with this client?

A. Use simple questions that call for a response.


B. Encourage discussion of feelings.
C. Look through a photo album together.
D. Bring up neutral topics.
50. Which of the following nursing approach is most important in a client with
depression?

A. Deemphasizing preoccupation with elimination, nourishment, and sleep.


B. Protecting against harm to others.
C. Providing motor outlets for aggressive, hostile feelings.
D. Reducing interpersonal contacts.
Answers and Rationales
1. B. This shows a weak sense of moral consciousness. According to
Freudian theory, personality disorders stem from a weak superego.
2. C. The client needs to have his or her feelings acknowledged, with
encouragement to discuss feelings, and be reassured about the nurse’s
presence.
3. D. Part of the definition of a crisis is a time span of 4-6 weeks.
4. C. The client is most likely confused, rather than exhibiting acting-out,
hostile behavior. Frequent toileting will allow urination in an appropriate
place.
5. D. The client needs basic, simple orientation that directly relates to the
here-and-now, and does not require verbal interaction.
6. A. Although all options may appear correct. A is the best because it
focuses on a range of possible positive reinforcers, a basis for an
effective behavior modification program. It can lead to concrete, specific
nursing interventions right away and provides a therapeutic use of
“control” for the 16-year-old.
7. A. The nurse needs to wait and see: do not “jump the gun”; do not
assume that the client wants to know now.
8. D. The woman is experiencing an actual loss and will probably exhibit
many of the same symptoms as a person who has lost someone to death.
9. C. This option is an example of pointing out reality- the nurse’s
perception.
10. B. This response asks information that the nurse can use. If the client
understands the statement, the nurse can support the therapist when
focusing on connection between food, love, and mother. If the client does
not understand thestatement, the nurse can help get clarification from the
therapist.
11. C. This option redirects the client to talk to her husband.
12. D. This is an example of reaction formation, a coping mechanism.
13. A. Often the verbalized ideas are jumbled, but the underlying feelings
are discernible and must be acknowledged.
14. D. Autistic children do best with solitary play because they typically do
not interact with others in a socially comprehensible and acceptable way.
15. A. The nurse is asking the client to clarify and further discuss feelings.
16. C. Denial is the act of avoiding disagreeable realities by ignoring them.
17. B. Reactions when told of a life-threatening illness stem from Kübler-
Ross’ ideas on death and dying. Denial is a typical grief response, and
usually is a first reaction.
18. D. Shock and anger are commonly the primary initial reactions.
19. A. This option avoids external stimuli, yet channels the excess motor
activity that is often part of the manic phase.
20. B. The patient needs a brief, factual answer.
21. C. Trihexyphenidyl HCI (Artane) is often used to counteract side effect
of pseudoparkinsonism, which often accompanies the use of
phenothiazine, such as chlorpromazine HCI (Thorazine or Trifluoperazine
HCI (Stelazine).
22. D. Persons with dementia needs sameness, consistency, structure,
routine, and predictability.
23. A. This is a false belief developed in response to an emotional need.
24. D. The client must be constantly observed.
25. B. The appetite is diminished and sleeping is affected to a client with
depression.
26. A. It is important to acknowledge a statement, even if it is not
understood.
27. D. Rationalization is the process of constructing plausible reasons for
one’s responses.
28. C. The client has ideas that someone is out to kill her.
29. D. A serious side effect of Imipramine HCI (Tofranil) is urinary retention
(voiding problems)
30. A. This allows the parents/family to grieve over the loss of the child, by
going through the steps of leave taking.
31. B. Assess for suicidal tendencies, especially during early therapy. There
is an increased risk of seizures in debilitated client and those with a
history of seizures.
32. D. The nurse helps to activate by doing something with the client.
33. C. Preschool children commonly experience fears and fantasies
regarding invasive procedures. The nurse should attempts to momentarily
distract the child with a simple task that can be easily accomplished while
the child remains in the side-lying position. The suppository can be slipped
into place while the child is counting, and then the nurse can praise the
child for cooperating, while holding the buttocks together to prevent
expulsion of the suppository.
34. A. This is the more inclusive answer, although diet restrictions
(answer1) are important, their purpose is to prevent hypertensive crisis
(answer 2).
35. D. This comes closest to beginning to focus on family-centered
approach to intervene in the “conspiracy of silence”. This is therefore the
best among the options.
36. A. The nurse should neither challenge nor use logic to dispel an
irrational belief.
37. D. Any threat to the safety of oneself or other should always be taken
seriously and never disregarded by the nurse.
38. C. The likelihood of multiple contributing factors may make this person
at higher risk for suicide. Some factors that may exist are physical illness
related to alcoholism, emotional factors ( anxiety, guilt, remorse), social
isolation due to impaired relationships and economic problems related to
employment.
39. B. A ritual, such as compulsive handwashing, is an attempt to allay
anxiety caused by unconscious impulses that are frightening.
40. D. This is the most neutral answer by process of elimination.
41. B. This option allows a normal grief response (anger).
42. C. Personality disorders stem from a weak superego, implying a lack of
adequate controls.
43. C. The client will be easier to care for if he has his hearing aid and
glasses.
44. A. Frequent contacts at times of stress are important, especially when a
client is isolated.
45. B. This response is aimed at redirecting the inappropriate behavior.
46. D. This option is best to decrease confusion and disorientation to place
and time.
47. C. This option helps the woman to think through and elaborate on her
own thoughts and prognosis.
48. B. It is mostly likely that grief would be expressed because of object
loss.
49. D. Neutral, nonthreatening topics are best in attempting to encourage a
response.
50. C. It is important to externalize the anger away from self.
PNLE I for Foundation of Nursing
1. Which element in the circular chain of infection can be eliminated
by preserving skin integrity?
A. Host
B. Reservoir
C. Mode of transmission
D. Portal of entry
2. Which of the following will probably result in a break in sterile technique
for respiratory isolation?
A. Opening the patient’s window to the outside environment
B. Turning on the patient’s room ventilator
C. Opening the door of the patient’s room leading into the hospital corridor
D. Failing to wear gloves when administering a bed bath
3. Which of the following patients is at greater risk for contracting
an infection?
A. A patient with leukopenia
B. A patient receiving broad-spectrum antibiotics
C. A postoperative patient who has undergone orthopedic surgery
D. A newly diagnosed diabetic patient
4. Effective hand washing requires the use of:
A. Soap or detergent to promote emulsification
B. Hot water to destroy bacteria
C. A disinfectant to increase surface tension
D. All of the above
5. After routine patient contact, hand washing should last at least:
A. 30 seconds
B. 1 minute
C. 2 minute
D. 3 minutes
6. Which of the following procedures always requires surgical asepsis?
A. Vaginal instillation of conjugated estrogen
B. Urinary catheterization
C. Nasogastric tube insertion
D. Colostomy irrigation
7. Sterile technique is used whenever:
A. Strict isolation is required
B. Terminal disinfection is performed
C. Invasive procedures are performed
D. Protective isolation is necessary
8. Which of the following constitutes a break in sterile technique
while preparing a sterile field for a dressing change?
A. Using sterile forceps, rather than sterile gloves, to handle a sterile item
B. Touching the outside wrapper of sterilized material without
sterile gloves
C. Placing a sterile object on the edge of the sterile field
D. Pouring out a small amount of solution (15 to 30 ml) before pouring the
solution into a sterile container
9. A natural body defense that plays an active role in preventing infection is:
A. Yawning
B. Body hair
C. Hiccupping
D. Rapid eye movements
10. All of the following statement are true about donning sterile gloves except:
A. The first glove should be picked up by grasping the inside of the cuff.
B. The second glove should be picked up by inserting the gloved fingers
under the cuff outside the glove.
C. The gloves should be adjusted by sliding the gloved fingers under the
sterile cuff and pulling the glove over the wrist
D. The inside of the glove is considered sterile
11.When removing a contaminated gown, the nurse should be careful that the
first thing she touches is the:
A. Waist tie and neck tie at the back of the gown
B. Waist tie in front of the gown
C. Cuffs of the gown
D. Inside of the gown
12.Which of the following nursing interventions is considered the
most effective form or universal precautions?
A. Cap all used needles before removing them from their syringes
B. Discard all used uncapped needles and syringes in an impenetrable
protective container
C. Wear gloves when administering IM injections
D. Follow enteric precautions
13.All of the following measures are recommended to prevent pressure
ulcers except:
A. Massaging the reddened are with lotion
B. Using a water or air mattress
C. Adhering to a schedule for positioning and turning
D. Providing meticulous skin care
14.Which of the following blood tests should be performed before a
blood transfusion?
A. Prothrombin and coagulation time
B. Blood typing and cross-matching
C. Bleeding and clotting time
D. Complete blood count (CBC) and electrolyte levels.
15.The primary purpose of a platelet count is to evaluate the:
A. Potential for clot formation
B. Potential for bleeding
C. Presence of an antigen-antibody response
D. Presence of cardiac enzymes
16.Which of the following white blood cell (WBC) counts clearly
indicates leukocytosis?
A. 4,500/mm³
B. 7,000/mm³
C. 10,000/mm³
D. 25,000/mm³
17. After 5 days of diuretic therapy with 20mg of furosemide (Lasix) daily,
a patient begins to exhibit fatigue, muscle cramping and muscle
weakness. These symptoms probably indicate that the patient is experiencing:
A. Hypokalemia
B. Hyperkalemia
C. Anorexia
D. Dysphagia
18.Which of the following statements about chest X-ray is false?
A. No contradictions exist for this test
B. Before the procedure, the patient should remove all jewelry, metallic
objects, and buttons above the waist
C. A signed consent is not required
D. Eating, drinking, and medications are allowed before this test
19.The most appropriate time for the nurse to obtain a sputum specimen
for culture is:
A. Early in the morning
B. After the patient eats a light breakfast
C. After aerosol therapy
D. After chest physiotherapy
20.A patient with no known allergies is to receive penicillin every 6
hours. When administering the medication, the nurse observes a fine rash on
the
patient’s skin. The most appropriate nursing action would be to:
A. Withhold the moderation and notify the physician
B. Administer the medication and notify the physician
C. Administer the medication with an antihistamine
D. Apply corn starch soaks to the rash
21.All of the following nursing interventions are correct when using the
Ztrack method of drug injection except:
A. Prepare the injection site with alcohol
B. Use a needle that’s a least 1” long
C. Aspirate for blood before injection
D. Rub the site vigorously after the injection to promote absorption
22.The correct method for determining the vastus lateralis site for
I.M. injection is to:
A. Locate the upper aspect of the upper outer quadrant of the
buttock about 5 to 8 cm below the iliac crest
B. Palpate the lower edge of the acromion process and the midpoint lateral
aspect of the arm
C. Palpate a 1” circular area anterior to the umbilicus
D. Divide the area between the greater femoral trochanter and the lateral
femoral condyle into thirds, and select the middle third on the anterior of
the thigh
23.The mid-deltoid injection site is seldom used for I.M. injections because it:
A. Can accommodate only 1 ml or less of medication
B. Bruises too easily
C. Can be used only when the patient is lying down
D. Does not readily parenteral medication
24.The appropriate needle size for insulin injection is:
A. 18G, 1 ½” long
B. 22G, 1” long
C. 22G, 1 ½” long
D. 25G, 5/8” long
25.The appropriate needle gauge for intradermal injection is:
A. 20G
B. 22G
C. 25G
D. 26G
26.Parenteral penicillin can be administered as an:
A. IM injection or an IV solution
B. IV or an intradermal injection
C. Intradermal or subcutaneous injection
D. IM or a subcutaneous injection
27.The physician orders gr 10 of aspirin for a patient. The equivalent dose
in milligrams is:
A. 0.6 mg
B. 10 mg
C. 60 mg
D. 600 mg
28.The physician orders an IV solution of dextrose 5% in water at 100ml/hour.
What would the flow rate be if the drop factor is 15 gtt = 1 ml?
A. 5 gtt/minute
B. 13 gtt/minute
C. 25 gtt/minute
D. 50 gtt/minute
29.Which of the following is a sign or symptom of a hemolytic reaction
to blood transfusion?
A. Hemoglobinuria
B. Chest pain
C. Urticaria
D. Distended neck veins
30.Which of the following conditions may require fluid restriction?
A. Fever
B. Chronic Obstructive Pulmonary Disease
C. Renal Failure
D. Dehydration
31.All of the following are common signs and symptoms of phlebitis except:
A. Pain or discomfort at the IV insertion site
B. Edema and warmth at the IV insertion site
C. A red streak exiting the IV insertion site
D. Frank bleeding at the insertion site
32.The best way of determining whether a patient has learned to instill
ear medication properly is for the nurse to:
A. Ask the patient if he/she has used ear drops before
B. Have the patient repeat the nurse’s instructions using her own words
C. Demonstrate the procedure to the patient and encourage to
ask questions
D. Ask the patient to demonstrate the procedure
33.Which of the following types of medications can be administered
via gastrostomy tube?
A. Any oral medications
B. Capsules whole contents are dissolve in water
C. Enteric-coated tablets that are thoroughly dissolved in water
D. Most tablets designed for oral use, except for extended-
duration compounds
34.A patient who develops hives after receiving an antibiotic is
exhibiting drug:
A. Tolerance
B. Idiosyncrasy
C. Synergism
D. Allergy
35.A patient has returned to his room after femoral arteriography. All of
the following are appropriate nursing interventions except:
A. Assess femoral, popliteal, and pedal pulses every 15 minutes for
2 hours
B. Check the pressure dressing for sanguineous drainage
C. Assess a vital signs every 15 minutes for 2 hours
D. Order a hemoglobin and hematocrit count 1 hour after the arteriography
36.The nurse explains to a patient that a cough:
A. Is a protective response to clear the respiratory tract of irritants
B. Is primarily a voluntary action
C. Is induced by the administration of an antitussive drug
D. Can be inhibited by “splinting” the abdomen
37.An infected patient has chills and begins shivering. The best
nursing intervention is to:
A. Apply iced alcohol sponges
B. Provide increased cool liquids
C. Provide additional bedclothes
D. Provide increased ventilation
38.A clinical nurse specialist is a nurse who has:
A. Been certified by the National League for Nursing
B. Received credentials from the Philippine Nurses’ Association
C. Graduated from an associate degree program and is a
registered professional nurse
D. Completed a master’s degree in the prescribed clinical area and is a
registered professional nurse.
39.The purpose of increasing urine acidity through dietary means is to:
A. Decrease burning sensations
B. Change the urine’s color
C. Change the urine’s concentration
D. Inhibit the growth of microorganisms
40.Clay colored stools indicate:
A. Upper GI bleeding
B. Impending constipation
C. An effect of medication
D. Bile obstruction
41.In which step of the nursing process would the nurse ask a patient if
the medication she administered relieved his pain?
A. Assessment
B. Analysis
C. Planning
D. Evaluation
42.All of the following are good sources of vitamin A except:
A. White potatoes
B. Carrots
C. Apricots
D. Egg yolks
43.Which of the following is a primary nursing intervention necessary for
all patients with a Foley Catheter in place?
A. Maintain the drainage tubing and collection bag level with the patient’s
bladder
B. Irrigate the patient with 1% Neosporin solution three times a daily
C. Clamp the catheter for 1 hour every 4 hours to maintain the bladder’s
elasticity
D. Maintain the drainage tubing and collection bag below bladder level to
facilitate drainage by gravity
44.The ELISA test is used to:
A. Screen blood donors for antibodies to human immunodeficiency virus
(HIV)
B. Test blood to be used for transfusion for HIV antibodies
C. Aid in diagnosing a patient with AIDS
D. All of the above
45.The two blood vessels most commonly used for TPN infusion are the:
A. Subclavian and jugular veins
B. Brachial and subclavian veins
C. Femoral and subclavian veins
D. Brachial and femoral veins
46.Effective skin disinfection before a surgical procedure includes which
of the following methods?
A. Shaving the site on the day before surgery
B. Applying a topical antiseptic to the skin on the evening before surgery
C. Having the patient take a tub bath on the morning of surgery
D. Having the patient shower with an antiseptic soap on the
evening v=before and the morning of surgery
47.When transferring a patient from a bed to a chair, the nurse should
use which muscles to avoid back injury?
A. Abdominal muscles
B. Back muscles
C. Leg muscles
D. Upper arm muscles
48.Thrombophlebitis typically develops in patients with which of the
following conditions?
A. Increases partial thromboplastin time
B. Acute pulsus paradoxus
C. An impaired or traumatized blood vessel wall
D. Chronic Obstructive Pulmonary Disease (COPD)
49.In a recumbent, immobilized patient, lung ventilation can become
altered, leading to such respiratory complications as:
A. Respiratory acidosis, ateclectasis, and hypostatic pneumonia
B. Appneustic breathing, atypical pneumonia and respiratory alkalosis
C. Cheyne-Strokes respirations and spontaneous pneumothorax
D. Kussmail’s respirations and hypoventilation
50.Immobility impairs bladder elimination, resulting in such disorders as
A. Increased urine acidity and relaxation of the perineal muscles, causing
incontinence
B. Urine retention, bladder distention, and infection
C. Diuresis, natriuresis, and decreased urine specific gravity
D. Decreased calcium and phosphate levels in the urine
Answers and Rationales
1. D. In the circular chain of infection, pathogens must be able to leave
their reservoir and be transmitted to a susceptible host through a portal
of entry, such as broken skin.
2. C. Respiratory isolation, like strict isolation, requires that the door to
the door patient’s room remain closed. However, the patient’s room
should be well ventilated, so opening the window or turning on the
ventricular is desirable. The nurse does not need to wear gloves for
respiratory isolation, but good hand washing is important for all types of
isolation.
3. A. Leukopenia is a decreased number of leukocytes (white blood
cells), which are important in resisting infection. None of the other
situations would put the patient at risk for contracting an infection; taking
broadspectrum antibiotics might actually reduce the infection risk.
4. A. Soaps and detergents are used to help remove bacteria because
of their ability to lower the surface tension of water and act as
emulsifying agents. Hot water may lead to skin irritation or burns.
5. A. Depending on the degree of exposure to pathogens, hand
washing may last from 10 seconds to 4 minutes. After routine patient
contact, hand washing for 30 seconds effectively minimizes the risk of
pathogen transmission.
6. B. The urinary system is normally free of microorganisms except at
the urinary meatus. Any procedure that involves entering this system
must use surgically aseptic measures to maintain a bacteria-free state.
7. C. All invasive procedures, including surgery, catheter insertion,
and administration of parenteral therapy, require sterile technique to
maintain a sterile environment. All equipment must be sterile, and the
nurse and the physician must wear sterile gloves and maintain surgical
asepsis. In the operating room, the nurse and physician are required to
wear sterile gowns, gloves, masks, hair covers, and shoe covers for all
invasive procedures. Strict isolation requires the use of clean gloves,
masks, gowns and equipment to prevent the transmission of highly
communicable diseases by contact or by airborne routes. Terminal
disinfection is the disinfection of all contaminated supplies and
equipment after a patient has been discharged to prepare them for reuse
by another patient. The purpose of protective (reverse) isolation is to
prevent a person with seriously impaired resistance from coming into
contact who potentially pathogenic organisms.
8. C. The edges of a sterile field are considered contaminated. When
sterile items are allowed to come in contact with the edges of the field, the
sterile items also become contaminated.
9. B. Hair on or within body areas, such as the nose, traps and
holds particles that contain microorganisms. Yawning and hiccupping do
not prevent microorganisms from entering or leaving the body. Rapid
eye movement marks the stage of sleep during which dreaming occurs.
10. D. The inside of the glove is always considered to be clean, but not
sterile.
11. A. The back of the gown is considered clean, the front is
contaminated. So, after removing gloves and washing hands, the nurse
should untie the back of the gown; slowly move backward away from the
gown, holding the inside of the gown and keeping the edges off the floor;
turn and fold the gown inside out; discard it in a contaminated linen
container; then wash her hands again.
12. B. According to the Centers for Disease Control (CDC), blood-to-
blood contact occurs most commonly when a health care worker
attempts to cap a used needle. Therefore, used needles should never be
recapped; instead they should be inserted in a specially designed puncture
resistant, labeled container. Wearing gloves is not always necessary
when administering an I.M. injection. Enteric precautions prevent the
transfer of pathogens via feces.
13. A. Nurses and other health care professionals previously believed
that massaging a reddened area with lotion would promote venous return
and reduce edema to the area. However, research has shown that
massage only increases the likelihood of cellular ischemia and necrosis to
the area.
14. B. Before a blood transfusion is performed, the blood of the donor
and recipient must be checked for compatibility. This is done by blood
typing (a test that determines a person’s blood type) and cross-matching
(a procedure that determines the compatibility of the donor’s and
recipient’s blood after the blood types has been matched). If the blood
specimens are incompatible, hemolysis and antigen-antibody reactions
will occur.
15. A. Platelets are disk-shaped cells that are essential for blood
coagulation. A platelet count determines the number of thrombocytes in
blood available for promoting hemostasis and assisting with blood
coagulation after injury. It also is used to evaluate the patient’s potential
for bleeding; however, this is not its primary purpose. The normal count
ranges from 150,000 to 350,000/mm3. A count of 100,000/mm3 or less
indicates a potential for bleeding; count of less than 20,000/mm3 is
associated with spontaneous bleeding.
16. D. Leukocytosis is any transient increase in the number of white
blood cells (leukocytes) in the blood. Normal WBC counts range from
5,000 to 100,000/mm3. Thus, a count of 25,000/mm3 indicates
leukocytosis.
17. A. Fatigue, muscle cramping, and muscle weaknesses are symptoms
of hypokalemia (an inadequate potassium level), which is a potential
side effect of diuretic therapy. The physician usually orders
supplemental potassium to prevent hypokalemia in patients receiving
diuretics. Anorexia is another symptom of hypokalemia. Dysphagia means
difficulty swallowing.
18. A. Pregnancy or suspected pregnancy is the only contraindication for
a chest X-ray. However, if a chest X-ray is necessary, the patient can
wear a lead apron to protect the pelvic region from radiation. Jewelry,
metallic objects, and buttons would interfere with the X-ray and thus
should not be worn above the waist. A signed consent is not required
because a chest X-ray is not an invasive examination. Eating, drinking and
medications are allowed because the X-ray is of the chest, not the
abdominal region.
19. A. Obtaining a sputum specimen early in this morning ensures
an adequate supply of bacteria for culturing and decreases the risk
of contamination from food or medication.
20. A. Initial sensitivity to penicillin is commonly manifested by a skin
rash, even in individuals who have not been allergic to it previously.
Because of the danger of anaphylactic shock, he nurse should withhold
the drug and notify the physician, who may choose to substitute another
drug. Administering an antihistamine is a dependent nursing intervention
that requires a written physician’s order. Although applying corn starch to
the rash may relieve discomfort, it is not the nurse’s top priority in such
a potentially life-threatening situation.
21. D. The Z-track method is an I.M. injection technique in which the
patient’s skin is pulled in such a way that the needle track is sealed off
after the injection. This procedure seals medication deep into the muscle,
thereby minimizing skin staining and irritation. Rubbing the injection site
is contraindicated because it may cause the medication to extravasate
into the skin.
22. D. The vastus lateralis, a long, thick muscle that extends the full length
of the thigh, is viewed by many clinicians as the site of choice for
I.M. injections because it has relatively few major nerves and blood
vessels. The middle third of the muscle is recommended as the injection
site. The patient can be in a supine or sitting position for an injection into
this site.
23. A. The mid-deltoid injection site can accommodate only 1 ml or less
of medication because of its size and location (on the deltoid muscle of
the arm, close to the brachial artery and radial nerve).
24. D. A 25G, 5/8” needle is the recommended size for insulin
injection because insulin is administered by the subcutaneous route. An
18G, 1 ½” needle is usually used for I.M. injections in children, typically in
the vastus lateralis. A 22G, 1 ½” needle is usually used for adult I.M.
injections, which are typically administered in the vastus lateralis or
ventrogluteal site.
25. D. Because an intradermal injection does not penetrate deeply into
the skin, a small-bore 25G needle is recommended. This type of injection
is used primarily to administer antigens to evaluate reactions for allergy
or sensitivity studies. A 20G needle is usually used for I.M. injections of
oilbased medications; a 22G needle for I.M. injections; and a 25G needle,
for I.M. injections; and a 25G needle, for subcutaneous insulin injections.
26. A. Parenteral penicillin can be administered I.M. or added to a
solution and given I.V. It cannot be administered subcutaneously or
intradermally.
27. D. gr 10 x 60mg/gr 1 = 600 mg
28. C. 100ml/60 min X 15 gtt/ 1 ml = 25 gtt/minute
29. A. Hemoglobinuria, the abnormal presence of hemoglobin in the
urine, indicates a hemolytic reaction (incompatibility of the donor’s
and recipient’s blood). In this reaction, antibodies in the recipient’s
plasma combine rapidly with donor RBC’s; the cells are hemolyzed in
either circulatory or reticuloendothelial system. Hemolysis occurs more
rapidly in ABO incompatibilities than in Rh incompatibilities. Chest pain
and urticaria may be symptoms of impending anaphylaxis. Distended neck
veins are an indication of hypervolemia.
30. C. In real failure, the kidney loses their ability to effectively
eliminate wastes and fluids. Because of this, limiting the patient’s intake
of oral and I.V. fluids may be necessary. Fever, chronic obstructive
pulmonary disease, and dehydration are conditions for which fluids should
be encouraged.
31. D. Phlebitis, the inflammation of a vein, can be caused by
chemical irritants (I.V. solutions or medications), mechanical irritants (the
needle or catheter used during venipuncture or cannulation), or a localized
allergic reaction to the needle or catheter. Signs and symptoms of
phlebitis include pain or discomfort, edema and heat at the I.V. insertion
site, and a red streak going up the arm or leg from the I.V. insertion site.
32. D. Return demonstration provides the most certain evidence for
evaluating the effectiveness of patient teaching.
33. D. Capsules, enteric-coated tablets, and most extended duration
or sustained release products should not be dissolved for use in
a gastrostomy tube. They are pharmaceutically manufactured in these
forms for valid reasons, and altering them destroys their purpose. The
nurse should seek an alternate physician’s order when an ordered
medication is inappropriate for delivery by tube.
34. D. A drug-allergy is an adverse reaction resulting from an
immunologic response following a previous sensitizing exposure to the
drug. The reaction can range from a rash or hives to anaphylactic shock.
Tolerance to a drug means that the patient experiences a decreasing
physiologic response to repeated administration of the drug in the same
dosage. Idiosyncrasy is an individual’s unique hypersensitivity to a drug,
food, or other substance; it appears to be genetically determined.
Synergism, is a drug interaction in which the sum of the drug’s combined
effects is greater than that of their separate effects.
35. D. A hemoglobin and hematocrit count would be ordered by the
physician if bleeding were suspected. The other answers are appropriate
nursing interventions for a patient who has undergone femoral
arteriography.
36. A. Coughing, a protective response that clears the respiratory tract
of irritants, usually is involuntary; however it can be voluntary, as when
a patient is taught to perform coughing exercises. An antitussive
drug inhibits coughing. Splinting the abdomen supports the abdominal
muscles when a patient coughs.
37. C. In an infected patient, shivering results from the body’s attempt
to increase heat production and the production of neutrophils
and phagocytotic action through increased skeletal muscle tension
and contractions. Initial vasoconstriction may cause skin to feel cold to
the touch. Applying additional bed clothes helps to equalize the
body temperature and stop the chills. Attempts to cool the body result in
further shivering, increased metabloism, and thus increased heat
production.
38. D. A clinical nurse specialist must have completed a master’s degree in
a clinical specialty and be a registered professional nurse. The
National League of Nursing accredits educational programs in nursing and
provides a testing service to evaluate student nursing competence but it
does not certify nurses. The American Nurses Association identifies
requirements for certification and offers examinations for certification in
many areas of nursing., such as medical surgical nursing. These
certification (credentialing) demonstrates that the nurse has the
knowledge and the ability to provide high quality nursing care in the area
of her certification. A graduate of an associate degree program is not a
clinical nurse specialist: however, she is prepared to provide bed side
nursing with a high degree of knowledge and skill. She must successfully
complete the licensing examination to become a registered professional
nurse.
39. D. Microorganisms usually do not grow in an acidic environment.
40. D. Bile colors the stool brown. Any inflammation or obstruction that
impairs bile flow will affect the stool pigment, yielding light, clay-colored
stool. Upper GI bleeding results in black or tarry stool. Constipation
is characterized by small, hard masses. Many medications and foods
will discolor stool – for example, drugs containing iron turn stool black.;
beets turn stool red.
41. D. In the evaluation step of the nursing process, the nurse must
decide whether the patient has achieved the expected outcome that
was identified in the planning phase.
42. A. The main sources of vitamin A are yellow and green vegetables
(such as carrots, sweet potatoes, squash, spinach, collard greens,
broccoli, and cabbage) and yellow fruits (such as apricots, and
cantaloupe). Animal sources include liver, kidneys, cream, butter, and egg
yolks.
43. D. Maintaing the drainage tubing and collection bag level with the
patient’s bladder could result in reflux of urine into the kidney. Irrigating
the bladder with Neosporin and clamping the catheter for 1 hour every 4
hours must be prescribed by a physician.
44. D. The ELISA test of venous blood is used to assess blood and
potential blood donors to human immunodeficiency virus (HIV). A positive
ELISA test combined with various signs and symptoms helps to
diagnose acquired immunodeficiency syndrome (AIDS)
45. D. Tachypnea (an abnormally rapid rate of breathing) would indicate
that the patient was still hypoxic (deficient in oxygen).The partial
pressures of arterial oxygen and carbon dioxide listed are within the
normal range. Eupnea refers to normal respiration.
46. D. Studies have shown that showering with an antiseptic soap
before surgery is the most effective method of removing microorganisms
from the skin. Shaving the site of the intended surgery might cause breaks
in the skin, thereby increasing the risk of infection; however, if
indicated, shaving, should be done immediately before surgery, not the day
before. A topical antiseptic would not remove microorganisms and would
be beneficial only after proper cleaning and rinsing. Tub bathing
might transfer organisms to another body site rather than rinse them
away.
47. C. The leg muscles are the strongest muscles in the body and should
bear the greatest stress when lifting. Muscles of the abdomen, back, and
upper arms may be easily injured.
48. C. The factors, known as Virchow’s triad, collectively predispose a
patient to thromboplebitis; impaired venous return to the heart,
blood hypercoagulability, and injury to a blood vessel wall. Increased
partial thromboplastin time indicates a prolonged bleeding time during
fibrin clot formation, commonly the result of anticoagulant (heparin)
therapy. Arterial blood disorders (such as pulsus paradoxus) and lung
diseases (such as COPD) do not necessarily impede venous return of
injure vessel walls.
49. A. Because of restricted respiratory movement, a recumbent,
immobilize patient is at particular risk for respiratory acidosis from poor
gas exchange; atelectasis from reduced surfactant and accumulated
mucus in the bronchioles, and hypostatic pneumonia from bacterial
growth caused by stasis of mucus secretions.
50. B. The immobilized patient commonly suffers from urine retention
caused by decreased muscle tone in the perineum. This leads to
bladder distention and urine stagnation, which provide an excellent
medium for bacterial growth leading to infection. Immobility also results in
more alkaline urine with excessive amounts of calcium, sodium and
phosphate, a gradual decrease in urine production, and an increased
specific gravity.

PNLE II for Maternal and Child


Health
1. For the client who is using oral contraceptives, the nurse informs the
client about the need to take the pill at the same time each day to
accomplish which of the following?
A. Decrease the incidence of nausea
B. Maintain hormonal levels
C. Reduce side effects
D. Prevent drug interactions
2. When teaching a client about contraception. Which of the following
would the nurse include as the most effective method for preventing
sexually transmitted infections?
A. Spermicides
B. Diaphragm
C. Condoms
D. Vasectomy
3. When preparing a woman who is 2 days postpartum for
discharge, recommendations for which of the following contraceptive
methods would be avoided?
A. Diaphragm
B. Female condom
C. Oral contraceptives
D. Rhythm method
4. For which of the following clients would the nurse expect that
an intrauterine device would not be recommended?
A. Woman over age 35
B. Nulliparous woman
C. Promiscuous young adult
D. Postpartum client
5. A client in her third trimester tells the nurse, “I’m constipated all the
time!” Which of the following should the nurse recommend?
A. Daily enemas
B. Laxatives
C. Increased fiber intake
D. Decreased fluid intake
6. Which of the following would the nurse use as the basis for the
teaching plan when caring for a pregnant teenager concerned about gaining
too much weight during pregnancy?
A. 10 pounds per trimester
B. 1 pound per week for 40 weeks
C. ½ pound per week for 40 weeks
D. A total gain of 25 to 30 pounds
7. The client tells the nurse that her last menstrual period started on
January 14 and ended on January 20. Using Nagele’s rule, the nurse
determines her EDD to be which of the following?
A. September 27
B. October 21
C. November 7
D. December 27
8. When taking an obstetrical history on a pregnant client who states, “I had a
son born at 38 weeks gestation, a daughter born at 30 weeks gestation and I
lost a baby at about 8 weeks,” the nurse should record her obstetrical history
as which of the following?
A. G2 T2 P0 A0 L2
B. G3 T1 P1 A0 L2
C. G3 T2 P0 A0 L2
D. G4 T1 P1 A1 L2
9. When preparing to listen to the fetal heart rate at 12 weeks’ gestation,
the nurse would use which of the following?
A. Stethoscope placed midline at the umbilicus
B. Doppler placed midline at the suprapubic region
C. Fetoscope placed midway between the umbilicus and the
xiphoid process
D. External electronic fetal monitor placed at the umbilicus
10.When developing a plan of care for a client newly diagnosed
with gestational diabetes, which of the following instructions would be
the priority?
A. Dietary intake
B. Medication
C. Exercise
D. Glucose monitoring
11.A client at 24 weeks gestation has gained 6 pounds in 4 weeks. Which
of the following would be the priority when assessing the client?
A. Glucosuria
B. Depression
C. Hand/face edema
D. Dietary intake
12. A client 12 weeks’ pregnant come to the emergency department
with abdominal cramping and moderate vaginal bleeding.
Speculum examination reveals 2 to 3 cms cervical dilation. The nurse
would document these findings as which of the following?
A. Threatened abortion
B. Imminent abortion
C. Complete abortion
D. Missed abortion
13.Which of the following would be the priority nursing diagnosis for a
client with an ectopic pregnancy?
A. Risk for infection
B. Pain
C. Knowledge Deficit
D. Anticipatory Grieving
14.Before assessing the postpartum client’s uterus for firmness and
position in relation to the umbilicus and midline, which of the following should
the nurse do first?
A. Assess the vital signs
B. Administer analgesia
C. Ambulate her in the hall
D. Assist her to urinate
15.Which of the following should the nurse do when a primipara who
is lactating tells the nurse that she has sore nipples?
A. Tell her to breast feed more frequently
B. Administer a narcotic before breast feeding
C. Encourage her to wear a nursing brassiere
D. Use soap and water to clean the nipples
16.The nurse assesses the vital signs of a client, 4 hours’ postpartum that
are as follows: BP 90/60; temperature 100.4ºF; pulse 100 weak, thready; R
20 per minute. Which of the following should the nurse do first?
A. Report the temperature to the physician
B. Recheck the blood pressure with another cuff
C. Assess the uterus for firmness and position
D. Determine the amount of lochia
17.The nurse assesses the postpartum vaginal discharge (lochia) on
four clients. Which of the following assessments would warrant notification
of the physician?
A. A dark red discharge on a 2-day postpartum client
B. A pink to brownish discharge on a client who is 5 days postpartum
C. Almost colorless to creamy discharge on a client 2 weeks after delivery
D. A bright red discharge 5 days after delivery
18.A postpartum client has a temperature of 101.4ºF, with a uterus that
is tender when palpated, remains unusually large, and not descending
as normally expected. Which of the following should the nurse assess next?
A. Lochia
B. Breasts
C. Incision
D. Urine
19.Which of the following is the priority focus of nursing practice with
the current early postpartum discharge?
A. Promoting comfort and restoration of health
B. Exploring the emotional status of the family
C. Facilitating safe and effective self-and newborn care
D. Teaching about the importance of family planning
20. Which of the following actions would be least effective in maintaining
a neutral thermal environment for the newborn?
A. Placing infant under radiant warmer after bathing
B. Covering the scale with a warmed blanket prior to weighing
C. Placing crib close to nursery window for family viewing
D. Covering the infant’s head with a knit stockinette
21.A newborn who has an asymmetrical Moro reflex response should
be further assessed for which of the following?
A. Talipes equinovarus
B. Fractured clavicle
C. Congenital hypothyroidism
D. Increased intracranial pressure
22.During the first 4 hours after a male circumcision, assessing for which
of the following is the priority?
A. Infection
B. Hemorrhage
C. Discomfort
D. Dehydration
23.The mother asks the nurse. “What’s wrong with my son’s breasts? Why are
they so enlarged?” Whish of the following would be the best response by the
nurse?
A. “The breast tissue is inflamed from the trauma experienced with birth”
B. “A decrease in material hormones present before birth
causes enlargement,”
C. “You should discuss this with your doctor. It could be a malignancy”
D. “The tissue has hypertrophied while the baby was in the uterus”
24. Immediately after birth the nurse notes the following on a male
newborn: respirations 78; apical hearth rate 160 BPM, nostril flaring; mild
intercostal
retractions; and grunting at the end of expiration. Which of the
following should the nurse do?
A. Call the assessment data to the physician’s attention
B. Start oxygen per nasal cannula at 2 L/min.
C. Suction the infant’s mouth and nares
D. Recognize this as normal first period of reactivity
25.The nurse hears a mother telling a friend on the telephone about
umbilical cord care. Which of the following statements by the mother
indicates effective teaching?
A. “Daily soap and water cleansing is best”
B. ‘Alcohol helps it dry and kills germs”
C. “An antibiotic ointment applied daily prevents infection”
D. “He can have a tub bath each day”
26.A newborn weighing 3000 grams and feeding every 4 hours needs
120 calories/kg of body weight every 24 hours for proper growth
and development. How many ounces of 20 cal/oz formula should this
newborn receive at each feeding to meet nutritional needs?
A. 2 ounces
B. 3 ounces
C. 4 ounces
D. 6 ounces
27.The postterm neonate with meconium-stained amniotic fluid needs
care designed to especially monitor for which of the following?
A. Respiratory problems
B. Gastrointestinal problems
C. Integumentary problems
D. Elimination problems
28.When measuring a client’s fundal height, which of the following
techniques denotes the correct method of measurement used by the nurse?
A. From the xiphoid process to the umbilicus
B. From the symphysis pubis to the xiphoid process
C. From the symphysis pubis to the fundus
D. From the fundus to the umbilicus
29.A client with severe preeclampsia is admitted with of BP
160/110, proteinuria, and severe pitting edema. Which of the following would
be most important to include in the client’s plan of care?
A. Daily weights
B. Seizure precautions
C. Right lateral positioning
D. Stress reduction
30. A postpartum primipara asks the nurse, “When can we have
sexual intercourse again?” Which of the following would be the nurse’s
best response?
A. “Anytime you both want to.”
B. “As soon as choose a contraceptive method.”
C. “When the discharge has stopped and the incision is healed.”
D. “After your 6 weeks examination.”
31.When preparing to administer the vitamin K injection to a neonate,
the nurse would select which of the following sites as appropriate for
the injection?
A. Deltoid muscle
B. Anterior femoris muscle
C. Vastus lateralis muscle
D. Gluteus maximus muscle
32.When performing a pelvic examination, the nurse observes a red
swollen area on the right side of the vaginal orifice. The nurse would
document this as enlargement of which of the following?
A. Clitoris
B. Parotid gland
C. Skene’s gland
D. Bartholin’s gland
33.To differentiate as a female, the hormonal stimulation of the embryo
that must occur involves which of the following?
A. Increase in maternal estrogen secretion
B. Decrease in maternal androgen secretion
C. Secretion of androgen by the fetal gonad
D. Secretion of estrogen by the fetal gonad
34.A client at 8 weeks’ gestation calls complaining of slight nausea in
the morning hours. Which of the following client interventions should the
nurse question?
A. Taking 1 teaspoon of bicarbonate of soda in an 8-ounce glass of water
B. Eating a few low-sodium crackers before getting out of bed
C. Avoiding the intake of liquids in the morning hours
D. Eating six small meals a day instead of thee large meals
35.The nurse documents positive ballottement in the client’s prenatal
record. The nurse understands that this indicates which of the following?
A. Palpable contractions on the abdomen
B. Passive movement of the unengaged fetus
C. Fetal kicking felt by the client
D. Enlargement and softening of the uterus
36.During a pelvic exam the nurse notes a purple-blue tinge of the cervix. The
nurse documents this as which of the following?
A. Braxton-Hicks sign
B. Chadwick’s sign
C. Goodell’s sign
D. McDonald’s sign
37.During a prenatal class, the nurse explains the rationale for
breathing techniques during preparation for labor based on the understanding
that breathing techniques are most important in achieving which of
the following?
A. Eliminate pain and give the expectant parents something to do
B. Reduce the risk of fetal distress by increasing uteroplacental perfusion
C. Facilitate relaxation, possibly reducing the perception of pain
D. Eliminate pain so that less analgesia and anesthesia are needed
38.After 4 hours of active labor, the nurse notes that the contractions of
a primigravida client are not strong enough to dilate the cervix. Which of the
following would the nurse anticipate doing?
A. Obtaining an order to begin IV oxytocin infusion
B. Administering a light sedative to allow the patient to rest for
several hour
C. Preparing for a cesarean section for failure to progress
D. Increasing the encouragement to the patient when pushing begins
39.A multigravida at 38 weeks’ gestation is admitted with painless, bright
red bleeding and mild contractions every 7 to 10 minutes. Which of
the following assessments should be avoided?
A. Maternal vital sign
B. Fetal heart rate
C. Contraction monitoring
D. Cervical dilation
40.Which of the following would be the nurse’s most appropriate response
to a client who asks why she must have a cesarean delivery if she has
a complete placenta previa?
A. “You will have to ask your physician when he returns.”
B. “You need a cesarean to prevent hemorrhage.”
C. “The placenta is covering most of your cervix.”
D. “The placenta is covering the opening of the uterus and blocking your
baby.”
41.The nurse understands that the fetal head is in which of the
following positions with a face presentation?
A. Completely flexed
B. Completely extended
C. Partially extended
D. Partially flexed
42.With a fetus in the left-anterior breech presentation, the nurse would expect
the fetal heart rate would be most audible in which of the following areas?
A. Above the maternal umbilicus and to the right of midline
B. In the lower-left maternal abdominal quadrant
C. In the lower-right maternal abdominal quadrant
D. Above the maternal umbilicus and to the left of midline
43.The amniotic fluid of a client has a greenish tint. The nurse interprets
this to be the result of which of the following?
A. Lanugo
B. Hydramnio
C. Meconium
D. Vernix
44.A patient is in labor and has just been told she has a breech
presentation. The nurse should be particularly alert for which of the following?
A. Quickening
B. Ophthalmia neonatorum
C. Pica
D. Prolapsed umbilical cord
45.When describing dizygotic twins to a couple, on which of the
following would the nurse base the explanation?
A. Two ova fertilized by separate sperm
B. Sharing of a common placenta
C. Each ova with the same genotype
D. Sharing of a common chorion
46.Which of the following refers to the single cell that reproduces itself
after conception?
A. Chromosome
B. Blastocyst
C. Zygote
D. Trophoblast
47.In the late 1950s, consumers and health care professionals
began challenging the routine use of analgesics and anesthetics during
childbirth. Which of the following was an outgrowth of this concept?
A. Labor, delivery, recovery, postpartum (LDRP)
B. Nurse-midwifery
C. Clinical nurse specialist
D. Prepared childbirth
48.A client has a midpelvic contracture from a previous pelvic injury due to
a motor vehicle accident as a teenager. The nurse is aware that this could
prevent a fetus from passing through or around which structure
during childbirth?
A. Symphysis pubis
B. Sacral promontory
C. Ischial spines
D. Pubic arch
49.When teaching a group of adolescents about variations in the length of
the menstrual cycle, the nurse understands that the underlying mechanism is
due to variations in which of the following phases?
A. Menstrual phase
B. Proliferative phase
C. Secretory phase
D. Ischemic phase
50.When teaching a group of adolescents about male hormone
production, which of the following would the nurse include as being produced
by the Leydig cells?
A. Follicle-stimulating hormone
B. Testosterone
C. Leuteinizing hormone
D. Gonadotropin releasing hormone
Answers and Rationales
1. B . Regular timely ingestion of oral contraceptives is necessary to
maintain hormonal levels of the drugs to suppress the action of the
hypothalamus and anterior pituitary leading to inappropriate secretion of
FSH and LH. Therefore, follicles do not mature, ovulation is inhibited, and
pregnancy is prevented. The estrogen content of the oral site
contraceptive may cause the nausea, regardless of when the pill is taken.
Side effects and drug interactions may occur with oral contraceptives
regardless of the time the pill is taken.
2. C . Condoms, when used correctly and consistently, are the most
effective contraceptive method or barrier against bacterial and viral
sexually transmitted infections. Although spermicides kill sperm, they do
not provide reliable protection against the spread of sexually
transmitted infections, especially intracellular organisms such as HIV.
Insertion and removal of the diaphragm along with the use of the
spermicides may cause vaginal irritations, which could place the client at
risk for infection transmission. Male sterilization eliminates spermatozoa
from the ejaculate, but it does not eliminate bacterial and/or viral
microorganisms that can cause sexually transmitted infections.
3. A . The diaphragm must be fitted individually to ensure
effectiveness. Because of the changes to the reproductive structures
during pregnancy and following delivery, the diaphragm must be refitted,
usually at the 6 weeks’ examination following childbirth or after a weight
loss of 15 lbs or more. In addition, for maximum effectiveness,
spermicidal jelly should be placed in the dome and around the rim.
However, spermicidal jelly should not be inserted into the vagina until
involution is completed at approximately 6 weeks. Use of a female
condom protects the reproductive system from the introduction of semen
or spermicides into the vagina and may be used after childbirth. Oral
contraceptives may be started within the first postpartum week to ensure
suppression of ovulation. For the couple who has determined the female’s
fertile period, using the rhythm method, avoidance of intercourse during
this period, is safe and effective.
4. C . An IUD may increase the risk of pelvic inflammatory disease,
especially in women with more than one sexual partner, because of the
increased risk of sexually transmitted infections. An UID should not be
used if the woman has an active or chronic pelvic infection, postpartum
infection, endometrial hyperplasia or carcinoma, or uterine abnormalities.
Age is not a factor in determining the risks associated with IUD use. Most
IUD users are over the age of 30. Although there is a slightly higher risk for
infertility in women who have never been pregnant, the IUD is an
acceptable option as long as the risk-benefit ratio is discussed. IUDs may
be inserted immediately after delivery, but this is not recommended
because of the increased risk and rate of expulsion at this time.
5. C . During the third trimester, the enlarging uterus places pressure on
the intestines. This coupled with the effect of hormones on smooth
muscle relaxation causes decreased intestinal motility (peristalsis).
Increasing fiber in the diet will help fecal matter pass more quickly through
the intestinal tract, thus decreasing the amount of water that is absorbed.
As a result, stool is softer and easier to pass. Enemas could precipitate
preterm labor and/or electrolyte loss and should be avoided. Laxatives
may cause preterm labor by stimulating peristalsis and may interfere with
the absorption of nutrients. Use for more than 1 week can also lead to
laxative dependency. Liquid in the diet helps provide a semisolid, soft
consistency to the stool. Eight to ten glasses of fluid per day are essential
to maintain hydration and promote stool evacuation.
6. D . To ensure adequate fetal growth and development during the 40
weeks of a pregnancy, a total weight gain 25 to 30 pounds is
recommended: 1.5 pounds in the first 10 weeks; 9 pounds by 30 weeks;
and 27.5 pounds by 40 weeks. The pregnant woman should gain less
weight in the first and second trimester than in the third. During the first
trimester, the client should only gain 1.5 pounds in the first 10 weeks, not
1 pound per week. A weight gain of ½ pound per week would be 20
pounds for the total pregnancy, less than the recommended amount.
7. B . To calculate the EDD by Nagele’s rule, add 7 days to the first day of
the last menstrual period and count back 3 months, changing the
year appropriately. To obtain a date of September 27, 7 days have been
added to the last day of the LMP (rather than the first day of the LMP),
plus 4 months (instead of 3 months) were counted back. To obtain the
date of November 7, 7 days have been subtracted (instead of added) from
the first day of LMP plus November indicates counting back 2 months
(instead of 3 months) from January. To obtain the date of December 27, 7
days were added to the last day of the LMP (rather than the first day of
the LMP) and December indicates counting back only 1 month (instead of
3 months) from January.
8. D. The client has been pregnant four times, including current
pregnancy (G). Birth at 38 weeks’ gestation is considered full term (T),
while birth form 20 weeks to 38 weeks is considered preterm (P). A
spontaneous abortion occurred at 8 weeks (A). She has two living children
(L).
9. B. At 12 weeks gestation, the uterus rises out of the pelvis and is
palpable above the symphysis pubis. The Doppler intensifies the sound of
the fetal pulse rate so it is audible. The uterus has merely risen out of the
pelvis into the abdominal cavity and is not at the level of the umbilicus.
The fetal heart rate at this age is not audible with a stethoscope. The
uterus at 12 weeks is just above the symphysis pubis in the abdominal
cavity, not midway between the umbilicus and the xiphoid process. At 12
weeks the FHR would be difficult to auscultate with a fetoscope. Although
the external electronic fetal monitor would project the FHR, the uterus has
not risen to the umbilicus at 12 weeks.
10. A . Although all of the choices are important in the management
of diabetes, diet therapy is the mainstay of the treatment plan and
should always be the priority. Women diagnosed with gestational
diabetes generally need only diet therapy without medication to control
their blood sugar levels. Exercise, is important for all pregnant women and
especially for diabetic women, because it burns up glucose, thus
decreasing blood sugar. However, dietary intake, not exercise, is the
priority. All pregnant women with diabetes should have periodic
monitoring of serum glucose. However, those with gestational diabetes
generally do not need daily glucose monitoring. The standard of care
recommends a fasting and 2- hour postprandial blood sugar level every 2
weeks.
11. C. After 20 weeks’ gestation, when there is a rapid weight
gain, preeclampsia should be suspected, which may be caused by
fluid retention manifested by edema, especially of the hands and face.
The three classic signs of preeclampsia are hypertension, edema,
and proteinuria. Although urine is checked for glucose at each clinic visit,
this is not the priority. Depression may cause either anorexia or
excessive food intake, leading to excessive weight gain or loss. This is not,
however, the priority consideration at this time. Weight gain thought to be
caused by excessive food intake would require a 24-hour diet recall.
However, excessive intake would not be the primary consideration for this
client at this time.
12. B. Cramping and vaginal bleeding coupled with cervical dilation
signifies that termination of the pregnancy is inevitable and cannot be
prevented. Thus, the nurse would document an imminent abortion. In a
threatened abortion, cramping and vaginal bleeding are present, but there
is no cervical dilation. The symptoms may subside or progress to
abortion. In a complete abortion all the products of conception are
expelled. A missed abortion is early fetal intrauterine death without
expulsion of the products of conception.
13. B . For the client with an ectopic pregnancy, lower abdominal pain,
usually unilateral, is the primary symptom. Thus, pain is the priority.
Although the potential for infection is always present, the risk is low in
ectopic pregnancy because pathogenic microorganisms have not been
introduced from external sources. The client may have a limited
knowledge of the pathology and treatment of the condition and will most
likely experience grieving, but this is not the priority at this time.
14. D. Before uterine assessment is performed, it is essential that the
woman empty her bladder. A full bladder will interfere with the accuracy of
the assessment by elevating the uterus and displacing to the side of
the midline. Vital sign assessment is not necessary unless an abnormality
in uterine assessment is identified. Uterine assessment should not
cause acute pain that requires administration of analgesia. Ambulating
the client is an essential component of postpartum care, but is not
necessary prior to assessment of the uterus.
15. A. Feeding more frequently, about every 2 hours, will decrease the
infant’s frantic, vigorous sucking from hunger and will decrease
breast engorgement, soften the breast, and promote ease of correct
latching-on for feeding. Narcotics administered prior to breast feeding are
passed through the breast milk to the infant, causing excessive
sleepiness. Nipple soreness is not severe enough to warrant narcotic
analgesia. All postpartum clients, especially lactating mothers, should
wear a supportive brassiere with wide cotton straps. This does not,
however, prevent or reduce nipple soreness. Soaps are drying to the skin
of the nipples and should not be used on the breasts of lactating mothers.
Dry nipple skin predisposes to cracks and fissures, which can become
sore and painful.
16. D. A weak, thready pulse elevated to 100 BPM may indicate
impending hemorrhagic shock. An increased pulse is a compensatory
mechanism of the body in response to decreased fluid volume. Thus, the
nurse should check the amount of lochia present. Temperatures up to
100.48F in the first 24 hours after birth are related to the dehydrating
effects of labor and are considered normal. Although rechecking the blood
pressure may be a correct choice of action, it is not the first action that
should be implemented in light of the other data. The data indicate a
potential impending hemorrhage. Assessing the uterus for firmness and
position in relation to the umbilicus and midline is important, but the nurse
should check the extent of vaginal bleeding first. Then it would be
appropriate to check the uterus, which may be a possible cause of the
hemorrhage.
17. D. Any bright red vaginal discharge would be considered abnormal,
but especially 5 days after delivery, when the lochia is typically pink
to brownish. Lochia rubra, a dark red discharge, is present for 2 to 3
days after delivery. Bright red vaginal bleeding at this time suggests
late postpartum hemorrhage, which occurs after the first 24 hours
following delivery and is generally caused by retained placental fragments
or bleeding disorders. Lochia rubra is the normal dark red
discharge occurring in the first 2 to 3 days after delivery, containing
epithelial cells, erythrocyes, leukocytes and decidua. Lochia serosa is a
pink to brownish serosanguineous discharge occurring from 3 to 10 days
after delivery that contains decidua, erythrocytes, leukocytes, cervical
mucus, and microorganisms. Lochia alba is an almost colorless to
yellowish discharge occurring from 10 days to 3 weeks after delivery and
containing leukocytes, decidua, epithelial cells, fat, cervical mucus,
cholesterol crystals, and bacteria.
18. A. The data suggests an infection of the endometrial lining of the
uterus. The lochia may be decreased or copious, dark brown in
appearance, and foul smelling, providing further evidence of a possible
infection. All the client’s data indicate a uterine problem, not a breast
problem. Typically, transient fever, usually 101ºF, may be present with
breast engorgement. Symptoms of mastitis include influenza-like
manifestations. Localized infection of an episiotomy or C-section incision
rarely causes systemic symptoms, and uterine involution would not be
affected. The client data do not include dysuria, frequency, or urgency,
symptoms of urinary tract infections, which would necessitate assessing
the client’s urine.
19. C. Because of early postpartum discharge and limited time for
teaching, the nurse’s priority is to facilitate the safe and effective care of
the client and newborn. Although promoting comfort and restoration of
health, exploring the family’s emotional status, and teaching about family
planning are important in postpartum/newborn nursing care, they are not
the priority focus in the limited time presented by early post-partum
discharge.
20. C. Heat loss by radiation occurs when the infant’s crib is placed too
near cold walls or windows. Thus placing the newborn’s crib close to
the viewing window would be least effective. Body heat is lost
through evaporation during bathing. Placing the infant under the radiant
warmer after bathing will assist the infant to be rewarmed. Covering the
scale with a warmed blanket prior to weighing prevents heat loss through
conduction. A knit cap prevents heat loss from the head a large head, a
large body surface area of the newborn’s body.
21. B. A fractured clavicle would prevent the normal Moro response
of symmetrical sequential extension and abduction of the arms followed
by flexion and adduction. In talipes equinovarus (clubfoot) the foot is
turned medially, and in plantar flexion, with the heel elevated. The feet are
not involved with the Moro reflex. Hypothyroiddism has no effect on
the primitive reflexes. Absence of the Moror reflex is the most
significant single indicator of central nervous system status, but it is not a
sign of increased intracranial pressure.
22. B. Hemorrhage is a potential risk following any surgical
procedure. Although the infant has been given vitamin K to facilitate
clotting, the prophylactic dose is often not sufficient to prevent bleeding.
Although infection is a possibility, signs will not appear within 4 hours
after the surgical procedure. The primary discomfort of circumcision
occurs during the surgical procedure, not afterward. Although feedings are
withheld prior to the circumcision, the chances of dehydration are
minimal.
23. B . The presence of excessive estrogen and progesterone in the
maternal fetal blood followed by prompt withdrawal at birth precipitates
breast engorgement, which will spontaneously resolve in 4 to 5 days after
birth. The trauma of the birth process does not cause inflammation of
the newborn’s breast tissue. Newborns do not have breast malignancy.
This reply by the nurse would cause the mother to have undue anxiety.
Breast tissue does not hypertrophy in the fetus or newborns.
24. D . The first 15 minutes to 1 hour after birth is the first period of
reactivity involving respiratory and circulatory adaptation to extrauterine
life. The data given reflect the normal changes during this time period. The
infant’s assessment data reflect normal adaptation. Thus, the physician
does not need to be notified and oxygen is not needed. The data do not
indicate the presence of choking, gagging or coughing, which are signs of
excessive secretions. Suctioning is not necessary.
25. B. Application of 70% isopropyl alcohol to the cord
minimizes microorganisms (germicidal) and promotes drying. The cord
should be kept dry until it falls off and the stump has healed. Antibiotic
ointment should only be used to treat an infection, not as a prophylaxis.
Infants should not be submerged in a tub of water until the cord falls off
and the stump has completely healed.
26. B. To determine the amount of formula needed, do the
following mathematical calculation. 3 kg x 120 cal/kg per day = 360
calories/day feeding q 4 hours = 6 feedings per day = 60 calories per
feeding: 60 calories per feeding; 60 calories per feeding with formula 20
cal/oz = 3 ounces per feeding. Based on the calculation. 2, 4 or 6 ounces
are incorrect.
27. A. Intrauterine anoxia may cause relaxation of the anal sphincter
and emptying of meconium into the amniotic fluid. At birth some of
the meconium fluid may be aspirated, causing mechanical obstruction
or chemical pneumonitis. The infant is not at increased risk
for gastrointestinal problems. Even though the skin is stained with
meconium, it is noninfectious (sterile) and nonirritating. The postterm
meconiumstained infant is not at additional risk for bowel or urinary
problems.
28. C . The nurse should use a nonelastic, flexible, paper measuring
tape, placing the zero point on the superior border of the symphysis pubis
and stretching the tape across the abdomen at the midline to the top of
the fundus. The xiphoid and umbilicus are not appropriate landmarks to
use when measuring the height of the fundus (McDonald’s measurement).
29. B . Women hospitalized with severe preeclampsia need decreased
CNS stimulation to prevent a seizure. Seizure precautions
provide environmental safety should a seizure occur. Because of edema,
daily weight is important but not the priority. Preclampsia causes
vasospasm and therefore can reduce utero-placental perfusion. The client
should be placed on her left side to maximize blood flow, reduce blood
pressure, and promote diuresis. Interventions to reduce stress and anxiety
are very important to facilitate coping and a sense of control, but
seizure precautions are the priority.
30. C. Cessation of the lochial discharge signifies healing of the
endometrium. Risk of hemorrhage and infection are minimal 3 weeks after
a normal vaginal delivery. Telling the client anytime is inappropriate
because this response does not provide the client with the specific
information she is requesting. Choice of a contraceptive method is
important, but not the specific criteria for safe resumption of sexual
activity. Culturally, the 6- weeks’ examination has been used as the time
frame for resuming sexual activity, but it may be resumed earlier.
31. C . The middle third of the vastus lateralis is the preferred injection site
for vitamin K administration because it is free of blood vessels and
nerves and is large enough to absorb the medication. The deltoid muscle
of a newborn is not large enough for a newborn IM injection. Injections
into this muscle in a small child might cause damage to the radial nerve.
The anterior femoris muscle is the next safest muscle to use in a newborn
but is not the safest. Because of the proximity of the sciatic nerve, the
gluteus maximus muscle should not be until the child has been walking 2
years.
32. D . Bartholin’s glands are the glands on either side of the vaginal
orifice. The clitoris is female erectile tissue found in the perineal area
above the urethra. The parotid glands are open into the mouth. Skene’s
glands open into the posterior wall of the female urinary meatus.
33. D . The fetal gonad must secrete estrogen for the embryo to
differentiate as a female. An increase in maternal estrogen secretion does
not effect differentiation of the embryo, and maternal estrogen secretion
occurs in every pregnancy. Maternal androgen secretion remains the
same as before pregnancy and does not effect differentiation. Secretion
of androgen by the fetal gonad would produce a male fetus.
34. A . Using bicarbonate would increase the amount of sodium
ingested, which can cause complications. Eating low-sodium crackers
would be appropriate. Since liquids can increase nausea avoiding them in
the morning hours when nausea is usually the strongest is appropriate.
Eating six small meals a day would keep the stomach full, which often
decrease nausea.
35. B . Ballottement indicates passive movement of the unengaged
fetus. Ballottement is not a contraction. Fetal kicking felt by the client
represents quickening. Enlargement and softening of the uterus is known
as Piskacek’s sign.
36. B . Chadwick’s sign refers to the purple-blue tinge of the cervix.
Braxton Hicks contractions are painless contractions beginning around
the 4th month. Goodell’s sign indicates softening of the cervix. Flexibility
of the uterus against the cervix is known as McDonald’s sign.
37. C . Breathing techniques can raise the pain threshold and reduce
the perception of pain. They also promote relaxation. Breathing
techniques do not eliminate pain, but they can reduce it. Positioning, not
breathing, increases uteroplacental perfusion.
38. A . The client’s labor is hypotonic. The nurse should call the physical
and obtain an order for an infusion of oxytocin, which will assist the uterus
to contact more forcefully in an attempt to dilate the cervix.
Administering light sedative would be done for hypertonic uterine
contractions. Preparing for cesarean section is unnecessary at this time.
Oxytocin would increase the uterine contractions and hopefully progress
labor before a cesarean would be necessary. It is too early to anticipate
client pushing with contractions.
39. D . The signs indicate placenta previa and vaginal exam to
determine cervical dilation would not be done because it could cause
hemorrhage. Assessing maternal vital signs can help determine maternal
physiologic status. Fetal heart rate is important to assess fetal well-being
and should be done. Monitoring the contractions will help evaluate the
progress of labor.
40. D . A complete placenta previa occurs when the placenta covers
the opening of the uterus, thus blocking the passageway for the baby.
This response explains what a complete previa is and the reason the
baby cannot come out except by cesarean delivery. Telling the client to
ask the physician is a poor response and would increase the patient’s
anxiety. Although a cesarean would help to prevent hemorrhage, the
statement does not explain why the hemorrhage could occur. With a
complete previa, the placenta is covering all the cervix, not just most of it.
41. B . With a face presentation, the head is completely extended. With
a vertex presentation, the head is completely or partially flexed. With a
brow (forehead) presentation, the head would be partially extended.
42. D . With this presentation, the fetal upper torso and back face the left
upper maternal abdominal wall. The fetal heart rate would be most
audible above the maternal umbilicus and to the left of the middle. The
other positions would be incorrect.
43. C. The greenish tint is due to the presence of meconium. Lanugo is
the soft, downy hair on the shoulders and back of the fetus.
Hydramnios represents excessive amniotic fluid. Vernix is the white,
cheesy substance covering the fetus.
44. D . In a breech position, because of the space between the presenting
part and the cervix, prolapse of the umbilical cord is common. Quickening
is the woman’s first perception of fetal movement. Ophthalmia
neonatorum usually results from maternal gonorrhea and is conjunctivitis.
Pica refers to the oral intake of nonfood substances.
45. A . Dizygotic (fraternal) twins involve two ova fertilized by separate
sperm. Monozygotic (identical) twins involve a common placenta, same
genotype, and common chorion.
46. C . The zygote is the single cell that reproduces itself after conception.
The chromosome is the material that makes up the cell and is gained
from each parent. Blastocyst and trophoblast are later terms for the
embryo after zygote.
47. D . Prepared childbirth was the direct result of the 1950’s challenging of
the routine use of analgesic and anesthetics during childbirth. The LDRP
was a much later concept and was not a direct result of the challenging
of routine use of analgesics and anesthetics during childbirth. Roles
for nurse midwives and clinical nurse specialists did not develop from
this challenge.
48. C . The ischial spines are located in the mid-pelvic region and could
be narrowed due to the previous pelvic injury. The symphysis pubis,
sacral promontory, and pubic arch are not part of the mid-pelvis.
49. B . Variations in the length of the menstrual cycle are due to variations
in the proliferative phase. The menstrual, secretory and ischemic phases
do not contribute to this variation.
50. B . Testosterone is produced by the Leyding cells in the
seminiferous tubules. Follicle-stimulating hormone and leuteinzing
hormone are released by the anterior pituitary gland. The hypothalamus is
responsible for releasing gonadotropin-releasing hormone.
PNLE III for Medical Surgical Nursing
1. Marco who was diagnosed with brain tumor was scheduled for craniotomy.
In preventing the development of cerebral edema after surgery, the nurse
should expect the use of:

A. Diuretics
B. Antihypertensive
C. Steroids
D. Anticonvulsants
2. Halfway through the administration of blood, the female client complains of
lumbar pain. After stopping the infusion Nurse Hazel should:

A. Increase the flow of normal saline


B. Assess the pain further
C. Notify the blood bank
D. Obtain vital signs.
3. Nurse Maureen knows that the positive diagnosis for HIV infection is made
based on which of the following:

A. A history of high risk sexual behaviors.


B. Positive ELISA and western blot tests
C. Identification of an associated opportunistic infection
D. Evidence of extreme weight loss and high fever
4. Nurse Maureen is aware that a client who has been diagnosed with chronic
renal failure recognizes an adequate amount of high-biologic-value protein
when the food the client selected from the menu was:

A. Raw carrots
B. Apple juice
C. Whole wheat bread
D. Cottage cheese
5. Kenneth who has diagnosed with uremic syndrome has the potential to
develop complications. Which among the following complications should the
nurse anticipates:

A. Flapping hand tremors


B. An elevated hematocrit level
C. Hypotension
D. Hypokalemia
6. A client is admitted to the hospital with benign prostatic hyperplasia, the
nurse most relevant assessment would be:

A. Flank pain radiating in the groin


B. Distention of the lower abdomen
C. Perineal edema
D. Urethral discharge
7. A client has undergone with penile implant. After 24 hrs of surgery, the
client’s scrotum was edematous and painful. The nurse should:

A. Assist the client with sitz bath


B. Apply war soaks in the scrotum
C. Elevate the scrotum using a soft support
D. Prepare for a possible incision and drainage.
8. Nurse hazel receives emergency laboratory results for a client with chest
pain and immediately informs the physician. An increased myoglobin level
suggests which of the following?
A. Liver disease
B. Myocardial damage
C. Hypertension
D. Cancer
9. Nurse Maureen would expect the a client with mitral stenosis would
demonstrate symptoms associated with congestion in the:

A. Right atrium
B. Superior vena cava
C. Aorta
D. Pulmonary
10. A client has been diagnosed with hypertension. The nurse priority nursing
diagnosis would be:

A. Ineffective health maintenance


B. Impaired skin integrity
C. Deficient fluid volume
D. Pain
11. Nurse Hazel teaches the client with angina about common expected side
effects of nitroglycerin including:

A. high blood pressure


B. stomach cramps
C. headache
D. shortness of breath
12. The following are lipid abnormalities. Which of the following is a risk
factor for the development of atherosclerosis and PVD?

A. High levels of low density lipid (LDL) cholesterol


B. High levels of high density lipid (HDL) cholesterol
C. Low concentration triglycerides
D. Low levels of LDL cholesterol.
13. Which of the following represents a significant risk immediately after
surgery for repair of aortic aneurysm?

A. Potential wound infection


B. Potential ineffective coping
C. Potential electrolyte balance
D. Potential alteration in renal perfusion
14. Nurse Josie should instruct the client to eat which of the following foods
to obtain the best supply of Vitamin B12?

A. dairy products
B. vegetables
C. Grains
D. Broccoli
15. Karen has been diagnosed with aplastic anemia. The nurse monitors for
changes in which of the following physiologic functions?

A. Bowel function
B. Peripheral sensation
C. Bleeding tendencies
D. Intake and out put
16. Lydia is scheduled for elective splenectomy. Before the clients goes to
surgery, the nurse in charge final assessment would be:

A. signed consent
B. vital signs
C. name band
D. empty bladder
17. What is the peak age range in acquiring acute lymphocytic leukemia
(ALL)?

A. 4 to 12 years.
B. 20 to 30 years
C. 40 to 50 years
D. 60 60 70 years
18. Marie with acute lymphocytic leukemia suffers from nausea and
headache. These clinical manifestations may indicate all of the following
except

A. effects of radiation
B. chemotherapy side effects
C. meningeal irritation
D. gastric distension
19. A client has been diagnosed with Disseminated Intravascular Coagulation
(DIC). Which of the following is contraindicated with the client?

A. Administering Heparin
B. Administering Coumadin
C. Treating the underlying cause
D. Replacing depleted blood products
20. Which of the following findings is the best indication that fluid
replacement for the client with hypovolemic shock is adequate?

A. Urine output greater than 30ml/hr


B. Respiratory rate of 21 breaths/minute
C. Diastolic blood pressure greater than 90 mmhg
D. Systolic blood pressure greater than 110 mmhg
21. Which of the following signs and symptoms would Nurse Maureen include
in teaching plan as an early manifestation of laryngeal cancer?

A. Stomatitis
B. Airway obstruction
C. Hoarseness
D. Dysphagia
22. Karina a client with myasthenia gravis is to receive immunosuppressive
therapy. The nurse understands that this therapy is effective because it:

A. Promotes the removal of antibodies that impair the transmission of


impulses
B. Stimulates the production of acetylcholine at the neuromuscular
junction.
C. Decreases the production of autoantibodies that attack the
acetylcholine receptors.
D. Inhibits the breakdown of acetylcholine at the neuromuscular junction.
23. A female client is receiving IV Mannitol. An assessment specific to safe
administration of the said drug is:

A. Vital signs q4h


B. Weighing daily
C. Urine output hourly
D. Level of consciousness q4h
24. Patricia a 20 year old college student with diabetes mellitus requests
additional information about the advantages of using a pen like insulin
delivery devices. The nurse explains that the advantages of these devices over
syringes includes:

A. Accurate dose delivery


B. Shorter injection time
C. Lower cost with reusable insulin cartridges
D. Use of smaller gauge needle.
25. A male client’s left tibia is fractures in an automobile accident, and a cast
is applied. To assess for damage to major blood vessels from the fracture
tibia, the nurse in charge should monitor the client for:

A. Swelling of the left thigh


B. Increased skin temperature of the foot
C. Prolonged reperfusion of the toes after blanching
D. Increased blood pressure
26. After a long leg cast is removed, the male client should:

A. Cleanse the leg by scrubbing with a brisk motion


B. Put leg through full range of motion twice daily
C. Report any discomfort or stiffness to the physician
D. Elevate the leg when sitting for long periods of time.
27. While performing a physical assessment of a male client with gout of the
great toe, NurseVivian should assess for additional tophi (urate deposits) on
the:

A. Buttocks
B. Ears
C. Face
D. Abdomen
28. Nurse Katrina would recognize that the demonstration of crutch walking
with tripod gait was understood when the client places weight on the:

A. Palms of the hands and axillary regions


B. Palms of the hand
C. Axillary regions
D. Feet, which are set apart
29. Mang Jose with rheumatoid arthritis states, “the only time I am without
pain is when I lie in bed perfectly still”. During the convalescent stage, the
nurse in charge with Mang Jose should encourage:

A. Active joint flexion and extension


B. Continued immobility until pain subsides
C. Range of motion exercises twice daily
D. Flexion exercises three times daily
30. A male client has undergone spinal surgery, the nurse should:

A. Observe the client’s bowel movement and voiding patterns


B. Log-roll the client to prone position
C. Assess the client’s feet for sensation and circulation
D. Encourage client to drink plenty of fluids
31. Marina with acute renal failure moves into the diuretic phase after one
week of therapy. During this phase the client must be assessed for signs of
developing:

A. Hypovolemia
B. renal failure
C. metabolic acidosis
D. hyperkalemia
32. Nurse Judith obtains a specimen of clear nasal drainage from a client with
a head injury. Which of the following tests differentiates mucus from
cerebrospinal fluid (CSF)?

A. Protein
B. Specific gravity
C. Glucose
D. Microorganism
33. A 22 year old client suffered from his first tonic-clonic seizure. Upon
awakening the client asks the nurse, “What caused me to have a
seizure? Which of the following would the nurse include in the primary cause
of tonic clonic seizures in adults more the 20 years?
A. Electrolyte imbalance
B. Head trauma
C. Epilepsy
D. Congenital defect
34. What is the priority nursing assessment in the first 24 hours after
admission of the client with thrombotic CVA?

A. Pupil size and papillary response


B. cholesterol level
C. Echocardiogram
D. Bowel sounds
35. Nurse Linda is preparing a client with multiple sclerosis for discharge from
the hospital to home. Which of the following instruction is most appropriate?

A. “Practice using the mechanical aids that you will need when future
disabilities arise”.
B. “Follow good health habits to change the course of the disease”.
C. “Keep active, use stress reduction strategies, and avoid fatigue.
D. “You will need to accept the necessity for a quiet and inactive lifestyle”.
36. The nurse is aware the early indicator of hypoxia in the unconscious client
is:

A. Cyanosis
B. Increased respirations
C. Hypertension
D. Restlessness
37. A client is experiencing spinal shock. Nurse Myrna should expect the
function of the bladder to be which of the following?

A. Normal
B. Atonic
C. Spastic
D. Uncontrolled
38. Which of the following stage the carcinogen is irreversible?

A. Progression stage
B. Initiation stage
C. Regression stage
D. Promotion stage
39. Among the following components thorough pain assessment, which is the
most significant?

A. Effect
B. Cause
C. Causing factors
D. Intensity
40. A 65 year old female is experiencing flare up of pruritus. Which of the
client’s action could aggravate the cause of flare ups?

A. Sleeping in cool and humidified environment


B. Daily baths with fragrant soap
C. Using clothes made from 100% cotton
D. Increasing fluid intake
41. Atropine sulfate (Atropine) is contraindicated in all but one of the
following client?

A. A client with high blood


B. A client with bowel obstruction
C. A client with glaucoma
D. A client with U.T.I
42. Among the following clients, which among them is high risk for potential
hazards from the surgical experience?

A. 67-year-old client
B. 49-year-old client
C. 33-year-old client
D. 15-year-old client
43. Nurse Jon assesses vital signs on a client undergone epidural anesthesia.
Which of the following would the nurse assess next?

A. Headache
B. Bladder distension
C. Dizziness
D. Ability to move legs
44. Nurse Katrina should anticipate that all of the following drugs may be
used in the attempt to control the symptoms of Meniere’s disease except:

A. Antiemetics
B. Diuretics
C. Antihistamines
D. Glucocorticoids
45. Which of the following complications associated with tracheostomy tube?

A. Increased cardiac output


B. Acute respiratory distress syndrome (ARDS)
C. Increased blood pressure
D. Damage to laryngeal nerves
46. Nurse Faith should recognize that fluid shift in an client with burn injury
results from increase in the:

A. Total volume of circulating whole blood


B. Total volume of intravascular plasma
C. Permeability of capillary walls
D. Permeability of kidney tubules
47. An 83-year-old woman has several ecchymotic areas on her right arm. The
bruises are probably caused by:

A. increased capillary fragility and permeability


B. increased blood supply to the skin
C. self inflicted injury
D. elder abuse
48. Nurse Anna is aware that early adaptation of client with renal carcinoma
is:

A. Nausea and vomiting


B. flank pain
C. weight gain
D. intermittent hematuria
49. A male client with tuberculosis asks Nurse Brian how long the
chemotherapy must be continued. Nurse Brian’s accurate reply would be:
A. 1 to 3 weeks
B. 6 to 12 months
C. 3 to 5 months
D. 3 years and more
50. A client has undergone laryngectomy. The immediate nursing priority
would be:

A. Keep trachea free of secretions


B. Monitor for signs of infection
C. Provide emotional support
D. Promote means of communication
Answers and Rationales
1. C . Glucocorticoids (steroids) are used for their anti-inflammatory
action, which decreases the development of edema.
2. A . The blood must be stopped at once, and then normal saline should
be infused to keep the line patent and maintain blood volume.
3. B . These tests confirm the presence of HIV antibodies that occur in
response to the presence of the human immunodeficiency virus (HIV).
4. D . One cup of cottage cheese contains approximately 225 calories, 27
g of protein, 9 g of fat, 30 mg cholesterol, and 6 g of carbohydrate.
Proteins of high biologic value (HBV) contain optimal levels of amino
acids essential for life.
5. A . Elevation of uremic waste products causes irritation of the nerves,
resulting in flapping hand tremors.
6. B . This indicates that the bladder is distended with urine, therefore
palpable.
7. C . Elevation increases lymphatic drainage, reducing edema and pain.
8. B . Detection of myoglobin is a diagnostic tool to determine
whether myocardial damage has occurred.
9. D . When mitral stenosis is present, the left atrium has difficulty
emptying its contents into the left ventricle because there is no valve to
prevent back ward flow into the pulmonary vein, the pulmonary circulation
is under pressure.
10. A . Managing hypertension is the priority for the client with
hypertension. Clients with hypertension frequently do not experience pain,
deficient volume, or impaired skin integrity. It is the asymptomatic nature
of hypertension that makes it so difficult to treat.
11. C . Because of its widespread vasodilating effects, nitroglycerin
often produces side effects such as headache, hypotension and dizziness.
12. A. An increased in LDL cholesterol concentration has been documented
at risk factor for the development of atherosclerosis. LDL cholesterol is
not broken down into the liver but is deposited into the wall of the blood
vessels.
13. D . There is a potential alteration in renal perfusion manifested by
decreased urine output. The altered renal perfusion may be related to
renal artery embolism, prolonged hypotension, or prolonged aortic cross-
clamping during the surgery.
14. A . Good source of vitamin B12 are dairy products and meats.
15. C . Aplastic anemia decreases the bone marrow production of RBC’s,
white blood cells, and platelets. The client is at risk for bruising and
bleeding tendencies.
16. B . An elective procedure is scheduled in advance so that all
preparations can be completed ahead of time. The vital signs are the final
check that must be completed before the client leaves the room so that
continuity of care and assessment is provided for.
17. A . The peak incidence of Acute Lymphocytic Leukemia (ALL) is 4 years
of age. It is uncommon after 15 years of age.
18. D . Acute Lymphocytic Leukemia (ALL) does not cause gastric
distention. It does invade the central nervous system, and clients
experience headaches and vomiting from meningeal irritation.
19. B . Disseminated Intravascular Coagulation (DIC) has not been found
to respond to oral anticoagulants such as Coumadin.
20. A . Urine output provides the most sensitive indication of the client’s
response to therapy for hypovolemic shock. Urine output should be
consistently greater than 30 to 35 mL/hr.
21. C . Early warning signs of laryngeal cancer can vary depending on
tumor location. Hoarseness lasting 2 weeks should be evaluated because
it is one of the most common warning signs.
22. C . Steroids decrease the body’s immune response thus decreasing
the production of antibodies that attack the acetylcholine receptors at
the neuromuscular junction
23. C . The osmotic diuretic mannitol is contraindicated in the presence
of inadequate renal function or heart failure because it increases
the intravascular volume that must be filtered and excreted by the kidney.
24. A . These devices are more accurate because they are easily to used
and have improved adherence in insulin regimens by young people
because the medication can be administered discreetly.
25. C . Damage to blood vessels may decrease the circulatory perfusion of
the toes, this would indicate the lack of blood supply to the extremity.
26. D . Elevation will help control the edema that usually occurs.
27. B . Uric acid has a low solubility, it tends to precipitate and form
deposits at various sites where blood flow is least active, including
cartilaginous tissue such as the ears.
28. B . The palms should bear the client’s weight to avoid damage to the
nerves in the axilla.
29. A . Active exercises, alternating extension, flexion, abduction, and
adduction, mobilize exudates in the joints relieves stiffness and pain.
30. C . Alteration in sensation and circulation indicates damage to the
spinal cord, if these occurs notify physician immediately.
31. A . In the diuretic phase fluid retained during the oliguric phase is
excreted and may reach 3 to 5 liters daily, hypovolemia may occur and
fluids should be replaced.
32. C . The constituents of CSF are similar to those of blood plasma.
An examination for glucose content is done to determine whether a body
fluid is a mucus or a CSF. A CSF normally contains glucose.
33. B . Trauma is one of the primary cause of brain damage and seizure
activity in adults. Other common causes of seizure activity in adults
include neoplasms, withdrawal from drugs and alcohol, and vascular
disease.
34. A . It is crucial to monitor the pupil size and papillary response to
indicate changes around the cranial nerves.
35. C . The nurse most positive approach is to encourage the client with
multiple sclerosis to stay active, use stress reduction techniques and
avoid fatigue because it is important to support the immune system while
remaining active.
36. D . Restlessness is an early indicator of hypoxia. The nurse should
suspect hypoxia in unconscious client who suddenly becomes restless.
37. B . In spinal shock, the bladder becomes completely atonic and will
continue to fill unless the client is catheterized.
38. A . Progression stage is the change of tumor from the preneoplastic
state or low degree of malignancy to a fast growing tumor that cannot be
reversed.
39. D . Intensity is the major indicative of severity of pain and it is important
for the evaluation of the treatment.
40. B . The use of fragrant soap is very drying to skin hence causing the
pruritus.
41. C . Atropine sulfate is contraindicated with glaucoma patients because
it increases intraocular pressure.
42. A . A 67 year old client is greater risk because the older adult client is
more likely to have a less-effective immune system.
43. B . The last area to return sensation is in the perineal area, and the nurse
in charge should monitor the client for distended bladder.
44. D . Glucocorticoids play no significant role in disease treatment.
45. D . Tracheostomy tube has several potential complications including
bleeding, infection and laryngeal nerve damage.
46. C . In burn, the capillaries and small vessels dilate, and cell damage
cause the release of a histamine-like substance. The substance causes
the capillary walls to become more permeable and significant quantities
of fluid are lost.
47. A . Aging process involves increased capillary fragility and permeability.
Older adults have a decreased amount of subcutaneous fat and cause an
increased incidence of bruise like lesions caused by collection of
extravascular blood inloosely structured dermis.
48. D . Intermittent pain is the classic sign of renal carcinoma. It is primarily
due to capillary erosion by the cancerous growth.
49. B . Tubercle bacillus is a drug resistant organism and takes a long time
to be eradicated. Usually a combination of three drugs is used for
minimum of 6 months and at least six months beyond culture conversion.
50. A . Patent airway is the most priority; therefore removal of secretions
is necessary.
PNLE IV for Psychiatric Nursing
1. Marco approached Nurse Trish asking for advice on how to deal with his
alcohol addiction. Nurse Trish should tell the client that the only effective
treatment for alcoholism is:

A. Psychotherapy
B. Alcoholics anonymous (A.A.)
C. Total abstinence
D. Aversion Therapy
2. Nurse Hazel is caring for a male client who experience false
sensory perceptions with no basis in reality. This perception is known as:

A. Hallucinations
B. Delusions
C. Loose associations
D. Neologisms
3. Nurse Monet is caring for a female client who has suicidal tendency. When
accompanying the client to the restroom, Nurse Monet should…

A. Give her privacy


B. Allow her to urinate
C. Open the window and allow her to get some fresh air
D. Observe her
4. Nurse Maureen is developing a plan of care for a female client
with anorexia nervosa. Which action should the nurse include in the plan?

A. Provide privacy during meals


B. Set-up a strict eating plan for the client
C. Encourage client to exercise to reduce anxiety
D. Restrict visits with the family
5. A client is experiencing anxiety attack. The most appropriate
nursing intervention should include?

A. Turning on the television


B. Leaving the client alone
C. Staying with the client and speaking in short sentences
D. Ask the client to play with other clients
6. A female client is admitted with a diagnosis of delusions of
GRANDEUR. This diagnosis reflects a belief that one is:

A. Being Killed
B. Highly famous and important
C. Responsible for evil world
D. Connected to client unrelated to oneself
7. A 20 year old client was diagnosed with dependent personality
disorder. Which behavior is not likely to be evidence of ineffective individual
coping?

A. Recurrent self-destructive behavior


B. Avoiding relationship
C. Showing interest in solitary activities
D. Inability to make choices and decision without advise
8. A male client is diagnosed with schizotypal personality disorder.
Which signs would this client exhibit during social situation?

A. Paranoid thoughts
B. Emotional affect
C. Independence need
D. Aggressive behavior
9. Nurse Claire is caring for a client diagnosed with bulimia. The
most appropriate initial goal for a client diagnosed with bulimia is?

A. Encourage to avoid foods


B. Identify anxiety causing situations
C. Eat only three meals a day
D. Avoid shopping plenty of groceries
10. Nurse Tony was caring for a 41 year old female client. Which behavior
by the client indicates adult cognitive development?

A. Generates new levels of awareness


B. Assumes responsibility for her actions
C. Has maximum ability to solve problems and learn new skills
D. Her perception are based on reality
11.A neuromuscular blocking agent is administered to a client before
ECT therapy. The Nurse should carefully observe the client for?

A. Respiratory difficulties
B. Nausea and vomiting
C. Dizziness
D. Seizures
12.A 75 year old client is admitted to the hospital with the diagnosis
of dementia of the Alzheimer’s type and depression. The symptom that
is unrelated to depression would be?

A. Apathetic response to the environment


B. “I don’t know” answer to questions
C. Shallow of labile effect
D. Neglect of personal hygiene
13.Nurse Trish is working in a mental health facility; the nurse priority
nursing intervention for a newly admitted client with bulimia nervosa would be
to?

A. Teach client to measure I & O


B. Involve client in planning daily meal
C. Observe client during meals
D. Monitor client continuously
14.Nurse Patricia is aware that the major health complication associated
with intractable anorexia nervosa would be?

A. Cardiac dysrhythmias resulting to cardiac arrest


B. Glucose intolerance resulting in protracted hypoglycemia
C. Endocrine imbalance causing cold amenorrhea
D. Decreased metabolism causing cold intolerance
15.Nurse Anna can minimize agitation in a disturbed client by?

A. Increasing stimulation
B. limiting unnecessary interaction
C. increasing appropriate sensory perception
D. ensuring constant client and staff contact
16.A 39 year old mother with obsessive-compulsive disorder has
become immobilized by her elaborate hand washing and walking rituals.
Nurse Trish recognizes that the basis of O.C. disorder is often:

A. Problems with being too conscientious


B. Problems with anger and remorse
C. Feelings of guilt and inadequacy
D. Feeling of unworthiness and hopelessness
17.Mario is complaining to other clients about not being allowed by staff
to keep food in his room. Which of the following interventions would be
most appropriate?

A. Allowing a snack to be kept in his room


B. Reprimanding the client
C. Ignoring the clients behavior
D. Setting limits on the behavior
18.Conney with borderline personality disorder who is to be discharge
soon threatens to “do something” to herself if discharged. Which of the
following actions by the nurse would be most important?

A. Ask a family member to stay with the client at home temporarily


B. Discuss the meaning of the client’s statement with her
C. Request an immediate extension for the client
D. Ignore the clients statement because it’s a sign of manipulation
19.Joey a client with antisocial personality disorder belches loudly. A
staff member asks Joey, “Do you know why people find you repulsive?”
this statement most likely would elicit which of the following client reaction?

A. Depensiveness
B. Embarrassment
C. Shame
D. Remorsefulness
20.Which of the following approaches would be most appropriate to use
with a client suffering from narcissistic personality disorder when
discrepancies exist between what the client states and what actually exist?

A. Rationalization
B. Supportive confrontation
C. Limit setting
D. Consistency
21.Cely is experiencing alcohol withdrawal exhibits tremors, diaphoresis
and hyperactivity. Blood pressure is 190/87 mmhg and pulse is 92 bpm.
Which of the medications would the nurse expect to administer?

A. Naloxone (Narcan)
B. Benzlropine (Cogentin)
C. Lorazepam (Ativan)
D. Haloperidol (Haldol)
22.Which of the following foods would the nurse Trish eliminate from the
diet of a client in alcohol withdrawal?

A. Milk
B. Orange Juice
C. Soda
D. Regular Coffee
23.Which of the following would Nurse Hazel expect to assess for a
client who is exhibiting late signs of heroin withdrawal?

A. Yawning & diaphoresis


B. Restlessness & Irritability
C. Constipation & steatorrhea
D. Vomiting and Diarrhea
24.To establish open and trusting relationship with a female client who
has been hospitalized with severe anxiety, the nurse in charge should?

A. Encourage the staff to have frequent interaction with the client


B. Share an activity with the client
C. Give client feedback about behavior
D. Respect client’s need for personal space
25. Nurse Monette recognizes that the focus of environmental
(MILIEU) therapy is to:

A. Manipulate the environment to bring about positive changes in behavior


B. Allow the client’s freedom to determine whether or not they will
be involved in activities
C. Role play life events to meet individual needs
D. Use natural remedies rather than drugs to control behavior
26.Nurse Trish would expect a child with a diagnosis of reactive
attachment disorder to:

A. Have more positive relation with the father than the mother
B. Cling to mother & cry on separation
C. Be able to develop only superficial relation with the others
D. Have been physically abuse
27.When teaching parents about childhood depression Nurse Trina
should say?

A. It may appear acting out behavior


B. Does not respond to conventional treatment
C. Is short in duration & resolves easily
D. Looks almost identical to adult depression
28.Nurse Perry is aware that language development in autistic
child resembles:

A. Scanning speech
B. Speech lag
C. Shuttering
D. Echolalia
29.A 60 year old female client who lives alone tells the nurse at
the community health center “I really don’t need anyone to talk to”. The TV
is my best friend. The nurse recognizes that the client is using the
defense mechanism known as?

A. Displacement
B. Projection
C. Sublimation
D. Denial
30.When working with a male client suffering phobia about black cats,
Nurse Trish should anticipate that a problem for this client would be?

A. Anxiety when discussing phobia


B. Anger toward the feared object
C. Denying that the phobia exist
D. Distortion of reality when completing daily routines
31.Linda is pacing the floor and appears extremely anxious. The duty
nurse approaches in an attempt to alleviate Linda’s anxiety. The
most therapeutic question by the nurse would be?

A. Would you like to watch TV?


B. Would you like me to talk with you?
C. Are you feeling upset now?
D. Ignore the client
32.Nurse Penny is aware that the symptoms that distinguish post
traumatic stress disorder from other anxiety disorder would be:

A. Avoidance of situation & certain activities that resemble the stress


B. Depression and a blunted affect when discussing the
traumatic situation
C. Lack of interest in family & others
D. Re-experiencing the trauma in dreams or flashback
33.Nurse Benjie is communicating with a male client with substance-
induced persisting dementia; the client cannot remember facts and fills in the
gaps with imaginary information. Nurse Benjie is aware that this is typical of?

A. Flight of ideas
B. Associative looseness
C. Confabulation
D. Concretism
34.Nurse Joey is aware that the signs & symptoms that would be
most specific for diagnosis anorexia are?

A. Excessive weight loss, amenorrhea & abdominal distension


B. Slow pulse, 10% weight loss & alopecia
C. Compulsive behavior, excessive fears & nausea
D. Excessive activity, memory lapses & an increased pulse
35.A characteristic that would suggest to Nurse Anne that an adolescent
may have bulimia would be:

A. Frequent regurgitation & re-swallowing of food


B. Previous history of gastritis
C. Badly stained teeth
D. Positive body image
36.Nurse Monette is aware that extremely depressed clients seem to do
best in settings where they have:

A. Multiple stimuli
B. Routine Activities
C. Minimal decision making
D. Varied Activities
37.To further assess a client’s suicidal potential. Nurse Katrina should
be especially alert to the client expression of:

A. Frustration & fear of death


B. Anger & resentment
C. Anxiety & loneliness
D. Helplessness & hopelessness
38.A nursing care plan for a male client with bipolar I disorder should include:

A. Providing a structured environment


B. Designing activities that will require the client to maintain contact with
reality
C. Engaging the client in conversing about current affairs
D. Touching the client provide assurance
39.When planning care for a female client using ritualistic behavior,
Nurse Gina must recognize that the ritual:

A. Helps the client focus on the inability to deal with reality


B. Helps the client control the anxiety
C. Is under the client’s conscious control
D. Is used by the client primarily for secondary gains
40.A 32 year old male graduate student, who has become
increasingly withdrawn and neglectful of his work and personal hygiene, is
brought to the psychiatric hospital by his parents. After detailed assessment,
a diagnosis of schizophrenia is made. It is unlikely that the client
will demonstrate:

A. Low self esteem


B. Concrete thinking
C. Effective self boundaries
D. Weak ego
41.A 23 year old client has been admitted with a diagnosis of
schizophrenia says to the nurse “Yes, its march, March is little woman”. That’s
literal you know”. These statement illustrate:

A. Neologisms
B. Echolalia
C. Flight of ideas
D. Loosening of association
42.A long term goal for a paranoid male client who has unjustifiably
accused his wife of having many extramarital affairs would be to help the
client develop:

A. Insight into his behavior


B. Better self control
C. Feeling of self worth
D. Faith in his wife
43.A male client who is experiencing disordered thinking about food
being poisoned is admitted to the mental health unit. The nurse uses
which communication technique to encourage the client to eat dinner?
A. Focusing on self-disclosure of own food preference
B. Using open ended question and silence
C. Offering opinion about the need to eat
D. Verbalizing reasons that the client may not choose to eat
44.Nurse Nina is assigned to care for a client diagnosed with
Catatonic Stupor. When Nurse Nina enters the client’s room, the client is
found lying on the bed with a body pulled into a fetal position. Nurse Nina
should?

A. Ask the client direct questions to encourage talking


B. Rake the client into the dayroom to be with other clients
C. Sit beside the client in silence and occasionally ask open-
ended question
D. Leave the client alone and continue with providing care to the
other clients
45.Nurse Tina is caring for a client with delirium and states that “look at
the spiders on the wall”. What should the nurse respond to the client?

A. “You’re having hallucination, there are no spiders in this room at all”


B. “I can see the spiders on the wall, but they are not going to hurt you”
C. “Would you like me to kill the spiders”
D. “I know you are frightened, but I do not see spiders on the wall”
46.Nurse Jonel is providing information to a community group about
violence in the family. Which statement by a group member would indicate a
need to provide additional information?

A. “Abuse occurs more in low-income families”


B. “Abuser Are often jealous or self-centered”
C. “Abuser use fear and intimidation”
D. “Abuser usually have poor self-esteem”
47.During electroconvulsive therapy (ECT) the client receives oxygen by mask
via positive pressure ventilation. The nurse assisting with this procedure
knows that positive pressure ventilation is necessary because?

A. Anesthesia is administered during the procedure


B. Decrease oxygen to the brain increases confusion and disorientation
C. Grand mal seizure activity depresses respirations
D. Muscle relaxations given to prevent injury during seizure
activity depress respirations.
48.When planning the discharge of a client with chronic anxiety, Nurse
Chris evaluates achievement of the discharge maintenance goals. Which
goal would be most appropriately having been included in the plan of
care requiring evaluation?

A. The client eliminates all anxiety from daily situations


B. The client ignores feelings of anxiety
C. The client identifies anxiety producing situations
D. The client maintains contact with a crisis counselor
49.Nurse Tina is caring for a client with depression who has not responded
to antidepressant medication. The nurse anticipates that what
treatment procedure may be prescribed.

A. Neuroleptic medication
B. Short term seclusion
C. Psychosurgery
D. Electroconvulsive therapy
50.Mario is admitted to the emergency room with drug-included
anxiety related to over ingestion of prescribed antipsychotic medication. The
most important piece of information the nurse in charge should obtain initially
is the:

A. Length of time on the med.


B. Name of the ingested medication & the amount ingested
C. Reason for the suicide attempt
D. Name of the nearest relative & their phone number
Answers and Rationales
1. C . Total abstinence is the only effective treatment for alcoholism
2. A . Hallucinations are visual, auditory, gustatory, tactile or
olfactory perceptions that have no basis in reality.
3. D . The Nurse has a responsibility to observe continuously the acutely
suicidal client. The Nurse should watch for clues, such as communicating
suicidal thoughts, and messages; hoarding medications and talking about
death.
4. B . Establishing a consistent eating plan and monitoring client’s weight
are important to this disorder.
5. C . Appropriate nursing interventions for an anxiety attack include using
short sentences, staying with the client, decreasing stimuli, remaining
calm and medicating as needed.
6. B . Delusion of grandeur is a false belief that one is highly famous
and important.
7. D . Individual with dependent personality disorder typically
shows indecisiveness submissiveness and clinging behavior so that
others will make decisions with them.
8. A . Clients with schizotypal personality disorder experience excessive
social anxiety that can lead to paranoid thoughts
9. B . Bulimia disorder generally is a maladaptive coping response to
stress and underlying issues. The client should identify anxiety causing
situation that stimulate the bulimic behavior and then learn new ways of
coping with the anxiety.
10. A . An adult age 31 to 45 generates new level of awareness.
11. A . Neuromuscular Blocker, such as SUCCINYLCHOLINE
(Anectine) produces respiratory depression because it inhibits
contractions of respiratory muscles.
12. C . With depression, there is little or no emotional involvement therefore
little alteration in affect.
13. D . These clients often hide food or force vomiting; therefore they must
be carefully monitored.
14. A . These clients have severely depleted levels of sodium and
potassium because of their starvation diet and energy expenditure, these
electrolytes are necessary for cardiac functioning.
15. B . Limiting unnecessary interaction will decrease stimulation and
agitation.
16. C . Ritualistic behavior seen in this disorder is aimed at controlling guilt
and inadequacy by maintaining an absolute set pattern of behavior.
17. D . The nurse needs to set limits in the client’s manipulative behavior to
help the client control dysfunctional behavior. A consistent approach by
the staff is necessary to decrease manipulation.
18. B . Any suicidal statement must be assessed by the nurse. The nurse
should discuss the client’s statement with her to determine its meaning in
terms of suicide.
19. A . When the staff member ask the client if he wonders why others find
him repulsive, the client is likely to feel defensive because the question
is belittling. The natural tendency is to counterattack the threat to self
image.
20. B . The nurse would specifically use supportive confrontation with the
client to point out discrepancies between what the client states and what
actually exists to increase responsibility for self.
21. C . The nurse would most likely administer benzodiazepine, such as
lorazepan (ativan) to the client who is experiencing symptom: The client’s
experiences symptoms of withdrawal because of the rebound
phenomenon when the sedation of the CNS from alcohol begins to
decrease.
22. D . Regular coffee contains caffeine which acts as psychomotor
stimulants and leads to feelings of anxiety and agitation. Serving coffee
top the client may add to tremors or wakefulness.
23. D . Vomiting and diarrhea are usually the late signs of heroin
withdrawal, along with muscle spasm, fever, nausea, repetitive, abdominal
cramps and backache.
24. D . Moving to a client’s personal space increases the feeling of threat,
which increases anxiety.
25. A . Environmental (MILIEU) therapy aims at having everything in the
client’s surrounding area toward helping the client.
26. C . Children who have experienced attachment difficulties with
primary caregiver are not able to trust others and therefore relate
superficially
27. A . Children have difficulty verbally expressing their feelings, acting
out behavior, such as temper tantrums, may indicate underlying
depression.
28. D . The autistic child repeat sounds or words spoken by others.
29. D . The client statement is an example of the use of denial, a defense
that blocks problem by unconscious refusing to admit they exist
30. A . Discussion of the feared object triggers an emotional response to
the object.
31. B . The nurse presence may provide the client with support & feeling
of control.
32. D . Experiencing the actual trauma in dreams or flashback is the
major symptom that distinguishes post traumatic stress disorder from
other anxiety disorder.
33. C . Confabulation or the filling in of memory gaps with imaginary facts is
a defense mechanism used by people experiencing memory deficits.
34. A . These are the major signs of anorexia nervosa. Weight loss is
excessive (15% of expected weight)
35. C . Dental enamel erosion occurs from repeated self-induced vomiting.
36. B . Depression usually is both emotional & physical. A simple daily
routine is the best, least stressful and least anxiety producing.
37. D . The expression of these feeling may indicate that this client is
unable to continue the struggle of life.
38. A . Structure tends to decrease agitation and anxiety and to increase
the client’s feeling of security.
39. B . The rituals used by a client with obsessive compulsive disorder
help control the anxiety level by maintaining a set pattern of action.
40. C . A person with this disorder would not have adequate self-boundaries
41. D . Loose associations are thoughts that are presented without the
logical connections usually necessary for the listening to interpret the
message.
42. C . Helping the client to develop feeling of self worth would reduce the
client’s need to use pathologic defenses.
43. B . Open ended questions and silence are strategies used to
encourage clients to discuss their problem in descriptive manner.
44. C . Clients who are withdrawn may be immobile and mute, and
require consistent, repeated interventions. Communication with withdrawn
clients requires much patience from the nurse. The nurse facilitates
communication with the client by sitting in silence, asking open-ended
question and pausing to provide opportunities for the client to respond.
45. D . When hallucination is present, the nurse should reinforce reality with
the client.
46. A . Personal characteristics of abuser include low self-esteem,
immaturity, dependence, insecurity and jealousy.
47. D . A short acting skeletal muscle relaxant such as succinylcholine
(Anectine) is administered during this procedure to prevent injuries during
seizure.
48. C . Recognizing situations that produce anxiety allows the client to
prepare to cope with anxiety or avoid specific stimulus.
49. D . Electroconvulsive therapy is an effective treatment for depression
that has not responded to medication
50. B . In an emergency, lives saving facts are obtained first. The name and
the amount of medication ingested are of outmost important in treating
this potentially life threatening situation.

PNLE: FON Practice Exam for


History , Concepts and Theories
1. The four major concepts in nursing theory are the

A. Person, Environment, Nurse, Health


B. Nurse, Person, Environment, Cure
C. Promotive, Preventive, Curative, Rehabilitative
D. Person, Environment, Nursing, Health
2. The act of utilizing the environment of the patient to assist him in his
recovery is theorized by

A. Nightingale
B. Benner
C. Swanson
D. King
3. For her, Nursing is a theoretical system of knowledge that prescribes a
process of analysis and action related to care of the ill person

A. King
B. Henderson
C. Roy
D. Leininger
4. According to her, Nursing is a helping or assistive profession to persons
who are wholly or partly dependent or when those who are supposedly caring
for them are no longer able to give care.

A. Henderson
B. Orem
C. Swanson
D. Neuman
5. Nursing is a unique profession, Concerned with all the variables affecting
an individual’s response to stressors, which are intra, inter and extra personal
in nature.

A. Neuman
B. Johnson
C. Watson
D. Parse
6. The unique function of the nurse is to assist the individual, sick or well, in
the performance of those activities contributing to health that he would
perform unaided if he has the necessary strength, will and knowledge, and do
this in such a way as to help him gain independence as rapidly as possible.

A. Henderson
B. Abdellah
C. Levin
D. Peplau
7. Caring is the essence and central unifying, a dominant domain that
distinguishes nursing from other health disciplines. Care is an essential
human need.

A. Benner
B. Watson
C. Leininger
D. Swanson
8. Caring involves 5 processes, KNOWING, BEING WITH, DOING FOR,
ENABLING and MAINTAINING BELIEF.

A. Benner
B. Watson
C. Leininger
D. Swanson
9. Caring is healing, it is communicated through the consciousness of the
nurse to the individual being cared for. It allows access to higher human spirit.

A. Benner
B. Watson
C. Leininger
D. Swanson
10. Caring means that person, events, projects and things matter to people. It
reveals stress and coping options. Caring creates responsibility. It is an
inherent feature of nursing practice. It helps the nurse assist clients to recover
in the face of the illness.

A. Benner
B. Watson
C. Leininger
D. Swanson
11. Which of the following is NOT TRUE about profession according to Marie
Jahoda?

A. A profession is an organization of an occupational group based on the


application of special knowledge
B. It serves specific interest of a group
C. It is altruistic
D. Quality of work is of greater importance than the rewards
12. Which of the following is NOT an attribute of a professional?

A. Concerned with quantity


B. Self directed
C. Committed to spirit of inquiry
D. Independent
13. The most unique characteristic of nursing as a profession is

A. Education
B. Theory
C. Caring
D. Autonomy
14. This is the distinctive individual qualities that differentiate a person to
another

A. Philosophy
B. Personality
C. Charm
D. Character
15. Refers to the moral values and beliefs that are used as guides to personal
behavior and actions

A. Philosophy
B. Personality
C. Charm
D. Character
16. As a nurse manager, which of the following best describes this function?

A. Initiate modification on client’s lifestyle


B. Protect client’s right
C. Coordinates the activities of other members of the health team in
managing patient care
D. Provide in service education programs, Use accurate nursing audit,
formulate philosophy and vision of the institution
17. What best describes nurses as a care provider?

A. Determine client’s need


B. Provide direct nursing care
C. Help client recognize and cope with stressful psychological situation
D. Works in combined effort with all those involved in patient’s care
18. The nurse questions a doctors order of Morphine sulfate 50 mg, IM for a
client with pancreatitis. Which role best fit that statement?

A. Change agent
B. Client advocate
C. Case manager
D. Collaborator
19. These are nursing intervention that requires knowledge, skills and
expertise of multiple health professionals.

A. Dependent
B. Independent
C. Interdependent
D. Intradependent
20. What type of patient care model is the most common for student nurses
and private duty nurses?

A. Total patient care


B. Team nursing
C. Primary Nursing
D. Case management
21. This is the best patient care model when there are many nurses but few
patients.

A. Functional nursing
B. Team nursing
C. Primary nursing
D. Total patient care
22. This patient care model works best when there are plenty of patient but
few nurses

A. Functional nursing
B. Team nursing
C. Primary nursing
D. Total patient care
23. RN assumes 24 hour responsibility for the client to maintain continuity of
care across shifts, days or visits.
A. Functional nursing
B. Team nursing
C. Primary nursing
D. Total patient care
24. Who developed the first theory of nursing?

A. Hammurabi
B. Alexander
C. Fabiola
D. Nightingale
25. She introduces the NATURE OF NURSING MODEL.

A. Henderson
B. Nightingale
C. Parse
D. Orlando
26. She described the four conservation principle.

A. Levin
B. Leininger
C. Orlando
D. Parse
27. Proposed the HEALTH CARE SYSTEM MODEL.

A. Henderson
B. Orem
C. Parse
D. Neuman
28. Conceptualized the BEHAVIORAL SYSTEM MODEL

A. Orem
B. Johnson
C. Henderson
D. Parse
29. Developed the CLINICAL NURSING – A HELPING ART MODEL

A. Swanson
B. Hall
C. Weidenbach
D. Zderad
30. Developed the ROLE MODELING and MODELING theory

A. Erickson,Tomlin,Swain
B. Neuman
C. Newman
D. Benner and Wrubel
31. Proposed the GRAND THEORY OF NURSING AS CARING
A. Erickson, Tomlin, Swain
B. Peterson,Zderad
C. Bnner,Wrubel
D. Boykin,Schoenhofer
32. Postulated the INTERPERSONAL ASPECT OF NURSING
A. Travelbee
B. Swanson
C. Zderad
D. Peplau
33. He proposed the theory of morality that is based on MUTUAL TRUST
A. Freud
B. Erikson
C. Kohlberg
D. Peters
34. He proposed the theory of morality based on PRINCIPLES
A. Freud
B. Erikson
C. Kohlberg
D. Peters
35. Freud postulated that child adopts parental standards and traits through
A. Imitation
B. Introjection
C. Identification
D. Regression
36. According to them, Morality is measured of how people treat human being
and that a moral child strives to be kind and just
A. Zderad and Peterson
B. Benner and Wrubel
C. Fowler and Westerhoff
D. Schulman and Mekler
37. Postulated that FAITH is the way of behaving. He developed four theories
of faith and development based on his experience.
A. Giligan
B. Westerhoff
C. Fowler
D. Freud
38. He described the development of faith. He suggested that faith is a
spiritual dimension that gives meaning to a persons life. Faith according to
him, is a relational phenomenon.
A. Giligan
B. Westerhoff
C. Fowler
D. Freud
39. Established in 1906 by the Baptist foreign mission society of America.
Miss rose nicolet, was it’s first superintendent.
A. St. Paul Hospital School of nursing
B. Iloilo Mission Hospital School of nursing
C. Philippine General Hospital School of nursing
D. St. Luke’s Hospital School of nursing
40. Anastacia Giron-Tupas was the first Filipino nurse to occupy the position
of chief nurse in this hospital.
A. St. Paul Hospital
B. Iloilo Mission Hospital
C. Philippine General Hospital
D. St. Luke’s Hospital
41. She was the daughter of Hungarian kings, who feed 300-900 people
everyday in their gate, builds hospitals, and care of the poor and sick herself.
A. Elizabeth
B. Catherine
C. Nightingale
D. Sairey Gamp
42. She dies of yellow fever in her search for truth to prove that yellow fever is
carried by a mosquitoes.
A. Clara louise Maas
B. Pearl Tucker
C. Isabel Hampton Robb
D. Caroline Hampton Robb
43. He was called the father of sanitation.
A. Abraham
B. Hippocrates
C. Moses
D. Willam Halstead
44. The country where SHUSHURUTU originated
A. China
B. Egypt
C. India
D. Babylonia
45. They put girls clothes on male infants to drive evil forces away
A. Chinese
B. Egyptian
C. Indian
D. Babylonian
46. In what period of nursing does people believe in TREPHINING to drive evil
forces away?
A. Dark period
B. Intuitive period
C. Contemporary period
D. Educative period
47. This period ended when Pastor Fliedner, build Kaiserwerth institute for the
training of Deaconesses
A. Apprentice period
B. Dark period
C. Contemporary period
D. Educative period
48. Period of nursing where religious Christian orders emerged to take care of
the sick
A. Apprentice period
B. Dark period
C. Contemporary period
D. Educative period
49. Founded the second order of St. Francis of Assisi
A. St. Catherine
B. St. Anne
C. St. Clare
D. St. Elizabeth
50. This period marked the religious upheaval of Luther, Who questions the
Christian faith.
A. Apprentice period
B. Dark period
C. Contemporary period
D. Educative period
51. According to the Biopsychosocial and spiritual theory of Sister Callista
Roy, Man, As a SOCIAL being is
A. Like all other men
B. Like some other men
C. Like no other men
D. Like men
52. She conceptualized that man, as an Open system is in constant interaction
and transaction with a changing environment.
A. Roy
B. Levin
C. Neuman
D. Newman
53. In a CLOSED system, which of the following is true?
A. Affected by matter
B. A sole island in vast ocean
C. Allows input
D. Constantly affected by matter, energy, information
54. Who postulated the WHOLISTIC concept that the totality is greater than
sum of its parts?
A. Roy
B. Rogers
C. Henderson
D. Johnson
55. She theorized that man is composed of sub and supra systems.
Subsystems are cells, tissues, organs and systems while the suprasystems
are family, society and community.
A. Roy
B. Rogers
C. Henderson
D. Johnson
56. Which of the following is not true about the human needs?
A. Certain needs are common to all people
B. Needs should be followed exactly in accordance with their hierarchy
C. Needs are stimulated by internal factors
D. Needs are stimulated by external factors
57. Which of the following is TRUE about the human needs?
A. May not be deferred
B. Are not interrelated
C. Met in exact and rigid way
D. Priorities are alterable
58. According to Maslow, which of the following is NOT TRUE about a self
actualized person?
A. Understands poetry, music, philosophy, science etc.
B. Desires privacy, autonomous
C. Follows the decision of the majority, uphold justice and truth
D. Problem centered
59. According to Maslow, which of the following is TRUE about a self
actualized person?
A. Makes decision contrary to public opinion
B. Do not predict events
C. Self centered
D. Maximum degree of self conflict
60. This is the essence of mental health
A. Self awareness
B. Self actualization
C. Self esteem
D. Self worth
61. Florence nightingale is born in
A. Germany
B. Britain
C. France
D. Italy
62. Which is unlikely of Florence Nightingale?
A. Born May 12, 1840
B. Built St. Thomas school of nursing when she was 40 years old
C. Notes in nursing
D. Notes in hospital
63. What country did Florence Nightingale train in nursing?
A. Belgium
B. US
C. Germany
D. England
64. Which of the following is recognized for developing the concept of HIGH
LEVEL WELLNESS?
A. Erikson
B. Madaw
C. Peplau
D. Dunn
65. One of the expectations is for nurses to join professional association
primarily because of
A. Promotes advancement and professional growth among its members
B. Works for raising funds for nurse’s benefit
C. Facilitate and establishes acquaintances
D. Assist them and securing jobs abroad
66. Founder of the PNA
A. Julita Sotejo
B. Anastacia Giron Tupas
C. Eufemia Octaviano
D. Anesia Dionisio
67. Which of the following provides that nurses must be a member of a
national nurse organization?
A. R.A 877
B. 1981 Code of ethics approved by the house of delegates and the PNA
C. Board resolution No. 1955 Promulgated by the BON
D. RA 7164
68. Which of the following best describes the action of a nurse who
documents her nursing diagnosis?
A. She documents it and charts it whenever necessary
B. She can be accused of malpractice
C. She does it regularly as an important responsibility
D. She charts it only when the patient is acutely ill
69. Which of the following does not govern nursing practice?
A. RA 7164
B. RA 9173
C. BON Res. Code Of Ethics
D. BON Res. Scope of Nursing Practice
70. A nurse who is maintaining a private clinic in the community renders
service on maternal and child health among the neighborhood for a fee is:
A. Primary care nurse
B. Independent nurse practitioner
C. Nurse-Midwife
D. Nurse specialist
71. When was the PNA founded?
A. September 22, 1922
B. September 02, 1920
C. October 21, 1922
D. September 02, 1922
72. Who was the first president of the PNA ?
A. Anastacia Giron-Tupas
B. Loreto Tupas
C. Rosario Montenegro
D. Ricarda Mendoza
73. Defines health as the ability to maintain internal milieu. Illness according
to him/her/them is the failure to maintain internal environment.
A. Cannon
B. Bernard
C. Leddy and Pepper
D. Roy
74. Postulated that health is a state and process of being and becoming an
integrated and whole person.
A. Cannon
B. Bernard
C. Dunn
D. Roy
75. What regulates HOMEOSTASIS according to the theory of Walter Cannon?
A. Positive feedback
B. Negative feedback
C. Buffer system
D. Various mechanisms
76. Stated that health is WELLNESS. A termed define by the culture or an
individual.
A. Roy
B. Henderson
C. Rogers
D. King
77. Defined health as a dynamic state in the life cycle, and Illness as
interference in the life cycle.
A. Roy
B. Henderson
C. Rogers
D. King
78. She defined health as the soundness and wholness of developed human
structure and bodily mental functioning.
A. Orem
B. Henderson
C. Neuman
D. Clark
79. According to her, Wellness is a condition in which all parts and subparts of
an individual are in harmony with the whole system.
A. Orem
B. Henderson
C. Neuman
D. Johnson
80. Postulated that health is reflected by the organization, interaction,
interdependence and integration of the subsystem of the behavioral system.
A. Orem
B. Henderson
C. Neuman
D. Johnson
81. According to them, Well being is a subjective perception of BALANCE,
HARMONY and VITALITY
A. Leavell and Clark
B. Peterson and Zderad
C. Benner and Wruber
D. Leddy and Pepper
82. He describes the WELLNESS-ILLNESS Continuum as interaction of the
environment with well being and illness.
A. Cannon
B. Bernard
C. Dunn
D. Clark
83. An integrated method of functioning that is oriented towards maximizing
one’s potential within the limitation of the environment.
A. Well being
B. Health
C. Low level Wellness
D. High level Wellness
84. What kind of illness precursor, according to DUNN is cigarette smoking?
A. Heredity
B. Social
C. Behavioral
D. Environmental
85. According to DUNN, Overcrowding is what type of illness precursor?
A. Heredity
B. Social
C. Behavioral
D. Environmental
86. Health belief model was formulated in 1975 by who?
A. Becker
B. Smith
C. Dunn
D. Leavell and Clark
87. In health belief model, Individual perception matters. Which of the
following is highly UNLIKELY to influence preventive behavior?
A. Perceived susceptibility to an illness
B. Perceived seriousness of an illness
C. Perceived threat of an illness
D. Perceived curability of an illness
88. Which of the following is not a PERCEIVED BARRIER in preventive action?
A. Difficulty adhering to the lifestyle
B. Economic factors
C. Accessibility of health care facilities
D. Increase adherence to medical therapies
89. Conceptualizes that health is a condition of actualization or realization of
person’s potential. Avers that the highest aspiration of people is fulfillment
and complete development actualization.
A. Clinical Model
B. Role performance Model
C. Adaptive Model
D. Eudaemonistic Model
90. Views people as physiologic system and Absence of sign and symptoms
equates health.
A. Clinical Model
B. Role performance Model
C. Adaptive Model
D. Eudaemonistic Model
91. Knowledge about the disease and prior contact with it is what type of
VARIABLE according to the health belief model?
A. Demographic
B. Sociopsychologic
C. Structural
D. Cues to action
92. It includes internal and external factors that leads the individual to seek
help
A. Demographic
B. Sociopsychologic
C. Structural
D. Cues to action
93. Influence from peers and social pressure is included in what variable of
HBM?
A. Demographic
B. Sociopsychologic
C. Structural
D. Cues to action
94. Age, Sex, Race etc. is included in what variable of HBM?
A. Demographic
B. Sociopsychologic
C. Structural
D. Cues to action
95. According to Leavell and Clark’s ecologic model, All of this are factors that
affects health and illness except
A. Reservoir
B. Agent
C. Environment
D. Host
96. Is a multi dimensional model developed by PENDER that describes the
nature of persons as they interact within the environment to pursue health
A. Ecologic Model
B. Health Belief Model
C. Health Promotion Model
D. Health Prevention Model
97. Defined by Pender as all activities directed toward increasing the level of
well being and self actualization.
A. Health prevention
B. Health promotion
C. Health teaching
D. Self actualization
98. Defined as an alteration in normal function resulting in reduction of
capacities and shortening of life span.
A. Illness
B. Disease
C. Health
D. Wellness
99. Personal state in which a person feels unhealthy
A. Illness
B. Disease
C. Health
D. Wellness
100. According to her, Caring is defined as a nurturant way of responding to a
valued client towards whom the nurse feels a sense of commitment and
responsibility.
A. Benner
B. Watson
C. Leininger
D. Swanson
Answers and Rationales
1. D. Person, Environment, Nursing, Health. This is an actual board exam
question and is a common board question. Theorist always describes The
nursing profession by first defining what is NURSING, followed by the
PERSON, ENVIRONMENT and HEALTH CONCEPT. The most popular
theory was perhaps Nightingale’s. She defined nursing as the utilization of
the persons environment to assist him towards recovery. She defined the
person as somebody who has a reparative capabilities mediated and
enhanced by factors in his environment. She describes the environment as
something that would facilitate the person’s reparative process and
identified different factors like sanitation, noise, etc. that affects a
person’s reparative state.
2.  A. Nightingale. Florence nightingale do not believe in the germ theory,
and perhaps this was her biggest mistake. Yet, her theory was the first in
nursing. She believed that manipulation of environment that includes
appropriate noise, nutrition, hygiene, light, comfort, sanitation etc. could
provide the client’s body the nurturance it needs for repair and recovery.
3.  C. Roy. Remember the word “ THEOROYTICAL “ For Callista Roy,
Nursing is a theoretical body of knowledge that prescribes analysis and
action to care for an ill person. She introduced the ADAPTATION MODEL
and viewed person as a BIOSPSYCHOSOCIAL BEING. She believed that by
adaptation, Man can maintain homeostasis.
4. B. Orem. In self care deficit theory, Nursing is defined as A helping or
assistive profession to person who are wholly or partly dependent or when
people who are to give care to them are no longer available. Self care, are
the activities that a person do for himself to maintain health, life and well
being.
5. A. Neuman. Neuman divided stressors as either intra, inter and extra
personal in nature. She said that NURSING is concerned with eliminating
these stressors to obtain a maximum level of wellness. The nurse helps
the client through PRIMARY, SECONDARY AND TERTIARY prevention
modes. Please do not confuse this with LEAVELL and CLARK’S level of
prevention.
6. A. Henderson. This was an actual board question. Remember this
definition and associate it with Virginia Henderson. Henderson also
describes the NATURE OF NURSING theory. She identified 14 basic needs
of the client. She describes nursing roles as SUBSTITUTIVE : Doing
everything for the client, SUPPLEMENTARY : Helping the client and
COMPLEMENTARY : Working with the client. Breathing normally,
Eliminating waste, Eating and drinking adquately, Worship and Play are
some of the basic needs according to her.
7. C. Leininger. There are many theorist that describes nursing as CARE.
The most popular was JEAN WATSON’S Human Caring Model. But this
question pertains to Leininger’s definition of caring. CUD I LIE IN GER?
[ Could I Lie In There ] Is the Mnemonics I am using not to get confused. C
stands for CENTRAL , U stands for UNIFYING, D stands for DOMINANT
DOMAIN. I emphasize on this matter due to feedback on the last June
2006 batch about a question about CARING.
8. D. Swanson . Caring according to Swanson involves 5 processes.
Knowing means understanding the client. Being with emphasizes the
Physical presence of the nurse for the patient. Doing for means doing
things for the patient when he is incapable of doing it for himself. Enabling
means helping client transcend maturational and developmental stressors
in life while Maintaining belief is the ability of the Nurse to inculcate
meaning to these events.
9. B. Watson. The deepest and spiritual definition of Caring came from
Jean watson. For her, Caring expands the limits of openess and allows
access to higher human spirit.
10. A. Benner.  I think of CARE BEAR to facilitate retainment of BENNER. As
in, Care Benner. For her, Caring means being CONNECTED or making
things matter to people. Caring according to Benner give meaning to
illness and re establish connection.
11. B. It serves specific interest of a group.Believe it or not, you should know
the definition of profession according to Jahoda because it is asked in the
Local boards. A profession should serve the WHOLE COMMUNITY and not
just a specific intrest of a group. Everything else, are correct.
12. A. Concerned with quantity. A professional is concerned with QUALITY
and not QUANTITY. In nursing, We have methods of quality assurance and
control to evaluate the effectiveness of nursing care. Nurses, are never
concerned with QUANTITY of care provided.
13. C. Caring. Caring and caring alone, is the most unique quality of the
Nursing Profession. It is the one the delineate Nursing from other
professions.
14. B. Personality. Personality are qualities that make us different from each
other. These are impressions that we made, or the footprints that we leave
behind. This is the result of the integration of one’s talents, behavior,
appearance, mood, character, morals and impulses into one harmonious
whole. Philosophy is the basic truth that fuel our soul and give our life a
purpose, it shapes the facets of a person’s character. Charm is to attract
other people to be a change agent. Character is our moral values and
belief that guides our actions in life.
15. D. Character.Rationale: Refer to number 14
16. D. Provide in service education programs, Use accurate nursing audit,
formulate philosophy and vision of the institution .  A refers to being a change
agent. B is a role of a patient advocate. C is a case manager while D
basically summarized functions of a nurse manager. If you haven’t read
Lydia Venzon’s Book : NURSING MANAGEMENT TOWARDS QUALITY
CARE, I suggest reading it in advance for your management subjects in the
graduate school. Formulating philosophy and vision is in PLANNING.
Nursing Audit is in CONTROLLING, In service education programs are
included in DIRECTING. These are the processes of Nursing Management,
I just forgot to add ORGANIZING which includes formulating an
organizational structure and plans, Staffing and developing qualifications
and job descriptions.
17. A. Determine client’s need.You can never provide nursing care if you don’t
know what are the needs of the client. How can you provide an effective
postural drainage if you do not know where is the bulk of the client’s
secretion. Therefore, the best description of a care provider is the
accurate and prompt determination of the client’s need to be able to
render an appropriate nursing care.
18. B. Client advocate. As a client’s advocate, Nurses are to protect the
client’s right and promotes what is best for the client. Knowing that
Morphine causes spasm of the sphincter of Oddi and will lead to further
increase in the client’s pain, The nurse knew that the best treatment option
for the client was not provided and intervene to provide the best possible
care.
19. C. Interdependent. Interdependent functions are those that needs
expertise and skills of multiple health professionals. Example is when A
child was diagnosed with nephrotic syndrome and the doctor ordered a
high protein diet, Budek then work together with the dietician about the
age appropriate high protein foods that can be given to the child, Including
the preparation to entice the child into eating the food. NOTE : It is still
debated if the diet in NS is low, moderate or high protein, In the U.S,
Protein is never restricted and can be taken in moderate amount. As far as
the local examination is concerned, answer LOW PROTEIN HIGH CALORIC
DIET.
20. A. Total patient care. This is also known as case nursing. It is a method of
nursing care wherein, one nurse is assigned to one patient for the delivery
of total care. These are the method use by Nursing students, Private duty
nurses and those in critical or isolation units.
21. D. Total patient care .Total patient care works best if there are many
nurses but few patients.
22. A. Functional nursing. Functional nursing is task oriented, One nurse is
assigned on a particular task leading to task expertise and efficiency. The
nurse will work fast because the procedures are repetitive leading to task
mastery. This care is not recommended as this leads fragmented nursing
care.
23. C. Primary nursing. Your keyword in Primary nursing is the 24 hours. This
does not necessarily means the nurse is awake for 24 hours, She can have
a SECONDARY NURSES that will take care of the patient in shifts where
she is not arround.
24. D. Nightingale .  Refer to question # 2. Hammurabi is the king of babylon
that introduces the LEX TALIONES law, If you kill me, you should be killed…
If you rob me, You should be robbed, An eye for an eye and a tooth for a
tooth. Alexander the great was the son of King Philip II and is from
macedonia but he ruled Greece including Persia and Egypt. He is known to
use a hammer to pierce a dying soldier’s medulla towards speedy death
when he thinks that the soldier will die anyway, just to relieve their
suffering. Fabiola was a beautiful roman matron who converted her house
into a hospital.
25. A. Henderson. Refer to question # 6.
26. A. Levin. Myra Levin described the 4 Conservation principles which are
concerned with the Unity and Integrity of an individual. These are
ENERGY : Our output to facilitate meeting of our needs. STRUCTURAL
INTEGRITY : We mus maintain the integrity of our organs, tissues and
systems to be able to function and prevent harmful agents entering our
body. PERSONAL INTEGRITY : These refers to our self esteem, self worth,
self concept, identify and personality. SOCIAL INTEGRITY : Reflects our
societal roles to our society, community, family, friends and fellow
individuals.
27. D. Neuman . Betty Neuman asserted that nursing is a unique profession
and is concerned with all the variables affecting the individual’s response
to stressors. These are INTRA or within ourselves, EXTRA or outside the
individual, INTER means between two or more people. She proposed the
HEALTH CARE SYSTEM MODEL which states that by PRIMARY,
SECONDARY and TERTIARY prevention, The nurse can help the client
maintain stability against these stressors.
28. B. Johnson. According to Dorothy Johnson, Each person is a behavioral
system that is composed of 7 subsystems. Man adjust or adapt to
stressors by a using a LEARNED PATTERN OF RESPONSE. Man uses his
behavior to meet the demands of the environment, and is able to modified
his behavior to support these demands.
29. C. Weidenbach.Just remember ERNESTINE WEIDENBACHLINICAL.
30. A. Erickson,Tomlin,Swain
31. D. Boykin,Schoenhofer . This theory was called GRAND THEORY because
boykin and schoenofer thinks that ALL MAN ARE CARING, And that
nursing is a response to this unique call. According to them, CARING IS A
MORAL IMPERATIVE, meaning, ALL PEOPLE will tend to help a man who
fell down the stairs even if he is not trained to do so.
32. A. Travelbee. Travelbee’s theory was referred to as INTERPERSONAL
theory because she postulated that NURSING is to assist the individual
and all people that affects this individual to cope with illness, recover and
FIND MEANING to this experience. For her, Nursing is a HUMAN TO
HUMAN relationship that is formed during illness. To her, an individual is a
UNIQUE and irreplaceable being in continuous process of becoming,
evolving and changing. PLEASE do remember, that it is PARSE who
postulated the theory of HUMAN BECOMING and not TRAVELBEE, for I
read books that say it was TRAVELBEE and not PARSE.
33. C. Kohlberg. Kohlber states that relationships are based on mutual trust.
He postulated the levels of morality development. At the first stage called
the PREMORAL or preconventional, A child do things and label them as
BAD or GOOD depending on the PUNISHMENT or REWARD they get. They
have no concept of justice, fairness and equity, for them, If I punch this kid
and mom gets mad, thats WRONG. But if I dance and sing, mama smiles
and give me a new toy, then I am doing something good. In the
Conventional level, The individual actuates his act based on the response
of the people around him. He will follow the rules, regulations, laws and
morality the society upholds. If the law states that I should not resuscitate
this man with a DNR order, then I would not. However, in the Post
conventional level or the AUTONOMOUS level, the individual still follows
the rules but can make a rule or bend part of these rules according to his
own MORALITY. He can change the rules if he thinks that it is needed to
be changed. Example is that, A nurse still continue resuscitating the client
even if the client has a DNR order because he believes that the client can
still recover and his mission is to save lives, not watch patients die.
34. D. Peters . Remember PETERS for PRINCIPLES. P is to P. He believes
that morality has 3 components : EMOTION or how one feels,
JUDGEMENT or how one reason and BEHAVIOR or how one actuates his
EMOTION and JUDGEMENT. He believes that MORALITY evolves with the
development of PRINCPLES or the person’s vitrue and traits. He also
believes in AUTOMATICITY of virtues or he calls HABIT, like kindness,
charity, honesty, sincerity and thirft which are innate to a person and
therfore, will be performed automatically.
35. C. Identification. A child, according to Freud adopts parental standards,
traits, habits and norms through identication. A good example is the
corned beef commercial ” WALK LIKE A MAN, TALK LIKE A MAN ” Where
the child identifies with his father by wearing the same clothes and doing
the same thing.
36. D. Schulman and Mekler . According to Schulman and Mekler, there are 2
components that makes an action MORAL : The intention should be good
and the Act must be just. A good example is ROBIN HOOD, His intention is
GOOD but the act is UNJUST, which makes his action IMMORAL.
37. B. Westerhoff. There are only 2 theorist of FAITH that might be asked in
the board examinations. Fowler and Westerhoff. What differs them is that,
FAITH of fowler is defined abstractly, Fowler defines faith as a FORCE that
gives a meaning to a person’s life while Westerhoff defines faith as a
behavior that continuously develops through time.
38. C. Fowler. Rationale: Refer to # 37
39. B. Iloilo Mission Hospital School of nursing
40. C. Philippine General Hospital
41. A. Elizabeth.Saint Elizabeth of Hungary was a daughter of a King and is
the patron saint of nurses. She build hospitals and feed hungry people
everyday using the kingdom’s money. She is a princess, but devoted her
life in feeding the hungry and serving the sick.
42. A. Clara louise Maas. Clara Louise Maas sacrificed her life in research of
YELLOW FEVER. People during her time do not believe that yellow fever
was brought by mosquitoes. To prove that they are wrong, She allowed
herself to be bitten by the vector and after days, She died.
43. C. Moses
44. C. India
45. A. Chinese. Chinese believes that male newborns are demon magnets.
To fool those demons, they put female clothes to their male newborn.
46. B. Intuitive period.Egyptians believe that a sick person is someone with
an evil force or demon that is inside their heads. To release these evil
spirits, They would tend to drill holes on the patient’s skull and it is called
TREPHINING.
47. A. Apprentice period.What dilineates apprentice period among others is
that, it ENDED when formal schools were established. During the
apprentice period, There is no formal educational institution for nurses.
Most of them receive training inside the convent or church. Some of them
are trained just for the purpose of nursing the wounded soldiers. But
almost all of them are influenced by the christian faith to serve and nurse
the sick. When Fliedner build the first formal school for nurses, It marked
the end of the APPRENTICESHIP period.
48. A. Apprentice period. Apprentice period is marked by the emergence of
religious orders the are devoted to religious life and the practice of
nursing.
49. C. St. Clare. The poor clares, is the second order of St. Francis of assisi.
The first order was founded by St. Francis himself. St. Catherine of Siena
was the first lady with the lamp. St. Anne is the mother of mama mary. St.
Elizabeth is the patron saint of Nursing.
50. B. Dark period. Protestantism emerged with Martin Luther questions the
Pope and Christianity. This started the Dark period of nursing when the
christian faith was smeared by controversies. These leads to closure of
some hospital and schools run by the church. Nursing became the work of
prostitutes, slaves, mother and least desirable of women.
51. B. Like some other men.According to ROY, Man as a social being is like
some other man. As a spiritual being and Biologic being, Man are all alike.
As a psychologic being, No man thinks alike. This basically summarized
her BIOPSYHOSOCIAL theory which is included in our licensure exam
coverage.
52. A. Roy. OPEN system theory is ROY. As an open system, man
continuously allows input from the environment. Example is when you tell
me Im good looking, I will be happy the entire day, Because I am an open
system and continuously interact and transact with my environment. A
close system is best exemplified by a CANDLE. When you cover the candle
with a glass, it will die because it will eventually use all the oxygen it needs
inside the glass for combustion. A closed system do not allow inputs and
output in its environment.
53. B. A sole island in vast ocean
54. B. Rogers. The wholistic theory by Martha Rogers states that MAN is
greater than the sum of all its parts and that his dignity and worth will not
be lessen even if one of this part is missing. A good example is ANNE
BOLEYN, The mother of Queen Elizabeth and the wife of King Henry VIII.
She was beheaded because Henry wants to mary another wife and that
his divorce was not approved by the pope. Outraged, He insisted on the
separation of the Church and State and divorce Anne himself by making
everyone believe that Anne is having an affair to another man. Anne was
beheaded while her lips is still saying a prayer. Even without her head,
People still gave respect to her diseased body and a separate head. She
was still remembered as Anne boleyn, Mother of Elizabeth who lead
england to their GOLDEN AGE.
55. B. Rogers. According to Martha Rogers, Man is composed of 2 systems :
SUB which includes cells, tissues, organs and system and SUPRA which
includes our famly, community and society. She stated that when any of
these systems are affected, it will affect the entire individual.
56. B. Needs should be followed exactly in accordance with their hierarchy.Needs
can be deferred. I can urinate later as not to miss the part of the movie’s
climax. I can save my money that are supposedly for my lunch to watch
my idols in concert. The physiologic needs can be meet later for some
other needs and need not be strictly followed according to their hierarchy.
57. D. Priorities are alterable. Refer to question # 56.
58. C. Follows the decision of the majority, uphold justice and truth. A,B and D
are all qualities of a self actualized person. A self actualized person do not
follow the decision of majority but is self directed and can make decisions
contrary to a popular opinion.
59. A. Makes decision contrary to public opinion. Refer to question # 58.
60. B. Self actualization. The peak of maslow’s hierarchy is the essence of
mental health.
61. D. Italy. Florence Nightingale was born in Florence, Italy, May 12, 1820.
Studied in Germany and Practiced in England.
62. A. Born May 12, 1840
63. C. Germany
64. D. Dunn. According to Dunn, High level wellness is the ability of an
individual to maximize his full potential with the limitations imposed by his
environment. According to him, An individual can be healthy or ill in both
favorable and unfavorable environment.
65. A. Promotes advancement and professional growth among its members
66. B. Anastacia Giron Tupas
67. C. Board resolution No. 1955 Promulgated by the BON.  This is an old board
resolution. The new Board resolution is No. 220 series of 2004 also known
as the Nursing Code Of ethics which states that [ SECTION 17, A ] A nurse
should be a member of an accredited professional organization which is
the PNA.
68. C. She does it regularly as an important responsibility
69. A. RA 7164. 7164 is an old law. This is the 1991 Nursing Law which was
repealed by the newer 9173.
70. B. Independent nurse practitioner
71. D. September 02, 1922.  According to the official PNA website, they are
founded September 02, 1922.
72. C. Rosario Montenegro. Anastacia Giron Tupas founded the FNA, the
former name of the PNA but the first President was Rosario Montenegro.
73. B. Bernard. According to Bernard, Health is the ability to maintain and
Internal Milieu and Illness is the failure to maintain the internal
environment.
74. D. Roy. According to ROY, Health is a state and process of becoming a
WHOLE AND INTEGRATED Person.
75. B. Negative feedback. The theory of Health as the ability to maintain
homeostasis was postulated by Walter Cannon. According to him, There
are certain FEEDBACK Mechanism that regulates our Homeostasis. A
good example is that when we overuse our arm, it will produce pain. PAIN
is a negative feedback that signals us that our arm needs a rest.
76. C. Rogers. Martha Rogers states that HEALTH is synonymous with
WELLNESS and that HEALTH and WELLNESS is subjective depending on
the definition of one’s culture.
77. D. King .Emogene King states that health is a state in the life cycle and
Illness is any interference on this cycle. I enjoyed the Movie LION KING
and like what Mufasa said that they are all part of the CIRCLE OF LIFE, or
the Life cycle.
78. A. Orem. Orem defined health as the SOUNDNESS and WHOLENESS of
developed human structure and of bodily and mental functioning.
79. C. Neuman. Neuman believe that man is composed of subparts and
when this subparts are in harmony with the whole system, Wellness
results. Please do not confuse this with the SUB and SUPRA systems of
martha rogers.
80. D. Johnson . Once you see the phrase BEHAVIORAL SYSTEM, answer
Dorothy Johnson.
81. D. Leddy and Pepper .According to Leddy and Pepper, Wellness is
subjective and depends on an individuals perception of balance, harmony
and vitality. Leavell and Clark postulared the ecologic model of health and
illness or the AGENT-HOST-ENVIRONMENT model. Peterson and Zderad
developed the HUMANISTIC NURSING PRACTICE theory while Benner and
Wruber postulate the PRIMACY OF CARING MODEL.
82. C. Dunn
83. D. High level Wellness
84. C. Behavioral. Behavioral precursors includes smoking, alcoholism, high
fat intake and other lifestyle choices. Environmental factors involved poor
sanitation and over crowding. Heridity includes congenital and diseases
acquired through the genes. There are no social precursors according to
DUNN.
85. D. Environmental
86. A. Becker. According to Becker, The belief of an individual greatly affects
his behavior. If a man believes that he is susceptible to an illness, He will
alter his behavior in order to prevent its occurence. For example, If a man
thinks that diabetes is acquired through high intake of sugar and simple
carbohydrates, then he will limit the intake of foods rich in these
components.
87. D. Perceived curability of an illness . If a man think he is susceptibe to a
certain disease, thinks that the disease is serious and it is a threat to his
life and functions, he will use preventive behaviors to avoid the occurence
of this threat.
88. A. Difficulty adhering to the lifestyle and B. Economic factors. Perceived
barriers are those factors that affects the individual’s health preventive
actions. Both A and B can affect the individual’s ability to prevent the
occurence of diseases. C and D are called Preventive Health Behaviors
which enhances the individual’s preventive capabilities.
89. D. Eudaemonistic Model . Smith formulated 5 models of health. Clinical
model simply states that when people experience sign and symptoms,
they would think that they are unhealthy therefore, Health is the absence
of clinical sign and symptoms of a disease. Role performance model
states that when a person does his role and activities without deficits, he
is healthy and the inability to perform usual roles means that the person is
ill. Adaptive Model states that if a person adapts well with his
environment, he is healthy and maladaptation equates illness.
Eudaemonistic Model of health according to smith is the actualization of a
person’s fullest potential. If a person functions optimally and develop self
actualization, then, no doubt that person is healthy.
90. A. Clinical Model. Rationale: Refer to question # 89.
91. C. Structural. Modifying variables in Becker’s health belief model
includes DEMOGRAPHIC : Age, sex, race etc. SOCIOPSYCHOLOGIC :
Social and Peer influence. STRUCTURAL : Knowledge about the disease
and prior contact with it and CUES TO ACTION : Which are the sign and
symptoms of the disease or advice from friends, mass media and others
that forces or makes the individual seek help.
92. D. Cues to action . Refer to question # 91.
93. B. Sociopsychologic. Refer to question # 91.
94. A. Demographic. Refer to question # 91.
95. A. Reservoir. According to L&C’s Ecologic model, there are 3 factors that
affect health and illness. These are the AGENT or the factor the leads to
illness, either a bacteria or an event in life. HOST are persons that may or
may not be affected by these agents. ENVIRONMENT are factors external
to the host that may or may not predispose him to the AGENT.
96. C. Health Promotion Model. Pender developed the concept of HEALTH
PROMOTION MODEL which postulated that an individual engages in
health promotion activities to increase well being and attain self
actualization. These includes exercise, immunization, healthy lifestyle,
good food, self responsibility and all other factors that minimize if not
totally eradicate risks and threats of health.
97. B. Health promotion. Refer to question # 96.
98. B. Disease. Disease are alteration in body functions resulting in reduction
of capabilities or shortening of life span.
99. A. Illness. Illness is something PERSONAL. Unlike disease, Illness are
personal state in which person feels unhealthy. An old person might think
he is ILL but in fact, he is not due, to diminishing functions and
capabilities, people might think they are ILL. Disease however, is
something with tangible basis like lab results, X ray films or clinical sign
and symptoms.
100. B. Watson. This is Jean Watson’s definition of Nursing as caring. This
was asked word per word last June 06′ NLE. Benner defines caring as
something that matters to people. She postulated the responsibility
created by Caring in nursing. She was also responsible for the PRIMACY
OF CARING MODEL. Leininger defind the 4 conservation principle while
Swanson introduced the 5 processes of caring.

PNLE: FON Practice Exam for


Infection, Asepsis, Basic concept of
Stress and Illness
1. When the General adaptation syndrome is activated, FLIGHT OR FIGHT
response sets in. Sympathetic nervous system releases norepinephrine while
the adrenal medulla secretes epinephrine. Which of the following is true with
regards to that statement?

A. Pupils will constrict


B. Client will be lethargic
C. Lungs will bronchodilate
D. Gastric motility will increase
2. Which of the following response is not expected to a person whose GAS is
activated and the FIGHT OR FLIGHT response sets in?

A. The client will not urinate due to relaxation of the detrusor muscle
B. The client will be restless and alert
C. Clients BP will increase, there will be vasodilation
D. There will be increase glycogenolysis, Pancrease will decrease insulin
secretion
3. State in which a person’s physical, emotional, intellectual and social
development or spiritual functioning is diminished or impaired compared with
a previous experience.

A. Illness
B. Disease
C. Health
D. Wellness
4. This is the first stage of illness wherein, the person starts to believe that
something is wrong. Also known as the transition phase from wellness to
illness.

A. Symptom Experience
B. Assumption of sick role
C. Medical care contact
D. Dependent patient role
5. In this stage of illness, the person accepts or rejects a professionals
suggestion. The person also becomes passive and may regress to an earlier
stage.
A. Symptom Experience
B. Assumption of sick role
C. Medical care contact
D. Dependent patient role
6. In this stage of illness, The person learns to accept the illness.

A. Symptom Experience
B. Assumption of sick role
C. Medical care contact
D. Dependent patient role
7. In this stage, the person tries to find answers for his illness. He wants his
illness to be validated, his symptoms explained and the outcome reassured or
predicted

A. Symptom Experience
B. Assumption of sick role
C. Medical care contact
D. Dependent patient role
8. The following are true with regards to aspect of the sick role except

A. One should be held responsible for his condition


B. One is excused from his societal role
C. One is obliged to get well as soon as possible
D. One is obliged to seek competent help
9. Refers to conditions that increases vulnerability of individual or group to
illness or accident

A. Predisposing factor
B. Etiology
C. Risk factor
D. Modifiable Risks
10. Refers to the degree of resistance the potential host has against a certain
pathogen

A. Susceptibility
B. Immunity
C. Virulence
D. Etiology
11. A group of symptoms that sums up or constitute a disease

A. Syndrome
B. Symptoms
C. Signs
D. Etiology
12. A woman undergoing radiation therapy developed redness and burning of
the skin around the best. This is best classified as what type of disease?

A. Neoplastic
B. Traumatic
C. Nosocomial
D. Iatrogenic
13. The classification of CANCER according to its etiology Is best described
as:

1. Nosocomial
2. Idiopathic
3. Neoplastic
4. Traumatic
5. Congenital
6. Degenrative
A. 5 and 2
B. 2 and 3
C. 3 and 4
D. 3 and 5
14. Term to describe the reactiviation and recurrence of pronounced
symptoms of a disease

A. Remission
B. Emission
C. Exacerbation
D. Sub acute
15. A type of illness characterized by periods of remission and exacerbation

A. Chronic
B. Acute
C. Sub acute
D. Sub chronic
16. Diseases that results from changes in the normal structure, from
recognizable anatomical changes in an organ or body tissue is termed as

A. Functional
B. Occupational
C. Inorganic
D. Organic
17. It is the science of organism as affected by factors in their environment. It
deals with the relationship between disease and geographical environment.

A. Epidemiology
B. Ecology
C. Statistics
D. Geography
18. This is the study of the patterns of health and disease. Its occurrence and
distribution in man, for the purpose of control and prevention of disease.

A. Epidemiology
B. Ecology
C. Statistics
D. Geography
19. Refers to diseases that produced no anatomic changes but as a result
from abnormal response to a stimuli.

A. Functional
B. Occupational
C. Inorganic
D. Organic
20. In what level of prevention according to Leavell and Clark does the nurse
support the client in obtaining OPTIMAL HEALTH STATUS after a disease or
injury?

A. Primary
B. Secondary
C. Tertiary
D. None of the above
21. In what level of prevention does the nurse encourage optimal health and
increases person’s susceptibility to illness?

A. Primary
B. Secondary
C. Tertiary
D. None of the above
22. Also known as HEALTH MAINTENANCE prevention.

A. Primary
B. Secondary
C. Tertiary
D. None of the above
23. PPD In occupational health nursing is what type of prevention?

A. Primary
B. Secondary
C. Tertiary
D. None of the above
24. BCG in community health nursing is what type of prevention?

A. Primary
B. Secondary
C. Tertiary
D. None of the above
25. A regular pap smear for woman every 3 years after establishing normal
pap smear for 3 consecutive years Is advocated. What level of prevention
does this belongs?

A. Primary
B. Secondary
C. Tertiary
D. None of the above
26. Self monitoring of blood glucose for diabetic clients is on what level of
prevention?
A. Primary
B. Secondary
C. Tertiary
D. None of the above
27. Which is the best way to disseminate information to the public?

A. Newspaper
B. School bulletins
C. Community bill boards
D. Radio and Television
28. Who conceptualized health as integration of parts and subparts of an
individual?

A. Newman
B. Neuman
C. Watson
D. Rogers
29. The following are concept of health:

1. Health is a state of complete physical, mental and social wellbeing and


not merely an absence of disease or infirmity.
2. Health is the ability to maintain balance
3. Health is the ability to maintain internal milieu
4. Health is integration of all parts and subparts of an individual
A. 1,2,3
B. 1,3,4
C. 2,3,4
D. 1,2,3,4
30. The theorist the advocated that health is the ability to maintain dynamic
equilibrium is

A. Bernard
B. Selye
C. Cannon
D. Rogers
31. Excessive alcohol intake is what type of risk factor?
A. Genetics
B. Age
C. Environment
D. Lifestyle
32. Osteoporosis and degenerative diseases like Osteoarthritis belongs to
what type of risk factor?

A. Genetics
B. Age
C. Environment
D. Lifestyle
33. Also known as STERILE TECHNIQUE

A. Surgical Asepsis
B. Medical Asepsis
C. Sepsis
D. Asepsis
34. This is a person or animal, who is without signs of illness but harbors
pathogen within his body and can be transferred to another

A. Host
B. Agent
C. Environment
D. Carrier
35. Refers to a person or animal, known or believed to have been exposed to a
disease.

A. Carrier
B. Contact
C. Agent
D. Host
36. A substance usually intended for use on inanimate objects, that destroys
pathogens but not the spores.

A. Sterilization
B. Disinfectant
C. Antiseptic
D. Autoclave
37. This is a process of removing pathogens but not their spores

A. Sterilization
B. Auto claving
C. Disinfection
D. Medical asepsis
38. The third period of infectious processes characterized by development of
specific signs and symptoms

A. Incubation period
B. Prodromal period
C. Illness period
D. Convalescent period
39. A child with measles developed fever and general weakness after being
exposed to another child with rubella. In what stage of infectious process
does this child belongs?

A. Incubation period
B. Prodromal period
C. Illness period
D. Convalescent period
40. A 50 year old mailman carried a mail with anthrax powder in it. A minute
after exposure, he still hasn’t developed any signs and symptoms of anthrax.
In what stage of infectious process does this man belongs?

A. Incubation period
B. Prodromal period
C. Illness period
D. Convalescent period
41. Considered as the WEAKEST LINK in the chain of infection that nurses can
manipulate to prevent spread of infection and diseases

A. Etiologic/Infectious agent
B. Portal of Entry
C. Susceptible host
D. Mode of transmission
42. Which of the following is the exact order of the infection chain?

1. Susceptible host
2. Portal of entry
3. Portal of exit
4. Etiologic agent
5. Reservoir
6. Mode of transmission
A. 1,2,3,4,5,6
B. 5,4,2,3,6,1
C. 4,5,3,6,2,1
D. 6,5,4,3,2,1
43. Markee, A 15 year old high school student asked you. What is the mode of
transmission of Lyme disease. You correctly answered him that Lyme disease
is transmitted via

A. Direct contact transmission


B. Vehicle borne transmission
C. Air borne transmission
D. Vector borne transmission
44. The ability of the infectious agent to cause a disease primarily depends on
all of the following except

A. Pathogenicity
B. Virulence
C. Invasiveness
D. Non Specificity
45. Contact transmission of infectious organism in the hospital is usually
cause by

A. Urinary catheterization
B. Spread from patient to patient
C. Spread by cross contamination via hands of caregiver
D. Cause by unclean instruments used by doctors and nurses
46. Transmission occurs when an infected person sneezes, coughs or laugh
that is usually projected at a distance of 3 feet.
A. Droplet transmission
B. Airborne transmission
C. Vehicle transmission
D. Vector borne transmission
47. Considered as the first line of defense of the body against infection

A. Skin
B. WBC
C. Leukocytes
D. Immunization
48. All of the following contributes to host susceptibility except

A. Creed
B. Immunization
C. Current medication being taken
D. Color of the skin
49. Graciel has been injected TT5, her last dosed for tetanus toxoid
immunization. Graciel asked you, what type of immunity is TT Injections? You
correctly answer her by saying Tetanus toxoid immunization is a/an

A. Natural active immunity


B. Natural passive immunity
C. Artificial active immunity
D. Artificial passive immunity
50. Agatha, was hacked and slashed by a psychotic man while she was
crossing the railway. She suffered multiple injuries and was injected Tetanus
toxoid Immunoglobulin. Agatha asked you, What immunity does TTIg
provides? You best answered her by saying TTIg provides

A. Natural active immunity


B. Natural passive immunity
C. Artificial active immunity
D. Artificial passive immunity
51. This is the single most important procedure that prevents cross
contamination and infection

A. Cleaning
B. Disinfecting
C. Sterilizing
D. Handwashing
52. This is considered as the most important aspect of handwashing

A. Time
B. Friction
C. Water
D. Soap
53. In handwashing by medical asepsis, Hands are held ….

A. Above the elbow, The hands must always be above the waist
B. Above the elbow, The hands are cleaner than the elbow
C. Below the elbow, Medical asepsis do not require hands to be above the
waist
D. Below the elbow, Hands are dirtier than the lower arms
54. The suggested time per hand on handwashing using the time method is

A. 5 to 10 seconds each hand


B. 10 to 15 seconds each hand
C. 15 to 30 seconds each hand
D. 30 to 60 seconds each hand
55. The minimum time in washing each hand should never be below

A. 5 seconds
B. 10 seconds
C. 15 seconds
D. 30 seconds
56. How many ml of liquid soap is recommended for handwashing procedure?

A. 1-2 ml
B. 2-3 ml
C. 2-4 ml
D. 5-10 ml
57. Which of the following is not true about sterilization, cleaning and
disinfection?
A. Equipment with small lumen are easier to clean
B. Sterilization is the complete destruction of all viable microorganism
including spores
C. Some organism are easily destroyed, while other, with coagulated
protein requires longer time
D. The number of organism is directly proportional to the length of time
required for sterilization
58. Karlita asked you, How long should she boil her glass baby bottle in water?
You correctly answered her by saying

A. The minimum time for boiling articles is 5 minutes


B. Boil the glass baby bottler and other articles for atleast 10 minutes
C. For boiling to be effective, a minimum of 15 minutes is required
D. It doesn’t matter how long you boil the articles, as long as the water
reached 100 degree Celsius
59. This type of disinfection is best done in sterilizing drugs, foods and other
things that are required to be sterilized before taken in by the human body

A. Boiling Water
B. Gas sterilization
C. Steam under pressure
D. Radiation
60. A TB patient was discharged in the hospital. A UV Lamp was placed in the
room where he stayed for a week. What type of disinfection is this?

A. Concurrent disinfection
B. Terminal disinfection
C. Regular disinfection
D. Routine disinfection
61. Which of the following is not true in implementing medical asepsis

A. Wash hand before and after patient contact


B. Keep soiled linens from touching the clothings
C. Shake the linens to remove dust
D. Practice good hygiene
62. Which of the following is true about autoclaving or steam under pressure?
A. All kinds of microorganism and their spores are destroyed by autoclave
machine
B. The autoclaved instruments can be used for 1 month considering the
bags are still intact
C. The instruments are put into unlocked position, on their hinge, during
the autoclave
D. Autoclaving different kinds of metals at one time is advisable
63. Which of the following is true about masks?

A. Mask should only cover the nose


B. Mask functions better if they are wet with alcohol
C. Masks can provide durable protection even when worn for a long time
and after each and every patient care
D. N95 Mask or particulate masks can filter organism as mall as 1
micromillimeter
64. Where should you put a wet adult diaper?

A. Green trashcan
B. Black trashcan
C. Orange trashcan
D. Yellow trashcan
65. Needles, scalpels, broken glass and lancets are considered as injurious
wastes. As a nurse, it is correct to put them at disposal via a/an

A. Puncture proof container


B. Reused PET Bottles
C. Black trashcan
D. Yellow trashcan with a tag “INJURIOUS WASTES”
66. Miranda Priestly, An executive of RAMP magazine, was diagnosed with
cancer of the cervix. You noticed that the radioactive internal implant
protrudes to her vagina where supposedly, it should be in her cervix. What
should be your initial action?

A. Using a long forceps, Push it back towards the cervix then call the
physician
B. Wear gloves, remove it gently and place it on a lead container
C. Using a long forceps, Remove it and place it on a lead container
D. Call the physician, You are not allowed to touch, re insert or remove it
67. After leech therapy, Where should you put the leeches?

A. In specially marked BIO HAZARD Containers


B. Yellow trashcan
C. Black trashcan
D. Leeches are brought back to the culture room, they are not thrown away
for they are reusable
68. Which of the following should the nurse AVOID doing in preventing spread
of infection?

A. Recapping the needle before disposal to prevent injuries


B. Never pointing a needle towards a body part
C. Using only Standard precaution to AIDS Patients
D. Do not give fresh and uncooked fruits and vegetables to Mr. Gatchie,
with Neutropenia
69. Where should you put Mr. Alejar, with Category II TB?

A. In a room with positive air pressure and atleast 3 air exchanges an hour
B. In a room with positive air pressure and atleast 6 air exchanges an hour
C. In a room with negative air pressure and atleast 3 air exchanges an hour
D. In a room with negative air pressure and atleast 6 air exchanges an hour
70. A client has been diagnosed with RUBELLA. What precaution is used for
this patient?

A. Standard precaution
B. Airborne precaution
C. Droplet precaution
D. Contact precaution
71. A client has been diagnosed with MEASLES. What precaution is used for
this patient?

A. Standard precaution
B. Airborne precaution
C. Droplet precaution
D. Contact precaution
72. A client has been diagnosed with IMPETIGO. What precaution is used for
this patient?

A. Standard precaution
B. Airborne precaution
C. Droplet precaution
D. Contact precaution
73. The nurse is to insert an NG Tube when suddenly, she accidentally dip the
end of the tube in the client’s glass containing distilled drinking water which is
definitely not sterile. As a nurse, what should you do?

A. Don’t mind the incident, continue to insert the NG Tube


B. Obtain a new NG Tube for the client
C. Disinfect the NG Tube before reinserting it again
D. Ask your senior nurse what to do
74. All of the following are principle of SURGICAL ASEPSIS except

A. Microorganism travels to moist surfaces faster than with dry surfaces


B. When in doubt about the sterility of an object, consider it not sterile
C. Once the skin has been sterilized, considered it sterile
D. If you can reach the object by overreaching, just move around the sterile
field to pick it rather than reaching for it
75. Which of the following is true in SURGICAL ASEPSIS?

A. Autoclaved linens and gowns are considered sterile for about 4 months
as long as the bagging is intact
B. Surgical technique is a sole effort of each nurse
C. Sterile conscience, is the best method to enhance sterile technique
D. If a scrubbed person leaves the area of the sterile field, He/she must do
handwashing and gloving again, but the gown need not be changed.
76. In putting sterile gloves, Which should be gloved first?

A. The dominant hand


B. The non dominant hand
C. The left hand
D. No specific order, Its up to the nurse for her own convenience
77. As the scrubbed nurse, when should you apply the goggles, shoe cap and
mask prior to the operation?

A. Immediately after entering the sterile field


B. After surgical hand scrub
C. Before surgical hand scrub
D. Before entering the sterile field
78. Which of the following should the nurse do when applying gloves prior to a
surgical procedure?

A. Slipping gloved hand with all fingers when picking up the second glove
B. Grasping the first glove by inserting four fingers, with thumbs up
underneath the cuff
C. Putting the gloves into the dominant hand first
D. Adjust only the fitting of the gloves after both gloves are on
79. Which gloves should you remove first?

A. The glove of the non dominant hand


B. The glove of the dominant hand
C. The glove of the left hand
D. Order in removing the gloves Is unnecessary
80. Before a surgical procedure, Give the sequence on applying the protective
items listed below

1. Eye wear or goggles


2. Cap
3. Mask
4. Gloves
5. Gown
A. 3,2,1,5,4
B. 3,2,1,4,5
C. 2,3,1,5,4
D. 2,3,1,4,5
81. In removing protective devices, which should be the exact sequence?

1. Eye wear or goggles


2. Cap
3. Mask
4. Gloves
5. Gown
A. 4,3,5,1,2
B. 2,3,1,5,4
C. 5,4,3,2,1
D. 1,2,3,4,5
82. In pouring a plain NSS into a receptacle located in a sterile field, how high
should the nurse hold the bottle above the receptacle?

A. 1 inch
B. 3 inches
C. 6 inches
D. 10 inches
83. The tip of the sterile forceps is considered sterile. It is used to manipulate
the objects in the sterile field using the non sterile hands. How should the
nurse hold a sterile forceps?

A. The tip should always be lower than the handle


B. The tip should always be above the handle
C. The handle and the tip should be at the same level
D. The handle should point downward and the tip, always upward
84. The nurse enters the room of the client on airborne precaution due to
tuberculosis. Which of the following are appropriate actions by the nurse?

1. She wears mask, covering the nose and mouth


2. She washes her hands before and after removing gloves, after
suctioning the client’s secretion
3. She removes gloves and hands before leaving the client’s room
4. She discards contaminated suction catheter tip in trashcan found in the
clients room
A. 1,2
B. 1,2,3
C. 1,2,3,4
D. 1,3
85. When performing surgical hand scrub, which of the following nursing
action is required to prevent contamination?

1. Keep fingernail short, clean and with nail polish


2. Open faucet with knee or foot control
3. Keep hands above the elbow when washing and rinsing
4. Wear cap, mask, shoe cover after you scrubbed
A. 1,2
B. 2,3
C. 1,2,3
D. 2,3,4
86. When removing gloves, which of the following is an inappropriate nursing
action?

A. Wash gloved hand first


B. Peel off gloves inside out
C. Use glove to glove skin to skin technique
D. Remove mask and gown before removing gloves
87. Which of the following is TRUE in the concept of stress?

A. Stress is not always present in diseases and illnesses


B. Stress are only psychological and manifests psychological symptoms
C. All stressors evoke common adaptive response
D. Hemostasis refers to the dynamic state of equilibrium
88. According to this theorist, in his modern stress theory, Stress is the non
specific response of the body to any demand made upon it.

A. Hans Selye
B. Walter Cannon
C. Claude Bernard
D. Martha Rogers
89. Which of the following is NOT TRUE with regards to the concept of
Modern Stress Theory?

A. Stress is not a nervous energy


B. Man, whenever he encounters stresses, always adapts to it
C. Stress is not always something to be avoided
D. Stress does not always lead to distress
90. Which of the following is TRUE with regards to the concept of Modern
Stress Theory?

A. Stress is essential
B. Man does not encounter stress if he is asleep
C. A single stress can cause a disease
D. Stress always leads to distress
91. Which of the following is TRUE in the stage of alarm of general adaptation
syndrome?

A. Results from the prolonged exposure to stress


B. Levels or resistance is increased
C. Characterized by adaptation
D. Death can ensue
92. The stage of GAS where the adaptation mechanism begins

A. Stage of Alarm
B. Stage of Resistance
C. Stage of Homeostasis
D. Stage of Exhaustion
93. Stage of GAS Characterized by adaptation

A. Stage of Alarm
B. Stage of Resistance
C. Stage of Homeostasis
D. Stage of Exhaustion
94. Stage of GAS wherein, the Level of resistance are decreased

A. Stage of Alarm
B. Stage of Resistance
C. Stage of Homeostasis
D. Stage of Exhaustion
95. Where in stages of GAS does a person moves back into HOMEOSTASIS?

A. Stage of Alarm
B. Stage of Resistance
C. Stage of Homeostasis
D. Stage of Exhaustion
96. Stage of GAS that results from prolonged exposure to stress. Here, death
will ensue unless extra adaptive mechanisms are utilized

A. Stage of Alarm
B. Stage of Resistance
C. Stage of Homeostasis
D. Stage of Exhaustion
97. All but one is a characteristic of adaptive response

A. This is an attempt to maintain homeostasis


B. There is a totality of response
C. Adaptive response is immediately mobilized, doesn’t require time
D. Response varies from person to person
98. Andy, a newly hired nurse, starts to learn the new technology and
electronic devices at the hospital. Which of the following mode of adaptation
is Andy experiencing?

A. Biologic/Physiologic adaptive mode


B. Psychologic adaptive mode
C. Sociocultural adaptive mode
D. Technological adaptive mode
99. Andy is not yet fluent in French, but he works in Quebec where majority
speaks French. He is starting to learn the language of the people. What type of
adaptation is Andy experiencing?

A. Biologic/Physiologic adaptive mode


B. Psychologic adaptive mode
C. Sociocultural adaptive mode
D. Technological adaptive mode
100. Andy made an error and his senior nurse issued a written warning. Andy
arrived in his house mad and kicked the door hard to shut it off. What
adaptation mode is this?

A. Biologic/Physiologic adaptive mode


B. Psychologic adaptive mode
C. Sociocultural adaptive mode
D. Technological adaptive mode
Answers and Rationales
1. C. Lungs will bronchodilate. To better understand the concept : The autonomic
nervous system is composed of SYMPATHETIC and PARASYMPATHETIC
Nervous system. It is called AUTONOMIC Because it is Involuntary and stimuli
based. You cannot tell your heart to kindly beat for 60 per minute, Nor, Tell your
blood vessels, Please constrict, because you need to wear skirt today and your
varicosities are bulging. Sympathetic Nervous system is the FIGHT or FLIGHT
mechanism. When people FIGHT or RUN, we tend to stimulate the ANS and
dominate over SNS. Just Imagine a person FIGHTING and RUNNING to get the
idea on the signs of SNS Domination. Imagine a resting and digesting person to
get a picture of PNS Domination. A person RUNNING or FIGHTING Needs to
bronchodilate, because the oxygen need is increased due to higher demand of the
body. Pupils will DILATE to be able to see the enemy clearly. Client will be fully
alert to dodge attacks and leap through obstacles during running. The client’s
gastric motility will DECREASE Because you cannot afford to urinate or defecate
during fighting nor running.
2. C. Clients BP will increase, there will be vasodilation. If vasodilation will
occur, The BP will not increase but decrease. It is true that Blood pressure
increases during SNS Stimulation due to the fact that we need more BLOOD to
circulate during the FIGHT or FLIGHT Response because the oxygen demand
has increased, but this is facilitated by vasoconstriction and not vasodilation. A,B
and D are all correct. The liver will increase glycogenolysis or glycogen store
utilization due to a heightened demand for energy. Pancrease will decrease insulin
secretion because almost every aspect of digestion that is controlled by
Parasympathetic nervous system is inhibited when the SNS dominates.
3. A. Illness.  Disease is a PROVEN FACT based on a medical theory,
standards, diagnosis and clinical feature while ILLNESS Is a subjective
state of not feeling well based on subjective appraisal, previous
experience, peer advice etc.
4. A. Symptom Experience. A favorite board question are Stages of Illness.
When a person starts to believe something is wrong, that person is
experiencing signs and symptoms of an illness. The patient will then
ASSUME that he is sick. This is called assumption of the sick role where
the patient accepts he is Ill and try to give up some activities. Since the
client only ASSUMES his illness, he will try to ask someone to validate if
what he is experiencing is a disease, This is now called as MEDICAL CARE
CONTACT. The client seeks professional advice for validation,
reassurance, clarification and explanation of the symptoms he is
experiencing. client will then start his dependent patient role of receiving
care from the health care providers. The last stage of Illness is the
RECOVERY stage where the patient gives up the sick role and assumes
the previous normal gunctions.
5. D. Dependent patient role. In the dependent patient role stage, Client needs
professionals for help. They have a choice either to accept or reject the
professional’s decisions but patients are usually passive and accepting.
Regression tends to occur more in this period.
6. B. Assumption of sick role. Acceptance of illness occurs in the Assumption of
sick role phase of illness.
7. C. Medical care contact. At this stage, The patient seeks for validation of his
symptom experience. He wants to find out if what he feels are normal or not
normal. He wants someone to explain why is he feeling these signs and symptoms
and wants to know the probable outcome of this experience.
8.  A. One should be held responsible for his condition. The nurse should not
judge the patient and not view the patient as the cause or someone
responsible for his illness. A sick client is excused from his societal roles,
Oblige to get well as soon as possible and Obliged to seek competent
help.
9. C. Risk factor
10. A. Susceptibility. Immunity is the ABSOLUTE Resistance to a pathogen
considering that person has an INTACT IMMUNITY while susceptibility is
the DEGREE of resistance. Degree of resistance means how well would
the individual combat the pathogens and repel infection or invasion of
these disease causing organisms. A susceptible person is someone who
has a very low degree of resistance to combat pathogens. An Immune
person is someone that can easily repel specific pathogens. However,
Remember that even if a person is IMMUNE [ Vaccination ] Immunity can
always be impaired in cases of chemotherapy, HIV, Burns, etc.
11.  A. Syndrome. Symptoms are individual manifestation of a certain
disease. For example, In Tourette syndrome, patient will manifest TICS,
but this alone is not enough to diagnose the patient as other diseases has
the same tic manifestation. Syndrome means COLLECTION of these
symptoms that occurs together to characterize a certain disease. Tics
with coprolalia, echolalia, palilalia, choreas or other movement disorders
are characteristics of TOURETTE SYNDROME.
12. D. Iatrogenic. Iatrogenic diseases refers to those that resulted from treatment of
a certain disease. For example, A child frequently exposed to the X-RAY
Machine develops redness and partial thickness burns over the chest area.
Neoplastic are malignant diseases cause by proliferation of abnormally growing
cells. Traumatic are brought about by injuries like Motor vehicular accidents.
Nosocomial are infections that acquired INSIDE the hospital. Example is UTI
Because of catheterization, This is commonly caused by E.Coli.
13.  B. 2 and 3. Aside from being NEOPLASTIC, Cancer is considered as
IDIOPATHIC because the cause is UNKNOWN.
14. C. Exacerbation
15. A. Chronic. A good example is Multiple sclerosis that characterized by
periods of remissions and exacerbation and it is a CHRONIC Disease. An
acute and sub acute diseases occurs too short to manifest remissions.
Chronic diseases persists longer than 6 months that is why remissions
and exacerbation are observable.
16. D. Organic. As the word implies, ORGANIC Diseases are those that
causes a CHANGE in the structure of the organs and systems. Inorganic
diseases is synonymous with FUNCTIONAL diseases wherein, There is no
evident structural, anatomical or physical change in the structure of the
organ or system but function is altered due to other causes, which is
usually due to abnormal response of the organ to stressors. Therefore,
ORGANIC BRAIN SYNDROME are anatomic and physiologic change in the
BRAIN that is NON PROGRESSIVE BUT IRREVERSIBLE caused by
alteration in structure of the brain and it’s supporting structure which
manifests different sign and symptoms of neurological, physiologic and
psychologic alterations. Mental disorders manifesting symptoms of
psychoses without any evident organic or structural damage are termed
as INORGANIC PSYCHOSES while alteration in the organ structures that
causes symptoms of bizaare pyschotic behavior is termed as ORGANIC
PSYCHOSES.
17. B. Ecology. Ecology is the science that deals with the ECOSYSTEM and its
effects on living things in the biosphere. It deals with diseases in relationship with
the environment. Epidimiology is simply the Study of diseases and its occurence
and distribution in man for the purpose of controlling and preventing diseases.
This was asked during the previous boards.
18. A. Epidemiology. Refer to number 17.
19. C. Inorganic. Refer to number 16.
20. C. Tertiary. Perhaps one of the easiest concept but asked frequently in the NLE.
Primary refers to preventions that aims in preventing the disease. Examples are
healthy lifestyle, good nutrition, knowledge seeking behaviors etc. Secondary
prevention are those that deals with early diagnostics, case finding and treatments.
Examples are monthly breast self exam, Chest X-RAY, Antibiotic treatment to
cure infection, Iron therapy to treat anemia etc. Tertiary prevention aims on
maintaining optimum level of functioning during or after the impact of a disease
that threatens to alter the normal body functioning. Examples are prosthetis fitting
for an amputated leg after an accident, Self monitoring of glucose among
diabetics, TPA Therapy after stroke etc. The confusing part is between the
treatment in secondary and treatment in tertiary. To best differentiate the
two, A client with ANEMIA that is being treated with ferrous sulfate is
considered being in the SECONDARY PREVENTION because ANEMIA once
treated, will move the client on PRE ILLNESS STATE again. However, In
cases of ASPIRING Therapy in cases of stroke, ASPIRING no longer cure
the patient or PUT HIM IN THE PRE ILLNESS STATE. ASA therapy is done
in order to prevent coagulation of the blood that can lead to thrombus
formation and a another possible stroke. You might wonder why I spelled
ASPIRIN as ASPIRING, Its side effect is OTOTOXICITY [ CN VIII ] that leads
to TINNITUS or ringing of the ears.
21.  D. None of the above. The nurse never increases the person’s susceptibility to
illness but rather, LESSEN the person’s susceptibility to illness.
22. B. Secondary. Secondary prevention is also known as HEALTH
MAINTENANCE Prevention. Here, The person feels signs and symptoms and
seeks Diagnosis and treatment in order to prevent deblitating complications. Even
if the person feels healthy, We are required to MAINTAIN our health by monthly
check ups, Physical examinations, Diagnostics etc.
23. A. Primary. PPD or PERSONAL PROTECTIVE DEVICES are worn by the
workes in a hazardous environment to protect them from injuries and
hazards. This is considered as a PRIMARY prevention because the nurse
prevents occurence of diseases and injuries.
24. A. Primary
25. B. Secondary
26. C. Tertiary
27. D. Radio and Television. An actual board question, The best way to
disseminate information to the public is by TELEVISION followed by RADIO.
This is how the DOH establish its IEC Programs other than publising posters,
leaflets and brochures. An emerging new way to disseminate is through the
internet.
28. B. Neuman. The supra and subsystems are theories of Martha Rogers but the
parts and subparts are Betty Neuman’s. She stated that HEALTH is a state where
in all parts and subparts of an individual are in harmony with the whole system.
Margarex Newman defined health as an EXPANDING CONSCIOUSNESS. Her
name is Margaret not Margarex, I just used that to help you remember her theory
of health.
29. D. 1,2,3,4. All of the following are correct statement about health. The first one
is the definition by WHO, The second one is from Walter Cannon’s homeostasis
theory. Third one is from Claude Bernard’s concept of Health as Internal Milieu
and the last one is Neuman’s Theory.
30. C. Cannon. Walter Cannon advocated health as HOMEOSTASIS or the ability
to maintain dynamic equilibrium. Hans Selye postulated Concepts about Stress
and Adaptation. Bernard defined health as the ability to maintain internal milieu
and Rogers defined Health as Wellness that is influenced by individual’s culture.
31. D. Lifestyle
32. B. Age
33. A. Surgical Asepsis. Surgical Asepsis is also known as STERILE
TECHNIQUE while Medical Asepsis is synonymous with CLEAN
TECHNIQUE.
34. D. Carrier
35. B. Contact
36. B. Disinfectant. Disinfectants are used on inanimate objects while
Antiseptics are intended for use on persons and other living things. Both
can kill and inhibit growth of microorganism but cannot kill their spores.
That is when autoclaving or steam under pressure gets in, Autoclaving can
kill almost ALL type of microoganism including their spores.
37. C. Disinfection. Both A and B are capable on killing spores. Autoclaving is a
form of Sterilization. Medical Asepsis is a PRACTICE designed to minimize or
reduce the transfer of pathogens, also known as your CLEAN TECHNIQUE.
Disinfection is the PROCESS of removing pathogens but not their spores.
38. C. Illness period. In incubation period, The disease has been introduced to the
body but no sign and symptom appear because the pathogen is not yet strong
enough to cause it and may still need to multiply. The second period is called
prodromal period. This is when the appearance of non specific signs and
symptoms sets in, This is when the sign and symptoms starts to appear. Illness
period is characterized by the appearance of specific signs and symptoms or refer
tp as time with the greatest symptom experience. Acme is the PEAK of illness
intensity while the convalescent period is characterized by the abatement of the
disease process or it’s gradual disappearance.
39. B. Prodromal period. To be able to categorize MEASLES in the Illness period,
the specific signs of Fever, Koplik’s Spot and Rashes must appear. In the situation
above, Only general signs and symptoms appeared and the Specific signs and
symptoms is yet to appear, therefore, the illness is still in the Prodromal period.
Signs and symptoms of measles during the prodromal phase are Fever, fatigue,
runny nose, cough and conjunctivitis. Koplik’s spot heralds the Illness period and
cough is the last symptom to disappear. All of this processes take place in 10 days
that is why, Measles is also known as 10 day measles.
40. A. Incubation period. Anthrax can have an incubation period of hours to 7
days with an average of 48 hours. Since the question stated exposure, we
can now assume that the mailman is in the incubation period.
41. D. Mode of transmission. Mode of transmission is the weakest link in the chain
of infection. It is easily manipulated by the Nurses using the tiers of prevention,
either by instituting transmission based precautions, Universal precaution or
Isolation techniques.
42. C. 4,5,3,6,2,1. Chain of infection starts with the SOURCE : The etiologic agent
itself. It will first proliferate on a RESERVOIR and will need a PORTAL OF
EXIT to be able to TRANSMIT irslef using a PORTAL OF ENTRY to a
SUSCEPTIBLE HOST. A simple way to understand the process is by looking at
the lives of a young queen ant that is starting to build her colony. Imagine the
QUEEN ANT as a SOURCE or the ETIOLOGIC AGENT. She first need to build
a COLONY, OR the RESERVOIR where she will start to lay the first eggs to be
able to produce her worker ants and soldier ants to be able to defend and sustain
the new colony. They need to EXIT [PORTAL OF EXIT] their colony and crawl
[MODE OF TRANSMISSION] in search of foods by ENTERING / INVADING
[PORTAL OF ENTRY] our HOUSE [SUSCEPTIBLE HOST]. By imagining the
Ant’s life cycle, we can easily arrange the chain of infection.
43. D. Vector borne transmission. Lyme disease is caused by Borrelia Burdorferi
and is transmitted by a TICK BITE.
44. D. Non Specificity. To be able to cause a disease, A pathogen should have a
TARGET ORGAN/S. The pathogen should be specific to these organs to cause an
infection. Mycobacterium Avium is NON SPECIFIC to human organs and
therefore, not infective to humans but deadly to birds. An immunocompromised
individual, specially AIDS Patient, could be infected with these NON SPECIFIC
diseases due to impaired immune system.
45. C. Spread by cross contamination via hands of caregiver. The hands of the
caregiver like nurses, is the main cause of cross contamination in hospital setting.
That is why HANDWASHING is the single most important procedure to prevent
the occurence of cross contamination and nosocomial infection. D refers to
Nosocomial infection and UTI is the most common noscomial infection in the
hospital caused by urinary catheterization. E.Coli seems to be the major cause of
this incident. B best fits Cross Contamination, It is the spread of microogranisms
from patient o patient.
46. A. Droplet transmission
47.  A. Skin. Remember that intact skin and mucus membrane is our first
line of defense against infection.
48. A. Creed. Creed, Faith or religious belief do not affect person’s
susceptibility to illness. Medication like corticosteroids could supress a
person’s immune system that will lead to increase susceptibility. Color of
the skin could affect person’s susceptibility to certain skin diseases. A
dark skinned person has lower risk of skin cancer than a fair skinned
person. Fair skinned person also has a higher risk for cholecystitis and
cholelithiasis.
49. C. Artificial active immunity. TT1 ti TT2 are considered the primary dose,
while TT3 to TT5 are the booster dose. A woman with completed immunization
of DPT need not receive TT1 and TT2. Tetanus toxoid is the actual toxin produce
by clostridium tetani but on its WEAK and INACTIVATED form. It is Artificial
because it did not occur in the course of actual illness or infection, it is Active
because what has been passed is an actual toxin and not a ready made
immunoglobulin.
50. D. Artificial passive immunity. In this scenario, Agatha was already wounded
and has injuries. Giving the toxin [TT Vaccine] itself would not help Agatha
because it will take time before the immune system produce antitoxin. What
agatha needs now is a ready made anti toxin in the form of ATS or TTIg. This is
artificial, because the body of agatha did not produce it. It is passive because her
immune system is not stimulated but rather, a ready made Immune globulin is
given to immediately supress the invasion.
51. D. Handwashing. When you see the word HANDWASHING as one of the
options, 90% Chance it is the correct answer in the local board. Or should I say,
100% because I have yet to see question from 1988 to 2005 board questions that
has option HANDWASHING on it but is not the correct answer.
52. B. Friction. The most important aspect of handwashing is FRICTION. The rest,
will just enhance friction. The use of soap lowers the surface tension thereby
increasing the effectiveness of friction. Water helps remove transient bacteria by
working with soap to create the lather that reduces surface tension. Time is of
essence but friction is the most essential aspect of handwashing.
53. D. Below the elbow, Hands are dirtier than the lower arms. Hands are held
BELOW the elbow in medical asepsis in contrast with surgical asepsis, wherein,
nurses are required to keep the hands above the waist. The rationale is because in
medical asepsis, Hands are considered dirtier than the elbow and therefore, to
limit contamination of the lower arm, The hands should always be below the
elbow.
54. C. 15 to 30 seconds each hand. Each hands requires atleast 15 to 30 seconds of
handwashing to effectively remove transient microorganisms.
55. B. 10 seconds. According to Kozier, The minimum time required for watching
each hands is 10 seconds and should not be lower than that. The recommended
time, again, is 15 to 30 seconds.
56. C. 2-4 ml. If a liquid soap is to be used, 1 tsp [ 5ml ] of liquid soap is
recommended for handwashing procedure.
57. A. Equipment with small lumen are easier to clean. Equipments with LARGE
LUMEN are easier to clean than those with small lumen. B C and D are all
correct.
58. C. For boiling to be effective, a minimum of 15 minutes is required. Boiling is
the most common and least expensive method of sterilization used in home. For it
to be effective, you should boil articles for atleast 15 minutes.
59. D. Radiation. Imagine foods and drugs that are being sterilized by a boiling
water, ethylene oxide gas and autoclave or steam under pressure, They will be
inactivated by these methods. Ethylene oxide gas used in gas sterlization is
TOXIC to humans. Boiling the food will alter its consistency and nutrients.
Autoclaving the food is never performed. Radiation using microwave oven or
Ionizing radiation penetrates to foods and drugs thus, sterilizing them.
60. B. Terminal disinfection. Terminal disinfection refers to practices to remove
pathogens that stayed in the belongings or immediate environemnt of an infected
client who has been discharged. An example would be Killing airborne TB Bacilli
using UV Light. Concurrent disinfection refers to ongoing efforts implented
during the client’s stay to remove or limit pathogens in his supplies, belongings,
immediate environment in order to control the spread of the disease. An example
is cleaning the bedside commode of a client with radium implant on her cervix
with a bleach disinfectant after each voiding.
61. C. Shake the linens to remove dust. NEVER shake the linens. Once soiled, fold
it inwards clean surface out. Shaking the linen will further spread pathogens that
has been harbored by the fabric.
62. C. The instruments are put into unlocked position, on their hinge, during the
autoclave. Only C is correct. Metals with locks, like clamps and scissors should
be UNLOCKED in order to minimize stiffening caused by autoclave to the hinges
of these metals. NOT ALL microorganism are destroyed by autoclaving. There
are recently discovered microorganism that is invulnarable to extreme heat.
Autoclaved instruments are to be used within 2 weeks. Only the same type of
metals should be autoclaved as this will alteration in plating of these metals.
63. D. N95 Mask or particulate masks can filter organism as mall as 1
micromillimeter.  Mask should cover both nose and mouth. Masks will not
function optimally when wet. Masks should be worn not greater than 4
hours, as it will lose effectiveness after 4 hours. N95 mask or particulate
mask can filter organism as small as 1 micromillimeter.
64. D. Yellow trashcan. Infectious waste like blood and blood products, wet
diapers and dressings are thrown in yellow trashcans.
65. A. Puncture proof container. Needles, scalpels and other sharps are to be
disposed in a puncture proof container.
66. C. Using a long forceps, Remove it and place it on a lead container. A
dislodged radioactive cervical implant in brachytherapy are to be picked by a
LONG FORCEP and stored in a LEAD CONTAINER in order to prevent damage
on the client’s normal tissue. Calling the physician is the second most appropriate
action among the choices. A nurse should never attempt to put it back nor, touch
it with her bare hands.
67. A. In specially marked BIO HAZARD Containers. Leeches, in leech therapy
or LEECH PHLEBOTOMY are to be disposed on a BIO HAZARD container.
They are never re used as this could cause transfer of infection. These
leeches are hospital grown and not the usual leeches found in swamps.
68. A. Recapping the needle before disposal to prevent injuries. Never recap
needles. They are directly disposed in a puncture proof container after
used. Recapping the needles could cause injury to the nurse and spread of
infection. B C and D are all appropriate. Standard precaution is sufficient
for an HIV patient. A client with neutropenia are not given fresh and
uncooked fruits and vegetables for even the non infective organisms
found in these foods could cause severe infection on an
immunocompromised patients.
69. D. In a room with negative air pressure and atleast 6 air exchanges an hour. TB
patients should have a private room with negative air pressure and atleast
6 to 12 air exhanges per hour. Negative pressure room will prevent air
inside the room from escaping. Air exchanges are necessary since the
client’s room do not allow air to get out of the room.
70. C. Droplet precaution. Droplet precaution is sufficient on client’s with
RUBELLA or german measles.
71. B. Airborne precaution. Measles is highly communicable and more contagious
than Rubella, It requires airborne precaution as it is spread by small particle
droplets that remains suspended in air and disperesed by air movements.
72. D. Contact precaution. Impetigo causes blisters or sores in the skin. It is
generally caused by GABS or Staph Aureaus. It is spread by skin to skin contact
or by scratching the lesions and touching another person’s skin.
73. A. Don’t mind the incident, continue to insert the NG Tube. The digestive
tract is not sterile, and therefore, simple errors like this would not cause
harm to the patient. NGT tube need not be sterile, and so is colostomy and
rectal tubes. Clean technique is sufficient during NGT and colostomy care.
74. C. Once the skin has been sterilized, considered it sterile. Human skin is
impossible to be sterilized. It contains normal flora of microorganism. A B and D
are all correct.
75.  C. Sterile conscience, is the best method to enhance sterile technique.  Sterile
conscience, or the moral imperative of a nurse to be honest in practicing sterile
technique, is the best method to enhance sterile technique. Autoclaved linens are
considered sterile only within 2 weeks even if the bagging is intact. Surgical
technique is a team effort of each nurse. If a scrubbed person leave the sterile
field and area, he must do the process all over again.
76. B. The non dominant hand. Gloves are put on the non dominant hands first and
then, the dominant hand. The rationale is simply because humans tend to use the
dominant hand first before the non dominant hand. Out of 10 humans that will put
on their sterile gloves, 8 of them will put the gloves on their non dominant hands
first.
77. C. Before surgical hand scrub. The nurse should put his goggles, cap and mask
prior to washing the hands. If he wash his hands prior to putting all these
equipments, he must wash his hands again as these equipments are said to be
UNSTERILE.
78. D. Adjust only the fitting of the gloves after both gloves are on. The nurse
should only adjust fitting of the gloves when they are both on the hands. Not
doing so will break the sterile technique. Only 4 gingers are slipped when picking
up the second gloves. You cannot slip all of your fingers as the cuff is limited and
the thumb would not be able to enter the cuff. The first glove is grasp by simply
picking it up with the first 2 fingers and a thumb in a pinching motion. Gloves are
put on the non dominant hands first.
79. A. The glove of the non dominant hand. Gloves are worn in the non
dominant hand first, and is removed also from the non dominant hand
first. Rationale is simply because in 10 people removing gloves, 8 of them
will use the dominant hand first and remove the gloves of the non
dominant hand.
80. D. 2,3,1,4,5. The nurse should use CaMEy Hand and Body Lotion in
moisturizing his hand before surgical procedure and after handwashing. Ca stands
for CAP, Mstands for MASK, Ey stands for eye goggles. The nurse will do
handwashing and then [HAND], Don the gloves first and wear the Gown
[BODY]. I created this mnemonic and I advise you use it because you can never
forget Camey hand and body lotion. [ Yes, I know it is spelled as CAMAY ]]
81. A. 4,3,5,1,2. When the nurse is about to remove his protective devices,
The nurse will remove the GLOVES first followed by the MASK and GOWN
then, other devices like cap, shoe cover, etc. This is to prevent
contamination of hair, neck and face area.
82. C. 6 inches. Even if you do not know the answer to this question, you can
answer it correctly by imagining. If you pour the NSS into a receptacle 1 to 3 inch
above it, Chances are, The mouth of the NSS bottle would dip into the receptacle
as you fill it, making it contaminated. If you pour the NSS bottle into a receptacle
10 inches above it, that is too high, chances are, as you pour the NSS, most will
spill out because the force will be too much for the buoyant force to handle. It will
also be difficult to pour something precisely into a receptacle as the height
increases between the receptacle and the bottle. 6 inches is the correct answer. It
is not to low nor too high.
83. A. The tip should always be lower than the handle. A sterile forcep is usually
dipped into a disinfectant or germicidal solution. Imagine, if the tip is
HIGHER than the handle, the solution will go into the handle and into your
hands and as you use the forcep, you will eventually lower its tip making
the solution in your hand go BACK into the tip thus contaminating the
sterile area of the forcep. To prevent this, the tip should always be lower
than the handle. In situation questions like this, IMAGINATION is very
important.
84. C. 1,2,3,4. All soiled equipments use in an infectious client are disposed
INSIDE the client’s room to prevent contamination outside the client’s room. The
nurse is correct in using Mask the covers both nose and mouth. Hands are washed
before and after removing the gloves and before and after you enter the client’s
room. Gloves and contaminated suction tip are thrown in trashcan found in the
clients room.
85. C. 1,2,3. Cap, mask and shoe cover are worn BEFORE scrubbing.
86. D. Remove mask and gown before removing gloves. Gloves are the dirtiest
protective item nurses are wearing and therefore, the first to be removed to
prevent spread of microorganism as you remove the mask and gown.
87.  C. All stressors evoke common adaptive response. All stressors evoke
common adaptive response. A psychologic fear like nightmare and a real fear or
real perceive threat evokes common manifestation like tachycardia, tachypnea,
sweating, increase muscle tension etc. ALL diseases and illness causes stress.
Stress can be both REAL or IMAGINARY. Hemostasis refers to the ARREST of
blood flowing abnormally through a damage vessel. Homeostasis is the one that
refers to dynamic state of equilibrium according to Walter Cannon.
88. A. Hans Selye. Hans Selye is the only theorist who proposed an intriguing
theory about stress that has been widely used and accepted by
professionals today. He conceptualized two types of human response to
stress, The GAS or general adaptation syndrome which is characterized by
stages of ALARM, RESISTANCE and EXHAUSTION. The Local adaptation
syndrome controls stress through a particular body part. Example is when
you have been wounded in your finger, it will produce PAIN to let you know
that you should protect that particular damaged area, it will also produce
inflammation to limit and control the spread of injury and facilitate healing
process. Another example is when you are frequently lifting heavy objects,
eventually, you arm, back and leg muscles hypertorphies to adapt to the
stress of heavy lifting.
89. B. Man, whenever he encounters stresses, always adapts to it. Man, do not
always adapt to stress. Sometimes, stress can lead to exhaustion and eventually,
death. A,C and D are all correct.
90. A. Stress is essential. Stress is ESSENTIAL. No man can live normally
without stress. It is essential because it is evoked by the body’s normal
pattern of response and leads to a favorable adaptive mechanism that are
utilized in the future when more stressors are encountered by the body.
Man can encounter stress even while asleep, example is nightmare.
Disease are multifactorial, No diseases are caused by a single stressors.
Stress are sometimes favorable and are not always a cause for distress.
An example of favorable stress is when a carpenter meets the demand
and stress of everyday work. He then develops calluses on the hand to
lessen the pressure of the hammer against the tissues of his hand. He
also develop larger muscle and more dense bones in the arm, thus, a
stress will lead to adaptations to decrease that particular stress.
91. D. Death can ensue. Death can ensue as early as the stage of alarm. Exhaustion
results to a prolonged exposure to stress. Resistance is when the levels of
resistance increases and characterized by being able to adapt.
92. A. Stage of Alarm. Adaptation mechanisms begin in the stage of alarm.
This is when the adaptive mechanism are mobilized. When someone
shouts SUNOG!!! your heart will begin to beat faster, you vessels
constricted and bp increased.
93. B. Stage of Resistance
94.  A. Stage of Alarm. Resistance are decreased in the stage of alarm.
Resistance is absent in the stage of exhaustion. Resistance is increased in
the stage of resistance.
95. B. Stage of Resistance
96. D. Stage of Exhaustion
97. C. Adaptive response is immediately mobilized, doesn’t require time. Aside
from having limits that leads to exhaustion. Adaptive response requires time for it
to act. It requires energy, physical and psychological taxes that needs time for our
body to mobilize and utilize.
98. D. Technological adaptive mode
99. C. Sociocultural adaptive mode. Sociocultural adaptive modes include
language, communication, dressing, acting and socializing in line with the social
and cultural standard of the people around the adapting individual.
100. B. Psychologic adaptive mode
PNLE: FON Practice Exam for
Stress, Crisis, Crisis Intervention,
Communication, Recording,
Learning and Documentation
1. The coronary vessels, unlike any other blood vessels in the body, respond to
sympathetic stimulation by

A. Vasoconstriction
B. Vasodilatation
C. Decreases force of contractility
D. Decreases cardiac output
2. What stress response can you expect from a patient with blood sugar of 50
mg / dl?
A. Body will try to decrease the glucose level
B. There will be a halt in release of sex hormones
C. Client will appear restless
D. Blood pressure will increase
3. All of the following are purpose of inflammation except

A. Increase heat, thereby produce abatement of phagocytosis


B. Localized tissue injury by increasing capillary permeability
C. Protect the issue from injury by producing pain
D. Prepare for tissue repair
4. The initial response of tissue after injury is

A. Immediate Vasodilation
B. Transient Vasoconstriction
C. Immediate Vasoconstriction
D. Transient Vasodilation
5. The last expected process in the stages of inflammation is characterized by

A. There will be sudden redness of the affected part


B. Heat will increase on the affected part
C. The affected part will loss its normal function
D. Exudates will flow from the injured site
6. What kind of exudates is expected when there is an antibody-antigen
reaction as a result of microorganism infection?

A. Serous
B. Serosanguinous
C. Purulent
D. Sanguinous
7. The first manifestation of inflammation is

A. Redness on the affected area


B. Swelling of the affected area
C. Pain, which causes guarding of the area
D. Increase heat due to transient vasodilation
8. The client has a chronic tissue injury. Upon examining the client’s antibody
for a particular cellular response, Which of the following WBC component is
responsible for phagocytosis in chronic tissue injury?

A. Neutrophils
B. Basophils
C. Eosinophils
D. Monocytes
9. Which of the following WBC component proliferates in cases of
Anaphylaxis?

A. Neutrophils
B. Basophils
C. Eosinophil
D. Monocytes
10. Icheanne, ask you, her Nurse, about WBC Components. She got an injury
yesterday after she twisted her ankle accidentally at her gymnastic class. She
asked you, which WBC Component is responsible for proliferation at the
injured site immediately following an injury. You answer:

A. Neutrophils
B. Basophils
C. Eosinophils
D. Monocytes
11. Icheanne then asked you, what is the first process that occurs in the
inflammatory response after injury, You tell her:

A. Phagocytosis
B. Emigration
C. Pavementation
D. Chemotaxis
12. Icheanne asked you again, What is that term that describes the magnetic
attraction of injured tissue to bring phagocytes to the site of injury?

A. Icheanne, you better sleep now, you asked a lot of questions


B. It is Diapedesis
C. We call that Emigration
D. I don’t know the answer, perhaps I can tell you after I find it out later
13. This type of healing occurs when there is a delayed surgical closure of
infected wound

A. First intention
B. Second intention
C. Third intention
D. Fourth intention
14. Type of healing when scars are minimal due to careful surgical incision
and good healing

A. First intention
B. Second intention
C. Third intention
D. Fourth intention
15. Imelda, was slashed and hacked by an unknown suspects. She suffered
massive tissue loss and laceration on her arms and elbow in an attempt to
evade the criminal. As a nurse, you know that the type of healing that will most
likely occur to Miss Imelda is

A. First intention
B. Second intention
C. Third intention
D. Fourth intention
16. Imelda is in the recovery stage after the incident. As a nurse, you know
that the diet that will be prescribed to Miss Imelda is

A. Low calorie, High protein with Vitamin A and C rich foods


B. High protein, High calorie with Vitamin A and C rich foods
C. High calorie, Low protein with Vitamin A and C rich foods
D. Low calorie, Low protein with Vitamin A and C rich foods
17. Miss Imelda asked you, What is WET TO DRY Dressing method? Your best
response is

A. It is a type of mechanical debridement using Wet dressing that is


applied and left to dry to remove dead tissues
B. It is a type of surgical debridement with the use of Wet dressing to
remove the necrotic tissues
C. It is a type of dressing where in, The wound is covered with Wet or Dry
dressing to prevent contamination
D. It is a type of dressing where in, A cellophane or plastic is placed on the
wound over a wet dressing to stimulate healing of the wound in a wet
medium
18. The primary cause of pain in inflammation is

A. Release of pain mediators


B. Injury to the nerve endings
C. Compression of the local nerve endings by the edema fluids
D. Circulation is lessen, Supply of oxygen is insufficient
19. The client is in stress because he was told by the physician he needs to
undergo surgery for removal of tumor in his bladder. Which of the following
are effects of sympatho-adreno-medullary response by the client?

1. Constipation
2. Urinary frequency
3. Hyperglycemia
4. Increased blood pressure
A. 3,4
B. 1,3,4
C. 1,2,4
D. 1,4
20. The client is on NPO post midnight. Which of the following, if done by the
client, is sufficient to cancel the operation in the morning?

A. Eat a full meal at 10:00 P.M


B. Drink fluids at 11:50 P.M
C. Brush his teeth the morning before operation
D. Smoke cigarette around 3:00 A.M
21. The client place on NPO for preparation of the blood test. Adreno-cortical
response is activated and which of the following below is an expected
response?

A. Low BP
B. Decrease Urine output
C. Warm, flushed, dry skin
D. Low serum sodium levels
22. Which of the following is true about therapeutic relationship?

A. Directed towards helping an individual both physically and emotionally


B. Bases on friendship and mutual trust
C. Goals are set by the solely nurse
D. Maintained even after the client doesn’t need anymore of the Nurse’s
help
23. According to her, A nurse patient relationship is composed of 4 stages :
Orientation, Identification, Exploitation and Resolution

A. Roy
B. Peplau
C. Rogers
D. Travelbee
24. In what phase of Nurse patient relationship does a nurse review the
client’s medical records thereby learning as much as possible about the
client?

A. Pre Orientation
B. Orientation
C. Working
D. Termination
25. Nurse Aida has seen her patient, Roger for the first time. She establish a
contract about the frequency of meeting and introduce to Roger the expected
termination. She started taking baseline assessment and set interventions
and outcomes. On what phase of NPR Does Nurse Aida and Roger belong?

A. Pre Orientation
B. Orientation
C. Working
D. Termination
26. Roger has been seen agitated, shouting and running. As Nurse Aida
approaches, he shouts and swear, calling Aida names. Nurse Aida told Roger
“That is an unacceptable behavior Roger, Stop and go to your room now.” The
situation is most likely in what phase of NPR?

A. Pre Orientation
B. Orientation
C. Working
D. Termination
27. Nurse Aida, in spite of the incident, still consider Roger as worthwhile
simply because he is a human being. What major ingredient of a therapeutic
communication is Nurse Aida using?

A. Empathy
B. Positive regard
C. Comfortable sense of self
D. Self awareness
28. Nurse Irma saw Roger and told Nurse Aida “ Oh look at that psychotic
patient “ Nurse Aida should intervene and correct Nurse Irma because her
statement shows that she is lacking?

A. Empathy
B. Positive regard
C. Comfortable sense of self
D. Self awareness
29. Which of the following statement is not true about stress?

A. It is a nervous energy
B. It is an essential aspect of existence
C. It has been always a part of human experience
D. It is something each person has to cope
30. Martina, a Tennis champ was devastated after many new competitors
outpaced her in the Wimbledon event. She became depressed and always
seen crying. Martina is clearly on what kind of situation?

A. Martina is just stressed out


B. Martina is Anxious
C. Martina is in the exhaustion stage of GAS
D. Martina is in Crisis
31. Which of the following statement is not true with regards to anxiety?

A. It has physiologic component


B. It has psychologic component
C. The source of dread or uneasiness is from an unrecognized entity
D. The source of dread or uneasiness is from a recognized entity
32. Lorraine, a 27 year old executive was brought to the ER for an unknown
reason. She is starting to speak but her speech is disorganized and cannot be
understood. On what level of anxiety does this features belongs?

A. Mild
B. Moderate
C. Severe
D. Panic
33. Elton, 21 year old nursing student is taking the board examination. She is
sweating profusely, has decreased awareness of his environment and is
purely focused on the exam questions characterized by his selective
attentiveness. What anxiety level is Elton exemplifying?

A. Mild
B. Moderate
C. Severe
D. Panic
34. You noticed the patient chart : ANXIETY +3 What will you expect to see in
this client?

A. An optimal time for learning, Hearing and perception is greatly


increased
B. Dilated pupils
C. Unable to communicate
D. Palliative Coping Mechanism
35. When should the nurse starts giving XANAX?

A. When anxiety is +1
B. When the client starts to have a narrow perceptual field and selective
inattentiveness
C. When problem solving is not possible
D. When the client is immobile and disorganized
36. Which of the following behavior is not a sign or a symptom of Anxiety?

A. Frequent hand movement


B. Somatization
C. The client asks a question
D. The client is acting out
37. Which of the following intervention is inappropriate for client’s with
anxiety?

A. Offer choices
B. Provide a quiet and calm environment
C. Provide detailed explanation on each and every procedures and
equipments
D. Bring anxiety down to a controllable level
38. Which of the following statement, if made by the nurse, is considered not
therapeutic?

A. “How did you deal with your anxiety before?”


B. “It must be awful to feel anxious.”
C. “How does it feel to be anxious?”
D. “What makes you feel anxious?”
39. Marissa Salva, Uses Benson’s relaxation. How is it done?

A. Systematically tensing muscle groups from top to bottom for 5


seconds, and then releasing them
B. Concentrating on breathing without tensing the muscle, Letting go and
repeating a word or sound after each exhalation
C. Using a strong positive, feeling-rich statement about a desired change
D. Exercise combined with meditation to foster relaxation and mental
alacrity
40. What type of relaxation technique does Lyza uses if a machine is showing
her pulse rate, temperature and muscle tension which she can visualize and
assess?

A. Biofeedback
B. Massage
C. Autogenic training
D. Visualization and Imagery
41. This is also known as Self-suggestion or Self-hypnosis

A. Biofeedback
B. Meditation
C. Autogenic training
D. Visualization and Imagery
42. Which among these drugs is NOT an anxiolytic?

A. Valium
B. Ativan
C. Milltown
D. Luvox
43. Kenneth, 25 year old diagnosed with HIV felt that he had not lived up with
God’s expectation. He fears that in the course of his illness, God will be
punitive and not be supportive. What kind of spiritual crisis is Kenneth
experiencing?

1. Spiritual Pain
2. Spiritual Anxiety
3. Spiritual Guilt
4. Spiritual Despair
A. 1,2
B. 2,3
C. 3,4
D. 1,4
44. Grace, believes that her relationship with God is broken. She tried to go to
church to ask forgiveness everyday to remedy her feelings. What kind of
spiritual distress is Grace experiencing?

A. Spiritual Pan
B. Spiritual Alienation
C. Spiritual Guilt
D. Spiritual Despair
45. Remedios felt “EMPTY” She felt that she has already lost God’s favor and
love because of her sins. This is a type of what spiritual crisis?
A. Spiritual Anger
B. Spiritual Loss
C. Spiritual Despair
D. Spiritual Anxiety
46. Budek is working with a schizophrenic patient. He noticed that the client is
agitated, pacing back and forth, restless and experiencing Anxiety +3. Budek
said “You appear restless” What therapeutic technique did Budek used?

A. Offering general leads


B. Seeking clarification
C. Making observation
D. Encouraging description of perception
47. Rommel told Budek “ I SEE DEAD PEOPLE “ Budek responded “You see
dead people?” This Is an example of therapeutic communication technique?

A. Reflecting
B. Restating
C. Exploring
D. Seeking clarification
48. Rommel told Budek, “Do you think Im crazy?” Budek responded, “Do you
think your crazy?” Budek uses what example of therapeutic communication?

A. Reflecting
B. Restating
C. Exploring
D. Seeking clarification
49. Myra, 21 year old nursing student has difficulty sleeping. She told Nurse
Budek “I really think a lot about my x boyfriend recently” Budek told Myra “And
that causes you difficulty sleeping?” Which therapeutic technique is used in
this situation?

A. Reflecting
B. Restating
C. Exploring
D. Seeking clarification
50. Myra told Budek “I cannot sleep, I stay away all night” Budek told her “You
have difficulty sleeping” This is what type of therapeutic communication
technique?

A. Reflecting
B. Restating
C. Exploring
D. Seeking clarification
51. Myra said “I saw my dead grandmother here at my bedside a while ago”
Budek responded “Really? That is hard to believe, How do you feel about it?”
What technique did Budek used?

A. Disproving
B. Disagreeing
C. Voicing Doubt
D. Presenting Reality
52. Which of the following is a therapeutic communication in response to “I
am a GOD, bow before me Or ill summon the dreaded thunder to burn you and
purge you to pieces!”

A. “You are not a GOD, you are Professor Tadle and you are a PE Teacher,
not a Nurse. I am Glen, Your nurse.”
B. “Oh hail GOD Tadle, everyone bow or face his wrath!”
C. “Hello Mr. Tadle, You are here in the hospital, I am your nurse and you
are a patient here”
D. “How can you be a GOD Mr. Tadle? Can you tell me more about it?”
53. Erik John Senna, Told Nurse Budek “ I don’t want to that, I don’t want that
thing.. that’s too painful!” Which of the following response is NON
THERAPEUTIC

A. “ This must be difficult for you, But I need to inject you this for your own
good”
B. “ You sound afraid”
C. “Are you telling me you don’t want this injection?”
D. “Why are you so anxious? Please tell me more about your feelings Erik”
54. Legrande De Salvaje Y Cobrador La Jueteng, was caught by the bacolod
police because of his illegal activities. When he got home after paying for the
bail, He shouted at his son. What defense mechanism did Mr. La Jueteng
used?

A. Restitution
B. Projection
C. Displacement
D. Undoing
55. Later that day, he bought his son ice cream and food. What defense
mechanism is Legrande unconsciously doing?

A. Restitution
B. Conversion
C. Redoing
D. Reaction formation
56. Crisis is a sudden event in ones life that disturbs a person’s homeostasis.
Which of the following is NOT TRUE in crisis?

A. The person experiences heightened feeling of stress


B. Inability to function in the usual organized manner
C. Lasts for 4 months
D. Indicates unpleasant emotional feelings
57. Which of the following is a characteristic of crisis?

A. Lasts for an unlimited period of time


B. There is a triggering event
C. Situation is not dangerous to the person
D. Person totality is not involved
58. Levito Devin, The Italian prime minister, is due to retire next week. He feels
depressed due to the enormous loss of influence, power, fame and fortune.
What type of crisis is Devin experiencing?

A. Situational
B. Maturational
C. Social
D. Phenomenal
59. Estrada, The Philippine president, has been unexpectedly impeached and
was out of office before the end of his term. He is in what type of crisis?
A. Situational
B. Maturational
C. Social
D. Phenomenal
60. The tsunami in Thailand and Indonesia took thousands of people and
change million lives. The people affected by the Tsunami are saddened and
do not know how to start all over again. What type of crisis is this?

A. Situational
B. Maturational
C. Social
D. Phenomenal
61. Which of the following is the BEST goal for crisis intervention?

A. Bring back the client in the pre crisis state


B. Make sure that the client becomes better
C. Achieve independence
D. Provide alternate coping mechanism
62. What is the best intervention when the client has just experienced the
crisis and still at the first phase of the crisis?

A. Behavior therapy
B. Gestalt therapy
C. Cognitive therapy
D. Milieu Therapy
63. Therapeutic nurse client relationship is describes as follows

1. Based on friendship and mutual interest


2. It is a professional relationship
3. It is focused on helping the patient solve problems and achieve health-
related goals
4. Maintained only as long as the patient requires professional helpA. 1,2,3
B. 1,2,4
C. 2,3,4
D. 1,3,4
64. The client is scheduled to have surgical removal of the tumor on her left
breast. Which of the following manifestation indicates that she is
experiencing Mild Anxiety?

A. She has increased awareness of her environmental details


B. She focused on selected aspect of her illness
C. She experiences incongruence of action, thoughts and feelings
D. She experiences random motor activities
65. Which of the following nursing intervention would least likely be effective
when dealing with a client with aggressive behavior?

A. Approach him in a calm manner


B. Provide opportunities to express feelings
C. Maintain eye contact with the client
D. Isolate the client from others
66. Whitney, a patient of nurse Budek, verbalizes… “I have nothing, nothing…
nothing! Don’t make me close one more door, I don’t wanna hurt anymore!”
Which of the following is the most appropriate response by Budek?

A. Why are you singing?


B. What makes you say that?
C. Ofcourse you are everything!
D. What is that you said?
67. Whitney verbalizes that she is anxious that the diagnostic test might
reveal laryngeal cancer. Which of the following is the most appropriate
nursing intervention?

A. Tell the client not to worry until the results are in


B. Ask the client to express feelings and concern
C. Reassure the client everything will be alright
D. Advice the client to divert his attention by watching television and
reading newspapers
68. Considered as the most accurate expression of person’s thought and
feelings

A. Verbal communication
B. Non verbal communication
C. Written communication
D. Oral communication
69. Represents inner feeling that a person do not like talking about.

A. Overt communication
B. Covert communication
C. Verbal communication
D. Non verbal communication
70. Which of the following is NOT a characteristic of an effective Nurse-Client
relationship?

A. Focused on the patient


B. Based on mutual trust
C. Conveys acceptance
D. Discourages emotional bond
71. A type of record wherein , each person or department makes notation in
separate records. A nurse will use the nursing notes, The doctor will use the
Physician’s order sheet etc. Data is arranged according to information source.

A. POMR
B. POR
C. Traditional
D. Resource oriented
72. Type of recording that integrates all data about the problem, gathered by
members of the health team.

A. POMR
B. Traditional
C. Resource oriented
D. Source oriented
73. These are data that are monitored by using graphic charts or graphs that
indicated the progression or fluctuation of client’s Temperature and Blood
pressure.

A. Progress notes
B. Kardex
C. Flow chart
D. Flow sheet
74. Provides a concise method of organizing and recording data about the
client. It is a series of flip cards kept in portable file used in change of shift
reports.

A. Kardex
B. Progress Notes
C. SOAPIE
D. Change of shift report
75. You are about to write an information on the Kardex. There are 4 available
writing instruments to use. Which of the following should you use?

A. Mongol #2
B. Permanent Ink
C. A felt or fountain pen
D. Pilot Pentel Pen marker
76. The client has an allergy to Iodine based dye. Where should you put this
vital information in the client’s chart?

A. In the first page of the client’s chart


B. At the last page of the client’s chart
C. At the front metal plate of the chart
D. In the Kardex
77. Which of the following is NOT TRUE about the Kardex

A. It provides readily available information


B. It is a tool of end of shift reports
C. The primary basis of endorsement
D. Where Allergies information are written
78. Which of the following, if seen on the Nurses notes, violates characteristic
of good recording?

A. The client has a blood pressure of 120/80, Temperature of 36.6 C Pulse


rate of 120 and Respiratory rate of 22
B. Ate 50% of food served
C. Refused administration of betaxolol
D. Visited and seen By Dr. Santiago
79. The physician ordered : Mannerix a.c , what does a.c means?

A. As desired
B. Before meals
C. After meals
D. Before bed time
80. The physician ordered, Maalox, 2 hours p.c, what does p.c means?

A. As desired
B. Before meals
C. After meals
D. Before bed time
81. The physician ordered, Maxitrol, Od. What does Od means?

A. Left eye
B. Right eye
C. Both eye
D. Once a day
82. The physician orderd, Magnesium Hydroxide cc Aluminum Hydroxide.
What does cc means?

A. without
B. with
C. one half
D. With one half dose
83. Physician ordered, Paracetamol tablet ss. What does ss means?

A. without
B. with
C. one half
D. With one half dose
84. Which of the following indicates that learning has been achieved?

A. Matuts starts exercising every morning and eating a balance diet after
you taught her mag HL tayo program
B. Donya Delilah has been able to repeat the steps of insulin
administration after you taught it to her
C. Marsha said “ I understand “ after you a health teaching about family
planning
D. John rated 100% on your given quiz about smoking and alcoholism
85. In his theory of learning as a BEHAVIORISM, he stated that transfer of
knowledge occurs if a new situation closely resembles an old one.

A. Bloom
B. Lewin
C. Thorndike
D. Skinner
86. Which of the following is TRUE with regards to learning?

A. Start from complex to simple


B. Goals should be hard to achieve so patient can strive to attain
unrealistic goals
C. Visual learning is the best for every individual
D. Do not teach a client when he is in pain
87. According to Bloom, there are 3 domains in learning. Which of these
domains is responsible for the ability of Donya Delilah to inject insulin?

A. Cognitive
B. Affective
C. Psychomotor
D. Motivative
88. Which domains of learning is responsible for making John and Marsha
understand the different kinds of family planning methods?

A. Cognitive
B. Affective
C. Psychomotor
D. Motivative
89. Which of the following statement clearly defines therapeutic
communication?

A. Therapeutic communication is an interaction process which is primarily


directed by the nurse
B. It conveys feeling of warmth, acceptance and empathy from the nurse
to a patient in relaxed atmosphere
C. Therapeutic communication is a reciprocal interaction based on trust
and aimed at identifying patient needs and developing mutual goals
D. Therapeutic communication is an assessment component of the
nursing process
90. Which of the following concept is most important in establishing a
therapeutic nurse patient relationship?

A. The nurse must fully understand the patient’s feelings, perception and
reactions before goals can be established
B. The nurse must be a role model for health fostering behavior
C. The nurse must recognize that the patient may manifest maladaptive
behavior after illness
D. The nurse should understand that patients might test her before trust is
established
91. Which of the following communication skill is most effective in dealing
with covert communication?

A. Validation
B. Listening
C. Evaluation
D. Clarification
92. Which of the following are qualities of a good recording?

1. Brevity
2. Completeness and chronology
3. Appropriateness
4. Accuracy
A. 1,2
B. 3,4
C. 1,2,3
D. 1,2,3,4
93. All of the following chart entries are correct except

A. V/S 36.8 C,80,16,120/80


B. Complained of chest pain
C. Seems agitated
D. Able to ambulate without assistance
94. Which of the following teaching method is effective in client who needs to
be educated about self injection of insulin?

A. Detailed explanation
B. Demonstration
C. Use of pamphlets
D. Film showing
95. What is the most important characteristic of a nurse patient relationship?

A. It is growth facilitating
B. Based on mutual understanding
C. Fosters hope and confidence
D. Involves primarily emotional bond
96. Which of the following nursing intervention is needed before teaching a
client post spleenectomy deep breathing and coughing exercises?

A. Tell the patient that deep breathing and coughing exercises is needed to
promote good breathing, circulation and prevent complication
B. Tell the client that deep breathing and coughing exercises is needed to
prevent Thrombophlebitis, hydrostatic pneumonia and atelectasis
C. Medicate client for pain
D. Tell client that cooperation is vital to improve recovery
97. The client has an allergy with penicillin. What is the best way to
communicate this information?

A. Place an allergy alert in the Kardex


B. Notify the attending physician
C. Write it on the patient’s chart
D. Take note when giving medications
98. An adult client is on extreme pain. He is moaning and grimacing. What is
the best way to assess the client’s pain?

A. Perform physical assessment


B. Have the client rate his pain on the smiley pain rating scale
C. Active listening on what the patient says
D. Observe the client’s behavior
99. Therapeutic communication begins with?

A. Knowing your client


B. Knowing yourself
C. Showing empathy
D. Encoding
100. The PCS gave new guidelines including leaflets to educate cancer
patients. As a nurse, When using materials like this, what is your
responsibility?

A. Read it for the patient


B. Give it for the patient to read himself
C. Let the family member read the material for the patient
D. Read it yourself then, Have the client read the material
Answers
1. B. Vasodilatation
2. D. Blood pressure will increase
3. A. Increase heat, thereby produce abatement of phagocytosis
4. C. Immediate Vasoconstriction
5. C. The affected part will loss its normal function
6. C. Purulent
7. A. Redness on the affected area
8. D. Monocytes
9. C. Eosinophil
10. A. Neutrophils
11. C. Pavementation
12. D. I don’t know the answer, perhaps I can tell you after I find it out later
13. C. Third intention
14. A. First intention
15. B. Second intention
16. B. High protein, High calorie with Vitamin A and C rich foods
17. A. It is a type of mechanical debridement using Wet dressing that is applied
and left to dry to remove dead tissues
18. C. Compression of the local nerve endings by the edema fluids
19. B. 1,3,4
20. D. Smoke cigarette around 3:00 A.M
21. B. Decrease Urine output
22. A. Directed towards helping an individual both physically and emotionally
23. B. Peplau
24. A. Pre Orientation
25. B. Orientation
26. C. Working
27. B. Positive regard
28. B. Positive regard
29. A. It is a nervous energy
30.  D. Martina is in Crisis
31. D. The source of dread or uneasiness is from a recognized entity
32. D. Panic
33. B. Moderate
34. B. Dilated pupils
35. B. When the client starts to have a narrow perceptual field and selective
inattentiveness
36. C. The client asks a question
37. A. Offer choices
38. D. “What makes you feel anxious?”
39. B. Concentrating on breathing without tensing the muscle, Letting go and
repeating a word or sound after each exhalation
40. A. Biofeedback
41. C. Autogenic training
42. D. Luvox
43. B. 2,3
44. B. Spiritual Alienation
45. B. Spiritual Loss
46. C. Making observation
47. B. Restating
48. A. Reflecting
49. B. Restating
50. B. Restating
51. C. Voicing Doubt
52. C. “Hello Mr. Tadle, You are here in the hospital, I am your nurse and you are
a patient here”
53. D. “Why are you so anxious? Please tell me more about your feelings Erik”
54. C. Displacement
55. A. Restitution
56. C. Lasts for 4 months
57. B. There is a triggering event
58. B. Maturational
59. A. Situational
60. C. Social
61. D. Provide alternate coping mechanism
62. D. Milieu Therapy
63. C. 2,3,4
64. A. She has increased awareness of her environmental details
65. B. Provide opportunities to express feelings
66. D. What is that you said?
67. B. Ask the client to express feelings and concern
68. B. Non verbal communication
69. B. Covert communication
70. D. Discourages emotional bond
71. D. Resource oriented
72. A. POMR
73. D. Flow sheet
74. A. Kardex
75. A. Mongol #2
76. C. At the front metal plate of the chart
77. C. The primary basis of endorsement
78. A. The client has a blood pressure of 120/80, Temperature of 36.6 C Pulse rate
of 120 and Respiratory rate of 22
79. B. Before meals
80. C. After meals
81. D. Once a day
82. B. with
83. C. one half
84. A. Matuts starts exercising every morning and eating a balance diet after you
taught her mag HL tayo program
85. C. Thorndike
86. D. Do not teach a client when he is in pain
87. C. Psychomotor
88. A. Cognitive
89. C. Therapeutic communication is a reciprocal interaction based on trust and
aimed at identifying patient needs and developing mutual goals
90. D. The nurse should understand that patients might test her before trust is
established
91. A. Validation
92. D. 1,2,3,4
93. C. Seems agitated
94. B. Demonstration
95. A. It is growth facilitating
96. A. Tell the patient that deep breathing and coughing exercises is needed to
promote good breathing, circulation and prevent complication
97. B. Notify the attending physician
98. C. Active listening on what the patient says
99. B. Knowing yourself
100. D. Read it yourself then, Have the client read the material
PNLE: FON Practice Exam for
Nursing Process, Physical and
Health Assessment and Routine
Procedures
1. She is the first one to coin the term “NURSING PROCESS” She introduced 3
steps of nursing process which are Observation, Ministration and Validation.

A. Nightingale
B. Johnson
C. Rogers
D. Hall
2. The American Nurses association formulated an innovation of the Nursing
process. Today, how many distinct steps are there in the nursing process?

A. APIE – 4
B. ADPIE – 5
C. ADOPIE – 6
D. ADOPIER – 7
3. They are the first one to suggest a 4 step nursing process which are : APIE ,
or assessment, planning, implementation and evaluation.

1. Yura
2. Walsh
3. Roy
4. Knowles
A. 1,2
B. 1,3
C. 3,4
D. 2,3
4. Which characteristic of nursing process is responsible for proper utilization
of human resources, time and cost resources?

A. Organized and Systematic


B. Humanistic
C. Efficient
D. Effective
5. Which characteristic of nursing process addresses the INDIVIDUALIZED
care a client must receive?

A. Organized and Systematic


B. Humanistic
C. Efficient
D. Effective
6. A characteristic of the nursing process that is essential to promote client
satisfaction and progress. The care should also be relevant with the client’s
needs.

A. Organized and Systematic


B. Humanistic
C. Efficient
D. Effective
7. Rhina, who has Menieres disease, said that her environment is moving.
Which of the following is a valid assessment?

1. Rhina is giving an objective data


2. Rhina is giving a subjective data
3. The source of the data is primary
4. The source of the data is secondary
A. 1,3
B. 2,3
C. 2.4
D. 1,4
8. Nurse Angela, observe Joel who is very apprehensive over the impending
operation. The client is experiencing dyspnea, diaphoresis and asks lots of
questions. Angela made a diagnosis of ANXIETY R/T INTRUSIVE
PROCEDURE. This is what type of Nursing Diagnosis?
A. Actual
B. Probable
C. Possible
D. Risk
9. Nurse Angela diagnosed Mrs. Delgado, who have undergone a BKA. Her
diagnosis is SELF ESTEEM DISTURBANCE R/T CHANGE IN BODY IMAGE.
Although the client has not yet seen her lost leg, Angela already anticipated
the diagnosis. This is what type of Diagnosis?

A. Actual
B. Probable
C. Possible
D. Risk
10. Nurse Angela is about to make a diagnosis but very unsure because the
S/S the client is experiencing is not specific with her diagnosis of
POWERLESSNESS R/T DIFFICULTY ACCEPTING LOSS OF LOVED ONE. She
then focus on gathering data to refute or prove her diagnosis but her plans
and interventions are already ongoing for the diagnosis. Which type of
Diagnosis is this?

A. Actual
B. Probable
C. Possible
D. Risk
11. Nurse Angela knew that Stephen Lee Mu Chin, has just undergone an
operation with an incision near the diaphragm. She knew that this will
contribute to some complications later on. She then should develop what type
of Nursing diagnosis?

A. Actual
B. Probable
C. Possible
D. Risk
12. Which of the following Nursing diagnosis is INCORRECT?

A. Fluid volume deficit R/T Diarrhea


B. High risk for injury R/T Absence of side rails
C. Possible ineffective coping R/T Loss of loved one
D. Self esteem disturbance R/T Effects of surgical removal of the leg
13. Among the following statements, which should be given the HIGHEST
priority?

A. Client is in extreme pain


B. Client’s blood pressure is 60/40
C. Client’s temperature is 40 deg. Centigrade
D. Client is cyanotic
14. Which of the following need is given a higher priority among others?

A. The client has attempted suicide and safety precaution is needed


B. The client has disturbance in his body image because of the recent
operation
C. The client is depressed because her boyfriend left her all alone
D. The client is thirsty and dehydrated
15. Which of the following is TRUE with regards to Client Goals?

A. They are specific, measurable, attainable and time bounded


B. They are general and broadly stated
C. They should answer for WHO, WHAT ACTIONS, WHAT
CIRCUMSTANCES, HOW WELL and WHEN.
D. Example is : After discharge planning, Client demonstrated the proper
psychomotor skills for insulin injection.
16. Which of the following is a NOT a correct statement of an Outcome
criteria?

A. Ambulates 30 feet with a cane before discharge


B. Discusses fears and concerns regarding the surgical procedure
C. Demonstrates proper coughing and breathing technique after a teaching
session
D. Reestablishes a normal pattern of elimination
17. Which of the following is a OBJECTIVE data?

A. Dizziness
B. Chest pain
C. Anxiety
D. Blue nails
18. A patient’s chart is what type of data source?

A. Primary
B. Secondary
C. Tertiary
D. Can be A and B
19. All of the following are characteristic of the Nursing process except

A. Dynamic
B. Cyclical
C. Universal
D. Intrapersonal
20. Which of the following is true about the NURSING CARE PLAN?

A. It is nursing centered
B. Rationales are supported by interventions
C. Verbal
D. Atleast 2 goals are needed for every nursing diagnosis
21. A framework for health assessment that evaluates the effects of stressors
to the mind, body and environment in relation with the ability of the client to
perform ADL.

A. Functional health framework


B. Head to toe framework
C. Body system framework
D. Cephalocaudal framework
22. Client has undergone Upper GI and Lower GI series. Which type of health
assessment framework is used in this situation?

A. Functional health framework


B. Head to toe framework
C. Body system framework
D. Cephalocaudal framework
23. Which of the following statement is true regarding temperature?

A. Oral temperature is more accurate than rectal temperature


B. The bulb used in Rectal temperature reading is pear shaped or round
C. The older the person, the higher his BMR
D. When the client is swimming, BMR Decreases
24. A type of heat loss that occurs when the heat is dissipated by air current

A. Convection
B. Conduction
C. Radiation
D. Evaporation
25. Which of the following is TRUE about temperature?

A. The highest temperature usually occurs later in a day, around 8 P.M to


12 M.N
B. The lowest temperature is usually in the Afternoon, Around 12 P.M
C. Thyroxin decreases body temperature
D. Elderly people are risk for hyperthermia due to the absence of fats,
Decreased thermoregulatory control and sedentary lifestyle.
26. Hyperpyrexia is a condition in which the temperature is greater than

A. 40 degree Celsius
B. 39 degree Celsius
C. 100 degree Fahrenheit
D. 105.8 degree Fahrenheit
27. Tympanic temperature is taken from John, A client who was brought
recently into the ER due to frequent barking cough. The temperature reads
37.9 Degrees Celsius. As a nurse, you conclude that this temperature is

A. High
B. Low
C. At the low end of the normal range
D. At the high end of the normal range
28. John has a fever of 38.5 Deg. Celsius. It surges at around 40 Degrees and
go back to 38.5 degrees 6 times today in a typical pattern. What kind of fever
is John having?

A. Relapsing
B. Intermittent
C. Remittent
D. Constant
29. John has a fever of 39.5 degrees 2 days ago, But yesterday, he has a
normal temperature of 36.5 degrees. Today, his temperature surges to 40
degrees. What type of fever is John having?

A. Relapsing
B. Intermittent
C. Remittent
D. Constant
30. John’s temperature 10 hours ago is a normal 36.5 degrees. 4 hours ago,
He has a fever with a temperature of 38.9 Degrees. Right now, his temperature
is back to normal. Which of the following best describe the fever john is
having?

A. Relapsing
B. Intermittent
C. Remittent
D. Constant
31. The characteristic fever in Dengue Virus is characterized as:

A. Tricyclic
B. Bicyclic
C. Biphasic
D. Triphasic
32. When John has been given paracetamol, his fever was brought down
dramatically from 40 degrees Celsius to 36.7 degrees in a matter of 10
minutes. The nurse would assess this event as:

A. The goal of reducing john’s fever has been met with full satisfaction of
the outcome criteria
B. The desired goal has been partially met
C. The goal is not completely met
D. The goal has been met but not with the desired outcome criteria
33. What can you expect from Marianne, who is currently at the ONSET stage
of fever?
A. Hot, flushed skin
B. Increase thirst
C. Convulsion
D. Pale,cold skin
34. Marianne is now at the Defervescence stage of the fever, which of the
following is expected?

A. Delirium
B. Goose flesh
C. Cyanotic nail beds
D. Sweating
35. Considered as the most accessible and convenient method for
temperature taking

A. Oral
B. Rectal
C. Tympanic
D. Axillary
36. Considered as Safest and most non invasive method of temperature
taking

A. Oral
B. Rectal
C. Tympanic
D. Axillary
37. Which of the following is NOT a contraindication in taking ORAL
temperature?

A. Quadriplegic
B. Presence of NGT
C. Dyspnea
D. Nausea and Vomitting
38. Which of the following is a contraindication in taking RECTAL
temperature?

A. Unconscious
B. Neutropenic
C. NPO
D. Very young children
39. How long should the Rectal Thermometer be inserted to the clients anus?

A. 1 to 2 inches
B. 5 to 1.5 inches
C. 3 to 5 inches
D. 2 to 3 inches
40. In cleaning the thermometer after use, The direction of the cleaning to
follow Medical Asepsis is :

A. From bulb to stem


B. From stem to bulb
C. From stem to stem
D. From bulb to bulb
41. How long should the thermometer stay in the Client’s Axilla?

A. 3 minutes
B. 4 minutes
C. 7 minutes
D. 10 minutes
42. Which of the following statement is TRUE about pulse?

A. Young person have higher pulse than older persons


B. Males have higher pulse rate than females after puberty
C. Digitalis has a positive chronotropic effect
D. In lying position, Pulse rate is higher
43. The following are correct actions when taking radial pulse except:

A. Put the palms downward


B. Use the thumb to palpate the artery
C. Use two or three fingers to palpate the pulse at the inner wrist
D. Assess the pulse rate, rhythm, volume and bilateral quality
44. The difference between the systolic and diastolic pressure is termed as

A. Apical rate
B. Cardiac rate
C. Pulse deficit
D. Pulse pressure
45. Which of the following completely describes PULSUS PARADOXICUS?

A. A greater-than-normal increase in systolic blood pressure with


inspiration
B. A greater-than-normal decrease in systolic blood pressure with
inspiration
C. Pulse is paradoxically low when client is in standing position and high
when supine.
D. Pulse is paradoxically high when client is in standing position and low
when supine.
46. Which of the following is TRUE about respiration?

A. I:E 2:1
B. I:E : 4:3
C. I:E 1:1
D. I:E 1:2
47. Contains the pneumotaxic and the apneutic centers

A. Medulla oblongata
B. Pons
C. Carotid bodies
D. Aortic bodies
48. Which of the following is responsible for deep and prolonged inspiration

A. Medulla oblongata
B. Pons
C. Carotid bodies
D. Aortic bodies
49. Which of the following is responsible for the rhythm and quality of
breathing?

A. Medulla oblongata
B. Pons
C. Carotid bodies
D. Aortic bodies
50. The primary respiratory center

A. Medulla oblongata
B. Pons
C. Carotid bodies
D. Aortic bodies
51. Which of the following is TRUE about the mechanism of action of the
Aortic and Carotid bodies?

A. If the BP is elevated, the RR increases


B. If the BP is elevated, the RR decreases
C. Elevated BP leads to Metabolic alkalosis
D. Low BP leads to Metabolic acidosis
52. All of the following factors correctly influence respiration except one.
Which of the following is incorrect?

A. Hydrocodone decreases RR
B. Stress increases RR
C. Increase temperature of the environment, Increase RR
D. Increase altitude, Increase RR
53. When does the heart receives blood from the coronary artery?

A. Systole
B. Diastole
C. When the valves opens
D. When the valves closes
54. Which of the following is more life threatening?

A. BP = 180/100
B. BP = 160/120
C. BP = 90/60
D. BP = 80/50
55. Refers to the pressure when the ventricles are at rest

A. Diastole
B. Systole
C. Preload
D. Pulse pressure
56. Which of the following is TRUE about the blood pressure determinants?

A. Hypervolemia lowers BP
B. Hypervolemia increases GFR
C. HCT of 70% might decrease or increase BP
D. Epinephrine decreases BP
57. Which of the following do not correctly correlates the increase BP of Ms.
Aida, a 70 year old diabetic?

A. Females, after the age 65 tends to have lower BP than males


B. Disease process like Diabetes increase BP
C. BP is highest in the morning, and lowest during the night
D. Africans, have a greater risk of hypertension than Caucasian and
Asians.
58. How many minutes are allowed to pass if the client had engaged in
strenuous activities, smoked or ingested caffeine before taking his/her BP?

A. 5
B. 10
C. 15
D. 30
59. Too narrow cuff will cause what change in the Client’s BP?

A. True high reading


B. True low reading
C. False high reading
D. False low reading
60. Which is a preferable arm for BP taking?

A. An arm with the most contraptions


B. The left arm of the client with a CVA affecting the right brain
C. The right arm
D. The left arm
61. Which of the following is INCORRECT in assessing client’s BP?
A. Read the mercury at the upper meniscus, preferably at the eye level to
prevent error of parallax
B. Inflate and deflate slowly, 2-3 mmHg at a time
C. The sound heard during taking BP is known as KOROTKOFF sound
D. If the BP is taken on the left leg using the popliteal artery pressure, a BP
of 160/80 is normal.
62. Which of the following is the correct interpretation of the ERROR OF
PARALLAX

A. If the eye level is higher than the level of the meniscus, it will cause a
false high reading
B. If the eye level is higher than the level of the meniscus, it will cause a
false low reading
C. If the eye level is lower than the level of the meniscus, it will cause a
false low reading
D. If the eye level is equal to that of the level of the upper meniscus, the
reading is accurate
63. How many minute/s is/are allowed to pass before making a re-reading
after the first one?

A. 1
B. 5
C. 15
D. 30
64. Which of the following is TRUE about the auscultation of blood pressure?

A. Pulse + 4 is considered as FULL


B. The bell of the stethoscope is use in auscultating BP
C. Sound produced by BP is considered as HIGH frequency sound
D. Pulse +1 is considered as NORMAL
65. In assessing the abdomen, Which of the following is the correct sequence
of the physical assessment?

A. Inspection, Auscultation, Percussion, Palpation


B. Palpation, Auscultation, Percussion, Inspection
C. Inspection, Palpation, Auscultation, Percussion
D. Inspection, Auscultation, Palpation, Percussion
66. The sequence in examining the quadrants of the abdomen is:

A. RUQ,RLQ,LUQ,LLQ
B. RLQ,RUQ,LLQ,LUQ
C. RUQ,RLQ,LLQ,LUQ
D. RLQ,RUQ,LUQ,LLQ
67. In inspecting the abdomen, which of the following is NOT DONE?

A. Ask the client to void first


B. Knees and legs are straighten to relax the abdomen
C. The best position in assessing the abdomen is Dorsal recumbent
D. The knees and legs are externally rotated
68. Dr. Fabian De Las Santas, is about to conduct an ophthalmoscope
examination. Which of the following, if done by a nurse, is a Correct
preparation before the procedure?

A. Provide the necessary draping to ensure privacy


B. Open the windows, curtains and light to allow better illumination
C. Pour warm water over the ophthalmoscope to ensure comfort
D. Darken the room to provide better illumination
69. If the client is female, and the doctor is a male and the patient is about to
undergo a vaginal and cervical examination, why is it necessary to have a
female nurse in attendance?

A. To ensure that the doctor performs the procedure safely


B. To assist the doctor
C. To assess the client’s response to examination
D. To ensure that the procedure is done in an ethical manner
70. In palpating the client’s breast, Which of the following position is
necessary for the patient to assume before the start of the procedure?

A. Supine
B. Dorsal recumbent
C. Sitting
D. Lithotomy
71. When is the best time to collect urine specimen for routine urinalysis and
C/S?
A. Early morning
B. Later afternoon
C. Midnight
D. Before breakfast
72. Which of the following is among an ideal way of collecting a urine
specimen for culture and sensitivity?

A. Use a clean container


B. Discard the first flow of urine to ensure that the urine is not
contaminated
C. Collect around 30-50 ml of urine
D. Add preservatives, refrigerate the specimen or add ice according to the
agency’s protocol
73. In a 24 hour urine specimen started Friday, 9:00 A.M, which of the
following if done by a Nurse indicate a NEED for further procedural debriefing?

A. The nurse ask the client to urinate at 9:00 A.M, Friday and she included
the urine in the 24 hour urine specimen
B. The nurse discards the Friday 9:00 A M urine of the client
C. The nurse included the Saturday 9:00 A.M urine of the client to the
specimen collection
D. The nurse added preservatives as per protocol and refrigerates the
specimen
74. This specimen is required to assess glucose levels and for the presence of
albumin the the urine

A. Midstream clean catch urine


B. 24 hours urine collection
C. Postprandial urine collection
D. Second voided urine
75. When should the client test his blood sugar levels for greater accuracy?

A. During meals
B. In between meals
C. Before meals
D. 2 Hours after meals
76. In collecting a urine from a catheterized patient, Which of the following
statement indicates an accurate performance of the procedure?

A. Clamp above the port for 30 to 60 minutes before drawing the urine
from the port
B. Clamp below the port for 30 to 60 minutes before drawing the urine
from the port
C. Clamp above the port for 5 to 10 minutes before drawing the urine from
the port
D. Clamp below the port for 5 to 10 minutes before drawing the urine from
the port
77. A community health nurse should be resourceful and meet the needs of
the client. A villager ask him, Can you test my urine for glucose? Which of the
following technique allows the nurse to test a client’s urine for glucose
without the need for intricate instruments.

A. Acetic Acid test


B. Nitrazine paper test
C. Benedict’s test
D. Litmus paper test
78. A community health nurse is assessing client’s urine using the Acetic Acid
solution. Which of the following, if done by a nurse, indicates lack of correct
knowledge with the procedure?

A. The nurse added the Urine as the 2/3 part of the solution
B. The nurse heats the test tube after adding 1/3 part acetic acid
C. The nurse heats the test tube after adding 2/3 part of Urine
D. The nurse determines abnormal result if she noticed that the test tube
becomes cloudy
79. Which of the following is incorrect with regards to proper urine testing
using Benedict’s Solution?

A. Heat around 5ml of Benedict’s solution together with the urine in a test
tube
B. Add 8 to 10 drops of urine
C. Heat the Benedict’s solution without the urine to check if the solution is
contaminated
D. If the color remains BLUE, the result is POSITIVE
80. +++ Positive result after Benedicts test is depicted by what color?

A. Blue
B. Green
C. Yellow
D. Orange
81. Clinitest is used in testing the urine of a client for glucose. Which of the
following, If committed by a nurse indicates error?

A. Specimen is collected after meals


B. The nurse puts 1 clinitest tablet into a test tube
C. She added 5 drops of urine and 10 drops of water
D. If the color becomes orange or red, It is considered postitive
82. Which of the following nursing intervention is important for a client
scheduled to have a Guaiac Test?

A. Avoid turnips, radish and horseradish 3 days before procedure


B. Continue iron preparation to prevent further loss of Iron
C. Do not eat read meat 12 hours before procedure
D. Encourage caffeine and dark colored foods to produce accurate results
83. In collecting a routine specimen for fecalysis, Which of the following, if
done by a nurse, indicates inadequate knowledge and skills about the
procedure?

A. The nurse scoop the specimen specifically at the site with blood and
mucus
B. She took around 1 inch of specimen or a teaspoonful
C. Ask the client to call her for the specimen after the client wiped off his
anus with a tissue
D. Ask the client to defecate in a bedpan, Secure a sterile container
84. In a routine sputum analysis, Which of the following indicates proper
nursing action before sputum collection?

A. Secure a clean container


B. Discard the container if the outside becomes contaminated with the
sputum
C. Rinse the client’s mouth with Listerine after collection
D. Tell the client that 4 tablespoon of sputum is needed for each specimen
for a routine sputum analysis
85. Who collects Blood specimen?

A. The nurse
B. Medical technologist
C. Physician
D. Physical therapist
86. David, 68 year old male client is scheduled for Serum Lipid analysis. Which
of the following health teaching is important to ensure accurate reading?

A. Tell the patient to eat fatty meals 3 days prior to the procedure
B. NPO for 12 hours pre procedure
C. Ask the client to drink 1 glass of water 1 hour prior to the procedure
D. Tell the client that the normal serum lipase level is 50 to 140 U/L
87. The primary factor responsible for body heat production is the

A. Metabolism
B. Release of thyroxin
C. Muscle activity
D. Stress
88. The heat regulating center is found in the

A. Medulla oblongata
B. Thalamus
C. Hypothalamus
D. Pons
89. A process of heat loss which involves the transfer of heat from one
surface to another is

A. Radiation
B. Conduction
C. Convection
D. Evaporation
90. Which of the following is a primary factor that affects the BP?
A. Obesity
B. Age
C. Stress
D. Gender
91. The following are social data about the client except

A. Patient’s lifestyle
B. Religious practices
C. Family home situation
D. Usual health status
92. The best position for any procedure that involves vaginal and cervical
examination is

A. Dorsal recumbent
B. Side lying
C. Supine
D. Lithotomy
93. Measure the leg circumference of a client with bipedal edema is best done
in what position?

A. Dorsal recumbent
B. Sitting
C. Standing
D. Supine
94. In palpating the client’s abdomen, Which of the following is the best
position for the client to assume?

A. Dorsal recumbent
B. Side lying
C. Supine
D. Lithotomy
95. Rectal examination is done with a client in what position?

A. Dorsal recumbent
B. Sims position
C. Supine
D. Lithotomy
96. Which of the following is a correct nursing action when collecting urine
specimen from a client with an Indwelling catheter?

A. Collect urine specimen from the drainage bag


B. Detach catheter from the connecting tube and draw the specimen from
the port
C. Use sterile syringe to aspirate urine specimen from the drainage port
D. Insert the syringe straight to the port to allow self sealing of the port
97. Which of the following is inappropriate in collecting mid stream clean
catch urine specimen for urine analysis?

A. Collect early in the morning, First voided specimen


B. Do perineal care before specimen collection
C. Collect 5 to 10 ml for urine
D. Discard the first flow of the urine
98. When palpating the client’s neck for lymphadenopathy, where should the
nurse position himself?

A. At the client’s back


B. At the client’s right side
C. At the client’s left side
D. In front of a sitting client
99. Which of the following is the best position for the client to assume if the
back is to be examined by the nurse?

A. Standing
B. Sitting
C. Side lying
D. Prone
100. In assessing the client’s chest, which position best show chest
expansion as well as its movements?

A. Sitting
B. Prone
C. Sidelying
D. Supine
Answers
1. D. Hall
2. C. ADOPIE – 6
3. A. 1,2
4. C. Efficient
5. B. Humanistic
6. D. Effective
7. B. 2,3
8. A. Actual
9. D. Risk
10. C. Possible
11. D. Risk
12.  B. High risk for injury R/T Absence of side rails
13. D. Client is cyanotic
14. D. The client is thirsty and dehydrated
15. B. They are general and broadly stated
16. D. Reestablishes a normal pattern of elimination
17. D. Blue nails
18. B. Secondary
19. D. Intrapersonal
20. A. It is nursing centered
21. A. Functional health framework
22. C. Body system framework
23. B. The bulb used in Rectal temperature reading is pear shaped or round
24. A. Convection
25. A. The highest temperature usually occurs later in a day, around 8 P.M to 12
M.N
26. D. 105.8 degree Fahrenheit
27. D. At the high end of the normal range
28. C. Remittent
29. A. Relapsing
30. B. Intermittent
31. C. Biphasic
32. D. The goal has been met but not with the desired outcome criteria
33. D. Pale,cold skin
34. D. Sweating
35. A. Oral
36. D. Axillary
37. A. Quadriplegic
38. B. Neutropenic
39. B. .5 to 1.5 inches
40. B. From stem to bulb
41. C. 7 minutes
42. A. Young person have higher pulse than older persons
43.  B. Use the thumb to palpate the artery
44. D. Pulse pressure
45. B. A greater-than-normal decrease in systolic blood pressure with inspiration
46. D. I:E 1:2
47.  B. Pons
48. B. Pons
49. B. Pons
50. A. Medulla oblongata
51. B. If the BP is elevated, the RR decreases
52.  C. Increase temperature of the environment, Increase RR
53.  B. Diastole
54. B. BP = 160/120
55. A. Diastole
56. D. Epinephrine decreases BP
57. A. Females, after the age 65 tends to have lower BP than males
58.  D. 30
59.  C. False high reading
60. D. The left arm
61. A. Read the mercury at the upper meniscus, preferably at the eye level to
prevent error of parallax
62. B. If the eye level is higher than the level of the meniscus, it will cause a false
low reading
63. A. 1
64. B. The bell of the stethoscope is use in auscultating BP
65. A. Inspection, Auscultation, Percussion, Palpation
66. D. RLQ,RUQ,LUQ,LLQ
67. B. Knees and legs are straighten to relax the abdomen
68. D. Darken the room to provide better illumination
69. D. To ensure that the procedure is done in an ethical manner
70. A. Supine
71. A. Early morning
72. B. Discard the first flow of urine to ensure that the urine is not contaminated
73. A. The nurse ask the client to urinate at 9:00 A.M, Friday and she included the
urine in the 24 hour urine specimen 
74. D. Second voided urine
75. C. Before meals
76. B. Clamp below the port for 30 to 60 minutes before drawing the urine from the
port
77. C. Benedict’s test
78. B. The nurse heats the test tube after adding 1/3 part acetic acid
79. D. If the color remains BLUE, the result is POSITIVE
80. D. Orange
81. A. Specimen is collected after meals
82. A. Avoid turnips, radish and horseradish 3 days before procedure
83. C. Ask the client to call her for the specimen after the client wiped off his anus
with a tissue
84. C. Rinse the client’s mouth with Listerine after collection
85. B. Medical technologist
86. B. NPO for 12 hours pre procedure
87. A. Metabolism
88. C. Hypothalamus
89. B. Conduction
90. C. Stress
91. A. Patient’s lifestyle
92. D. Lithotomy
93. A. Dorsal recumbent
94. A. Dorsal recumbent
95. B. Sims position
96. C. Use sterile syringe to aspirate urine specimen from the drainage port
97. C. Collect 5 to 10 ml for urine
98. A. At the client’s back
99. A. Standing
100. A. Sitting
PNLE: FON Practice Exam for
Oxygenation and Nutrition
1. Which one of the following is NOT a function of the Upper airway?

A. For clearance mechanism such as coughing


B. Transport gases to the lower airways
C. Warming, Filtration and Humidification of inspired air
D. Protect the lower airway from foreign mater
2. It is the hair the lines the vestibule which function as a filtering mechanism
for foreign objects

A. Cilia
B. Nares
C. Carina
D. Vibrissae
3. This is the paranasal sinus found between the eyes and the nose that
extends backward into the skull

A. Ehtmoid
B. Sphenoid
C. Maxillary
D. Frontal
4. Which paranasal sinus is found over the eyebrow?

A. Ehtmoid
B. Sphenoid
C. Maxillary
D. Frontal
5. Gene De Vonne Katrouchuacheulujiki wants to change her surname to
something shorter, The court denied her request which depresses her and find
herself binge eating. She accidentally aspirate a large piece of nut and it
passes the carina. Probabilty wise, Where will the nut go?

A. Right main stem bronchus


B. Left main stem bronchus
C. Be dislodged in between the carina
D. Be blocked by the closed epiglottis
6. Which cell secretes mucus that help protect the lungs by trapping debris in
the respiratory tract?

A. Type I pneumocytes
B. Type II pneumocytes
C. Goblet cells
D. Adipose cells
7. How many lobes are there in the RIGHT LUNG?

A. One
B. Two
C. Three
D. Four
8. The presence of the liver causes which anatomical difference of the
Kidneys and the Lungs?

A. Left kidney slightly lower, Left lung slightly shorter


B. Left kidney slightly higher, Left lung slightly shorter
C. Right kidney lower, Right lung shorter
D. Right kidney higher, Right lung shorter
9. Surfactant is produced by what cells in the alveoli?

A. Type I pneumocytes
B. Type II pneumocytes
C. Goblet cells
D. Adipose cells
10. The normal L:S Ratio to consider the newborn baby viable is
A. 1:2
B. 2:1
C. 3:1
D. 1:3
11. Refers to the extra air that can be inhaled beyond the normal tidal volume

A. Inspiratory reserve volume


B. Expiratory reserve volume
C. Functional residual capacity
D. Residual volume
12. This is the amount of air remained in the lungs after a forceful expiration

A. Inspiratory reserve volume


B. Expiratory reserve volume
C. Functional residual capacity
D. Residual volume
13. Casssandra, A 22 year old grade Agnostic, Asked you, how many spikes of
bones are there in my ribs? Your best response is which of the following?

A. We have 13 pairs of ribs Cassandra


B. We have 12 pairs of ribs Cassandra
C. Humans have 16 pairs of ribs, and that was noted by Vesalius in 1543
D. Humans have 8 pairs of ribs. 4 of which are floating
14. Which of the following is considered as the main muscle of respiration?

A. Lungs
B. Intercostal Muscles
C. Diaphragm
D. Pectoralis major
15. Cassandra asked you : How many air is there in the oxygen and how many
does human requires? Which of the following is the best response :

A. God is good, Man requires 21% of oxygen and we have 21% available in
our air
B. Man requires 16% of oxygen and we have 35% available in our air
C. Man requires 10% of oxygen and we have 50% available in our air
D. Human requires 21% of oxygen and we have 21% available in our air
16. Which of the following is TRUE about Expiration?

A. A passive process
B. The length of which is half of the length of Inspiration
C. Stridor is commonly heard during expiration
D. Requires energy to be carried out
17. Which of the following is TRUE in postural drainage?

A. Patient assumes position for 10 to 15 minutes


B. Should last only for 60 minutes
C. Done best P.C
D. An independent nursing action
18. All but one of the following is a purpose of steam inhalation

A. Mucolytic
B. Warm and humidify air
C. Administer medications
D. Promote bronchoconstriction
19. Which of the following is NOT TRUE in steam inhalation?

A. It is a dependent nursing action


B. Spout is put 12-18 inches away from the nose
C. Render steam inhalation for atleast 60 minutes
D. Cover the client’s eye with wash cloth to prevent irritation
20. When should a nurse suction a client?

A. As desired
B. As needed
C. Every 1 hour
D. Every 4 hours
21. Ernest Arnold Hamilton, a 60 year old American client was mobbed by
teen gangsters near New york, Cubao. He was rushed to John John Hopio
Medical Center and was Unconscious. You are his nurse and you are to
suction his secretions. In which position should you place Mr. Hamilton?

A. High fowlers
B. Semi fowlers
C. Prone
D. Side lying
22. You are about to set the suction pressure to be used to Mr. Hamilton. You
are using a Wall unit suction machine. How much pressure should you set the
valve before suctioning Mr. Hamilton?

A. 50-95 mmHg
B. 200-350 mmHg
C. 100-120 mmHg
D. 10-15 mmHg
23. The wall unit is not functioning; You then try to use the portable suction
equipment available. How much pressure of suction equipment is needed to
prevent trauma to mucus membrane and air ways in case of portable suction
units?

A. 2-5 mmHg
B. 5-10 mmHg
C. 10-15 mmHg
D. 15-25 mmHg
24. There are four catheter sizes available for use, which one of these should
you use for Mr. Hamilton?

A. Fr. 18
B. Fr. 12
C. Fr. 10
D. Fr, 5
25. Which of the following, if done by the nurse, indicates incompetence
during suctioning an unconscious client?

A. Measure the length of the suction catheter to be inserted by measuring


from the tip of the nose, to the earlobe, to the xiphoid process
B. Use KY Jelly if suctioning nasopharyngeal secretion
C. The maximum time of suctioning should not exceed 15 seconds
D. Allow 30 seconds interval between suctioning
26. Which of the following is the initial sign of hypoxemia in an adult client?

1. Tachypnea
2. Tachycardia
3. Cyanosis
4. Pallor
5. Irritability
6. Flaring of NaresA. 1,2
B. 2,5
C. 2,6
D. 3,4
27. Which method of oxygenation least likely produces anxiety and
apprehension?

A. Nasal Cannula
B. Simple Face mask
C. Non Rebreather mask
D. Partial Rebreather mask
28. Which of the following oxygen delivery method can deliver 100% Oxygen at
15 LPM?

A. Nasal Cannula
B. Simple Face mask
C. Non Rebreather mask
D. Partial Rebreather mask
29. Which of the following is not true about OXYGEN?

A. Oxygen is odorless, tasteless and colorless gas.


B. Oxygen can irritate mucus membrane
C. Oxygen supports combustion
D. Excessive oxygen administration results in respiratory acidosis
30. Roberto San Andres, A new nurse in the hospital is about to administer
oxygen on patient with Respiratory distress. As his senior nurse, you should
intervene if Roberto will:

A. Uses venture mask in oxygen administration


B. Put a non rebreather mask in the patient before opening the oxygen
source
C. Use a partial rebreather mask to deliver oxygen
D. Check for the doctor’s order for Oxygen administration
31. Which of the following will alert the nurse as an early sign of hypoxia?

A. Client is tired and dyspneic


B. The client is coughing out blood
C. The client’s heart rate is 50 BPM
D. Client is frequently turning from side to side
32. Miguelito de balboa, An OFW presents at the admission with an A:P
Diameter ratio of 2:1, Which of the following associated finding should the
nurse expect?

A. Pancytopenia
B. Anemia
C. Fingers are Club-like
D. Hematocrit of client is decreased
33. The best method of oxygen administration for client with COPD uses:

A. Cannula
B. Simple Face mask
C. Non rebreather mask
D. Venturi mask
34. Mang dagul, a 50 year old chronic smoker was brought to the E.R because
of difficulty in breathing. Pleural effusion was the diagnosis and CTT was
ordered. What does C.T.T Stands for?

A. Chest tube thoracotomy


B. Chest tube thoracostomy
C. Closed tube thoracotomy
D. Closed tube thoracostmy
35. Where will the CTT be inserted if we are to drain fluids accumulated in
Mang dagul’s pleura?

A. 2nd ICS
B. 4th ICS
C. 5th ICS
D. 8th ICS
36. There is a continuous bubbling in the water sealed drainage system with
suction. And oscillation is observed. As a nurse, what should you do?
A. Consider this as normal findings
B. Notify the physician
C. Check for tube leak
D. Prepare a petrolatum gauze dressing
37. Which of the following is true about nutrition?

A. It is the process in which food are broken down, for the body to use in
growth and development
B. It is a process in which digested proteins, fats, minerals, vitamins and
carbohydrates are transported into the circulation
C. It is a chemical process that occurs in the cell that allows for energy
production, energy use, growth and tissue repair
D. It is the study of nutrients and the process in which they are use by the
body
38. The majority of the digestion processes take place in the

A. Mouth
B. Small intestine
C. Large intestine
D. Stomach
39. All of the following is true about digestion that occurs in the Mouth except

A. It is where the digestion process starts


B. Mechanical digestion is brought about by mastication
C. The action of ptyalin or the salivary tyrpsin breaks down starches into
maltose
D. Deglutition occurs after food is broken down into small pieces and well
mixed with saliva
40. Which of the following foods lowers the cardiac sphincter pressure?

A. Roast beef, Steamed cauliflower and Rice


B. Orange juice, Non fat milk, Dry crackers
C. Decaffeinated coffee, Sky flakes crackers, Suman
D. Coffee with coffee mate, Bacon and Egg
41. Where does the digestion of carbohydrates start?

A. Mouth
B. Esophagus
C. Small intestine
D. Stomach
42. Protein and Fat digestion begins where?

A. Mouth
B. Esophagus
C. Small intestine
D. Stomach
43. All but one is true about digestion that occurs in the Stomach

A. Carbohydrates are the fastest to be digested, in about an hour


B. Fat is the slowest to be digested, in about 5 hours
C. HCl inhibits absorption of Calcium in the gastric mucosa
D. HCl converts pepsinogen to pepsin, which starts the complex process
of protein digestion
44. Which of the following is NOT an enzyme secreted by the small intestine?

A. Sucrase
B. Enterokinase
C. Amylase
D. Enterokinase
45. The hormone secreted by the Small intestine that stimulates the
production of pancreatic juice which primarily aids in buffering the acidic
bolus passed by the Stomach

A. Enterogastrone
B. Cholecystokinin
C. Pancreozymin
D. Enterokinase
46. When the duodenal enzyme sucrase acts on SUCROSE, which 2
monosaccharides are formed?

A. Galactose + Galactose
B. Glucose + Fructose
C. Glucose + Galactose
D. Fructose + Fructose
47. This is the enzyme secreted by the pancrease that completes the protein
digestion

A. Trypsin
B. Enterokinase
C. Enterogastrone
D. Amylase
48. The end product of protein digestion or the “Building blocks of Protein” is
what we call

A. Nucleotides
B. Fatty acids
C. Glucose
D. Amino Acids
49. Enzyme secreted by the small intestine after it detects a bolus of fatty
food. This will contract the gallbladder to secrete bile and relax the sphincter
of Oddi to aid in the emulsification of fats and its digestion.

A. Lipase
B. Amylase
C. Cholecystokinin
D. Pancreozymin
50. Which of the following is not true about the Large Intestine?

A. It absorbs around 1 L of water making the feces around 75% water and
25% solid
B. The stool formed in the transverse colon is not yet well formed
C. It is a sterile body cavity
D. It is called large intestine because it is longer than the small intestine
51. This is the amount of heat required to raise the temperature of 1 kg water
to 1 degree Celsius

A. Calorie
B. Joules
C. Metabolism
D. Basal metabolic rate
52. Assuming a cup of rice provides 50 grams of carbohydrates. How many
calories are there in that cup of rice?

A. 150 calories
B. 200 calories
C. 250 calories
D. 400 calories
53. An average adult filipino requires how many calories in a day?

A. 1,000 calories
B. 1,500 calories
C. 2,000 calories
D. 2,500 calories
54. Which of the following is true about an individual’s caloric needs?

A. All individual have the same caloric needs


B. Females in general have higher BMR and therefore, require more
calories
C. During cold weather, people need more calories due to increase BMR
D. Dinner should be the heaviest meal of the day
55. Among the following people, who requires the greatest caloric intake?

A. An individual in a long state of gluconeogenesis


B. An individual in a long state of glycogenolysis
C. A pregnant individual
D. An adolescent with a BMI of 25
56. Which nutrient deficiency is associated with the development of Pellagra,
Dermatitis and Diarrhea?

A. Vitamin B1
B. Vitamin B2
C. Vitamin B3
D. Vitamin B6
57. Which Vitamin is not given in conjunction with the intake of LEVODOPA in
cases of Parkinson’s Disease due to the fact that levodopa increases its level
in the body?
A. Vitamin B1
B. Vitamin B2
C. Vitamin B3
D. Vitamin B6
58. A vitamin taken in conjunction with ISONIAZID to prevent peripheral
neuritis

A. Vitamin B1
B. Vitamin B2
C. Vitamin B3
D. Vitamin B6
59. The inflammation of the Lips, Palate and Tongue is associated in the
deficiency of this vitamin

A. Vitamin B1
B. Vitamin B2
C. Vitamin B3
D. Vitamin B6
60. Beri beri is caused by the deficiency of which Vitamin?

A. Vitamin B1
B. Vitamin B2
C. Vitamin B3
D. Vitamin C
61. Which of the following is the best source of Vitamin E?

A. Green leafy vegetables


B. Vegetable oil
C. Fortified Milk
D. Fish liver oil
62. Among the following foods, which food should you emphasize giving on
an Alcoholic client?

A. Pork liver and organ meats, Pork


B. Red meat, Eggs and Dairy products
C. Green leafy vegetables, Yellow vegetables, Cantaloupe and Dairy
products
D. Chicken, Peanuts, Bananas, Wheat germs and yeasts
63. Which food group should you emphasize giving on a pregnant mother in
first trimester to prevent neural tube defects?

A. Broccoli, Guava, Citrus fruits, Tomatoes


B. Butter, Sardines, Tuna, Salmon, Egg yolk
C. Wheat germ, Vegetable Oil, soybeans, corn, peanuts
D. Organ meats, Green leafy vegetables, Liver, Eggs
64. A client taking Coumadin is to be educated on his diet. As a nurse, which
of the following food should you instruct the client to avoid?

A. Spinach, Green leafy vegetables, Cabbage, Liver


B. Salmon, Sardines, Tuna
C. Butter, Egg yolk, breakfast cereals
D. Banana, Yeast, Wheat germ, Chicken
65. Vitamin E plus this mineral works as one of the best anti oxidant in the
body according to the latest research. They are combined with 5 Alpha
reductase inhibitor to reduce the risk of acquiring prostate cancer

A. Zinc
B. Iron
C. Selenium
D. Vanadium
66. Incident of prostate cancer is found to have been reduced on a population
exposed in tolerable amount of sunlight. Which vitamin is associated with this
phenomenon?

A. Vitamin A
B. Vitamin B
C. Vitamin C
D. Vitamin D
67. Micronutrients are those nutrients needed by the body in a very minute
amount. Which of the following vitamin is considered as a MICRONUTRIENT

A. Phosphorous
B. Iron
C. Calcium
D. Sodium
68. Deficiency of this mineral results in tetany, osteomalacia, osteoporosis
and rickets.

A. Vitamin D
B. Iron
C. Calcium
D. Sodium
69. Among the following foods, which has the highest amount of potassium
per area of their meat?

A. Cantaloupe
B. Avocado
C. Raisin
D. Banana
70. A client has HEMOSIDEROSIS. Which of the following drug would you
expect to be given to the client?

A. Acetazolamide
B. Deferoxamine
C. Calcium EDTA
D. Activated charcoal
71. Which of the following provides the richest source of Iron per area of their
meat?

A. Pork meat
B. Lean read meat
C. Pork liver
D. Green mongo
72. Which of the following is considered the best indicator of nutritional status
of an individual?

A. Height
B. Weight
C. Arm muscle circumference
D. BMI
73. Jose Miguel, a 50 year old business man is 6’0 Tall and weights 179 lbs.
As a nurse, you know that Jose Miguel is :

A. Overweight
B. Underweight
C. Normal
D. Obese
74. Jose Miguel is a little bit nauseous. Among the following beverages,
Which could help relieve JM’s nausea?

A. Coke
B. Sprite
C. Mirinda
D. Orange Juice or Lemon Juice
75. Which of the following is the first sign of dehydration?

A. Tachycardia
B. Restlessness
C. Thirst
D. Poor skin turgor
76. What Specific gravity lab result is compatible with a dehydrated client?

A. 1.007
B. 1.020
C. 1.039
D. 1.029
77. Which hematocrit value is expected in a dehydrated male client?

A. 67%
B. 50%
C. 36%
D. 45%
78. Which of the following statement by a client with prolonged vomiting
indicates the initial onset of hypokalemia?

A. My arm feels so weak


B. I felt my heart beat just right now
C. My face muscle is twitching
D. Nurse, help! My legs are cramping
79. Which of the following is not an anti-emetic?

A. Marinol
B. Dramamine
C. Benadryl
D. Alevaire
80. Which is not a clear liquid diet?

A. Hard candy
B. Gelatin
C. Coffee with Coffee mate
D. Bouillon
81. Which of the following is included in a full liquid diet?

A. Popsicles
B. Pureed vegetable meat
C. Pineapple juice with pulps
D. Mashed potato
82. Which food is included in a BLAND DIET?

A. Steamed broccoli
B. Creamed potato
C. Spinach in garlic
D. Sweet potato
83. Which of the following if done by the nurse, is correct during NGT
Insertion?

A. Use an oil based lubricant


B. Measure the amount of the tube to be inserted from the Tip of the nose,
to the earlobe, to the xiphoid process
C. Soak the NGT in a basin of ice water to facilitate easy insertion
D. Check the placement of the tube by introducing 10 cc of sterile water
and auscultating for bubbling sound
84. Which of the following is the BEST method in assessing for the correct
placement of the NGT?
A. X-Ray
B. Immerse tip of the tube in water to check for bubbles produced
C. Aspirating gastric content to check if the content is acidic
D. Instilling air in the NGT and listening for a gurgling sound at the
epigastric area
85. A terminally ill cancer patient is scheduled for an NGT feeding today. How
should you position the patient?

A. Semi fowlers in bed


B. Bring the client into a chair
C. Slightly elevated right side lying position
D. Supine in bed
86. A client is scheduled for NGT Feeding. Checking the residual volume, you
determined that he has 40 cc residual from the last feeding. You reinstill the
40 cc of residual volume and added the 250 cc of feeding ordered by the
doctor. You then instill 60 cc of water to clear the lumen and the tube. How
much will you put in the client’s chart as input?

A. 250 cc
B. 290 cc
C. 350 cc
D. 310 cc
87. Which of the following if done by a nurse indicates deviation from the
standards of NGT feeding?

A. Do not give the feeding and notify the doctor of residual of the last
feeding is greater than or equal to 50 ml
B. Height of the feeding should be 12 inches about the tube point of
insertion to allow slow introduction of feeding
C. Ask the client to position in supine position immediately after feeding to
prevent dumping syndrome
D. Clamp the NGT before all of the water is instilled to prevent air entry in
the stomach
88. What is the most common problem in TUBE FEEDING?

A. Diarrhea
B. Infection
C. Hyperglycemia
D. Vomiting
89. Which of the following is TRUE in colostomy feeding?

A. Hold the syringe 18 inches above the stoma and administer the feeding
slowly
B. Pour 30 ml of water before and after feeding administration
C. Insert the ostomy feeding tube 1 inch towards the stoma
D. A Pink stoma means that circulation towards the stoma is all well
90. A client with TPN suddenly develops tremors, dizziness, weakness and
diaphoresis. The client said “I feel weak” You saw that his TPN is already
empty and another TPN is scheduled to replace the previous one but its
provision is already 3 hours late. Which of the following is the probable
complication being experienced by the client?

A. Hyperglycemia
B. Hypoglycemia
C. Infection
D. Fluid overload
91. To assess the adequacy of food intake, which of the following
assessment parameters is best used?

A. Food likes and dislikes


B. Regularity of meal times
C. 3 day diet recall
D. Eating style and habits
92. The vomiting center is found in the

A. Medulla Oblongata
B. Pons
C. Hypothalamus
D. Cerebellum
93. The most threatening complication of vomiting in client’s with stroke is

A. Aspiration
B. Dehydration
C. Fluid and electrolyte imbalance
D. Malnutrition
94. Which among this food is the richest source of Iron?

A. Ampalaya
B. Broccoli
C. Mongo
D. Malunggay leaves
95. Which of the following is a good source of Vitamin A?

A. Egg yolk
B. Liver
C. Fish
D. Peanuts
96. The most important nursing action before gastrostomy feeding is

A. Check V/S
B. Assess for patency of the tube
C. Measure residual feeding
D. Check the placement of the tube
97. The primary advantage of gastrostomy feeding is

A. Ensures adequate nutrition


B. It prevents aspiration
C. Maintains Gastro esophageal sphincter integrity
D. Minimizes fluid-electrolyte imbalance
98. What is the BMI Of Budek, weighing 120 lbs and has a height of 5 feet 7
inches.

A. 20
B. 19
C. 15
D. 25
99. Which finding is consistent with PERNICIOUS ANEMIA?

A. Strawberry tongue
B. Currant Jelly stool
C. Beefy red tongue
D. Pale [ HYPOCHROMIC ] RBC
100. The nurse is browsing the chart of the patient and notes a normal serum
lipase level. Which of the following is a normal serum lipase value?

A. 10 U/L
B. 100 U/L
C. 200 U/L
D. 350 U/L
Answers
1. A. For clearance mechanism such as coughing 
2. D. Vibrissae
3. A. Ehtmoid
4. D. Frontal
5. A. Right main stem bronchus
6. C. Goblet cells
7. C. Three
8. C. Right kidney lower, Right lung shorter
9. B. Type II pneumocytes
10. B. 2:1
11. A. Inspiratory reserve volume
12. D. Residual volume
13. B. We have 12 pairs of ribs Cassandra
14. C. Diaphragm
15. D. Human requires 21% of oxygen and we have 21% available in our air
16. A. A passive process
17. A. Patient assumes position for 10 to 15 minutes
18. D. Promote bronchoconstriction
19. C. Render steam inhalation for atleast 60 minutes
20. B. As needed
21. D. Side lying
22. C. 100-120 mmHg
23. C. 10-15 mmHg
24. A. Fr. 18
25. A. Measure the length of the suction catheter to be inserted by measuring from
the tip of the nose, to the earlobe, to the xiphoid process
26. B. 2,5
27. A. Nasal Cannula
28. C. Non Rebreather mask
29. D. Excessive oxygen administration results in respiratory acidosis
30. B. Put a non rebreather mask in the patient before opening the oxygen source
31. D. Client is frequently turning from side to side
32. C. Fingers are Club-like
33. A. Cannula
34. C. Closed tube thoracotomy
35. D. 8th ICS
36. A. Consider this as normal findings
37. D. It is the study of nutrients and the process in which they are use by the body
38. B. Small intestine
39. C. The action of ptyalin or the salivary tyrpsin breaks down starches into
maltose
40. D. Coffee with coffee mate, Bacon and Egg
41. A. Mouth
42. D. Stomach
43. C. HCl inhibits absorption of Calcium in the gastric mucosa
44. C. Amylase
45. C. Pancreozymin
46. B. Glucose + Fructose
47. A. Trypsin
48. D. Amino Acids
49. C. Cholecystokinin
50. C. It is a sterile body cavity
51. A. Calorie
52. B. 200 calories
53. C. 2,000 calories
54. C. During cold weather, people need more calories due to increase BMR
55. A. An individual in a long state of gluconeogenesis
56. C. Vitamin B3
57. D. Vitamin B6
58. D. Vitamin B6
59. B. Vitamin B2
60. A. Vitamin B1
61. B. Vegetable oil
62. A. Pork liver and organ meats, Pork
63. D. Organ meats, Green leafy vegetables, Liver, Eggs
64. A. Spinach, Green leafy vegetables, Cabbage, Liver
65. C. Selenium
66. D. Vitamin D
67. B. Iron
68. C. Calcium
69. A. Cantaloupe
70. B. Deferoxamine
71. C. Pork liver
72. B. Weight
73. C. Normal
74. A. Coke
75. C. Thirst
76. C. 1.039
77. A. 67%
78. D. Nurse, help! My legs are cramping
79. D. Alevaire
80. C. Coffee with Coffee mate
81. A. Popsicles
82. B. Creamed potato
83. B. Measure the amount of the tube to be inserted from the Tip of the nose, to
the earlobe, to the xiphoid process
84. A. X-Ray
85. B. Bring the client into a chair
86. D. 310 cc
87. C. Ask the client to position in supine position immediately after feeding to
prevent dumping syndrome
88. A. Diarrhea
89. B. Pour 30 ml of water before and after feeding administration
90. B. Hypoglycemia
91. C. 3 day diet recall
92. A. Medulla Oblongata
93. A. Aspiration
94. C. Mongo
95. B. Liver
96. B. Assess for patency of the tube
97. C. Maintains Gastro esophageal sphincter integrity
98. B. 19
99. C. Beefy red tongue
100. C. 200 U/L

PNLE: Fundamentals in Nursing


Exam 1
1. Jake is complaining of shortness of breath. The nurse assesses his
respiratory rate to be 30 breaths per minute and documents that Jake is
tachypneic. The nurse understands that tachypnea means:

A. Pulse rate greater than 100 beats per minute


B. Blood pressure of 140/90
C. Respiratory rate greater than 20 breaths per minute
D. Frequent bowel sounds
2. The nurse listens to Mrs. Sullen’s lungs and notes a hissing sound or
musical sound. The nurse documents this as:

A. Wheezes
B. Rhonchi
C. Gurgles
D. Vesicular
3. The nurse in charge measures a patient’s temperature at 101 degrees F.
What is the equivalent centigrade temperature?

A. 36.3 degrees C
B. 37.95 degrees C
C. 40.03 degrees C
D. 38.01 degrees C
4. Which approach to problem solving tests any number of solutions until one
is found that works for that particular problem?

A. Intuition
B. Routine
C. Scientific method
D. Trial and error
5. What is the order of the nursing process?

A. Assessing, diagnosing, implementing, evaluating, planning


B. Diagnosing, assessing, planning, implementing, evaluating
C. Assessing, diagnosing, planning, implementing, evaluating
D. Planning, evaluating, diagnosing, assessing, implementing
6. During the planning phase of the nursing process, which of the following is
the outcome?
A. Nursing history
B. Nursing notes
C. Nursing care plan
D. Nursing diagnosis
7. What is an example of a subjective data?

A. Heart rate of 68 beats per minute


B. Yellowish sputum
C. Client verbalized, “I feel pain when urinating.”
D. Noisy breathing
8. Which expected outcome is correctly written?

A. “The patient will feel less nauseated in 24 hours.”


B. “The patient will eat the right amount of food daily.”
C. “The patient will identify all the high-salt food from a prepared list by
discharge.”
D. “The patient will have enough sleep.”
9. Which of the following behaviors by Nurse Jane Robles demonstrates that
she understands well th elements of effecting charting?

A. She writes in the chart using a no. 2 pencil.


B. She noted: appetite is good this afternoon.
C. She signs on the medication sheet after administering the medication.
D. She signs her charting as follow: J.R
10. What is the disadvantage of computerized documentation of the nursing
process?

A. Accuracy
B. Legibility
C. Concern for privacy
D. Rapid communication
11. The theorist who believes that adaptation and manipulation of stressors
are related to foster change is:

A. Dorothea Orem
B. Sister Callista Roy
C. Imogene King
D. Virginia Henderson
12. Formulating a nursing diagnosis is a joint function of:

A. Patient and relatives


B. Nurse and patient
C. Doctor and family
D. Nurse and doctor
13. Mrs. Caperlac has been diagnosed to have hypertension since 10 years
ago. Since then, she had maintained low sodium, low fat diet, to control her
blood pressure. This practice is viewed as:

A. Cultural belief
B. Personal belief
C. Health belief
D. Superstitious belief
14. Becky is on NPO since midnight as preparation for blood test. Adreno-
cortical response is activated. Which of the following is an expected
response?

A. Low blood pressure


B. Warm, dry skin
C. Decreased serum sodium levels
D. Decreased urine output
15. What nursing action is appropriate when obtaining a sterile urine
specimen from an indwelling catheter to prevent infection?

A. Use sterile gloves when obtaining urine.


B. Open the drainage bag and pour out the urine.
C. Disconnect the catheter from the tubing and get urine.
D. Aspirate urine from the tubing port using a sterile syringe.
16. A client is receiving 115 ml/hr of continuous IVF. The nurse notices that
the venipuncture site is red and swollen. Which of the following interventions
would the nurse perform first?

A. Stop the infusion


B. Call the attending physician
C. Slow that infusion to 20 ml/hr
D. Place a clod towel on the site
17. The nurse enters the room to give a prescribed medication but the patient
is inside the bathroom. What should the nurse do?

A. Leave the medication at the bedside and leave the room.


B. After few minutes, return to that patient’s room and do not leave until
the patient takes the medication.
C. Instruct the patient to take the medication and leave it at the bedside.
D. Wait for the patient to return to bed and just leave the medication at the
bedside.
18. Which of the following is inappropriate nursing action when administering
NGT feeding?

A. Place the feeding 20 inches above the pint if insertion of NGT.


B. Introduce the feeding slowly.
C. Instill 60ml of water into the NGT after feeding.
D. Assist the patient in fowler’s position.
19. A female patient is being discharged after thyroidectomy. After providing
the medication teaching. The nurse asks the patient to repeat the instructions.
The nurse is performing which professional role?

A. Manager
B. Caregiver
C. Patient advocate
D. Educator
20. Which data would be of greatest concern to the nurse when completing
the nursing assessment of a 68-year-old woman hospitalized due to
Pneumonia?

A. Oriented to date, time and place


B. Clear breath sounds
C. Capillary refill greater than 3 seconds and buccal cyanosis
D. Hemoglobin of 13 g/dl
21. During a change-of-shift report, it would be important for the nurse
relinquishing responsibility for care of the patient to communicate. Which of
the following facts to the nurse assuming responsibility for care of the
patient?
A. That the patient verbalized, “My headache is gone.”
B. That the patient’s barium enema performed 3 days ago was negative
C. Patient’s NGT was removed 2 hours ago
D. Patient’s family came for a visit this morning.
22. Which statement is the most appropriate goal for a nursing diagnosis of
diarrhea?

A. “The patient will experience decreased frequency of bowel elimination.”


B. “The patient will take anti-diarrheal medication.”
C. “The patient will give a stool specimen for laboratory examinations.”
D. “The patient will save urine for inspection by the nurse.
23. Which of the following is the most important purpose of planning care
with this patient?

A. Development of a standardized NCP.


B. Expansion of the current taxonomy of nursing diagnosis
C. Making of individualized patient care
D. Incorporation of both nursing and medical diagnoses in patient care
24. Using Maslow’s hierarchy of basic human needs, which of the following
nursing diagnoses has the highest priority?

A. Ineffective breathing pattern related to pain, as evidenced by shortness


of breath.
B. Anxiety related to impending surgery, as evidenced by insomnia.
C. Risk of injury related to autoimmune dysfunction
D. Impaired verbal communication related to tracheostomy, as evidenced
by inability to speak.
25. When performing an abdominal examination, the patient should be in a
supine position with the head of the bed at what position?

A. 30 degrees
B. 90 degrees
C. 45 degrees
D. 0 degree
Answers and Rationales
1. 1. (C) Respiratory rate greater than 20 breaths per minute. A respiratory rate
of greater than 20 breaths per minute is tachypnea. A blood pressure of
140/90 is considered hypertension. Pulse greater than 100 beats per
minute is tachycardia. Frequent bowel sounds refer to hyper-active bowel
sounds.
2. (A) Wheezes. Wheezes are indicated by continuous, lengthy, musical;
heard during inspiration or expiration. Rhonchi are usually coarse breath
sounds. Gurgles are loud gurgling, bubbling sound. Vesicular breath
sounds are low pitch, soft intensity on expiration. 

3. (B) 37.95 degrees C. To convert °F to °C use this formula, ( °F – 32 )


(0.55). While when converting °C to °F use this formula, ( °C x 1.8) + 32.
Note that 0.55 is 5/9 and 1.8 is 9/5.

4. (D) Trial and error. The trial and error method of problem solving isn’t
systematic (as in the scientific method of problem solving) routine, or
based on inner prompting (as in the intuitive method of problem solving).
5. (C) Assessing, diagnosing, planning, implementing, evaluating. The correct
order of the nursing process is assessing, diagnosing, planning,
implementing, evaluating.
6. (C) Nursing care plan. The outcome, or the product of the planning phase
of the nursing process is a Nursing care plan.
7. (C) Client verbalized, “I feel pain when urinating.”. Subjective data are
those that can be described only by the person experiencing it. Therefore,
only the patient can describe or verify whether he is experiencing pain or
not.
8. (C) “The patient will identify all the high-salt food from a prepared list by
discharge.”. Expected outcomes are specific, measurable, realistic
statements of goal attainment. The phrases “right amount”, “less
nauseated” and “enough sleep” are vague and not measurable.
9. (C) She signs on the medication sheet after administering the medication.A
nurse should record a nursing intervention (ex. Giving medications) after
performing the nursing intervention (not before). Recording should also be
done using a pen, be complete, and signed with the nurse’s full name and
title.
10. (C) Concern for privacy. A patient’s privacy may be violated if security
measures aren’t used properly or if policies and procedures aren’t in place
that determines what type of information can be retrieved, by whom, and
for what purpose.
11. (B) Sister Callista Roy. Sister Roy’s theory is called the adaptation theory
and she viewed each person as a unified biophysical system in constant
interaction with a changing environment. Orem’s theory is called self-care
deficit theory and is based on the belief that individual has a need for self-
care actions. King’s theory is the Goal attainment theory and described
nursing as a helping profession that assists individuals and groups in
society to attain, maintain, and restore health. Henderson introduced the
nature of nursing model and identified the 14 basic needs.
12. (B) Nurse and patient. Although diagnosing is basically the nurse’s
responsibility, input from the patient is essential to formulate the correct
nursing diagnosis.
13. (C) Health belief. Health belief of an individual influences his/her
preventive health behavior.
14. (D) Decreased urine output. Adreno-cortical response involves release of
aldosterone that leads to retention of sodium and water. This results to
decreased urine output.
15. (D) Aspirate urine from the tubing port using a sterile syringe. The nurse
should aspirate the urine from the port using a sterile syringe to obtain a
urine specimen. Opening a closed drainage system increase the risk of
urinary tract infection.
16. (A) Stop the infusion. The sign and symptoms indicate extravasation so
the IVF should be stopped immediately and put warm not cold towel on
the affected site.
17. (B) After few minutes, return to that patient’s room and do not leave until the
patient takes the medication. This is to verify or to make sure that the
medication was taken by the patient as directed.
18. (A) Place the feeding 20 inches above the pint if insertion of NGT. The height
of the feeding is above 12 inches above the point of insertion, bot 20
inches. If the height of feeding is too high, this results to very rapid
introduction of feeding. This may trigger nausea and vomiting.
19. (D) Educator. When teaching a patient about medications before
discharge, the nurse is acting as an educator. A caregiver provides direct
care to the patient. The nurse acts as s patient advocate when making the
patient’s wishes known to the doctor.
20. (C) Capillary refill greater than 3 seconds and buccal cyanosis. Capillary refill
greater than 3 seconds and buccal cyanosis indicate decreased oxygen to
the tissues which requires immediate attention/intervention. Oriented to
date, time and place, hemoglobin of 13 g/dl are normal data.
21. (C) Patient’s NGT was removed 2 hours ago. The change-of-shift report
should indicate significant recent changes in the patient’s condition that
the nurse assuming responsibility for care of the patient will need to
monitor. The other options are not critical enough to include in the report.
22. (A) “The patient will experience decreased frequency of bowel
elimination.” The goal is the opposite, healthy response of the problem
statement of the nursing diagnosis. In this situation, the problem
statement is diarrhea.
23. (C) Making of individualized patient care. To be effective, the nursing care
plan developed in the planning phase of the nursing process must reflect
the individualized needs of the patient.
24. (A) Ineffective breathing pattern related to pain, as evidenced by shortness of
breath.. Physiologic needs (ex. Oxygen, fluids, nutrition) must be met
before lower needs (such as safety and security, love and belongingness,
self-esteem and self-actualization) can be met. Therefore, physiologic
needs have the highest priority.
25. (D) 0 degree. The patient should be positioned with the head of the bed
completely flattened to perform an abdominal examination. If the head of
the bed is elevated, the abdominal muscles and organs can be bunched
up, altering the findings
PNLE: Fundamentals in Nursing
Exam 2
1. A patient is wearing a soft wrist-safety device. Which of the following
nursing assessment is considered abnormal?

A. Palpable radial pulse


B. Palpable ulnar pulse
C. Capillary refill within 3 seconds
D. Bluish fingernails, cool and pale fingers
2. Pia’s serum sodium level is 150 mEq/L. Which of the following food items
does the nurse instruct Pia to avoid?

A. broccoli
B. sardines
C. cabbage
D. tomatoes
3. Jason, 3 years old vomited. His mom stated, “He vomited 6 ounces of his
formula this morning.” This statement is an example of:

A. objective data from a secondary source


B. objective data from a primary source
C. subjective data from a primary source
D. subjective data from a secondary source
4. Which of the following is a nursing diagnosis?

A. Hypethermia
B. Diabetes Mellitus
C. Angina
D. Chronic Renal Failure
5. What is the characteristic of the nursing process?
A. stagnant
B. inflexible
C. asystematic
D. goal-oriented
6. A skin lesion which is fluid-filled, less than 1 cm in size is called:

A. papule
B. vesicle
C. bulla
D. macule
7. During application of medication into the ear, which of the following is
inappropriate nursing action?

A. In an adult, pull the pinna upward.


B. Instill the medication directly into the tympanic membrane.
C. Warm the medication at room or body temperature.
D. Press the tragus of the ear a few times to assist flow of medication into
the ear canal.
8. Which of the following is appropriate nursing intervention for a client who is
grieving over the death of her child?

A. Tell her not to cry and it will be better.


B. Provide opportunity to the client to tell their story.
C. Encourage her to accept or to replace the lost person.
D. Discourage the client in expressing her emotions.
9. It is the gradual decrease of the body’s temperature after death.

A. livor mortis
B. rigor mortis
C. algor mortis
D. none of the above
10. When performing an admission assessment on a newly admitted patient,
the nurse percusses resonance. The nurse knows that resonance heard on
percussion is most commonly heard over which organ?

A. thigh
B. liver
C. intestine
D. lung
11. The nurse is aware that Bell’s palsy affects which cranial nerve?

A. 2nd CN (Optic)
B. 3rd CN (Occulomotor)
C. 4th CN (Trochlear)
D. 7th CN (Facial)
12. Prolonged deficiency of Vitamin B9 leads to:

A. scurvy
B. pellagra
C. megaloblastic anemia
D. pernicious anemia
13. Nurse Cherry is teaching a 72 year old patient about a newly prescribed
medication. What could cause a geriatric patient to have difficulty retaining
knowledge about the newly prescribed medication?

A. Absence of family support


B. Decreased sensory functions
C. Patient has no interest on learning
D. Decreased plasma drug levels
14. When assessing a patient’s level of consciousness, which type of nursing
intervention is the nurse performing?

A. Independent
B. Dependent
C. Collaborative
D. Professional
15. Claire is admitted with a diagnosis of chronic shoulder pain. By definition,
the nurse understands that the patient has had pain for more than:

A. 3 months
B. 6 months
C. 9 months
D. 1 year
16. Which of the following statements regarding the nursing process is true?
A. It is useful on outpatient settings.
B. It progresses in separate, unrelated steps.
C. It focuses on the patient, not the nurse.
D. It provides the solution to all patient health problems.
17. Which of the following is considered significant enough to require
immediate communication to another member of the health care team?

A. Weight loss of 3 lbs in a 120 lb female patient.


B. Diminished breath sounds in patient with previously normal breath
sounds
C. Patient stated, “I feel less nauseated.”
D. Change of heart rate from 70 to 83 beats per minute.
18. To assess the adequacy of food intake, which of the following
assessment parameters is best used?

A. food preferences
B. regularity of meal times
C. 3-day diet recall
D. eating style and habits
19. Van Fajardo is a 55 year old who was admitted to the hospital with newly
diagnosed hepatitis. The nurse is doing a patient teaching with Mr. Fajardo.
What kind of role does the nurse assume?

A. talker
B. teacher
C. thinker
D. doer
20. When providing a continuous enteral feeding, which of the following action
is essential for the nurse to do?

A. Place the client on the left side of the bed.


B. Attach the feeding bag to the current tubing.
C. Elevate the head of the bed.
D. Cold the formula before administering it.
21. Kussmaul’s breathing is;

A. Shallow breaths interrupted by apnea.


B. Prolonged gasping inspiration followed by a very short, usually
inefficient expiration.
C. Marked rhythmic waxing and waning of respirations from very deep to
very shallow breathing and temporary apnea.
D. Increased rate and depth of respiration.
22. Presty has terminal cancer and she refuses to believe that loss is
happening ans she assumes artificial cheerfulness. What stage of grieving is
she in?

A. depression
B. bargaining
C. denial
D. acceptance
23. Immunization for healthy babies and preschool children is an example of
what level of preventive health care?

A. Primary
B. Secondary
C. Tertiary
D. Curative
24. Which is an example of a subjective data?

A. Temperature of 38 0C
B. Vomiting for 3 days
C. Productive cough
D. Patient stated, “My arms still hurt.”
25. The nurse is assessing the endocrine system. Which organ is part of the
endocrine system?

A. Heart
B. Sinus
C. Thyroid
D. Thymus
Answers and Rationales
1. (D) Bluish fingernails, cool and pale fingers. A safety device on the wrist
may impair blood circulation. Therefore, the nurse should assess the
patient for signs of impaired circulation such as bluish fingernails, cool
and pale fingers. Palpable radial and ulnar pulses, capillary refill within 3
seconds are all normal findings.
2. (B) sardines. The normal serum sodium level is 135 to 145 mEq/L, the
client is having hypernatremia. Pia should avoid food high in sodium like
processed food. Broccoli, cabbage and tomatoes are good source of
Vitamin C.
3. (A) objective data from a secondary source. Jason is the primary source; his
mother is a secondary source. The data is objective because it can be
perceived by the senses, verified by another person observing the same
patient, and tested against accepted standards or norms.
4. (A) Hypethermia. Hyperthermia is a NANDA-approved nursing diagnosis.
Diabetes Mellitus, Angina and Chronic Renal Failure are medical
diagnoses.
5. (D) goal-oriented. The nursing process is goal-oriented. It is also
systematic, patient-centered, and dynamic.
6. (B) vesicle. Vesicle is a circumscribed circulation containing serous fluid
or blood and less than 1 cm (ex. Blister, chicken pox).
7. (B) Instill the medication directly into the tympanic membrane. During the
application of medication it is inappropriate to instill the medication
directly into the tympanic membrane. The right thing to do is instill the
medication along the lateral wall of the auditory canal.
8. (B) Provide opportunity to the client to tell their story. Providing a grieving
person an opportunity to tell their story allows the person to express
feelings. This is therapeutic in assisting the client resolve grief.
9. (C) algor mortis. Algor mortis is the decrease of the body’s temperature
after death. Livor mortis is the discoloration of the skin after death. Rigor
mortis is the stiffening of the body that occurs about 2-4 hours after
death.
10. (D) lung. Resonance is loud, low-pitched and long duration that’s heard
most commonly over an air-filled tissue such as a normal lung.
11. (D) 7th CN (Facial). Bells’ palsy is the paralysis of the motor component
of the 7th caranial nerve, resulting in facial sag, inability to close the eyelid
or the mouth, drooling, flat nasolabial fold and loss of taste on the
affected side of the face.
12. (C) megaloblastic anemia. Prolonged Vitamin B9 deficiency will lead to
megaloblastic anemia while pernicious anemia results in deficiency in
Vitamin B12. Prolonged deficiency of Vitamin C leads to scurvy and
Pellagra results in deficiency in Vitamin B3.
13. (B) Decreased sensory functions. Decreased in sensory functions could
cause a geriatric patient to have difficulty retaining knowledge about the
newly prescribed medications. Absence of family support and no interest
on learning may affect compliance, not knowledge retention. Decreased
plasma levels do not alter patient’s knowledge about the drug.
14. (A) Independent. Independent nursing interventions involve actions that
nurses initiate based on their own knowledge and skills without the
direction or supervision of another member of the health care team.
15. (B) 6 months. Chronic pain s usually defined as pain lasting longer than 6
months.
16. (C) It focuses on the patient, not the nurse. The nursing process is patient-
centered, not nurse-centered. It can be use in any setting, and the steps
are related. The nursing process can’t solve all patient health problems.
17. (B) Diminished breath sounds in patient with previously normal breath
sounds. Diminished breath sound is a life threatening problem therefore it
is highly priority because they pose the greatest threat to the patient’s
well-being.
18. (C) 3-day diet recall. 3-day diet recall is an example of dietary history.
This is used to indicate the adequacy of food intake of the client.
19. (B) teacher. The nurse will assume the role of a teacher in this
therapeutic relationship. The other roles are inappropriate in this situation.
20. (C) Elevate the head of the bed. Elevating the head of the bed during an
enteral feeding prevents aspiration. The patient may be placed on the right
side to prevent aspiration. Enteral feedings are given at room temperature
to lessen GI distress. The enteral tubing should be changed every 24 hours
to limit microbial growth.
21. (D) Increased rate and depth of respiration. Kussmaul breathing is also
called as hyperventilation. Seen in metabolic acidosis and renal failure.
Option A refers to Biot’s breathing. Option B is apneustic breathing and
option C is the Cheyne-stokes breathing.
22. (C) denial. The client is in denial stage because she is unready to face
the reality that loss is happening and she assumes artificial cheerfulness.
23. (A) Primary. The primary level focuses on health promotion. Secondary
level focuses on health maintenance. Tertiary focuses on rehabilitation.
There is n Curative level of preventive health care problems.
24. (D) Patient stated, “My arms still hurt.”. Subjective data are apparent only
to the person affected and can or verified only by that person.
25. (C) Thyroid. The thyroid is part of the endocrine system. Heart, sinus and
thymus are not.
PNLE: Fundamentals in Nursing
Exam 3
1. Nurse Brenda is teaching a patient about a newly prescribed drug. What
could cause a geriatric patient to have difficulty retaining knowledge about
prescribed medications?

A. Decreased plasma drug levels


B. Sensory deficits
C. Lack of family support
D. History of Tourette syndrome
2. When examining a patient with abdominal pain the nurse in charge should
assess:

A. Any quadrant first


B. The symptomatic quadrant first
C. The symptomatic quadrant last
D. The symptomatic quadrant either second or third
3. The nurse is assessing a postoperative adult patient. Which of the following
should the nurse document as subjective data?

A. Vital signs
B. Laboratory test result
C. Patient’s description of pain
D. Electrocardiographic (ECG) waveforms
4. A male patient has a soft wrist-safety device. Which assessment finding
should the nurse consider abnormal?

A. A palpable radial pulse


B. A palpable ulnar pulse
C. Cool, pale fingers
D. Pink nail beds
5. Which of the following planes divides the body longitudinally into anterior
and posterior regions?

A. Frontal plane
B. Sagittal plane
C. Midsagittal plane
D. Transverse plane
6. A female patient with a terminal illness is in denial. Indicators of denial
include:

A. Shock dismay
B. Numbness
C. Stoicism
D. Preparatory grief
7. The nurse in charge is transferring a patient from the bed to a chair. Which
action does the nurse take during this patient transfer?

A. Position the head of the bed flat


B. Helps the patient dangle the legs
C. Stands behind the patient
D. Places the chair facing away from the bed
8. A female patient who speaks a little English has emergency gallbladder
surgery, during discharge preparation, which nursing action would best help
this patient understand wound care instruction?

A. Asking frequently if the patient understands the instruction


B. Asking an interpreter to replay the instructions to the patient.
C. Writing out the instructions and having a family member read them to
the patient
D. Demonstrating the procedure and having the patient return the
demonstration
9. Before administering the evening dose of a prescribed medication, the
nurse on the evening shift finds an unlabeled, filled syringe in the patient’s
medication drawer. What should the nurse in charge do?

A. Discard the syringe to avoid a medication error


B. Obtain a label for the syringe from the pharmacy
C. Use the syringe because it looks like it contains the same medication
the nurse was prepared to give
D. Call the day nurse to verify the contents of the syringe
10. When administering drug therapy to a male geriatric patient, the nurse
must stay especially alert for adverse effects. Which factor makes geriatric
patients to adverse drug effects?

A. Faster drug clearance


B. Aging-related physiological changes
C. Increased amount of neurons
D. Enhanced blood flow to the GI tract
11. A female patient is being discharged after cataract surgery. After
providing medication teaching, the nurse asks the patient to repeat the
instructions. The nurse is performing which professional role?

A. Manager
B. Educator
C. Caregiver
D. Patient advocate
12. A female patient exhibits signs of heightened anxiety. Which response by
the nurse is most likely to reduce the patient’s anxiety?

A. “Everything will be fine. Don’t worry.”


B. “Read this manual and then ask me any questions you may have.”
C. “Why don’t you listen to the radio?”
D. “Let’s talk about what’s bothering you.”
13. A scrub nurse in the operating room has which responsibility?

A. Positioning the patient


B. Assisting with gowning and gloving
C. Handling surgical instruments to the surgeon
D. Applying surgical drapes
14. A patient is in the bathroom when the nurse enters to give a prescribed
medication. What should the nurse in charge do?

A. Leave the medication at the patient’s bedside


B. Tell the patient to be sure to take the medication. And then leave it at
the bedside
C. Return shortly to the patient’s room and remain there until the patient
takes the medication
D. Wait for the patient to return to bed, and then leave the medication at
the bedside
15. The physician orders heparin, 7,500 units, to be administered
subcutaneously every 6 hours. The vial reads 10,000 units per millilitre. The
nurse should anticipate giving how much heparin for each dose?

A. ¼ ml
B. ½ ml
C. ¾ ml
D. 1 ¼ ml
16. The nurse in charge measures a patient’s temperature at 102 degrees F.
what is the equivalent Centigrade temperature?

A. 39 degrees C
B. 47 degrees C
C. 38.9 degrees C
D. 40.1 degrees C
17. To evaluate a patient for hypoxia, the physician is most likely to order
which laboratory test?

A. Red blood cell count


B. Sputum culture
C. Total hemoglobin
D. Arterial blood gas (ABG) analysis
18. The nurse uses a stethoscope to auscultate a male patient’s chest. Which
statement about a stethoscope with a bell and diaphragm is true?

A. The bell detects high-pitched sounds best


B. The diaphragm detects high-pitched sounds best
C. The bell detects thrills best
D. The diaphragm detects low-pitched sounds best
19. A male patient is to be discharged with a prescription for an analgesic that
is a controlled substance. During discharge teaching, the nurse should explain
that the patient must fill this prescription how soon after the date on which it
was written?

A. Within 1 month
B. Within 3 months
C. Within 6 months
D. Within 12 months
20. Which human element considered by the nurse in charge during
assessment can affect drug administration?

A. The patient’s ability to recover


B. The patient’s occupational hazards
C. The patient’s socioeconomic status
D. The patient’s cognitive abilities
21. When explaining the initiation of I.V. therapy to a 2-year-old child, the nurse
should:

A. Ask the child, “Do you want me to start the I.V. now?”
B. Give simple directions shortly before the I.V. therapy is to start
C. Tell the child, “This treatment is for your own good”
D. Inform the child that the needle will be in place for 10 days
22. All of the following parts of the syringe are sterile except the:

A. Barrel
B. Inside of the plunger
C. Needle tip
D. Barrel tip
23. The best way to instill eye drops is to:

A. Instruct the patient to lock upward, and drop the medication into the
center of the lower lid
B. Instruct the patient to look ahead, and drop the medication into the
center of the lower lid
C. Drop the medication into the inner canthus regardless of eye position
D. Drop the medication into the center of the canthus regardless of eye
position
24. The difference between an 18G needle and a 25G needle is the needle’s:
A. Length
B. Bevel angle
C. Thickness
D. Sharpness
25. A patient receiving an anticoagulant should be assessed for signs of:

A. Hypotension
B. Hypertension
C. An elevated hemoglobin count
D. An increased number of erythrocytes
Answers and Rationales
1. (B) Sensory deficits. Sensory deficits could cause a geriatric patient to
have difficulty retaining knowledge about prescribed medications.
Decreased plasma drug levels do not alter the patient’s knowledge about
the drug. A lack of family support may affect compliance, not knowledge
retention. Toilette syndrome is unrelated to knowledge retention.
2. (C) The symptomatic quadrant last. The nurse should systematically
assess all areas of the abdomen, if time and the patient’s condition permit,
concluding with the symptomatic area. Otherwise, the nurse may elicit
pain in the symptomatic area, causing the muscles in other areas to
tighten. This would interfere with further assessment.
3. (C) Patient’s description of pain. Subjective data come directly from the
patient and usually are recorded as direct quotations that reflect the
patient’s opinions or feelings about a situation. Vital signs, laboratory test
result, and ECG waveforms are examples of objective data.
4. (C) Cool, pale fingers. A safety device on the wrist may impair circulation
and restrict blood supply to body tissues. Therefore, the nurse should
assess the patient for signs of impaired circulation, such as cool, pale
fingers. A palpable radial or lunar pulse and pink nail beds are normal
findings.
5. (A) Frontal plane. Frontal or coronal plane runs longitudinally at a right
angle to a sagittal plane dividing the body in anterior and posterior
regions. A sagittal plane runs longitudinally dividing the body into right and
left regions; if exactly midline, it is called a midsagittal plane. A transverse
plane runs horizontally at a right angle to the vertical axis, dividing the
structure into superior and inferior regions.
6. (A) Shock dismay. Shock and dismay are early signs of denial-the first
stage of grief. The other options are associated with depression—a later
stage of grief.
7. (B) Helps the patient dangle the legs. After placing the patient in high
Fowler’s position and moving the patient to the side of the bed, the nurse
helps the patient sit on the edge of the bed and dangle the legs; the nurse
then faces the patient and places the chair next to and facing the head of
the bed.
8. (D) Demonstrating the procedure and having the patient return the
demonstration. Demonstrating by the nurse with a return demonstration by
the patient ensures that the patient can perform wound care correctly.
Patients may claim to understand discharge instruction when they do not.
An interpreter of family member may communicate verbal or written
instructions inaccurately.
9. (A) Discard the syringe to avoid a medication error. As a safety precaution,
the nurse should discard an unlabeled syringe that contains medication.
The other options are considered unsafe because they promote error.
10. (B) Aging-related physiological changes. Aging-related physiological
changes account for the increased frequency of adverse drug reactions in
geriatric patients. Renal and hepatic changes cause drugs to clear more
slowly in these patients. With increasing age, neurons are lost and blood
flow to the GI tract decreases.
11. (B) Educator. When teaching a patient about medications before
discharge, the nurse is acting as an educator. The nurse acts as a
manager when performing such activities as scheduling and making
patient care assignments. The nurse performs the care giving role when
providing direct care, including bathing patients and administering
medications and prescribed treatments. The nurse acts as a patient
advocate when making the patient’s wishes known to the doctor.
12. (D) “Let’s talk about what’s bothering you.” Anxiety may result from feeling
of helplessness, isolation, or insecurity. This response helps reduce
anxiety by encouraging the patient to express feelings. The nurse should
be supportive and develop goals together with the patient to give the
patient some control over an anxiety-inducing situation. Because the other
options ignore the patient’s feeling and block communication, they would
not reduce anxiety.
13. (C) Handling surgical instruments to the surgeon. The scrub nurse assist the
surgeon by providing appropriate surgical instruments and supplies,
maintaining strict surgical asepsis and, with the circulating nurse,
accounting for all gauze, sponges, needles, and instruments. The
circulating nurse assists the surgeon and scrub nurse, positions the
patient, applies appropriate equipment and surgical drapes, assists with
gowning and gloving, and provides the surgeon and scrub nurse with
supplies.
14. (C) Return shortly to the patient’s room and remain there until the patient takes
the medication. The nurse should return shortly to the patient’s room and
remain there until the patient takes the medication to verify that it was
taken as directed. The nurse should never leave medication at the
patient’s bedside unless specifically requested to do so.
15. (C) ¾ ml. The nurse solves the problem as follows: 10,000 units/7,500
units = 1 ml/X 10,000 X = 7,500 X= 7,500/10,000 or ¾ ml
16. (C) 38.9 degrees C. To convert Fahrenheit degrees to centigrade, use this
formula: C degrees = (F degrees – 32) x 5/9 C degrees = (102 – 32) 5/9 +
70 x 5/9 38.9 degrees C
17. (D) Arterial blood gas (ABG) analysis. All of these test help evaluate a
patient with respiratory problems. However, ABG analysis is the only test
evaluates gas exchange in the lungs, providing information about patient’s
oxygenation status.
18. (B) The diaphragm detects high-pitched sounds best. The diaphragm of a
stethoscope detects high-pitched sound best; the bell detects low pitched
sounds best. Palpation detects thrills best.
19. (C) Within 6 months. In most cases, an outpatient must fill a prescription
for a controlled substance within 6 months of the date on which the
prescription was written.
20. (D) The patient’s cognitive abilities. The nurse must consider the patient’s
cognitive abilities to understand drug instructions. If not, the nurse must
find a family member or significant other to take on the responsibility of
administering medications in the home setting. The patient’s ability to
recover, occupational hazards, and socioeconomic status do not affect
drug administration.
21. (B) Give simple directions shortly before the I.V. therapy is to start. Because
a 2-year-old child has limited understanding, the nurse should give simple
directions and explanations of what will occur shortly before the
procedure. She should try to avoid frightening the child with the
explanation and allow the child to make simple choices, such as choosing
the I.V. insertion site, if possible. However, she shouldn’t ask the child if he
wants the therapy, because the answer may be “No!” Telling the child that
the treatment is for his own good is ineffective because a 2-year-old
perceives pain as a negative sensation and cannot understand that a
painful procedure can have position results. Telling the child how long the
therapy will last is ineffective because the 2-year-old doesn’t have a good
understanding of time.
22. (A) Barrel. All syringes have three parts: a tip, which connects the needle
to the syringe; a barrel, the outer part on which the measurement scales
are printed; and a plunger, which fits inside the barrel to expel the
medication. The external part of the barrel and the plunger and (flange)
must be handled during the preparation and administration of the
injection. However, the inside and trip of the barrel, the inside (shaft) of the
plunger, and the needle tip must remain sterile until after the injection.
23. (A) Instruct the patient to lock upward, and drop the medication into the center
of the lower lid. Having the patient look upward reduces blinking and
protects the cornea. Instilling drops in the center of the lower lid promotes
absorption because the drops are less likely to run into the nasolacrimal
duct or out of the eye.
24. (C) Thickness. Gauge is a measure of the needle’s thickness: The higher
the number the thinner the shaft. Therefore, an 18G needle is considerably
thicker than a 25G needle.
25. (A) Hypotension. A major side effect of anticoagulant therapy is bleeding,
which can be identified by hypotension (a systolic blood pressure under
100 mm Hg). Anticoagulants do not result in the other three conditions.
PNLE : Maternal and Child Health
Nursing Exam 1
1. A client asks the nurse what a third degree laceration is. She was informed
that she had one. The nurse explains that this is:

A. that extended their anal sphincter


B. through the skin and into the muscles
C. that involves anterior rectal wall
D. that extends through the perineal muscle.
2. Betina 30 weeks AOG discharged with a diagnosis of placenta previa. The
nurse knows that the client understands her care at home when she says:
A. I am happy to note that we can have sex occasionally when I have no
bleeding.
B. I am afraid I might have an operation when my due comes
C. I will have to remain in bed until my due date comes
D. I may go back to work since I stay only at the office.
3. The uterus has already risen out of the pelvis and is experiencing farther
into the abdominal area at about the:

A. 8th week of pregnancy


B. 10th week of pregnancy
C. 12th week of pregnancy
D. 18th week of pregnancy
4. Which of the following urinary symptoms does the pregnant woman most
frequently experience during the first trimester:

A. frequency
B. dysuria
C. incontinence
D. burning
5. Mrs. Jimenez went to the health center for pre-natal check-up. the student
nurse took her weight and revealed 142 lbs. She asked the student nurse how
much should she gain weight in her pregnancy.

A. 20-30 lbs
B. 25-35 lbs
C. 30- 40 lbs
D. 10-15 lbs
6. The nurse is preparing Mrs. Jordan for cesarean delivery. Which of the
following key concept should the nurse consider when implementing nursing
care?

A. Explain the surgery, expected outcome and kind of anesthetics.


B. Modify preoperative teaching to meet the needs of either a planned or
emergency cesarean birth.
C. Arrange for a staff member of the anesthesia department to explain
what to expect post-operatively.
D. Instruct the mother’s support person to remain in the family lounge until
after the delivery.
7. Bettine Gonzales is hospitalized for the treatment of severe preecplampsia.
Which of the following represents an unusual finding for this condition?

A. generalized edema
B. proteinuria 4+
C. blood pressure of 160/110
D. convulsions
8. Nurse Geli explains to the client who is 33 weeks pregnant and is
experiencing vaginal bleeding that coitus:

A. Need to be modified in any way by either partner


B. Is permitted if penile penetration is not deep.
C. Should be restricted because it may stimulate uterine activity.
D. Is safe as long as she is in side-lying position.
9. Mrs. Precilla Abuel, a 32 year old mulripara is admitted to labor and
delivery. Her last 3 pregnancies in short stage one of labor. The nurses decide
to observe her closely. The physician determines that Mrs. Abuel’s cervix is
dilated to 6 cm. Mrs. Abuel states that she is extremely uncomfortable. To
lessen Mrs. Abuel’s discomfort, the nurse can advise her to:

A. lie face down


B. not drink fluids
C. practice holding breaths between contractions
D. assume Sim’s position
10. Which is true regarding the fontanels of the newborn?

A. The anterior is large in shape when compared to the posterior fontanel.


B. The anterior is triangular shaped; the posterior is diamond shaped.
C. The anterior is bulging; the posterior appears sunken.
D. The posterior closes at 18 months; the anterior closes at 8 to 12
months.
11. Mrs. Quijones gave birth by spontaneous delivery to a full term baby boy.
After a minute after birth, he is crying and moving actively. His birth weight is
6.8 lbs. What do you expect baby Quijones to weigh at 6 months?
A. 13 -14 lbs
B. 16 -17 lbs
C. 22 -23 lbs
D. 27 -28 lbs
12. During the first hours following delivery, the post partum client is given IVF
with oxytocin added to them. The nurse understands the primary reason for
this is:

A. To facilitate elimination
B. To promote uterine contraction
C. To promote analgesia
D. To prevent infection
13. Nurse Luis is assessing the newborn’s heart rate. Which of the following
would be considered normal if the newborn is sleeping?

A. 80 beats per minute


B. 100 beats per minute
C. 120 beats per minute
D. 140 beats per minute
14. The infant with Down Syndrome should go through which of the Erikson’s
developmental stages first?

A. Initiative vs. Self doubt


B. Industry vs. Inferiority
C. Autonomy vs. Shame and doubt
D. Trust vs. Mistrust
15. The child with phenylketonuria (PKU) must maintain a low phenylalanine
diet to prevent which of the following complications?

A. Irreversible brain damage


B. Kidney failure
C. Blindness
D. Neutropenia
16. Which age group is with imaginative minds and creates imaginary friends?

A. Toddler
B. Preschool
C. School
D. Adolescence
17. Which of the following situations would alert you to a potentially
developmental problem with a child?

A. Pointing to body parts at 15 months of age.


B. Using gesture to communicate at 18 months.
C. Cooing at 3 months.
D. Saying “mama” or “dada” for the first time at 18 months of age.
18. Isabelle, a 2 year old girl loves to move around and oftentimes manifests
negativism and temper tantrums. What is the best way to deal with her
behavior?

A. Tell her that she would not be loved by others is she behaves that way..
B. Withholding giving her toys until she behaves properly.
C. Ignore her behavior as long as she does not hurt herself and others.
D. Ask her what she wants and give it to pacify her.
19. Baby boy Villanueva, 4 months old, was seen at the pediatric clinic for his
scheduled check-up. By this period, baby Villanueva has already increased his
height by how many inches?

A. 3 inches
B. 4 inches
C. 5 inches
D. 6 inches
20. Alice, 10 years old was brought to the ER because of Asthma. She was
immediately put under aerosol administration of Terbutaline. After sometime,
you observe that the child does not show any relief from the treatment given.
Upon assessment, you noticed that both the heart and respiratory rate are still
elevated and the child shows difficulty of exhaling. You suspect:

A. Bronchiectasis
B. Atelectasis
C. Epiglotitis
D. Status Asthmaticus
21. Nurse Jonas assesses a 2 year old boy with a tentative diagnosis of
nephroblastoma. Symptoms the nurse observes that suggest this problem
include:

A. Lymphedema and nerve palsy


B. Hearing loss and ataxia
C. Headaches and vomiting
D. Abdominal mass and weakness
22. Which of the following danger sings should be reported immediately
during the antepartum period?

A. blurred vision
B. nasal stuffiness
C. breast tenderness
D. constipation
23. Nurse Jacob is assessing a 15 month old child with acute otitis media.
Which of the following symptoms would the nurse anticipate finding?

A. periorbital edema, absent light reflex and translucent tympanic


membrane
B. irritability, purulent drainage in middle ear, nasal congestion and cough
C. diarrhea, retracted tympanic membrane and enlarged parotid gland
D. Vomiting, pulling at ears and pearly white tympanic membrane
24. Which of the following is the most appropriate intervention to reduce
stress in a preterm infant at 33 weeks gestation?

A. Sensory stimulation including several senses at a time


B. tactile stimulation until signs of over stimulation develop
C. An attitude of extension when prone or side lying
D. Kangaroo care
25. The parent of a client with albinism would need to be taught which
preventive healthcare measure by the nurse:

A. Ulcerative colitis diet


B. Use of a high-SPF sunblock
C. Hair loss monitoring
D. Monitor for growth retardation
Answers and Rationales
1. (A) that extended their anal sphincter. Third degree laceration involves all in
the second degree laceration and the external sphincter of the rectum.
Options B, C and D are under the second degree laceration.
2. (C) I will have to remain in bed until my due date comes. Placenta previa
means that the placenta is the presenting part. On the first and second
trimester there is spotting. On the third trimester there is bleeding that is
sudden, profuse and painless.
3. (D) 18th week of pregnancy. On the 8th week of pregnancy, the uterus is
still within the pelvic area. On the 10th week, the uterus is still within the
pelvic area. On the 12th week, the uterus and placenta have grown,
expanding into the abdominal cavity. On the 18th week, the uterus has
already risen out of the pelvis and is expanding into the abdominal area.
4. (A) frequency. Pressure and irritation of the bladder by the growing
uterus during the first trimester is responsible for causing urinary
frequency. Dysuria, incontinence and burning are symptoms associated
with urinary tract infection.
5. (B) 25-35 lbs. A weight gain of 11. 2 to 15.9 kg (25 to 35 lbs) is currently
recommended as an average weight gain in pregnancy. This weight gain
consists of the following: fetus- 7.5 lb; placenta- 1.5 lb; amniotic fluid- 2 lb;
uterus- 2.5 lb; breasts- 1.5 to 3 lb; blood volume- 4 lb; body fat- 7 lb; body
fluid- 7 lb.
6. (B) Modify preoperative teaching to meet the needs of either a planned or
emergency cesarean birth. A key point to consider when preparing the client
for a cesarean delivery is to modify the preoperative teaching to meet the
needs of either planned or emergency cesarean birth, the depth and
breadth of instruction will depend on circumstances and time available.
7. (D) convulsions. Options A, B and C are findings of severe preeclampsia.
Convulsions is a finding of eclampsia—an obstetrical emergency.
8. (C) Should be restricted because it may stimulate uterine activity.. Coitus is
restricted when there is watery discharge, uterine contraction and vaginal
bleeding. Also those women with a history of spontaneous miscarriage
may be advised to avoid coitus during the time of pregnancy when a
previous miscarriage occurred.
9. (D) assume Sim’s position. When the woman is in Sim’s position, this puts
the weight of the fetus on bed, not on the woman and allows good
circulation in the lower extremities.
10. (A) The anterior is large in shape when compared to the posterior
fontanel.. The anterior fontanel is larger in size than the posterior fontanel.
Additionally, the anterior fontanel, which is diamond shaped closes at 18
month, whereas the posterior fontanel, which is triangular in shape closes
at 8 to 12 weeks. Neither fontanel should appear bulging, which may
indicate increases ICP or sunken, which may indicate hydration.
11. (A) 13 -14 lbs. The birth weight of an infant is doubled at 6 months and is
tripled at 12 months.
12. (B) To promote uterine contraction. Oxytocin is a hormone produced by the
pituitary gland that produces intermittent uterine contractions, helping to
promote uterine involution.
13. (B) 100 beats per minute. The normal heart rate for a newborn that is
sleeping is approximately 100 beats per minute. If the newborn was
awake, the normal heart rate would range from 120 to 160 beats per
minute.
14. (D) Trust vs. Mistrust. The child with Down syndrome will go through the
same first stage, trust vs. mistrust, only at a slow rate. Therefore, the
nurse should concentrate on developing on bond between the primary
caregiver and the child.
15. (A) Irreversible brain damage. The child with PKU must maintain a strict
low phenylalanine diet to prevent central nervous system damage,
seizures and eventual death.
16. (B) Preschool. During preschool, this is the time when children do
imitative play, imaginative play—the occurrence of imaginative playmates,
dramatic play where children like to act, dance and sing.
17. (D) Saying “mama” or “dada” for the first time at 18 months of age.. A child
should say “mama” or “dada” during 10 to 12 months of age. Options A, B
and C are all normal assessments of language development of a child.
18. (C) Ignore her behavior as long as she does not hurt herself and others.. If a
child is trying to get attention or trying to get something through tantrums
—ignore his/her behavior.
19. (B) 4 inches. From birth to 6 months, the infant grows 1 inch (2.5 cm) per
month. From 6 to 12 months, the infant grows ½ inch (1.25 cm) per
month.
20. (D) Status Asthmaticus. Status asthmaticus leads to respiratory distress
and bronchospasm despite of treatment and interventions. Mechanical
ventilation maybe needed due to respiratory failure.
21. (D) Abdominal mass and weakness. Nephroblastoma or Wilm’s tumor is
caused by chromosomal abnormalities, most common kidney cancer
among children characterized by abdominal mass, hematuria,
hypertension and fever.
22. (A) blurred vision. Danger signs that require prompt reporting are leaking
of amniotic fluid, blurred vision, vaginal bleeding, rapid weight gain and
elevated blood pressure. Nasal stuffiness, breast tenderness, and
constipation are common discomforts associated with pregnancy.
23. (B) irritability, purulent drainage in middle ear, nasal congestion and
cough. Irritability, purulent drainage in middle ear, nasal congestion and
cough, fever, loss of appetite, vomiting and diarrhea are clinical
manifestations of otitis media. Acute otitis media is common in children 6
months to 3 years old and 8 years old and above. Breast fed infants have
higher resistance due to protection of Eustachian tubes and middle ear
from breast milk.
24. (D) Kangaroo care. Kangaroo care is the use of skin-to-skin contact to
maintain body heat. This method of care not only supplies heat but also
encourages parent-child interaction.
25. (B) Use of a high-SPF sunblock. Without melanin production, the child with
albinism is at risk for severe sunburns. Maximum sun protection should
be taken, including use of hats, long sleeves, minimal time in the sun and
high-SPF sunblock, to prevent any problems.
PNLE : Maternal and Child Health
Nursing Exam 2
1. Nurse Bella explains to a 28 year old pregnant woman undergoing a non-
stress test that the test is a way of evaluating the condition of the fetus by
comparing the fetal heart rate with:

A. Fetal lie
B. Fetal movement
C. Maternal blood pressure
D. Maternal uterine contractions
2. During a 2 hour childbirth focusing on labor and delivery process for
primigravida. The nurse describes the second maneuver that the fetus goes
through during labor progress when the head is the presenting part as which
of the following:

A. Flexion
B. Internal rotation
C. Descent
D. External rotation
3. Mrs. Jovel Diaz went to the hospital to have her serum blood test for alpha-
fetoprotein. The nurse informed her about the result of the elevation of serum
AFP. The patient asked her what was the test for:

A. Congenital Adrenal Hyperplasia


B. PKU
C. Down Syndrome
D. Neural tube defects
4. Fetal heart rate can be auscultated with a fetoscope as early as:

A. 5 weeks of gestation
B. 10 weeks of gestation
C. 15 weeks of gestation
D. 20 weeks of gestation
5. Mrs. Bendivin states that she is experiencing aching swollen, leg veins. The
nurse would explain that this is most probably the result of which of the
following:

A. Thrombophlebitis
B. PIH
C. Pressure on blood vessels from the enlarging uterus
D. The force of gravity pulling down on the uterus
6. Mrs. Ella Santoros is a 25 year old primigravida who has Rheumatic heart
disease lesion. Her pregnancy has just been diagnosed. Her heart disease has
not caused her to limit physical activity in the past. Her cardiac disease and
functional capacity classification is:

A. Class I
B. Class II
C. Class III
D. class IV
7. The client asks the nurse, “When will this soft spot at the top of the head of
my baby will close?” The nurse should instruct the mother that the neonate’s
anterior fontanel will normally close by age:

A. 2-3 months
B. 6-8 months
C. 10-12 months
D. 12-18 months
8. When a mother bleeds and the uterus is relaxed, soft and non-tender, you
can account the cause to:

A. Atony of the uterus


B. Presence of uterine scar
C. Laceration of the birth canal
D. Presence of retained placenta fragments
9. Mrs. Pichie Gonzales’s LMP began April 4, 2010. Her EDD should be which
of the following:

A. February 11, 2011


B. January 11, 20111
C. December 12, 2010
D. Nowember 14, 2010
10. Which of the following prenatal laboratory test values would the nurse
consider as significant?

A. Hematocrit 33.5%
B. WBC 8,000/mm3
C. Rubella titer less than 1:8
D. One hour glucose challenge test 110 g/dL
11. Aling Patricia is a patient with preeclampsia. You advise her about her
condition, which would tell you that she has not really understood your
instructions?

A. “I will restrict my fat in my diet.”


B. “I will limit my activities and rest more frequently throughout the day.”
C. “I will avoid salty foods in my diet.”
D. “I will come more regularly for check-up.”
12. Mrs. Grace Evangelista is admitted with severe preeclampsia. What type
of room should the nurse select this patient?

A. A room next to the elevator.


B. The room farthest from the nursing station.
C. The quietest room on the floor.
D. The labor suite.
13. During a prenatal check-up, the nurse explains to a client who is Rh
negative that RhoGAM will be given:

A. Weekly during the 8th month because this is her third pregnancy.
B. During the second trimester, if amniocentesis indicates a problem.
C. To her infant immediately after delivery if the Coomb’s test is positive.
D. Within 72 hours after delivery if infant is found to be Rh positive.
14. A baby boy was born at 8:50pm. At 8:55pm, the heart rate was 99 bpm.
She has a weak cry, irregular respiration. She was moving all extremities and
only her hands and feet were still slightly blue. The nurse should enter the
APGAR score as:

A. 5
B. 6
C. 7
D. 8
15. Billy is a 4 year old boy who has an IQ of 140 which means:

A. average normal
B. very superior
C. above average
D. genius
16. A newborn is brought to the nursery. Upon assessment, the nurse finds
that the child has short palpebral fissures, thinned upper lip. Based on this
data, the nurse suspects that the newborn is MOST likely showing the effects
of:

A. Chronic toxoplasmosis
B. Lead poisoning
C. Congenital anomalies
D. Fetal alcohol syndrome
17. A priority nursing intervention for the infant with cleft lip is which of the
following:

A. Monitoring for adequate nutritional intake


B. Teaching high-risk newborn care
C. Assessing for respiratory distress
D. Preventing injury
18. Nurse Jacob is assessing a 12 year old who has hemophilia A. Which of
the following assessment findings would the nurse anticipate?

A. an excess of RBC
B. an excess of WBC
C. a deficiency of clotting factor VIII
D. a deficiency of clotting factor IX
19. Celine, a mother of a 2 year old tells the nurse that her child “cries and has
a fit when I have to leave him with a sitter or someone else.” Which of the
following statements would be the nurse’s most accurate analysis of the
mother’s comment?
A. The child has not experienced limit-setting or structure.
B. The child is expressing a physical need, such as hunger.
C. The mother has nurtured overdependence in the child.
D. The mother is describing her child’s separation anxiety.
20. Mylene Lopez, a 16 year old girl with scoliosis has recently received an
invitation to a pool party. She asks the nurse how she can disguise her
impairment when dressed in a bathing suit. Which nursing diagnosis can be
justified by Mylene’s statement?

A. Anxiety
B. Body image disturbance
C. Ineffective individual coping
D. Social isolation
21. The foul-smelling, frothy characteristic of the stool in cystic fibrosis
results from the presence of large amounts of which of the following:

A. sodium and chloride


B. undigested fat
C. semi-digested carbohydrates
D. lipase, trypsin and amylase
22. Which of the following would be a disadvantage of breast feeding?

A. involution occurs rapidly


B. the incidence of allergies increases due to maternal antibodies
C. the father may resent the infant’s demands on the mother’s body
D. there is a greater chance of error during preparation
23. A client is noted to have lymphedema, webbed neck and low posterior
hairline. Which of the following diagnoses is most appropriate?

A. Turner’s syndrome
B. Down’s syndrome
C. Marfan’s syndrome
D. Klinefelter’s syndrome
24. A 4 year old boy most likely perceives death in which way:

A. An insignificant event unless taught otherwise


B. Punishment for something the individual did
C. Something that just happens to older people
D. Temporary separation from the loved one.
25. Catherine Diaz is a 14 year old patient on a hematology unit who is being
treated for sickle cell crisis. During a crisis such as that seen in sickle cell
anemia, aldosterone release is stimulated. In what way might this influence
Catherine’s fluid and electrolyte balance?

A. sodium loss, water loss and potassium retention


B. sodium loss, water los and potassium loss
C. sodium retention, water loss and potassium retention
D. sodium retention, water retention and potassium loss
Answers and Rationales
1. (B) Fetal movement. Non-stress test measures response of the FHR to
the fetal movement. With fetal movement, FHR increase by 15 beats and
remain for 15 seconds then decrease to average rate. No increase means
poor oxygenation perfusion to fetus.
2. (A) Flexion. The 6 cardinal movements of labor are descent, flexion,
internal rotation, extension, external rotation and expulsion.
3. (D) Neural tube defects. Alpha-fetoprotein is a substance produces by the
fetal liver that is present in amniotic fluid and maternal serum. The level is
abnormally high in the maternal serum if the fetus has an open spinal or
abdominal defect because the open defect allows more AFP to appear.
4. (D) 20 weeks of gestation. The FHR can be auscultated with a fetoscope
at about 20 weeks of gestation. FHR is usually auscultated at the midline
suprapubic region with Doppler ultrasound at 10 to 12 weeks of gestation.
FHR cannot be heard any earlier than 10 weeks of gestation.
5. (C) Pressure on blood vessels from the enlarging uterus. Pressure of the
growing fetus on blood vessels results in an increase risk for venous
stasis in the lower extremities. Subsequently, edema and varicose vein
formation may occur.
6. (A) Class I. Clients under class I has no physical activity limitation. There
is a slight limitation of physical activity in class II, ordinary activity causes
fatigue, palpitation, dyspnea or angina. Class III is moderate limitation of
physical activity; less than ordinary activity causes fatigue. Unable to carry
on any activity without experiencing discomfort is under class IV.
7. (D) 12-18 months. Anterior fontanel closes at 12-18 months while
posterior fontanel closes at birth until 2 months.
8. (A) Atony of the uterus. Uterine atony, or relaxation of the uterus is the
most frequent cause of postpartal hemorrhage. It is the inability to
maintain the uterus in contracted state.
9. (B) January 11, 20111. Using the Nagel’s rule, he use this formula ( -3
calendar months + 7 days).
10. (C) Rubella titer less than 1:8. A rubella titer should be 1:8 or greater. Thus,
a finding of a titer less than 1:8 is significant, indicating that the client may
not possess immunity to rubella. A hematocrit of 33.5%, WBC of
8,000/mm3, and a 1 hour glucose challenge test of 110 g/dL are within
normal parameters.
11. (B) “I will limit my activities and rest more frequently throughout the
day.”Pregnant woman with preeclampsia should be in a complete bed
rest. When body is in recumbent position, sodium tends to be excreted at
a faster rate. It is the best method of aiding increased excretion of sodium
and encouraging diuresis. Rest should always be in a lateral recumbent
position to avoid uterine pressure on the vena cava and prevent supine
hypotension.
12. (C) The quietest room on the floor.A loud noise such as a crying baby, or a
dropped tray of equipment may be sufficient to trigger a seizure initiating
eclampsia, a woman with severe preeclampsia should be admiotted to a
private room so she can rest as undisturbed as possible. Darken the room
if possible because bright light can trigger seizures.
13. (D) Within 72 hours after delivery if infant is found to be Rh
positive. RhoGAM is given to Rh-negative mothers within 72 hours after
birth of Rh-positive baby to prevent development of antibodies in the
maternal blood stream, which will be fata to succeeding Rh-positive
offspring.
14. (B) 6. Heart rate of 99 bpm-1; weak cry-1; irregular respiration-1; moving
all extremities-2; extremities are slightly blue-1; with a total score of 6.
15. (D) genius. IQ= mental age/chronological age x 100. Mental age refers to
the typical intelligence level found for people at a give chronological age.
OQ of 140 and above is considered genius.
16. (D) Fetal alcohol syndrome. The newborn with fetal alcohol syndrome has
a number of possible problems at birth. Characteristics that mark the
syndrome include pre and postnatal growth retardation; CNS involvement
such as cognitive challenge, microcephally and cerebral palsy; and a
distinctive facial feature of a short palpebral fissure and thin upper lip.
17. (A) Monitoring for adequate nutritional intake. The infant with cleft lip is
unable to create an adequate seal for sucking. The child is at risk for
inadequate nutritional intake as well as aspiration.
18. (C) a deficiency of clotting factor VIII. Hemophillia A (classic hemophilia)
is a deficiency in factor VIII (an alpha globulin that stabilizes fibrin clots).
19. (D) The mother is describing her child’s separation anxiety. Before coming to
any conclusion, the nurse should ask the mother focused questions;
however, based on initial information, the analysis of separation anxiety
would be most valid. Separation anxiety is a normal toddler response.
When the child senses he is being sent away from those who most
provide him with love and security. Crying is one way a child expresses a
physical need; however, the nurse would be hasty in drawing this as first
conclusion based on what the mother has said. Nurturing overdependence
or not providing structure for the toddler are inaccurate conclusions based
on the information provided.
20. (B) Body image disturbance. Mylene is experiencing uneasiness about the
curvative of her spine, which will be more evident when she wears a
bathing suit. This data suggests a body image disturbance. There is no
evidence of anxiety or ineffective coping. The fact that Mylene is planning
to attend a pool party dispels a diagnosis of social isolation.
21. (B) undigested fat. The client with cystic fibrosis absorbs fat poorly
because of the think secretions blocking the pancreatic duct. The lack of
natural pancreatic enzyme leads to poor absorption of predominantly fats
in the duodenum. Foul-smelling, frothy stool is termed steatorrhea.
22. (C) the father may resent the infant’s demands on the mother’s body. With
breast feeding, the father’s body is not capable of providing the milk for
the newborn, which may interfere with feeding the newborn, providing
fewer chances for bonding, or he may be jealous of the infant’s demands
on his wife time and body. Breast feeding is advantageous because
uterine involution occurs more rapidly, thus minimizing blood loss. The
presence of maternal antibodies in breast milk helps decrease the
incidence of allergies in the newborn. A greater chance for error is
associated with bottle feeding. No preparation required for breast feeding.
23. (A) Turner’s syndrome. Lymphedema, webbed neck and low posterior
hairline, these are the 3 key assessment features in Turner’s syndrome. If
the child is diagnosed early in age, proper treatment can be offered to the
family. All newborns should be screened for possible congenital defects.
24. (D) Temporary separation from the loved one. The predominant perception
of death by preschool age children is that death is temporary separation.
Because that child is losing someone significant and will not see that
person again, it’s inaccurate to infer death is insignificant, regardless of
the child’s response.
25. (D) sodium retention, water retention and potassium loss. Stress stimulates
the adrenal cortex to increase the release of aldosterone. Aldosterone
promotes the resorption of sodium, the retention of water and the loss of
potassium
PNLE : Maternal and Child Health
Nursing Exam 3
1. A pregnant woman who is at term is admitted to the birthing unit in active
labor. The client has only progressed from 2cm to 3 cm in 8 hours. She is
diagnosed with hypotonic dystocia and the physician ordered Oxytocin
(Pitocin) to augment her contractions. Which of the following is the most
important aspect of nursing intervention at this time?

A. Timing and recording length of contractions.


B. Monitoring.
C. Preparing for an emergency cesarean birth.
D. Checking the perineum for bulging.
2. A client who hallucinates is not in touch with reality. It is important for the
nurse to:

A. Isolate the client from other patients.


B. Maintain a safe environment.
C. Orient the client to time, place, and person.
D. Establish a trusting relationship.
3. The nurse is caring to a child client who has had a tonsillectomy. The child
complains of having dryness of the throat. Which of the following would the
nurse give to the child?

A. Cola with ice


B. Yellow noncitrus Jello
C. Cool cherry Kool-Aid
D. A glass of milk
4. The physician ordered Phenylephrine (Neo-Synephrine) nasal spray to a 13-
year-old client. The nurse caring to the client provides instructions that the
nasal spray must be used exactly as directed to prevent the development of:

A. Increased nasal congestion.


B. Nasal polyps.
C. Bleeding tendencies.
D. Tinnitus and diplopia.
5. A client with tuberculosis is to be admitted in the hospital. The nurse who
will be assigned to care for the client must institute appropriate precautions.
The nurse should:

A. Place the client in a private room.


B. Wear an N 95 respirator when caring for the client.
C. Put on a gown every time when entering the room.
D. Don a surgical mask with a face shield when entering the room.
6. Which of the following is the most frequent cause of noncompliance to the
medical treatment of open-angle glaucoma?

A. The frequent nausea and vomiting accompanying use of miotic drug.


B. Loss of mobility due to severe driving restrictions.
C. Decreased light and near-vision accommodation due to miotic effects
of pilocarpine.
D. The painful and insidious progression of this type of glaucoma.
7. In the morning shift, the nurse is making rounds in the nursing care units.
The nurse enters in a client’s room and notes that the client’s tube has
become disconnected from the Pleurovac. What would be the initial nursing
action?

A. Apply pressure directly over the incision site.


B. Clamp the chest tube near the incision site.
C. Clamp the chest tube closer to the drainage system.
D. Reconnect the chest tube to the Pleurovac.
8. Which of the following complications during a breech birth the nurse needs
to be alarmed?

A. Abruption placenta.
B. Caput succedaneum.
C. Pathological hyperbilirubinemia.
D. Umbilical cord prolapse.
9. The nurse is caring to a client diagnosed with severe depression. Which of
the following nursing approach is important in depression?

A. Protect the client against harm to others.


B. Provide the client with motor outlets for aggressive, hostile feelings.
C. Reduce interpersonal contacts.
D. Deemphasizing preoccupation with elimination, nourishment, and sleep.
10. A 3-month-old client is in the pediatric unit. During assessment, the nurse
is suspecting that the baby may have hypothyroidism when mother states that
her baby does not:

A. Sit up.
B. Pick up and hold a rattle.
C. Roll over.
D. Hold the head up.
11. The physician calls the nursing unit to leave an order. The senior nurse
had conversation with the other staff. The newly hired nurse answers the
phone so that the senior nurses may continue their conversation. The new
nurse does not knowthe physician or the client to whom the order pertains.
The nurse should:

A. Ask the physician to call back after the nurse has read the hospital
policy manual.
B. Take the telephone order.
C. Refuse to take the telephone order.
D. Ask the charge nurse or one of the other senior staff nurses to take the
telephone order.
12. The staff nurse on the labor and delivery unit is assigned to care to a
primigravida in transition complicated by hypertension. A new pregnant
woman in active labor is admitted in the same unit. The nurse manager
assigned the same nurse to the second client. The nurse feels that the client
with hypertension requires one-to-one care. What would be the initial actionof
the nurse?

A. Accept the new assignment and complete an incident report describing


a shortage of nursing staff.
B. Report the incident to the nursing supervisor and request to be floated.
C. Report the nursing assessment of the client in transitional labor to the
nurse manager and discuss misgivings about the new assignment.
D. Accept the new assignment and provide the best care.
13. A newborn infant with Down syndrome is to be discharged today. The
nurse is preparing to give the discharge teaching regarding the proper care at
home. The nurse would anticipate that the mother is probably at the:

A. 40 years of age.
B. 20 years of age.
C. 35 years of age.
D. 20 years of age.
14. The emergency department has shortage of staff. The nurse manager
informs the staff nurse in the critical care unit that she has to float to the
emergency department. What should the staff nurse expect under these
conditions?

A. The float staff nurse will be informed of the situation before the shift
begins.
B. The staff nurse will be able to negotiate the assignments in the
emergency department.
C. Cross training will be available for the staff nurse.
D. Client assignments will be equally divided among the nurses.
15. The nurse is assigned to care for a child client admitted in the pediatrics
unit. The client is receiving digoxin. Which of the following questions will be
asked by the nurse to the parents of the child in order to assess the client’s
risk for digoxin toxicity?

A. “Has he been exposed to any childhood communicable diseases in the


past 2-3 weeks?”
B. “Has he been taking diuretics at home?”
C. “Do any of his brothers and sisters have history of cardiac problems?”
D. “Has he been going to school regularly?”
16. The nurse noticed that the signed consent form has an error. The form
states, “Amputation of the right leg” instead of the left leg that is to be
amputated. The nurse has administered already the preoperative medications.
What should the nurse do?

A. Call the physician to reschedule the surgery.


B. Call the nearest relative to come in to sign a new form.
C. Cross out the error and initial the form.
D. Have the client sign another form.
17. The nurse in the nursing care unit checks the fluctuation in the water-seal
compartment of a closed chest drainage system. The fluctuation has stopped,
the nurse would:

A. Vigorously strip the tube to dislodge a clot.


B. Raise the apparatus above the chest to move fluid.
C. Increase wall suction above 20 cm H2O pressure.
D. Ask the client to cough and take a deep breath.
18. The pediatric nurse in the neonatal unit was informed that the baby that is
brought to the mother in the hospital room is wrong. The nurse determines
that two babies were placed in the wrong cribs. The most appropriate nursing
action would be to:

A. Determine who is responsible for the mistake and terminate his or her
employment.
B. Record the event in an incident/variance report and notify the nursing
supervisor.
C. Reassure both mothers, report to the charge nurse, and do not record.
D. Record detailed notes of the event on the mother’s medical record.
19. Before the administration of digoxin, the nurse completes an assessment
to a toddler client for signs and symptoms of digoxin toxicity. Which of the
following is the earliest and most significant sign of digoxin toxicity?

A. Tinnitus
B. Nausea and vomiting
C. Vision problem
D. Slowing in the heart rate
20. Which of the following treatment modality is appropriate for a client with
paranoid tendency?

A. Activity therapy.
B. Individual therapy.
C. Group therapy.
D. Family therapy.
21. The client with rheumatoid arthritis is for discharge. In preparing the client
for discharge on prednisone therapy, the nurse should advise the client to:

A. Wear sunglasses if exposed to bright light for an extended period of


time.
B. Take oral preparations of prednisone before meals.
C. Have periodic complete blood counts while on the medication.
D. Never stop or change the amount of the medication without medical
advice.
22. A pregnant client tells the nurse that she is worried about having urinary
frequency. What will be the most appropriate nursing response?

A. “Try using Kegel (perineal) exercises and limiting fluids before bedtime.
If you have frequency associated with fever, pain on voiding, or blood in
the urine, call your doctor/nurse-midwife.
B. “Placental progesterone causes irritability of the bladder sphincter. Your
symptoms will go away after the baby comes.”
C. “Pregnant women urinate frequently to get rid of fetal wastes. Limit
fluids to 1L/daily.”
D. “Frequency is due to bladder irritation from concentrate urine and is
normal in pregnancy. Increase your daily fluid intake to 3L.”
23. Which of the following will help the nurse determine that the expression of
hostility is useful?

A. Expression of anger dissipates the energy.


B. Energy from anger is used to accomplish what needs to be done.
C. Expression intimidates others.
D. Degree of hostility is less than the provocation.
24. The nurse is providing an orientation regarding case management to the
nursing students. Which characteristics should the nurse include in the
discussion in understanding case management?

A. Main objective is a written plan that combines discipline-specific


processes used to measure outcomes of care.
B. Main purpose is to identify expected client, family and staff
performance against the timeline for clients with the same diagnosis.
C. Main focus is comprehensive coordination of client care, avoid
unnecessary duplication of services, improve resource utilization and
decrease cost.
D. Primary goal is to understand why predicted outcomes have not been
met and the correction of identified problems.
25. The physician orders a dose of IV phenytoin to a child client. In preparing
in the administration of the drug, which nursing action is not correct?

A. Infuse the phenytoin into a smaller vein to prevent purple glove


syndrome.
B. Check the phenytoin solution to be sure it is clear or light yellow in color,
never cloudy.
C. Plan to give phenytoin over 30-60 minutes, using an in-line filter.
D. Flush the IV tubing with normal saline before starting phenytoin.
26. The pregnant woman visits the clinic for check –up. Which assessment
findings will help the nurse determine that the client is in 8-week gestation?

A. Leopold maneuvers.
B. Fundal height.
C. Positive radioimmunoassay test (RIA test).
D. Auscultation of fetal heart tones.
27. Which of the following nursing intervention is essential for the client who
had pneumonectomy?

A. Medicate for pain only when needed.


B. Connect the chest tube to water-seal drainage.
C. Notify the physician if the chest drainage exceeds 100mL/hr.
D. Encourage deep breathing and coughing.
28. The nurse is providing a health teaching to a group of parents regarding
Chlamydia trachomatis. The nurse is correct in the statement, “Chlamydia
trachomatis is not only an intracellular bacterium that causes neonatal
conjunctivitis, but it also can cause:

A. Discoloration of baby and adult teeth.


B. Pneumonia in the newborn.
C. Snuffles and rhagades in the newborn.
D. Central hearing defects in infancy.
29. The nurse is assigned to care to a 17-year-old male client with a history of
substance abuse. The client asks the nurse, “Have you ever tried or used
drugs?” The most correct response of the nurse would be:

A. “Yes, once I tried grass.”


B. “No, I don’t think so.”
C. “Why do you want to know that?”
D. “How will my answer help you?”
30. Which of the following describes a health care team with the principles of
participative leadership?

A. Each member of the team can independently make decisions regarding


the client’s care without necessarily consulting the other members.
B. The physician makes most of the decisions regarding the client’s care.
C. The team uses the expertise of its members to influence the decisions
regarding the client’s care.
D. Nurses decide nursing care; physicians decide medical and other
treatment for the client.
31. A nurse is giving a health teaching to a woman who wants to breastfeed
her newborn baby. Which hormone, normally secreted during the postpartum
period, influences both the milk ejection reflex and uterine involution?

A. Oxytocin.
B. Estrogen.
C. Progesterone.
D. Relaxin.
32. One staff nurse is assigned to a group of 5 patients for the 12-hour shift.
The nurse is responsible for the overall planning, giving and evaluating care
during the entire shift. After the shift, same responsibility will be endorsed to
the next nurse in charge. This describes nursing care delivered via the:

A. Primary nursing method.


B. Case method.
C. Functional method.
D. Team method.
33. The ambulance team calls the emergency department that they are going
to bring a client who sustained burns in a house fire. While waiting for the
ambulance, the nurse will anticipate emergency care to include assessment
for:

A. Gas exchange impairment.


B. Hypoglycemia.
C. Hyperthermia.
D. Fluid volume excess.
34. Most couples are using “natural” family planning methods. Most
accidental pregnancies in couples preferred to use this method have been
related to unprotected intercourse before ovulation. Which of the following
factor explains why pregnancy may be achieved by unprotected intercourse
during the preovulatory period?

A. Ovum viability.
B. Tubal motility.
C. Spermatozoal viability.
D. Secretory endometrium.
35. An older adult client wakes up at 2 o’clock in the morning and comes to
the nurse’s station saying, “I am having difficulty in sleeping.” What is the best
nursing response to the client?

A. “I’ll give you a sleeping pill to help you get more sleep now.”
B. “Perhaps you’d like to sit here at the nurse’s station for a while.”
C. “Would you like me to show you where the bathroom is?”
D. “What woke you up?”
36. The nurse is taking care of a multipara who is at 42 weeks of gestation
and in active labor, her membranes ruptured spontaneously 2 hours ago.
While auscultating for the point of maximum intensity of fetal heart tones
before applying an external fetal monitor, the nurse counts 100 beats per
minute. The immediate nursing action is to:

A. Start oxygen by mask to reduce fetal distress.


B. Examine the woman for signs of a prolapsed cord.
C. Turn the woman on her left side to increase placental perfusion.
D. Take the woman’s radial pulse while still auscultating the FHR.
37. The nurse must instruct a client with glaucoma to avoid taking over-the-
counter medications like:

A. Antihistamines.
B. NSAIDs.
C. Antacids.
D. Salicylates.
38. A male client is brought to the emergency department due to motor
vehicle accident. While monitoring the client, the nurse suspects increasing
intracranial pressure when:

A. Client is oriented when aroused from sleep, and goes back to sleep
immediately.
B. Blood pressure is decreased from 160/90 to 110/70.
C. Client refuses dinner because of anorexia.
D. Pulse is increased from 88-96 with occasional skipped beat.
39. The nurse is conducting a lecture to a class of nursing students about
advance directives to preoperative clients. Which of the following statement
by the nurse js correct?

A. “The spouse, but not the rest of the family, may override the advance
directive.”
B. “An advance directive is required for a “do not resuscitate” order.”
C. “A durable power of attorney, a form of advance directive, may only be
held by a blood relative.”
D. “The advance directive may be enforced even in the face of opposition
by the spouse.”
40. A client diagnosed with schizophrenia is shouting and banging on the door
leading to the outside, saying, “I need to go to an appointment.” What is the
appropriate nursing intervention?
A. Tell the client that he cannot bang on the door.
B. Ignore this behavior.
C. Escort the client going back into the room.
D. Ask the client to move away from the door.
41. Which of the following action is an accurate tracheal suctioning
technique?

A. 25 seconds of continuous suction during catheter insertion.


B. 20 seconds of continuous suction during catheter insertion.
C. 10 seconds of intermittent suction during catheter withdrawal.
D. 15 seconds of intermittent suction during catheter withdrawal.
42. The client’s jaw and cheekbone is sutured and wired. The nurse
anticipates that the most important thing that must be ready at the bedside is:

A. Suture set.
B. Tracheostomy set.
C. Suction equipment.
D. Wire cutters.
43. A mother is in the third stage of labor. Which of the following signs will
help the nurse determine the signs of placental separation?

A. The uterus becomes globular.


B. The umbilical cord is shortened.
C. The fundus appears at the introitus.
D. Mucoid discharge is increased.
44. After therapy with the thrombolytic alteplase (t-PA), what observation will
the nurse report to the physician?

A. 3+ peripheral pulses.
B. Change in level of consciousness and headache.
C. Occasional dysrhythmias.
D. Heart rate of 100/bpm.
45. A client who undergone left nephrectomy has a large flank incision. Which
of the following nursing action will facilitate deep breathing and coughing?

A. Push fluid administration to loosen respiratory secretions.


B. Have the client lie on the unaffected side.
C. Maintain the client in high Fowler’s position.
D. Coordinate breathing and coughing exercise with administration of
analgesics.
46. The community nurse is teaching the group of mothers about the cervical
mucus method of natural family planning. Which characteristics are typical of
the cervical mucus during the “fertile” period of the menstrual cycle?

A. Absence of ferning.
B. Thin, clear, good spinnbarkeit.
C. Thick, cloudy.
D. Yellow and sticky.
47. A client with ruptured appendix had surgery an hour ago and is transferred
to the nursing care unit. The nurse placed the client in a semi-Fowler’s
position primarily to:

A. Facilitate movement and reduce complications from immobility.


B. Fully aerate the lungs.
C. Splint the wound.
D. Promote drainage and prevent subdiaphragmatic abscesses.
48. Which of the following will best describe a management function?

A. Writing a letter to the editor of a nursing journal.


B. Negotiating labor contracts.
C. Directing and evaluating nursing staff members.
D. Explaining medication side effects to a client.
49. The parents of an infant client ask the nurse to teach them how to
administer Cortisporin eye drops. The nurse is correct in advising the parents
to place the drops:

A. In the middle of the lower conjunctival sac of the infant’s eye.


B. Directly onto the infant’s sclera.
C. In the outer canthus of the infant’s eye.
D. In the inner canthus of the infant’s eye.
50. The nurse is assessing on the client who is admitted due to vehicle
accident. Which of the following findings will help the nurse that there is
internal bleeding?
A. Frank blood on the clothing.
B. Thirst and restlessness.
C. Abdominal pain.
D. Confusion and altered of consciousness.
51. The nurse is completing an assessment to a newborn baby boy. The nurse
observes that the skin of the newborn is dry and flaking and there are several
areas of an apparent macular rash. The nurse charts this as:

A. Icterus neonatorum
B. Multiple hemangiomas
C. Erythema toxicum
D. Milia
52. The client is brought to the emergency department because of serious
vehicle accident. After an hour, the client has been declared brain dead. The
nurse who has been with the client must now talk to the family about organ
donation. Which of the following consideration is necessary?

A. Include as many family members as possible.


B. Take the family to the chapel.
C. Discuss life support systems.
D. Clarify the family’s understanding of brain death.
53. The nurse is teaching exercises that are good for pregnant women
increasing tone and fitness and decreasing lower backache. Which of the
following should the nurse exclude in the exercise program?

A. Stand with legs apart and touch hands to floor three times per day.
B. Ten minutes of walking per day with an emphasis on good posture.
C. Ten minutes of swimming or leg kicking in pool per day.
D. Pelvic rock exercise and squats three times a day.
54. A client with obsessive-compulsive behavior is admitted in the psychiatric
unit. The nurse taking care of the client knows that the primary treatment goal
is to:

A. Provide distraction.
B. Support but limit the behavior.
C. Prohibit the behavior.
D. Point out the behavior.
55. After ileostomy, the nurse expects that the drainage appliance will be
applied to the stoma:

A. When the client is able to begin self-care procedures.


B. 24 hours later, when the swelling subsided.
C. In the operating room after the ileostomy procedure.
D. After the ileostomy begins to function.
56. A female client who has a 28-day menstrual cycle asks the community
health nurse when she get pregnant during her cycle. What will be the best
nursing response?

A. It is impossible to determine the fertile period reliably. So it is best to


assume that a woman is always fertile.
B. In a 28-day cycle, ovulation occurs at or about day 14. The egg lives for
about 24 hours and the sperm live for about 72 hours. The fertile period
would be approximately between day 11 and day 15.
C. In a 28- day cycle, ovulation occurs at or about day 14. The egg lives for
about 72 hours and the sperm live for about 24 hours. The fertile period
would be approximately between day 13 and 17.
D. In a 28-day cycle, ovulation occurs 8 days before the next period or at
about day 20. The fertile period is between day 20 and the beginning of
the next period.
57. Which of the following statement describes the role of a nurse as a client
advocate?

A. A nurse may override clients’ wishes for their own good.


B. A nurse has the moral obligation to prevent harm and do well for clients.
C. A nurse helps clients gain greater independence and self-determination.
D. A nurse measures the risk and benefits of various health situations
while factoring in cost.
58. A community health nurse is providing a health teaching to a woman
infected with herpes simplex 2. Which of the following health teaching must
the nurse include to reduce the chances of transmission of herpes simplex 2?

A. “Abstain from intercourse until lesions heal.”


B. “Therapy is curative.”
C. “Penicillin is the drug of choice for treatment.”
D. “The organism is associated with later development of hydatidiform
mole.
59. The nurse in the psychiatric ward informed the male client that he will be
attending the 9:00 AM group therapy sessions. The client tells the nurse that
he must wash his hands from 9:00 to 9:30 AM each day and therefore he
cannot attend. Which concept does the nursing staff need to keep in mind in
planning nursing intervention for this client?

A. Depression underlines ritualistic behavior.


B. Fear and tensions are often expressed in disguised form through
symbolic processes.
C. Ritualistic behavior makes others uncomfortable.
D. Unmet needs are discharged through ritualistic behavior.
60. The nurse assesses the health condition of the female client. The client
tells the nurse that she discovered a lump in the breast last year and hesitated
to seek medical advice. The nurse understands that, women who tend to delay
seeking medical advice after discovering the disease are displaying what
common defense mechanism?

A. Intellectualization.
B. Suppression.
C. Repression.
D. Denial.
61. Which of the following situations cannot be delegated by the registered
nurse to the nursing assistant?

A. A postoperative client who is stable needs to ambulate.


B. Client in soft restraint who is very agitated and crying.
C. A confused elderly woman who needs assistance with eating.
D. Routine temperature check that must be done for a client at end of shift.
62. In the admission care unit, which of the following client would the nurse
give immediate attention?

A. A client who is 3 days postoperative with left calf pain.


B. A client who is postoperative hip pinning who is complaining of pain.
C. New admitted client with chest pain.
D. A client with diabetes who has a glucoscan reading of 180.
63. A couple seeks medical advice in the community health care unit. A
couple has been unable to conceive; the man is being evaluated for possible
problems. The physician ordered semen analysis. Which of the following
instructions is correct regarding collection of a sperm specimen?

A. Collect a specimen at the clinic, place in iced container, and give to


laboratory personnel immediately.
B. Collect specimen after 48-72 hours of abstinence and bring to clinic
within 2 hours.
C. Collect specimen in the morning after 24 hours of abstinence and bring
to clinic immediately.
D. Collect specimen at night, refrigerate, and bring to clinic the next
morning.
64. The physician ordered Betamethasone to a pregnant woman at 34 weeks
of gestation with sign of preterm labor. The nurse expects that the drug will:

A. Treat infection.
B. Suppress labor contraction.
C. Stimulate the production of surfactant.
D. Reduce the risk of hypertension.
65. A tracheostomy cuff is to be deflated, which of the following nursing
intervention should be implemented before starting the procedures?

A. Suction the trachea and mouth.


B. Have the obdurator available.
C. Encourage deep breathing and coughing.
D. Do a pulse oximetry reading.
66. A client is diagnosed with Tuberculosis and respiratory isolation is
initiated. This means that:

A. Gloves are worn when handling the client’s tissue, excretions, and linen.
B. Both client and attending nurse must wear masks at all times.
C. Nurse and visitors must wear masks until chemotherapy is begun.
Client is instructed in cough and tissue techniques.
D. Full isolation; that is, caps and gowns are required during the period of
contagion.
67. A client with lung cancer is admitted in the nursing care unit. The husband
wants to know the condition of his wife. How should the nurse respond to the
husband?

A. Find out what information he already has.


B. Suggest that he discuss it with his wife.
C. Refer him to the doctor.
D. Refer him to the nurse in charge.
68. A hospitalized client cannot find his handkerchief and accuses other cient
in the room and the nurse of stealing them. Which is the most therapeutic
approach to this client?

A. Divert the client’s attention.


B. Listen without reinforcing the client’s belief.
C. Inject humor to defuse the intensity.
D. Logically point out that the client is jumping to conclusions.
69. After a cystectomy and formation of an ileal conduit, the nurse provides
instruction regarding prevention of leakage of the pouch and backflow of the
urine. The nurse is correct to include in the instruction to empty the urine
pouch:

A. Every 3-4 hours.


B. Every hour.
C. Twice a day.
D. Once before bedtime.
70. Which telephone call from a student’s mother should the school nurse
take care of at once?

A. A telephone call notifying the school nurse that the child’ pediatrician
has informed the mother that the child will need cardiac repair surgery
within the next few weeks.
B. A telephone call notifying the school nurse that the child’s pediatrician
has informed the mother that the child has head lice.
C. A telephone call notifying the school nurse that a child has a
temperature of 102ºF and a rash covering the trunk and upper extremities
of the body.
D. A telephone call notifying the school nurse that a child underwent an
emergency appendectomy during the previous night.
71. Which of the following signs and symptoms that require immediate
attention and may indicate most serious complications during pregnancy?

A. Severe abdominal pain or fluid discharge from the vagina.


B. Excessive saliva, “bumps around the areolae, and increased vaginal
mucus.
C. Fatigue, nausea, and urinary frequency at any time during pregnancy.
D. Ankle edema, enlarging varicosities, and heartburn.
72. The nurse is assessing the newborn boy. Apgar scores are 7 and 9. The
newborn becomes slightly cyanotic. What is the initial nursing action?

A. Elevate his head to promote gravity drainage of secretions.


B. Wrap him in another blanket, to reduce heat loss.
C. Stimulate him to cry,, to increase oxygenation.
D. Aspirate his mouth and nose with bulb syringe.
73. The nurse is formulating a plan of care to a client with a somatoform
disorder. The nurse needs to have knowledge of which psychodynamic
principle?

A. The symptoms of a somatoform disorder are an attempt to adjust to


painful life situations or to cope with conflicting sexual, aggressive, or
dependent feelings.
B. The major fundamental mechanism is regression.
C. The client’s symptoms are imaginary and the suffering is faked.
D. An extensive, prolonged study of the symptoms will be reassuring to the
client, who seeks sympathy, attention and love.
74. An infant is brought to the health care clinic for three immunizations at the
same time. The nurse knows that hepatitis B, DPT, and Haemophilus
influenzae type B immunizations should:

A. Be drawn in the same syringe and given in one injection.


B. Be mixed and inject in the same sites.
C. Not be mixed and the nurse must give three injections in three sites.
D. Be mixed and the nurse must give the injection in three sites.
75. A female client with cancer has radium implants. The nurse wants to
maintain the implants in the correct position. The nurse should position the
client:

A. Flat in bed.
B. On the side only.
C. With the foot of the bed elevated.
D. With the head elevated 45-degrees (semi-Fowler’s).
76. The nurse wants to know if the mother of a toddler understands the
instructions regarding the administration of syrup of ipecac. Which of the
following statement will help the nurse to know that the mother needs
additional teaching?

A. “I’ll give the medicine if my child gets into some toilet bowl cleaner.”
B. “I’ll give the medicine if my child gets into some aspirin.”
C. “I’ll give the medicine if my child gets into some plant bulbs.”
D. “I’ll give the medicine if my child gets into some vitamin pills.”
77. To assess if the cranial nerve VII of the client was damaged, which
changes would not be expected?

A. Drooling and drooping of the mouth.


B. Inability to open eyelids on operative side.
C. Sagging of the face on the operative side.
D. Inability to close eyelid on operative side.
78. The community health nurse makes a home visit to a family. During the
visit, the nurse observes that the mother is beating her child. What is the
priority nursing intervention in this situation?

A. Assess the child’s injuries.


B. Report the incident to protective agencies.
C. Refer the family to appropriate support group.
D. Assist the family to identify stressors and use of other coping
mechanisms to prevent further incidents.
79. The nurse in the neonatal care unit is supervising the actions of a certified
nursing assistant in giving care to the newborns. The nursing assistant
mistakenly gives a formula feeding to a newborn that is on water feeding only.
The nurse is responsible for the mistake of the nursing assistant:
A. Always, as a representative of the institution.
B. Always, because nurses who supervise less-trained individuals are
responsible for their mistakes.
C. If the nurse failed to determine whether the nursing assistant was
competent to take care of the client.
D. Only if the nurse agreed that the newborn could be fed formula.
80. The nurse is assigned to care for a client with urinary calculi. Fluid intake
of 2L/day is encouraged to the client. the primary reason for this is to:

A. Reduce the size of existing stones.


B. Prevent crystalline irritation to the ureter.
C. Reduce the size of existing stones
D. Increase the hydrostatic pressure in the urinary tract.
81. The nurse is counseling a couple in their mid 30’s who have been unable
to conceive for about 6 months. They are concerned that one or both of them
may be infertile. What is the best advice the nurse could give to the couple?

A. “it is no unusual to take 6-12 months to get pregnant, especially when


the partners are in their mid-30s. Eat well, exercise, and avoid stress.”
B. “Start planning adoption. Many couples get pregnant when they are
trying to adopt.”
C. “Consult a fertility specialist and start testing before you get any older.”
D. “Have sex as often as you can, especially around the time of ovulation,
to increase your chances of pregnancy.”
82. The nurse is caring for a cient who Is a retired nurse. A 24-hour urine
collection for Creatinine clearance is to be done. The client tells the nurse, “I
can’t remember what this test is for.” The best response by the nurse is:

A. “It provides a way to see if you are passing any protein in your urine.”
B. “It tells how well the kidneys filter wastes from the blood.”
C. “It tells if your renal insufficiency has affected your heart.”
D. “The test measures the number of particles the kidney filters.”
83. The nurse observes the female client in the psychiatric ward that she is
having a hard time sleeping at night. The nurse asks the client about it and the
client says, “I can’t sleep at night because of fear of dying.” What is the best
initial nursing response?
A. “It must be frightening for you to feel that way. Tell me more about it.”
B. “Don’t worry, you won’t die. You are just here for some test.”
C. “Why are you afraid of dying?”
D. “Try to sleep. You need the rest before tomorrow’s test.”
84. In the hospital lobby, the registered nurse overhears a two staff members
discussing about the health condition of her client. What would be the
appropriate action for the registered nurse to take?

A. Join in the conversation, giving her input about the case.


B. Ignore them, because they have the right to discuss anything they want
to.
C. Tell them it is not appropriate to discuss such things.
D. Report this incident to the nursing supervisor.
85. The client has had a right-sided cerebrovascular accident. In transferring
the client from the wheelchair to bed, in what position should a client be
placed to facilitate safe transfer?

A. Weakened (L) side of the cient next to bed.


B. Weakened (R) side of the client next to bed.
C. Weakened (L) side of the client away from bed.
D. Weakened (R) side of the cient away from bed.
86. The child client has undergone hip surgery and is in a spica cast. Which of
the following toy should be avoided to be in the child’s bed?

A. A toy gun.
B. A stuffed animal.
C. A ball.
D. Legos.
87. The LPN/LVN asks the registered nurse why oxytocin (Pitocin), 10 units
(IV or IM) must be given to a client after birth fo the fetus. The nurse is correct
to explain that oxytocin:

A. Minimizes discomfort from “afterpains.”


B. Suppresses lactation.
C. Promotes lactation.
D. Maintains uterine tone.
88. The nurse in the nursing care unit is aware that one of the medical staff
displays unlikely behaviors like confusion, agitation, lethargy and unkempt
appearance. This behavior has been reported to the nurse manager several
times, but no changes observed. The nurse should:

A. Continue to report observations of unusual behavior until the problem is


resolved.
B. Consider that the obligation to protect the patient from harm has been
met by the prior reports and do nothing further.
C. Discuss the situation with friends who are also nurses to get ideas .
D. Approach the partner of this medical staff member with these concerns.
89. The physician ordered tetracycline PO qid to a child client who weights
20kg. The recommended PO tetracycline dose is 25-50 mg/kg/day. What is
the maximum single dose that can be safely administered to this child?

A. 1 g
B. 500 mg
C. 250 mg
D. 125 mg
90. The nurse is completing an obstetric history of a woman in labor. Which
event in the obstetric history will help the nurse suspects dysfunctional labor
in the current pregnancy?

A. Total time of ruptured membranes was 24 hours with the second birth.
B. First labor lasting 24 hours.
C. Uterine fibroid noted at time of cesarean delivery.
D. Second birth by cesarean for face presentation.
91. The nurse is planning to talk to the client with an antisocial personality
disorder. What would be the most therapeutic approach?

A. Provide external controls.


B. Reinforce the client’s self-concept.
C. Give the client opportunities to test reality.
D. Gratify the client’s inner needs.
92. The nurse is teaching a group of women about fertility awareness, the
nurse should emphasize that basal body temperature:
A. Can be done with a mercury thermometer but no a digital one.
B. The average temperature taken each morning.
C. Should be recorded each morning before any activity.
D. Has a lower degree of accuracy in predicting ovulation than the cervical
mucus test.
93. The nursing applicant has given the chance to ask questions during a job
interview at a local hospital. What should be the most important question to
ask that can increase chances of securing a job offer?

A. Begin with questions about client care assignments, advancement


opportunities, and continuing education.
B. Decline to ask questions, because that is the responsibility of the
interviewer.
C. Ask as many questions about the facility as possible.
D. Clarify information regarding salary, benefits, and working hours first,
because this will help in deciding whether or not to take the job.
94. The nurse advised the pregnant woman that smoking and alcohol should
be avoided during pregnancy. The nurse takes into account that the
developing fetus is most vulnerable to environment teratogens that cause
malformation during:

A. The entire pregnancy.


B. The third trimester.
C. The first trimester.
D. The second trimester.
95. A male client tells the nurse that there is a big bug in his bed. The most
therapeutic nursing response would be:

A. Silence.
B. “Where’s the bug? I’ll kill it for you.”
C. “I don’t see a bug in your bed, but you seem afraid.”
D. “You must be seeing things.”
96. A pregnant client in late pregnancy is complaining of groin pain that
seems worse on the right side. Which of the following is the most likely cause
of it?

A. Beginning of labor.
B. Bladder infection.
C. Constipation.
D. Tension on the round ligament.
97. The nurse is conducting a lecture to a group of volunteer nurses. The
nurse is correct in imparting the idea that the Good Samaritan law protects
the nurse from a suit for malpractice when:

A. The nurse stops to render emergency aid and leaves before the
ambulance arrives.
B. The nurse acts in an emergency at his or her place of employment.
C. The nurse refuses to stop for an emergency outside of the scope of
employment.
D. The nurse is grossly negligent at the scene of an emergency.
98. A woman is hospitalized with mild preeclampsia. The nurse is formulating
a plan of care for this client, which nursing care is least likely to be done?

A. Deep-tendon reflexes once per shift.


B. Vital signs and FHR and rhythm q4h while awake.
C. Absolute bed rest.
D. Daily weight.
99. While feeding a newborn with an unrepaired cardiac defect, the nurse
keeps on assessing the condition of the client. The nurse notes that the
newborn’s respiration is 72 breaths per minute. What would be the initial
nursing action?

A. Burp the newborn.


B. Stop the feeding.
C. Continue the feeding.
D. Notify the physician.
100. A client who undergone appendectomy 3 days ago is scheduled for
discharge today. The nurse notes that the client is restless, picking at
bedclothes and saying, “I am late on my appointment,” and calling the nurse
by the wrong name. The nurse suspects:

A. Panic reaction.
B. Medication overdose.
C. Toxic reaction to an antibiotic.
D. Delirium tremens.
Answers and Rationales
1. A. The oxytocic effect of Pitocin increases the intensity and durations of
contractions; prolonged contractions will jeopardize the safetyof the fetus
and necessitate discontinuing the drug.
2. B. It is of paramount importance to prevent the client from hurting
himself or herself or others.
3. B. After tonsillectomy, clear, cool liquids should be given. Citrus,
carbonated, and hot or cold liquids should be avoided because they may
irritate the throat. Red liquids should be avoided because they give the
appearance of blood if the child vomits. Milk and milk products including
pudding are avoided because they coat the throat, cause the child to clear
the throat, and increase the risk of bleeding.
4. A. Phenylephrine, with frequent and continued use, can cause rebound
congestion of mucous membranes.
5. B. The N 95 respirator is a high-particulate filtration mask that meets the
CDC performance criteria for a tuberculosis respirator.
6. C. The most frequent cause of noncompliance to the treatment of
chronic, or open-angle glaucoma is the miotic effects of pilocarpine.
Pupillary constriction impedes normal accommodation, making night
driving difficult and hazardous, reducing the client’s ability to read for
extended periods and making participation in games with fast-moving
objects impossible.
7. B. This stops the sucking of air through the tube and prevents the entry
of contaminants. In addition, clamping near the chest wall provides for
some stability and may prevent the clamp from pulling on the chest tube.
8. D. Because umbilical cord’s insertion site is born before the fetal head,
the cord may be compressed by the after-coming head in a breech birth.
9. B. It is important to externalize the anger away from self.
10. D. Development normally proceeds cephalocaudally; so the first major
developmental milestone that the infant achieves is the ability to hold the
head up within the first 8-12 weeks of life. In hypothyroidism, the infant’s
muscle tone would be poor and the infant would not be able to achieve
this milestone.
11. D. Get a senior nurse who know s the policies, the client, and the doctor.
Generally speaking, a nurse should not accept telephone orders. However,
if it is necessary to take one, follow the hospital’s policy regarding
telephone orders. Failure to followhospital policy could be considered
negligence. In this case, the nurse was new and did not know the
hospital’s policy concerning telephone orders. The nurse was also
unfamiliar with the doctor and the client. Therefore the nurse should not
take the order unless a) no one else is available and b) it is an emergency
situation.
12. C. The nurse is obligated to inform the nurse manager about changes in
the condition of the client, which may change the decision made by the
nurse manager.
13. A. Perinatal risk factors for the development of Down syndrome include
advanced maternal age, especially with the first pregnancy.
14. B. Assignments should be based on scope of practice and expertise.
15. B. The child who is concurrently taking digoxin and diuretics is at
increased risk for digoxin toxicity due to the loss of potassium. The child
and parents should be taught what foods are high in potassium, and the
child should be encouraged to eat a high-potassium diet. In addition, the
child’s serum potassium level should be carefully monitored.
16. A. The responsible for an accurate informed consent is the physician.
An exception to this answer would be a life-threatening emergency, but
there are no data to support another response.
17. D. Asking the client to cough and take a deep breath will help determine
if the chest tube is kinked or if the lungs has reexpanded.
18. B. Every event that exposes a client to harm should be recorded in an
incident report, as well as reported to the appropriate supervisors in order
to resolve the current problems and permit the institution to prevent the
problem from happening again.
19. D. One of the earliest signs of digoxin toxicity is Bradycardia. For a
toddler, any heart rate that falls below the norm of about 100-120 bpm
would indicate Bradycardia and would necessitate holding the medication
and notifying the physician.
20. B. This option is least threatening.
21. D. In preparing the client for discharge that is receiving prednisone, the
nurse should caution the client to (a) take oral preparations after meals;
(b) remember that routine checks of vital signs, weight, and lab studies are
critical; (c) NEVER STOP OR CHANGE THE AMOUNT OF MEDICATION
WITHOUT MEDICAL ADVICE; (d) store the medication in a light-resistant
container.
22. A. Progesterone also reduces smooth muscle motility in the urinary
tract and predisposes the pregnant woman to urinary tract infections.
Women should contact their doctors if they exhibit signs of infection.
Kegel exercise will help strengthen the perineal muscles; limiting fluids at
bedtime reduces the possibility of being awakened by the necessity of
voiding.
23. B. This is the proper use of anger.
24. C. There are several models of case management, but the commonality
is comprehensive coordination of care to better predict needs of high-risk
clients, decrease exacerbations and continually monitor progress
overtime.
25. A. Phenytoin should be infused or injected into larger veins to avoid the
discoloration know as purple glove syndrome; infusing into a smaller vein
is not appropriate.
26. C. Serum radioimmunoassay (RIA) is accurate within 7days of
conception. This test is specific for HCG, and accuracy is not
compromised by confusion with LH.
27. D. Surgery and anesthesia can increase mucus production. Deep
breathing and coughing are essential to prevent atelectasis and
pneumonia in the client’s only remaining lung.
28. B. Newborns can get pneumonia (tachypnea, mild hypoxia, cough,
eosinophilia) and conjunctivitis from Chlamydia.
29. D. The client may perceive this as avoidance, but it is more important to
redirect back to the client, especially in light of the manipulative behavior
of drug abusers and adolescents.
30. C. It describes a democratic process in which all members have input in
the client’s care.
31. A. Contraction of the milk ducts and let-down reflex occur under the
stimulation of oxytocin released by the posterior pituitary gland.
32. B. In case management, the nurse assumes total responsibility for
meeting the needs of the client during the entire time on duty.
33. A. Smoke inhalation affects gas exchange.
34. C. Sperm deposited during intercourse may remain viable for about 3
days. If ovulation occurs during this period, conception may result.
35. B. This option shows acceptance (key concept) of this age-typical sleep
pattern (that of waking in the early morning).
36. D. Taking the mother’s pulse while listening to the FHR will differentiate
between the maternal and fetal heart rates and rule out fetal Bradycardia.
37. A. Antihistamines cause pupil dilation and should be avoided with
glaucoma.
38. A. This suggests that the level of consciousness is decreasing.
39. D. An advance directive is a form of informed consent, and only a
competent adult or the holder of a durable power of attorney has the right
to consent or refuse treatment. If the spouse does not hold the power of
attorney, the decisions of the holder, even if opposed by the spouse, are
enforced.
40. C. Gentle but firm guidance and nonverbal direction is needed to
intervene when a client with schizophrenic symptoms is being disruptive.
41. C. Suctioning is only done for 10 seconds, intermittently, as the catheter
is being withdrawn.
42. D. The priority for this client is being able to establish an airway.
43. A. Signs of placental separation include a change in the shape of the
uterus from ovoid to globular.
44. B. This could indicate intracranial bleeding. Alteplase is a thrombolytic
enzyme that lyses thrombi and emboli. Bleeding is an adverse effect.
Monitor clotting times and signs of any gastrointestinal or internal
bleeding.
45. D. Because flank incision in nephrectomy is directly below the
diaphragm, deep breathing is painful. Additionally, there is a greater
incisional pull each time the person moves than there is with abdominal
surgery. Incisional pain following nephrectomy generally requires
analgesics administration every 3-4 hours for 24-48 hours after surgery.
Therefore, turning, coughing and deep-breathing exercises should be
planned to maximize the analgesic effects.
46. B. Under high estrogen levels, during the period surrounding ovulation,
the cervical mucus becomes thin, clear, and elastic (spinnbarkeit),
facilitating sperm passage.
47. D. After surgery for a ruptured appendix, the client should be placed in a
semi-Fowler’s position to promote drainage and to prevent possible
complications.
48. C. Directing and evaluation of staff is a major responsibility of a nursing
manager.
49. A. The recommended procedure for administering eyedrops to any
client calls for the drops to be placed in the middle of the lower
conjunctival sac.
50. B. Thirst and restlessness indicate hypovolemia and hypoxemia.
Internal bleeding is difficult to recognized and evaluate because it is not
apparent.
51. C. Erythema toxicum is the normal, nonpathological macular newborn
rash.
52. D. The family needs to understand what brain death is before talking
about organ donation. They need time to accept the death of their family
member. An environment conducive to discussing an emotional issue is
needed.
53. A. Bending from the waist in pregnancy tends to make backache worse.
54. B. Support and limit setting decrease anxiety and provide external
control.
55. C. The stoma drainage bag is applied in the operating room. Drainage
from the ileostomy contains secretions that are rich in digestive enzymes
and highly irritating to the skin. Protection of the skin from the effects of
these enzymes is begun at once. Skin exposed to these enzymes even for
a short time becomes reddened, painful and excoriated.
56. B. It is the most accurate statement of physiological facts for a 28-day
menstrual cycle: ovulation at day 14, egg life span 24 hours, sperm life
span of 72 hours. Fertilization could occur from sperm deposited before
ovulation.
57. C. An advocate role encourage freedom of choice, includes speaking
out for the client, and supports the client’s best interests.
58. A. Abstinence will eliminate any unnecessary pain during intercourse
and will reduce the possibility of transmitting infection to one’s sexual
partner.
59. B. Anxiety is generated by group therapy at 9:00 AM. The ritualistic
behavioral defense of hand washing decreases anxiety by avoiding group
therapy.
60. D. Denial is a very strong defense mechanism used to allay the
emotional effects of discovering a potential threat. Although denial has
been found to be an effective mechanism for survival in some instances,
such as during natural disasters, it may in greater pathology in a woman
with potential breast carcinoma.
61. B. The registered nurse cannot delegate the responsibility for
assessment and evaluation of clients. The status of the client in restraint
requires further assessment to determine if there are additional causes
for the behavior.
62. C. The client with chest pain may be having a myocardial infarction, and
immediate assessment and intervention is a priority.
63. B. Is correct because semen analysis requires that a freshly
masturbated specimen be obtained after a rest (abstinence) period of 48-
72 hours.
64. C. Betamethasone, a form of cortisone, acts on the fetal lungs to
produce surfactant.
65. A. Secretions may have pooled above the tracheostomy cuff. If these
are not suctioned before deflation, the secretions may be aspirated.
66. C. Proper handling of sputum is essential to allay droplet transference
of bacilli in the air. Clients need to be taught to cover their nose and mouth
with tissues when sneezing or coughing. Chemotherapy generally renders
the client noninfectious within days to a few weeks, usually before
cultures for tubercle bacilli are negative. Until chemical isolation is
established, many institutions require the client to wear a mask when
visitors are in the room or when the nurse is in attendance. Client should
be in a well-ventilated room, without air recirculation, to prevent air
contamination.
67. A. It is best to establish baseline information first.
68. B. Listening is probably the most effective response of the four choices.
69. A. Urine flow is continuous. The pouch has an outlet valve for easy
drainage every 3-4 hours. (the pouch should be changed every 3-5 days, or
sooner if the adhesive is loose).
70. C. A high fever accompanied by a body rash could indicate that the child
has a communicable disease and would have exposed other students to
the infection. The school nurse would want to investigate this telephone
call immediately so that plans could be instituted to control the spread of
such infection.
71. A. Severe abdominal pain may indicate complications of pregnancy
such as abortion, ectopic pregnancy, or abruption placenta; fluid discharge
from the vagina may indicate premature rupture of the membrane.
72. D. Gentle aspiration of mucus helps maintain a patent airway, required
for effective gas exchange.
73. A. Somatoform disorders provide a way of coping with conflicts.
74. C. Immunization should never be mixed together in a syringe, thus
necessitating three separate injections in three sites. Note: some
manufacturers make a premixed combination of immunization that is safe
and effective.
75. A. Clients with radioactive implants should be positioned flat in bed to
prevent dislodgement of the vaginal packing. The client may roll to the
side for meals but the upper body should not be raised more than 20
degrees.
76. A. Syrup of ipecac is not administered when the ingested substances is
corrosive in nature. Toilet bowl cleaners, as a collective whole, are highly
corrosive substances. If the ingested substance “burned” the esophagus
going down, it will “burn” the esophagus coming back up when the child
begins to vomit after administration of syrup of ipecac.
77. B. Inability to open eyelids on operative side is seen with cranial nerve III
damage.
78. A. Assessment of physical injuries (like bruises, lacerations, bleeding
and fractures) is the first priority.
79. C. The nurse who is supervising others has a legal obligation to
determine that they are competent to perform the assignment, as well as
legal obligation to provide adequate supervision.
80. D. Increasing hydrostatic pressure in the urinary tract will facilitate
passage of the calculi.
81. A. Infertility is not diagnosed until atleast 12months of unprotected
intercourse has failed to produce a pregnancy. Older couples will
experience a longer time to get pregnant.
82. B. Determining how well the kidneys filter wastes states the purpose of
a Creatinine clearance test.
83. A. Acknowledging a feeling tone is the most therapeutic response and
provides a broad opening for the client to elaborate feelings.
84. C. The behavior should be stopped. The first is to remind the staff that
confidentiality maybe violated.
85. C. With a right-sided cerebrovascular accident the client would have left-
sided hemiplegia or weakness. The client’s good side should be closest to
the bed to facilitate the transfer.
86. D. Legos are small plastic building blocks that could easily slip under
the child’s cast and lead to a break in skin integrity and even infection.
Pencils, backscratchers, and marbles are some other narrow or small
items that could easily slip under the child’s cast and lead to a break in
skin integrity and infection.
87. D. Oxytocin (Pitocin) is used to maintain uterine tone.
88. B. The submission of reports about incidents that expose clients to
harm does not remove the obligation to report ongoing behavior as long
as the risk to the client continues.
89. C. The recommended dosage of tetracycline is 25-50mg/kg/day. If the
child weighs 20kg and the maximum dose is 50mg/kg, this would indicate
a total daily dose of 1000mg of tetracycline. In this case, the child is being
given this medication four times a day. Therefore the maximum single
dose that can be given is 250mg (1000 mg of tetracycline divided by four
doses.)
90. C. An abnormality in the uterine muscle could reduce the effectiveness
of uterine contractions and lengthen the duration of subsequent labors.
91. A. Personality disorders stem from a weak superego, implying a lack of
adequate controls.
92. C. The basal body temperature is the lowest body temperature of a
healthy person that is taken immediately after waking and before getting
out of bed. The BBT usually varies from 36.2 ºC to 36.3ºC during menses
and for about 5-7 days afterward. About the time of ovulation, a slight
drop in temperature may be seen, after ovulation in concert with the
increasing progesterone levels of the early luteal phase, the BBT rises 0.2-
0.4 ºC. This elevation remains until 2-3 days before menstruation, or if
pregnancy has occurred.
93. A. This choice implies concern for client care and self-improvement.
94. C. The first trimester is the period of organogenesis, that is, cell
differentiation into the various organs, tissues, and structures.
95. C. This response does not contradict the client’s perception, is honest,
and shows empathy.
96. D. Tension on round ligament occurs because of the erect human
posture and pressure exerted by the growing fetus.
97. D. The Good Samaritan Law does not impose a duty to stop at the
scene of an emergency outside of the scope of employment, therefore
nurses who do not stop are not liable for suit.
98. C. Although reducing environment stimuli and activity is necessary for a
woman with mild preeclampsia, she will most probably have bathroom
privileges.
99. B.  A normal respiratory rate for a newborn is 30-40 breaths per minute.
100. D. The behavior described is likely to be symptoms of delirium tremens,
or alcohol withdrawal (often unsuspected on a surgical unit.)

PNLE: Pediatric Nursing Exam


1. Which of the following is characterized the rate of growth during this
period?

A. most rapid period of growth


B. a decline in growth rate
C. growth spurt
D. slow uniform growth rate
2. In assessing Raphael’s growth and development, the nurse is guided by
principles of growth and development. Which is not included?

A. All individuals follow cephalo-caudal and proximo-distal


B. Different parts of the body grows at different rate
C. All individual follow standard growth rate
D. Rate and pattern of growth can be modified
3. What type of play will be ideal for Raphael at this period?

A. Make believe
B. Hide and seek
C. Peek-a-boo
D. Building blocks
4. Which of the following information indicate that Raphael is normal for his
age?

A. Determine own sense self


B. Develop sense of whether he can trust the world
C. Has the ability to try new things
D. Learn basic skills within his culture
5. Based on Kohlberg’s theory, what is the stage of moral development of
Raphael?

A. Punishment-obedience
B. “good boy-Nice girl”
C. naïve instrumental orientation
D. social contact
Situation 2 Baby boy Lacson delivered at 36 weeks gestation weighs
3,400 gm and height of 59 cm (6-10)
6. Baby boy Lacson’s height is

A. Long
B. Short
C. Average
D. Too short
7. Growth and development in a child progresses in the following ways
EXCEPT

A. From cognitive to psychosexual


B. From trunk to the tip of the extremities
C. From head to toe
D. From general to specific
8. As described by Erikson, the major psychosexual conflict of the above
situation is

A. Autonomy vs. Shame and doubt


B. Industry vs. Inferiority
C. Trust vs. mistrust
D. Initiation vs. guilt
9. Which of the following is true about Mongolian Spots?

A. Disappears in about a year


B. Are linked to pathologic conditions
C. Are managed by tropical steroids
D. Are indicative of parental abuse
10. Signs of cold stress that the nurse must be alert when caring for a
Newborn is:

A. Hypothermia
B. Decreased activity level
C. Shaking
D. Increased RR
Situation 3 Nursing care after delivery has an important aspect in
every stages of delivery
11. After the baby is delivered, the cord was cut between two clamps using a
sterile scissors and blade, then the baby is placed at the:

A. Mother’s breast
B. Mother’s side
C. Give it to the grandmother
D. Baby’s own mat or bed
12. The baby’s mother is RH(-). Which of the following laboratory tests will
probably be ordered for the newborn?

A. Direct Coomb’s
B. Indirect Coomb’s
C. Blood culture
D. Platelet count
13. Hypothermia is common in newborn because of their inability to control
heat. The following would be an appropriate nursing intervention to prevent
heat loss except:

A. Place the crib beside the wall


B. Doing Kangaroo care
C. By using mechanical pressure
D. Drying and wrapping the baby
14. The following conditions are caused by cold stress except

A. Hypoglycemia
B. Increase ICP
C. Metabolic acidosis
D. Cerebral palsy
15. During the feto-placental circulation, the shunt between two atria is called

A. Ductus venosous
B. Foramen Magnum
C. Ductus arteriosus
D. Foramen Ovale
16. What would cause the closure of the Foramen ovale after the baby had
been delivered?

A. Decreased blood flow


B. Shifting of pressures from right side to the left side of the heart
C. Increased PO2
D. Increased in oxygen saturation
17. Failure of the Foramen Ovale to close will cause what Congenital Heart
Disease?

A. Total anomalous Pulmunary Artery


B. Atrial Septal defect
C. Transposition of great arteries
D. Pulmunary Stenosis
Situation 4 Children are vulnerable to some minor health problems or
injuries hence the nurse should be able to teach mothers to give
appropriate home care.
18. A mother brought her child to the clinic with nose bleeding. The nurse
showed the mother the most appropriate position for the child which is:

A. Sitting up
B. With low back rest
C. With moderate back rest
D. Lying semi flat
19. A common problem in children is the inflammation of the middle ear. This
is related to the malfunctioning of the:

A. Tympanic membrane
B. Eustachian tube
C. Adenoid
D. Nasopharynx
20. For acute otitis media, the treatment is prompt antibiotic therapy. Delayed
treatment may result in complications of:

A. Tonsillitis
B. Eardrum Problems
C. Brain damage
D. Diabetes mellitus
21. When assessing gross motor development in a 3 year old, which of the
following activities would the nurse expect to finds?

A. Riding a tricycle
B. Hopping on one foot
C. Catching a ball
D. Skipping on alternate foot.
22. When assessing the weight of a 5-month old, which of the following
indicates healthy growth?

A. Doubling of birth weight


B. Tripling of birth weight
C. Quadrupling of birth weight
D. Stabilizing of birth weight
23. An appropriate toy for a 4 year old child is:

A. Push-pull toys
B. Card games
C. Doctor and nurse kits
D. Books and Crafts
24. Which of the following statements would the nurse expects a 5-year old
boy to say whose pet gerbil just died

A. “The boogieman got him”


B. “He’s just a bit dead”
C. “Ill be good from now own so I wont die like my gerbil”
D. “Did you hear the joke about…”
25. When assessing the fluid and electrolyte balance in an infant, which of the
following would be important to remember?

A. Infant can concentrate urine at an adult level


B. The metabolic rate of an infant is slower than in adults
C. Infants have more intracellular water that adult do
D. Infant have greater body surface area than adults
26. When assessing a child with aspirin overdose, which of the following will
be expected?

A. Metabolic alkalosis
B. Respiratory alkalosis
C. Metabolic acidosis
D. Respiratory acidosis
27. Which of the following is not a possible systemic clinical manifestation of
severe burns?

A. Growth retardation
B. Hypermetabolism
C. Sepsis
D. Blisters and edema
28. When assessing a family for potential child abuse risks, the nurse would
observe for which of the following?

A. Periodic exposure to stress


B. Low socio-economic status
C. High level of self esteem
D. Problematic pregnancies
29. Which of the following is a possible indicator of Munchausen syndrome by
proxy type of child abuse?

A. Bruises found at odd locations, with different stages of healing


B. STD’s and genital discharges
C. Unexplained symptoms of diarrhea, vomiting and apnea with no organic
basis
D. Constant hunger and poor hygiene
30. Which of the following is an inappropriate interventions when caring for a
child with HIV?

A. Teaching family about disease transmission


B. Offering large amount of fresh fruits and vegetables
C. Encouraging child to perform at optimal level
D. Teach proper hand washing technique
Situation 5 Agata, 2 years old is rushed to the ER due to cyanosis
precipitated by crying. Her mother observed that after playing she gets
tired. She was diagnosed with Tetralogy of Fallot.
31. The goal of nursing care fro Agata is to:

A. Prevent infection
B. Promote normal growth and development
C. Decrease hypoxic spells
D. Hydrate adequately
32. The immediate nursing intervention for cyanosis of Agata is:

A. Call up the pediatrician


B. Place her in knee chest position
C. Administer oxygen inhalation
D. Transfer her to the PICU
33. Agata was scheduled for a palliative surgery, which creates anastomosis
of the subclavian artery to the pulmonary artery. This procedure is:

A. Waterston-Cooley
B. Raskkind Procedure
C. Coronary artery bypass
D. Blalock-Taussig
34. Which of the following is not an indicator that Agata experiences
separation anxiety brought about her hospitalization?

A. Friendly with the nurse


B. Prolonged loud crying, consoled only by mother
C. Occasional temper tantrums and always says NO
D. Repeatedly verbalizes desire to go home
35. When Agata was brought to the OR, her parents where crying. What would
be the most appropriate nursing diagnosis?

A. Infective family coping r/t situational crisis


B. Anxiety r/t powerlessness
C. Fear r/t uncertain prognosis
D. Anticipatory grieving r/t gravity of child’s physical status
36. Which of the following respiratory condition is always considered a
medical emergency?

A. Laryngeotracheobronchitis (LTB)
B. Epiglottitis
C. Asthma
D. Cystic Fibrosis
37. Which of the following statements by the family of a child with asthma
indicates a need for additional teaching?

A. “We need to identify what things triggers his attacks”


B. “He is to use bronchodilator inhaler before steroid inhaler”
C. “We’ll make sure he avoids exercise to prevent asthma attacks”
D. “he should increase his fluid intake regularly to thin secretions”
38. Which of the following would require careful monitoring in the child with
ADHD who is receiving Methylphenidate (Ritalin)?

A. Dental health
B. Mouth dryness
C. Height and weight
D. Excessive appetite
Situation 6 Laura is assigned as the Team Leader during the
immunization day at the RHU
39. What program for the DOH is launched at 1976 in cooperation with WHO
and UNICEF to reduce morbidity and mortality among infants caused by
immunizable disease?

A. Patak day
B. Immunization day on Wednesday
C. Expanded program on immunization
D. Bakuna ng kabtaan
40. One important principle of the immunization program is based on?

A. Statistical occurrence
B. Epidemiologic situation
C. Cold chain management
D. Surveillance study
41. The main element of immunization program is one of the following?

A. Information, education and communication


B. Assessment and evaluation of the program
C. Research studies
D. Target setting
42. What does herd immunity means?

A. Interruption of transmission
B. All to be vaccinated
C. Selected group for vaccination
D. Shorter incubation
43. Measles vaccine can be given simultaneously. What is the combined
vaccine to be given to children starting at 15 months?

A. MCG
B. MMR
C. BCG
D. BBR
Situation 7: Braguda brought her 5-month old daughter in the nearest
RHU because her baby sleeps most of the time, with decreased
appetite, has colds and fever for more than a week. The physician
diagnosed pneumonia.
44. Based on this data given by Braguda, you can classify Braguda’s daughter
to have:

A. Pneumonia: cough and colds


B. Severe pneumonia
C. Very severe pneumonia
D. Pneumonia moderate
45. For a 3-month old child to be classified to have Pneumonia (not severe),
you would expect to find RR of:

A. 60 bpm
B. 40 bpm
C. 70 bpm
D. 50 pbm
46. You asked Braguda if her baby received all vaccines under EPI. What legal
basis is used in implementing the UN’s goal on Universal Child Immunization?

A. PD no. 996
B. PD no. 6
C. PD no. 46
D. RA 9173
47. Braguda asks you about Vitamin A supplementation. You responded that
giving Vitamin A starts when the infant reaches 6 months and the first dose
is”
A. 200,000 “IU”
B. 100,000 “IU”
C. 500,000 “IU”
D. 10,000 “IU”
48. As part of CARI program, assessment of the child is your main
responsibility. You could ask the following question to the mother except:

A. “How old is the child?”


B. “IS the child coughing? For how long?”
C. “Did the child have chest indrawing?”
D. “Did the child have fever? For how long?”
49. A newborn’s failure to pass meconium within 24 hours after birth may
indicate which of the following?

A. Aganglionic Mega colon


B. Celiac disease
C. Intussusception
D. Abdominal wall defect
50. The nurse understands that a good snack for a 2 year old with a diagnosis
of acute asthma would be:

A. Grapes
B. Apple slices
C. A glass of milk
D. A glass of cola
51. Which of the following immunizations would the nurse expect to
administer to a child who is HIV (+) and severely immunocomromised?

A. Varicella
B. Rotavirus
C. MMR
D. IPV
52. When assessing a newborn for developmental dysplasia of the hip, the
nurse would expect to assess which of the following?

A. Symmetrical gluteal folds


B. Trendelemburg sign
C. Ortolani’s sign
D. Characteristic limp
53. While assessing a male neonate whose mother desires him to be
circumcised, the nurse observes that the neonate’s urinary meatus appears to
be located on the ventral surface of the penis. The physician is notified
because the nurse would suspect which of the following?

A. Phimosis
B. Hydrocele
C. Epispadias
D. Hypospadias
54. When teaching a group of parents about seat belt use, when would the
nurse state that the child be safely restrained in a regular automobile
seatbelt?

A. 30 lb and 30 in
B. 35 lb and 3 y/o
C. 40 lb and 40 in
D. 60 lb and 6 y/o
55. When assessing a newborn with cleft lip, the nurse would be alert which of
the following will most likely be compromised?

A. Sucking ability
B. Respiratory status
C. Locomotion
D. GI function
56. For a child with recurring nephritic syndrome, which of the following areas
of potential disturbances should be a prime consideration when planning
ongoing nursing care?

A. Muscle coordination
B. Sexual maturation
C. Intellectual development
D. Body image
57. An inborn error of metabolism that causes premature destruction of RBC?

A. G6PD
B. Hemocystinuria
C. Phenylketonuria
D. Celiac Disease
58. Which of the following would be a diagnostic test for Phenylketonuria
which uses fresh urine mixed with ferric chloride?

A. Guthrie Test
B. Phenestix test
C. Beutler’s test
D. Coomb’s test
59. Dietary restriction in a child who has Hemocystenuria will include which of
the following amino acid?

A. Lysine
B. Methionine
C. Isolensine tryptophase
D. Valine
60. A milk formula that you can suggest for a child with Galactosemia:

A. Lofenalac
B. Lactum
C. Neutramigen
D. Sustagen
Answers and Rationales
1. B. a decline in growth rate.  During the Preschooler stage growth is very
minimal. Weight gain is only 4.5lbs (2kgs) per year and Height is 3.5in (6-
8cm) per year.
 Review:
 Most rapid growth and development- Infancy
 Slow growth- Toddler hood and Preschooler
 Slower growth- School age
 Rapid growth- Adolescence
2. D. Rate and pattern of growth can be modified. Growth and development
occurs in cephalo-caudal meaning development occurs through out the
body’s axis. Example: the child must be able to lift the head before he is
able to lift his chest. Proximo-distal is development that progresses from
center of the body to the extremities. Example: a child first develops arm
movement before fine-finger movement. Different parts of the body grows
at different range because some body tissue mature faster than the other
such as the neurologic tissues peaks its growth during the first years of
life while the genital tissue doesn’t till puberty. Also G&D is predictable in
the sequence which a child normally precedes such as motor skills and
behavior. Lastly G&D can never be modified .
3. A. Make believe. Make believe is most appropriate because it enhances
the imitative play and imagination of the preschooler. C and D are for
infants while letter A is B is recommended for schoolers because it
enhances competitive play.
4. C. Has the ability to try new things.  Erickson defines the developmental
task of a preschool period is learning Initiative vs. Guilt. Children can
initiate motor activities of various sorts on their own and no longer
responds to or imitate the actions of other children or of their parents. 
5. C. naïve instrumental orientation. According to Kohlber, a preschooler is
under Pre-conventional where a child learns about instrumental purpose
and exchange, that is they will something do for another if that that person
does something with the child in return. Letter A is applicable for Toddlers
and letter B is for a School age child.
6. A. Long. The average length of full-term babies at birth is 20 in. (51 cm),
although the normal range is 46 cm (18 in.) to 56 cm (22 in.).
7. A. From cognitive to psychosexual. Growth and development occurs in
cephalo-caudal (head to toe), proximo-distal (trunk to tips of the
extremities and general to specific, but it doesn’t occurs in cognitive to
psychosexual because they can develop at the same time.
8. C. Trust vs. mistrust. According to Erikson, children 0-18 months are
under the developmental task of Trust vs. Mistrust.
9. A. Disappears in about a year. Mongolian spots are stale grey or bluish
patches of discoloration commonly seen across the sacrum or buttocks
due to accumulation of melanocytes and they disappears in 1 year. They
are not linked to steroid use and pathologic conditions.
10. D. Increased RR. Hypothermia is inaccurate cause normally, temperature
of a newborn drop, Also a child under cold stress will kick and cry to
increase the metabolic rate thereby increasing heat so B isn’t a good
choice. A newborn doesn’t have the ability to shiver, so letter B and C is
wrong. A newborn will increase its RR because the NB will need more
oxygen because of too much activity.
11. A. Mother’s breast. Place it at the mother’s breast for latch-on. (Note: for
NSD breast feed ASAP while for CS delivery, breast feed after 4 hours)
12. A. Direct Coomb’s. Coomb’s test is the test to determine if RH antibodies
are present. Indirect Coomb’s is done to the mother and Direct Coomb’s is
the one don’t to the baby. Blood culture and Platelet count doesn’t help
detect RH antibodies.
13. A. Place the crib beside the wall. Placing the crib beside the wall is
inappropriate because it can provide heat loss by radiation. Doing
Kangaroo care or hugging the baby, mechanical pressure or incubators
and drying and wrapping the baby will help conserve heat.
14. B. Increase ICP. Hypoglycemia may occur due to increase metabolic rate,
and because of newborns are born slightly acidic, and they catabolize
brownfat which will produce ketones which is an acid will cause metabolic
acidosis. Also a NB with severe hypothermia is in high risk for kernicterus
(too much bilirubin in the brain) can lead to Cerebral palsy. There is no
connection in the increase of ICP with hypothermia. (NOTE: pathognomonic
sign of Kernicterus in adult- asterexis, or involuntary flapping of the hand.)
15. D. Foramen Ovale. Foramen ovale is opening between two atria, Ductus
venosus is the shunt from liver to the inferior vena cava, and your Ductus
Arteriosus is the shunt from the pulmonary artery to the aorta.
16. B. Shifting of pressures from right side to the left side of the heart. During
feto-placental circulation, the pressure in the heart is much higher in the
right side, but once breathing/crying is established, the pressure will shift
from the R to the L side, and will facilitate the closure of Foramen Ovale.
(Note: that is why you should position the NB in R side lying position to
increase pressure in the L side of the heart.)
 Review:
 Increase PO2-> closure of ductus arteriosus
 Decreased bloodflow -> closure of the ductus venosus
 Circulation in the lungs is initiated by -> lung expansion and
pulmonary ventilation
 What will sustain 1st breath-> decreased artery pressure
 What will complete circulation-> cutting of the cord
17. B. Atrial Septal defect. Foramen ovale is the opening between two Atria
so, if its will not close Atrial Septal defect can occur.
18. A. Sitting up. The correct position is making the child having an upright
sitting position with the head slightly tilted forward. This position will
minimize the amount of blood pressure in nasal vessels and keep blood
moving forward not back into the nasopharynx, which will have the
choking sensation and increase risk of aspiration. Choices b, c, d, are
inappropriate cause they can cause blood to enter the nasopharynx.
19. B. Eustachian tube. This is because children has short, horizontal
Eustachian tubes. The dysfunction in the Eustachian tube enables
bacterial invasion of the middle ear and obstructs drainage of secretions.
20. C. Brain damage. One of the complication of recurring acute otitis media
is risk for having Meningitis, thereby causing possible brain damage. That
is why patient must follow a complete treatment regimen and follow up
care. A,B and D are not complications of AOM.
21. A. Riding a tricycle. Answer is A, riding a tricycle is appropriate for a 3 y/o
child. Hopping on one foot can be done by a 4 y/o child, as well as
catching and throwing a ball over hand. Skipping can be done by a 5 y/o.
22. A. Doubling of birth weight. During the first 6 months of life the weight
from birth will be doubled and as soon as the baby reaches 1 year, its birth
weight is tripled.
23. C. Doctor and nurse kits. Letter C is appropriate because it will enhance
the creativity and imagination of a pre-school child. Letter B and D are
inappropriate because they are too complex for a 4 y/o. Push-pull toys are
recommended for infants.
24. B. “He’s just a bit dead”. A 5 y/o views death in “degrees”, so the child
most likely will say that “he is just a bit dead”. Personification of death like
boogeyman occurs in ages 7 to 9 as well as denying death can if they will
be good. Denying death using jokes and attributing life qualities to death
occurs during age 3-5.
25. D. Infant have greater body surface area than adults. Infants have greater
body surface area than adult, increasing their risk to F&E imbalances. Also
infants cant concentrate a urine at an adult level and their metabolic rate,
also called water turnover, is 2 to 3 times higher than adult. Plus more
fluids of the infants are at the ECF spaces not in the ICF spaces.
26. C. Metabolic acidosis. Remember that Aspirin is acid (Acetylsalicylic
ACID).
27. D. Blisters and edema. The question was asking for a SYSTEMIC clinical
manifestation, Letters A,B and C are systemic manifestations while
Blisters and Edema weren’t.
28. D. Problematic pregnancies. Typical factors that may be risk for Child
abuse are problematic pregnancies, chronic exposure to stress not
periodic, low level of self esteem not high level. Also child abuse can
happen in all socio-economic status not just on low socio-economic
status.
29. C. Unexplained symptoms of diarrhea, vomiting and apnea with no organic
basis. Munchausen syndrome by Proxy is the fabrication or inducement of
an illness by one person to another person, usually mother to child. It is
characterized by symptoms such as apnea and siezures, which may be
due to suffocation, drugs or poisoning, vomiting which can be induced
with poisons and diarrhea with the use of laxatives. Letter A can be seen
in a Physical abuse, Letter B for sexual abuse and Letter C is for Physical
Neglect.
30. B. Offering large amount of fresh fruits and vegetables. A child with HIV is
immunocompromised. Fresh fruits and vegetables, which may be
contaminated with organisms and pesticides can be harmful, if not fatal to
the child, therefore these items should be avoided.
31. C. Decrease hypoxic spells. The correct answer is letter C. Though letter B
would be a good answer too, this goal is too vague and not specific.
Nursing interventions will not solely promote normal G&D unless he will
undergo surgical repair. So decreasing Hypoxic Spells is more SMART.
Letter A and D are inappropriate.
32. B. Place her in knee chest position. The immediate intervention would be to
place her on knee-chest or “squatting” position because it traps blood into
the lower extremities. Though also letter C would be a good choice but the
question is asking for “Immediate” so letter B is more appropriate. Letter A
and D are incorrect because its normal for a child who have ToF to have
hypoxic or “tets” spells so there is no need to transfer her to the NICU or to
alert the Pediatrician.
33. D. Blalock-Taussig. Blalock-Taussig procedure its just a temporary or
palliative surgery which creates a shunt between the aorta and pulmonary
artery so that the blood can leave the aorta and enter the pulmonary artery
and thus oxygenating the lungs and return to the left side of the heart, then
to the aorta then to the body. This procedure also makes use of the
subclavian vein so pulse is not palpable at the right arm. The full repair for
ToF is called the Brock procedure. Raskkind is a palliative surgery for TOGA.
34. A. Friendly with the nurse. Because toddlers views hospitalization is
abandonment, separation anxiety is common. Its has 3 phases: PDD
(parang c puff daddy LOL) 1. Protest 2. despair 3. detachment (or denial).
Choices B, C, D are usually seen in a child with separation anxiety (usually
in the protest stage).
 REVIEW:
 Separation anxiety begin at: 9 months
 Peaks: 18 months
35. D. Anticipatory grieving r/t gravity of child’s physical status. In this item
letter A and be are inappropriate response so remove them. The possible
answers are C and D. Fear defined as the perceived threat (real or
imagined) that is consciously recognized as danger (NANDA) is applicable
in the situation but its defining characteristics are not applicable. Crying
per se can not be a subjective cue to signify fear, and most of the
symptoms of fear in NANDA are physiological. Anticipatory grieving on the
other hand are intellectual and EMOTIONAL responses based on a
potential loss. And remember that procedures like this cannot assure total
recovery. So letter D is a more appropriate Nursing diagnosis.
36. B. Epiglottitis. Acute and sever inflammation of the epiglottis can cause
life threatening airway obstruction, that is why its always treated as a
medical emergency. NSG intervention : Prepare tracheostomy set at bed
side. LTB, can also cause airway obstruction but its not an emergency.
Asthma is also not an emergency. CF is a chronic disease, so its not a
medical emergency.
37. C. “We’ll make sure he avoids exercise to prevent asthma attacks”. Asthmatic
children don’t have to avoid exercise. They can participate on physical
activities as tolerated. Using a bronchodilator before administering
steroids is correct because steroids are just anti-inflammatory and they
don’t have effects on the dilation of the bronchioles. OF course letters A
and B are obviously correct.
38. C. Height and weight. Dental problems are more likely to occur in children
under going TCA therapy. Mouth dryness is a expected side effects of
Ritalin since it activates the SNS. Also loss of appetite is more likely to
happen, not increase in appetite. The correct answer is letter C, because
Ritalin can affect the child’s G&D. Intervention: medication “holidays or
vacation”. (This means during weekends or holidays or school vacations,
where the child wont be in school, the drug can be withheld.)
39. C. Expanded program on immunization
40. B. Epidemiologic situation. Letters A, C and D are not included in the
principles of EPI.
41. D. Target setting
42. A. Interruption of transmission
43. B. MMR. MMR or Measles, Mumps, Rubella is a vaccine furnished in
one vial and is routinely given in one injection (Sub-Q). It can be given at
15 months but can also be given as early as 12th month.
44. B. Severe pneumonia. For a child aging 2months up to 5 years old can be
classified to have sever pneumonia when he have any of the following
danger signs:
 Not able to drink
 Convulsions
 Abnormally sleepy or difficult to wake
 Stridor in calm child or
 Severe under-nutrition
45. D. 50 pbm. A child can be classified to have Pneumonia (not severe) if:
 the young infant is less than 2 months- 60 bpm or more
 if the child is 2 months up to less than 12 months- 50 bpm or
more
 if the child is 12 months to 4 y/o- 40 bpm or more
46. B. PD no. 6 Presidential Proclamation no. 6 (April 3, 1986) is the
“Implementing a United Nations goal on Universal Child Immunization by
1990”. PD 996 (September 16, 1976) is “providing for compulsory basic
immunization for infants and children below 8 years of age. PD no. 46
(September 16, 1992) is the “Reaffirming the commitment of the
Philippines to the universal Child and Mother goal of the World Health
Assembly. RA 9173 is of course the “Nursing act of 2002”
47. B. 100,000 “IU”. An infant aging 6-11 months will be given Vitamin
supplementation of 100, 000 IU and for Preschoolers ages 12-83 months
200,000 “IU” will be given.
48. C. “Did the child have chest indrawing?”. The CARI program of the DOH
includes the “ASK” and “LOOK, LISTEN” as part of the assessment of the
child who has suspected Pneumonia. Choices A, B and D are included in
the “ASK” assessment while Chest indrawings is included in the “LOOK,
LISTEN” and should not be asked to the mother.
49. A. Aganglionic Mega colon. Failure to pass meconium of Newborn during
the first 24 hours of life may indicate Hirschsprung disease or Congenital
Aganglionic Megacolon, an anomaly resulting in mechanical obstruction
due to inadequate motility in an intestinal segment. B, C, and D are not
associated in the failure to pass meconium of the newborn.
50. B. Apple slices. Grapes is in appropriate because of its “balat” that can
cause choking. A glass of milk is not a good snack because it’s the most
common cause of Iron-deficiency anemia in children (milk contains few
iron), A glass of cola is also not appropriate cause it contains complex
sugar. (walang kinalaman ang asthma dahil ala naman itong diatery
restricted foods na nasa choices.)
51. D. IPV. IPV or Inactivated polio vaccine does not contain live micro
organisms which can be harmful to an immunocompromised child. Unlike
OPV, IPV is administered via IM route.
52. C. Ortolani’s sign. Correct answer is Ortolani’s sign; it is the abnormal
clicking sound when the hips are abducted. The sound is produced when
the femoral head enters the acetabulum. Letter A is wrong because its
should be “asymmetrical gluteal fold”. Letter B and C are not applicable for
newborns because they are seen in older children.
53. D. Hypospadias. Hypospadias is a c condition in which the urethral
opening is located below the glans penis or anywhere along the ventral
surface of the penile shaft. Epispadias, the urethral meatus is located at
the dorsal surface of the penile shaft. (Para di ka malilto, I-alphabetesize
mo Dorsal, (Above) eh mauuna sa Ventral (Below) , Epis mauuna sa
Hypo.)
54. C. 40 lb and 40 in. Basta tandaan ang rule of 4! 4 years old, 40 lbs and 40
in.
55. A. Sucking ability. Because of the defect, the child will be unable to form
the mouth adequately arounf the nipple thereby requiring special devices
to allow feeding and sucking gratification. Respiratory status may be
compromised when the child is fed improperly or during post op period.
56. D. Body image. Because of edema, associated with nephroitic syndrome,
potential self concept and body image disturbance related to changes in
appearance and social isolation should be considered.
57. A. G6PD. G6PD is the premature destruction of RBC when the blood is
exposed to antioxidants, ASA (ano un? Aspirin), legumes and flava beans.
58. B. Phenestix test. Phenestix test is a diagnostic test which uses a fresh
urine sample (diapers) and mixed with ferric chloride. If positive, there will
be a presence of green spots at the diapers. Guthrie test is another test for
PKU and is the one that mostly used. The specimen used is the blood and
it tests if CHON is converted to amino acid.
59. B. Methionine. Hemocystenuria is the elevated excretion of the amino
acid hemocystiene, and there is inability to convert the amino acid
methionine or cystiene. So dietary restriction of this amino acids is
advised. This disease can lead to mental retardation.
60. C. Neutramigen. Neutramien is suggested for a child with Galactosemia.
Lofenalac is suggested for a child with PKU.

PNLE: Community Health Nursing


Exam 1
1. Which is the primary goal of community health nursing?

A. To support and supplement the efforts of the medical profession in the


promotion of health and prevention of illness
B. To enhance the capacity of individuals, families and communities to
cope with their health needs
C. To increase the productivity of the people by providing them with
services that will increase their level of health
D. To contribute to national development through promotion of family
welfare, focusing particularly on mothers and children.
2. CHN is a community-based practice. Which best explains this statement?

A. The service is provided in the natural environment of people.


B. The nurse has to conduct community diagnosis to determine nursing
needs and problems.
C. The services are based on the available resources within the
community.
D. Priority setting is based on the magnitude of the health problems
identified.
3. Population-focused nursing practice requires which of the following
processes?

A. Community organizing
B. Nursing process
C. Community diagnosis
D. Epidemiologic process
4. R.A. 1054 is also known as the Occupational Health Act. Aside from
number of employees, what other factor must be considered in determining
the occupational health privileges to which the workers will be entitled?

A. Type of occupation: agricultural, commercial, industrial


B. Location of the workplace in relation to health facilities
C. Classification of the business enterprise based on net profit
D. Sex and age composition of employees
5. A business firm must employ an occupational health nurse when it has at
least how many employees?

A. 21
B. 101
C. 201
D. 301
6. When the occupational health nurse employs ergonomic principles, she is
performing which of her roles?

A. Health care provider


B. Health educator
C. Health care coordinator
D. Environmental manager
7. A garment factory does not have an occupational nurse. Who shall provide
the occupational health needs of the factory workers?

A. Occupational health nurse at the Provincial Health Office


B. Physician employed by the factory
C. Public health nurse of the RHU of their municipality
D. Rural sanitary inspector of the RHU of their municipality
8. “Public health services are given free of charge.” Is this statement true or
false?

A. The statement is true; it is the responsibility of government to provide


basic services.
B. The statement is false; people pay indirectly for public health services.
C. The statement may be true or false, depending on the specific service
required.
D. The statement may be true or false, depending on policies of the
government concerned.
9. According to C.E.Winslow, which of the following is the goal of Public
Health?

A. For people to attain their birthrights of health and longevity


B. For promotion of health and prevention of disease
C. For people to have access to basic health services
D. For people to be organized in their health efforts
10. We say that a Filipino has attained longevity when he is able to reach the
average lifespan of Filipinos. What other statistic may be used to determine
attainment of longevity?

A. Age-specific mortality rate


B. Proportionate mortality rate
C. Swaroop’s index
D. Case fatality rate
11. Which of the following is the most prominent feature of public health
nursing?

A. It involves providing home care to sick people who are not confined in
the hospital.
B. Services are provided free of charge to people within the catchment
area.
C. The public health nurse functions as part of a team providing a public
health nursing services.
D. Public health nursing focuses on preventive, not curative, services.
12. According to Margaret Shetland, the philosophy of public health nursing is
based on which of the following?

A. Health and longevity as birthrights


B. The mandate of the state to protect the birthrights of its citizens
C. Public health nursing as a specialized field of nursing
D. The worth and dignity of man
13. Which of the following is the mission of the Department of Health?

A. Health for all Filipinos


B. Ensure the accessibility and quality of health care
C. Improve the general health status of the population
D. Health in the hands of the Filipino people by the year 2020
14. Region IV Hospital is classified as what level of facility?

A. Primary
B. Secondary
C. Intermediate
D. Tertiary
15. Which is true of primary facilities?

A. They are usually government-run.


B. Their services are provided on an out-patient basis.
C. They are training facilities for health professionals.
D. A community hospital is an example of this level of health facilities.
16. Which is an example of the school nurse’s health care provider functions?

A. Requesting for BCG from the RHU for school entrant immunization
B. Conducting random classroom inspection during a measles epidemic
C. Taking remedial action on an accident hazard in the school playground
D. Observing places in the school where pupils spend their free time
17. When the nurse determines whether resources were maximized in
implementing Ligtas Tigdas, she is evaluating

A. Effectiveness
B. Efficiency
C. Adequacy
D. Appropriateness
18. You are a new B.S.N. graduate. You want to become a Public Health
Nurse. Where will you apply?

A. Department of Health
B. Provincial Health Office
C. Regional Health Office
D. Rural Health Unit
19. R.A. 7160 mandates devolution of basic services from the national
government to local government units. Which of the following is the major
goal of devolution?

A. To strengthen local government units


B. To allow greater autonomy to local government units
C. To empower the people and promote their self-reliance
D. To make basic services more accessible to the people
20. Who is the Chairman of the Municipal Health Board?

A. Mayor
B. Municipal Health Officer
C. Public Health Nurse
D. Any qualified physician
21. Which level of health facility is the usual point of entry of a client into the
health care delivery system?
A. Primary
B. Secondary
C. Intermediate
D. Tertiary
22. The public health nurse is the supervisor of rural health midwives. Which
of the following is a supervisory function of the public health nurse?

A. Referring cases or patients to the midwife


B. Providing technical guidance to the midwife
C. Providing nursing care to cases referred by the midwife
D. Formulating and implementing training programs for midwives
23. One of the participants in a hilot training class asked you to whom she
should refer a patient in labor who develops a complication. You will answer,
to the

A. Public Health Nurse


B. Rural Health Midwife
C. Municipal Health Officer
D. Any of these health professionals
24. You are the public health nurse in a municipality with a total population of
about 20,000. There are 3 rural health midwives among the RHU personnel.
How many more midwife items will the RHU need?

A. 1
B. 2
C. 3
D. The RHU does not need any more midwife item.
25. If the RHU needs additional midwife items, you will submit the request for
additional midwife items for approval to the

A. Rural Health Unit


B. District Health Office
C. Provincial Health Office
D. Municipal Health Board
26. As an epidemiologist, the nurse is responsible for reporting cases of
notifiable diseases. What law mandates reporting of cases of notifiable
diseases?
A. Act 3573
B. R.A. 3753
C. R.A. 1054
D. R.A. 1082
27. According to Freeman and Heinrich, community health nursing is a
developmental service. Which of the following best illustrates this statement?

A. The community health nurse continuously develops himself personally


and professionally.
B. Health education and community organizing are necessary in providing
community health services.
C. Community health nursing is intended primarily for health promotion
and prevention and treatment of disease.
D. The goal of community health nursing is to provide nursing services to
people in their own places of residence.
28. Which disease was declared through Presidential Proclamation No. 4 as a
target for eradication in the Philippines?

A. Poliomyelitis
B. Measles
C. Rabies
D. Neonatal tetanus
29. The public health nurse is responsible for presenting the municipal health
statistics using graphs and tables. To compare the frequency of the leading
causes of mortality in the municipality, which graph will you prepare?

A. Line
B. Bar
C. Pie
D. Scatter diagram
30. Which step in community organizing involves training of potential leaders
in the community?

A. Integration
B. Community organization
C. Community study
D. Core group formation
Answers and Rationales
1. Answer: (B) To enhance the capacity of individuals, families and communities
to cope with their health needs
2. Answer: (B) The nurse has to conduct community diagnosis to determine
nursing needs and problems.
3. Answer: (C) Community diagnosis. Population-focused nursing care
means providing care based on the greater need of the majority of the
population. The greater need is identified through community diagnosis.
4. Answer: (B) Location of the workplace in relation to health facilities. Based
on R.A. 1054, an occupational nurse must be employed when there are 30
to 100 employees and the workplace is more than 1 km. away from the
nearest health center.
5. Answer: (B) 101. Again, this is based on R.A. 1054.
6. Answer: (D) Environmental manager. Ergonomics is improving efficiency
of workers by improving the worker’s environment through appropriately
designed furniture, for example.
7. Answer: (C) Public health nurse of the RHU of their municipality. You’re
right! This question is based on R.A.1054.
8. Answer: (B) The statement is false; people pay indirectly for public health
services. Community health services, including public health services, are
pre-paid services, though taxation, for example.
9. Answer: (A) For people to attain their birthrights of health and
longevity. According to Winslow, all public health efforts are for people to
realize their birthrights of health and longevity.
10. Answer: (C) Swaroop’s index. Swaroop’s index is the percentage of the
deaths aged 50 years or older. Its inverse represents the percentage of
untimely deaths (those who died younger than 50 years).
11. Answer: (D) Public health nursing focuses on preventive, not curative,
services.The catchment area in PHN consists of a residential community,
many of whom are well individuals who have greater need for preventive
rather than curative services.
12. Answer: (D) The worth and dignity of man. This is a direct quote from Dr.
Margaret Shetland’s statements on Public Health Nursing.
13. Answer: (B) Ensure the accessibility and quality of health care
14. Answer: (D) Tertiary. Regional hospitals are tertiary facilities because
they serve as training hospitals for the region.
15. Answer: (B) Their services are provided on an out-patient basis. Primary
facilities government and non-government facilities that provide basic out-
patient services.
16. Answer: (B) Conducting random classroom inspection during a measles
epidemic. Random classroom inspection is assessment of pupils/students
and teachers for signs of a health problem prevalent in the community.
17. Answer: (B) Efficiency. Efficiency is determining whether the goals were
attained at the least possible cost.
18. Answer: (D) Rural Health Unit. R.A. 7160 devolved basic health services
to local government units (LGU’s ). The public health nurse is an employee
of the LGU.
19. Answer: (C) To empower the people and promote their self-reliance. People
empowerment is the basic motivation behind devolution of basic services
to LGU’s.
20. Answer: (A) Mayor. The local executive serves as the chairman of the
Municipal Health Board.
21. Answer: (A) Primary. The entry of a person into the health care delivery
system is usually through a consultation in out-patient services.
22. Answer: (B) Providing technical guidance to the midwife. The nurse provides
technical guidance to the midwife in the care of clients, particularly in the
implementation of management guidelines, as in Integrated Management
of Childhood Illness.
23. Answer: (C) Municipal Health Officer. A public health nurse and rural
health midwife can provide care during normal childbirth. A physician
should attend to a woman with a complication during labor.
24. Answer: (A) 1. Each rural health midwife is given a population
assignment of about 5,000.
25. Answer: (D) Municipal Health Board. As mandated by R.A. 7160, basic
health services have been devolved from the national government to local
government units.
26. Answer: (A) Act 3573. Act 3573, the Law on Reporting of Communicable
Diseases, enacted in 1929, mandated the reporting of diseases listed in
the law to the nearest health station.
27. Answer: (B) Health education and community organizing are necessary in
providing community health services. The community health nurse develops
the health capability of people through health education and community
organizing activities.
28. Answer: (B) Measles. Presidential Proclamation No. 4 is on the Ligtas
Tigdas Program.
29. Answer: (B) Bar. A bar graph is used to present comparison of values, a
line graph for trends over time or age, a pie graph for population
composition or distribution, and a scatter diagram for correlation of two
variables.
30. Answer: (D) Core group formation. In core group formation, the nurse is
able to transfer the technology of community organizing to the potential or
informal community leaders through a training program.
PNLE: Community Health Nursing
Exam 2
1. In which step are plans formulated for solving community problems?

A. Mobilization
B. Community organization
C. Follow-up/extension
D. Core group formation
2. The public health nurse takes an active role in community participation.
What is the primary goal of community organizing?

A. To educate the people regarding community health problems


B. To mobilize the people to resolve community health problems
C. To maximize the community’s resources in dealing with health
problems
D. To maximize the community’s resources in dealing with health
problems
3. An indicator of success in community organizing is when people are able to

A. Participate in community activities for the solution of a community


problem
B. Implement activities for the solution of the community problem
C. Plan activities for the solution of the community problem
D. Identify the health problem as a common concern
4. Tertiary prevention is needed in which stage of the natural history of
disease?

A. Pre-pathogenesis
B. Pathogenesis
C. Prodromal
D. Terminal
5. Isolation of a child with measles belongs to what level of prevention?

A. Primary
B. Secondary
C. Intermediate
D. Tertiary
6. On the other hand, Operation Timbang is _____ prevention.

A. Primary
B. Secondary
C. Intermediate
D. Tertiary
7. Which type of family-nurse contact will provide you with the best
opportunity to observe family dynamics?

A. Clinic consultation
B. Group conference
C. Home visit
D. Written communication
8. The typology of family nursing problems is used in the statement of nursing
diagnosis in the care of families. The youngest child of the de los Reyes family
has been diagnosed as mentally retarded. This is classified as a:

A. Health threat
B. Health deficit
C. Foreseeable crisis
D. Stress point
9. The de los Reyes couple have a 6-year old child entering school for the first
time. The de los Reyes family has a:

A. Health threat
B. Health deficit
C. Foreseeable crisis
D. Stress point
10. Which of the following is an advantage of a home visit?

A. It allows the nurse to provide nursing care to a greater number of


people.
B. It provides an opportunity to do first hand appraisal of the home
situation.
C. It allows sharing of experiences among people with similar health
problems.
D. It develops the family’s initiative in providing for health needs of its
members.
11. Which is CONTRARY to the principles in planning a home visit?

A. A home visit should have a purpose or objective.


B. The plan should revolve around family health needs.
C. A home visit should be conducted in the manner prescribed by the RHU.
D. Planning of continuing care should involve a responsible family
member.
12. The PHN bag is an important tool in providing nursing care during a home
visit. The most important principle of bag technique states that it

A. Should save time and effort.


B. Should minimize if not totally prevent the spread of infection.
C. Should not overshadow concern for the patient and his family.
D. May be done in a variety of ways depending on the home situation, etc.
13. To maintain the cleanliness of the bag and its contents, which of the
following must the nurse do?

A. Wash his/her hands before and after providing nursing care to the
family members.
B. In the care of family members, as much as possible, use only articles
taken from the bag.
C. Put on an apron to protect her uniform and fold it with the right side out
before putting it back into the bag.
D. At the end of the visit, fold the lining on which the bag was placed,
ensuring that the contaminated side is on the outside.
14. The public health nurse conducts a study on the factors contributing to the
high mortality rate due to heart disease in the municipality where she works.
Which branch of epidemiology does the nurse practice in this situation?

A. Descriptive
B. Analytical
C. Therapeutic
D. Evaluation
15. Which of the following is a function of epidemiology?

A. Identifying the disease condition based on manifestations presented by


a client
B. Determining factors that contributed to the occurrence of pneumonia in
a 3 year old
C. Determining the efficacy of the antibiotic used in the treatment of the 3
year old client with pneumonia
D. Evaluating the effectiveness of the implementation of the Integrated
Management of Childhood Illness
16. Which of the following is an epidemiologic function of the nurse during an
epidemic?

A. Conducting assessment of suspected cases to detect the


communicable disease
B. Monitoring the condition of the cases affected by the communicable
disease
C. Participating in the investigation to determine the source of the
epidemic
D. Teaching the community on preventive measures against the disease
17. The primary purpose of conducting an epidemiologic investigation is to

A. Delineate the etiology of the epidemic


B. Encourage cooperation and support of the community
C. Identify groups who are at risk of contracting the disease
D. Identify geographical location of cases of the disease in the community
18. Which is a characteristic of person-to-person propagated epidemics?

A. There are more cases of the disease than expected.


B. The disease must necessarily be transmitted through a vector.
C. The spread of the disease can be attributed to a common vehicle.
D. There is a gradual build up of cases before the epidemic becomes easily
noticeable.
19. In the investigation of an epidemic, you compare the present frequency of
the disease with the usual frequency at this time of the year in this
community. This is done during which stage of the investigation?
A. Establishing the epidemic
B. Testing the hypothesis
C. Formulation of the hypothesis
D. Appraisal of facts
20. The number of cases of Dengue fever usually increases towards the end
of the rainy season. This pattern of occurrence of Dengue fever is best
described as

A. Epidemic occurrence
B. Cyclical variation
C. Sporadic occurrence
D. Secular variation
21. In the year 1980, the World Health Organization declared the Philippines,
together with some other countries in the Western Pacific Region, “free” of
which disease?

A. Pneumonic plague
B. Poliomyelitis
C. Small pox
D. Anthrax
22. In the census of the Philippines in 1995, there were about 35,299,000
males and about 34,968,000 females. What is the sex ratio?

A. 99.06:100
B. 100.94:100
C. 50.23%
D. 49.76%
23. Primary health care is a total approach to community development. Which
of the following is an indicator of success in the use of the primary health care
approach?

A. Health services are provided free of charge to individuals and families.


B. Local officials are empowered as the major decision makers in matters
of health.
C. Health workers are able to provide care based on identified health needs
of the people.
D. Health programs are sustained according to the level of development of
the community.
24. Sputum examination is the major screening tool for pulmonary
tuberculosis. Clients would sometimes get false negative results in this exam.
This means that the test is not perfect in terms of which characteristic of a
diagnostic examination?

A. Effectiveness
B. Efficacy
C. Specificity
D. Sensitivity
25. Use of appropriate technology requires knowledge of indigenous
technology. Which medicinal herb is given for fever, headache and cough?

A. Sambong
B. Tsaang gubat
C. Akapulko
D. Lagundi
26. What law created the Philippine Institute of Traditional and Alternative
Health Care?

A. R.A. 8423
B. R.A. 4823
C. R.A. 2483
D. R.A. 3482
27. In traditional Chinese medicine, the yielding, negative and feminine force is
termed

A. Yin
B. Yang
C. Qi
D. Chai
28. What is the legal basis for Primary Health Care approach in the
Philippines?

A. Alma Ata Declaration on PHC


B. Letter of Instruction No. 949
C. Presidential Decree No. 147
D. Presidential Decree 996
29. Which of the following demonstrates intersectoral linkages?

A. Two-way referral system


B. Team approach
C. Endorsement done by a midwife to another midwife
D. Cooperation between the PHN and public school teacher
30. The municipality assigned to you has a population of about 20,000.
Estimate the number of 1-4 year old children who will be given Retinol capsule
200,000 I.U. every 6 months.

A. 1,500
B. 1,800
C. 2,000
D. 2,300
Answers and Rationales
1. Answer: (B) Community organization. Community organization is the step
when community assemblies take place. During the community assembly,
the people may opt to formalize the community organization and make
plans for community action to resolve a community health problem.
2. Answer: (D) To maximize the community’s resources in dealing with health
problems. Community organizing is a developmental service, with the goal
of developing the people’s self-reliance in dealing with community health
problems. A, B and C are objectives of contributory objectives to this goal.
3. Answer: (A) Participate in community activities for the solution of a
community problem. Participation in community activities in resolving a
community problem may be in any of the processes mentioned in the
other choices.
4. Answer: (D) Terminal. Tertiary prevention involves rehabilitation,
prevention of permanent disability and disability limitation appropriate for
convalescents, the disabled, complicated cases and the terminally ill
(those in the terminal stage of a disease)
5. Answer: (A) Primary. The purpose of isolating a client with a
communicable disease is to protect those who are not sick (specific
disease prevention).
6. Answer: (B) Secondary. Operation Timbang is done to identify members
of the susceptible population who are malnourished. Its purpose is early
diagnosis and, subsequently, prompt treatment.
7. Answer: (C) Home visit. Dynamics of family relationships can best be
observed in the family’s natural environment, which is the home.
8. Answer: (B) Health deficit. Failure of a family member to develop
according to what is expected, as in mental retardation, is a health deficit.
9. Answer: (C) Foreseeable crisis. Entry of the 6-year old into school is an
anticipated period of unusual demand on the family.
10. Answer: (B) It provides an opportunity to do first hand appraisal of the home
situation.. Choice A is not correct since a home visit requires that the nurse
spend so much time with the family. Choice C is an advantage of a group
conference, while choice D is true of a clinic consultation.
11. Answer: (C) A home visit should be conducted in the manner prescribed by the
RHU.The home visit plan should be flexible and practical, depending on
factors, such as the family’s needs and the resources available to the
nurse and the family.
12. Answer: (B) Should minimize if not totally prevent the spread of infection. Bag
technique is performed before and after handling a client in the home to
prevent transmission of infection to and from the client.
13. Answer: (A) Wash his/her hands before and after providing nursing care to the
family members. Choice B goes against the idea of utilizing the family’s
resources, which is encouraged in CHN. Choices C and D goes against the
principle of asepsis of confining the contaminated surface of objects.
14. Answer: (B) Analytical. Analytical epidemiology is the study of factors or
determinants affecting the patterns of occurrence and distribution of
disease in a community.
15. Answer: (D) Evaluating the effectiveness of the implementation of the
Integrated Management of Childhood Illness. Epidemiology is used in the
assessment of a community or evaluation of interventions in community
health practice.
16. Answer: (C) Participating in the investigation to determine the source of the
epidemic. Epidemiology is the study of patterns of occurrence and
distribution of disease in the community, as well as the factors that affect
disease patterns. The purpose of an epidemiologic investigation is to
identify the source of an epidemic, i.e., what brought about the epidemic.
17. Answer: (A) Delineate the etiology of the epidemic. Delineating the etiology
of an epidemic is identifying its source.
18. Answer: (D) There is a gradual build up of cases before the epidemic becomes
easily noticeable. A gradual or insidious onset of the epidemic is usually
observable in person-to-person propagated epidemics.
19. Answer: (A) Establishing the epidemic. Establishing the epidemic is
determining whether there is an epidemic or not. This is done by
comparing the present number of cases with the usual number of cases
of the disease at the same time of the year, as well as establishing the
relatedness of the cases of the disease.
20. Answer: (B) Cyclical variation. A cyclical variation is a periodic fluctuation
in the number of cases of a disease in the community.
21. Answer: (C) Small pox. The last documented case of Small pox was in
1977 at Somalia.
22. Answer: (B) 100.94:100. Sex ratio is the number of males for every 100
females in the population.
23. Answer: (D) Health programs are sustained according to the level of
development of the community. Primary health care is essential health care
that can be sustained in all stages of development of the community.
24. Answer: (D) Sensitivity. Sensitivity is the capacity of a diagnostic
examination to detect cases of the disease. If a test is 100% sensitive, all
the cases tested will have a positive result, i.e., there will be no false
negative results.
25. Answer: (D) Lagundi. Sambong is used as a diuretic. Tsaang gubat is
used to relieve diarrhea. Akapulko is used for its antifungal property.
26. Answer: (A) R.A. 8423
27. Answer: (A) Yin. Yang is the male dominating, positive and masculine
force.
28. Answer: (B) Letter of Instruction No. 949. Letter of Instruction 949 was
issued by then President Ferdinand Marcos, directing the formerly called
Ministry of Health, now the Department of Health, to utilize Primary Health
Care approach in planning and implementing health programs.
29. Answer: (D) Cooperation between the PHN and public school
teacher. Intersectoral linkages refer to working relationships between the
health sector and other sectors involved in community development.
30. Answer: (D) 2,300. Based on the Philippine population composition, to
estimate the number of 1-4 year old children, multiply total population by
11.5%.

PNLE: Community Health Nursing


Exam 3
1. Estimate the number of pregnant women who will be given tetanus toxoid
during an immunization outreach activity in a barangay with a population of
about 1,500.

A. 265
B. 300
C. 375
D. 400
2. To describe the sex composition of the population, which demographic tool
may be used?

A. Sex ratio
B. Sex proportion
C. Population pyramid
D. Any of these may be used.
3. Which of the following is a natality rate?

A. Crude birth rate


B. Neonatal mortality rate
C. Infant mortality rate
D. General fertility rate
4. You are computing the crude death rate of your municipality, with a total
population of about 18,000, for last year. There were 94 deaths. Among those
who died, 20 died because of diseases of the heart and 32 were aged 50 years
or older. What is the crude death rate?

A. 4.2/1,000
B. 5.2/1,000
C. 6.3/1,000
D. 7.3/1,000
5. Knowing that malnutrition is a frequent community health problem, you
decided to conduct nutritional assessment. What population is particularly
susceptible to protein energy malnutrition (PEM)?

A. Pregnant women and the elderly


B. Under-5 year old children
C. 1-4 year old children
D. School age children
6. Which statistic can give the most accurate reflection of the health status of
a community?

A. 1-4 year old age-specific mortality rate


B. Infant mortality rate
C. Swaroop’s index
D. Crude death rate
7. In the past year, Barangay A had an average population of 1655. 46 babies
were born in that year, 2 of whom died less than 4 weeks after they were born.
There were 4 recorded stillbirths. What is the neonatal mortality rate?

A. 27.8/1,000
B. 43.5/1,000
C. 86.9/1,000
D. 130.4/1,000
8. Which statistic best reflects the nutritional status of a population?

A. 1-4 year old age-specific mortality rate


B. Proportionate mortality rate
C. Infant mortality rate
D. Swaroop’s index
9. What numerator is used in computing general fertility rate?

A. Estimated midyear population


B. Number of registered live births
C. Number of pregnancies in the year
D. Number of females of reproductive age
10. You will gather data for nutritional assessment of a purok. You will gather
information only from families with members who belong to the target
population for PEM. What method of data gathering is best for this purpose?

A. Census
B. Survey
C. Record review
D. Review of civil registry
11. In the conduct of a census, the method of population assignment based
on the actual physical location of the people is termed

A. De jure
B. De locus
C. De facto
D. De novo
12. The Field Health Services and Information System (FHSIS) is the recording
and reporting system in public health care in the Philippines. The Monthly
Field Health Service Activity Report is a form used in which of the components
of the FHSIS?

A. Tally report
B. Output report
C. Target/client list
D. Individual health record
13. To monitor clients registered in long-term regimens, such as the Multi-
Drug Therapy, which component will be most useful?

A. Tally report
B. Output report
C. Target/client list
D. Individual health record
14. Civil registries are important sources of data. Which law requires
registration of births within 30 days from the occurrence of the birth?

A. P.D. 651
B. Act 3573
C. R.A. 3753
D. R.A. 3375
15. Which of the following professionals can sign the birth certificate?

A. Public health nurse


B. Rural health midwife
C. Municipal health officer
D. Any of these health professionals
16. Which criterion in priority setting of health problems is used only in
community health care?

A. Modifiability of the problem


B. Nature of the problem presented
C. Magnitude of the health problem
D. Preventive potential of the health problem
17. The Sentrong Sigla Movement has been launched to improve health
service delivery. Which of the following is/are true of this movement?

A. This is a project spearheaded by local government units.


B. It is a basis for increasing funding from local government units.
C. It encourages health centers to focus on disease prevention and
control.
D. Its main strategy is certification of health centers able to comply with
standards.
18. Which of the following women should be considered as special targets for
family planning?

A. Those who have two children or more


B. Those with medical conditions such as anemia
C. Those younger than 20 years and older than 35 years
D. Those who just had a delivery within the past 15 months
19. Freedom of choice is one of the policies of the Family Planning Program
of the Philippines. Which of the following illustrates this principle?

A. Information dissemination about the need for family planning


B. Support of research and development in family planning methods
C. Adequate information for couples regarding the different methods
D. Encouragement of couples to take family planning as a joint
responsibility
20. A woman, 6 months pregnant, came to the center for consultation. Which
of the following substances is contraindicated?

A. Tetanus toxoid
B. Retinol 200,000 IU
C. Ferrous sulfate 200 mg
D. Potassium iodate 200 mg. capsule
21. During prenatal consultation, a client asked you if she can have her
delivery at home. After history taking and physical examination, you advised
her against a home delivery. Which of the following findings disqualifies her
for a home delivery?

A. Her OB score is G5P3.


B. She has some palmar pallor.
C. Her blood pressure is 130/80.
D. Her baby is in cephalic presentation.
22. Inadequate intake by the pregnant woman of which vitamin may cause
neural tube defects?

A. Niacin
B. Riboflavin
C. Folic acid
D. Thiamine
23. You are in a client’s home to attend to a delivery. Which of the following
will you do first?

A. Set up the sterile area.


B. Put on a clean gown or apron.
C. Cleanse the client’s vulva with soap and water.
D. Note the interval, duration and intensity of labor contractions.
24. In preparing a primigravida for breastfeeding, which of the following will
you do?

A. Tell her that lactation begins within a day after delivery.


B. Teach her nipple stretching exercises if her nipples are everted.
C. Instruct her to wash her nipples before and after each breastfeeding.
D. Explain to her that putting the baby to breast will lessen blood loss after
delivery.
25. A primigravida is instructed to offer her breast to the baby for the first time
within 30 minutes after delivery. What is the purpose of offering the breast
this early?

A. To initiate the occurrence of milk letdown


B. To stimulate milk production by the mammary acini
C. To make sure that the baby is able to get the colostrum
D. To allow the woman to practice breastfeeding in the presence of the
health worker
26. In a mothers’ class, you discuss proper breastfeeding technique. Which is
of these is a sign that the baby has “latched on” to the breast properly?
A. The baby takes shallow, rapid sucks.
B. The mother does not feel nipple pain.
C. The baby’s mouth is only partly open.
D. Only the mother’s nipple is inside the baby’s mouth.
27. You explain to a breastfeeding mother that breast milk is sufficient for all
of the baby’s nutrient needs only up to ____.

A. 3 months
B. 6 months
C. 1 year
D. 2 years
28. What is given to a woman within a month after the delivery of a baby?

A. Malunggay capsule
B. Ferrous sulfate 100 mg. OD
C. Retinol 200,000 I.U., 1 capsule
D. Potassium iodate 200 mg, 1 capsule
29. Which biological used in Expanded Program on Immunization (EPI) is
stored in the freezer?

A. DPT
B. Tetanus toxoid
C. Measles vaccine
D. Hepatitis B vaccine
30. Unused BCG should be discarded how many hours after reconstitution?

A. 2
B. 4
C. 6
D. At the end of the day
Answers and Rationales
1. Answer: (A) 265. To estimate the number of pregnant women, multiply
the total population by 3.5%.
2. Answer: (D) Any of these may be used. Sex ratio and sex proportion are
used to determine the sex composition of a population. A population
pyramid is used to present the composition of a population by age and
sex.
3. Answer: (A) Crude birth rate. Natality means birth. A natality rate is a birth
rate.
4. Answer: (B) 5.2/1,000. To compute crude death rate divide total number
of deaths (94) by total population (18,000) and multiply by 1,000.
5. Answer: (C) 1-4 year old children. Preschoolers are the most susceptible
to PEM because they have generally been weaned. Also, this is the
population who, unable to feed themselves, are often the victims of poor
intrafamilial food distribution.
6. Answer: (C) Swaroop’s index. Swaroop’s index is the proportion of deaths
aged 50 years and above. The higher the Swaroop’s index of a population,
the greater the proportion of the deaths who were able to reach the age of
at least 50 years, i.e., more people grew old before they died.
7. Answer: (B) 43.5/1,000. To compute for neonatal mortality rate, divide the
number of babies who died before reaching the age of 28 days by the total
number of live births, then multiply by 1,000.
8. Answer: (A) 1-4 year old age-specific mortality rate. Since preschoolers are
the most susceptible to the effects of malnutrition, a population with poor
nutritional status will most likely have a high 1-4 year old age-specific
mortality rate, also known as child mortality rate.
9. Answer: (B) Number of registered live births. To compute for general or
total fertility rate, divide the number of registered live births by the number
of females of reproductive age (15-45 years), then multiply by 1,000.
10. Answer: (B) Survey. A survey, also called sample survey, is data
gathering about a sample of the population.
11. Answer: (C) De facto. The other method of population assignment, de
jure, is based on the usual place of residence of the people.
12. Answer: (A) Tally report. A tally report is prepared monthly or quarterly by
the RHU personnel and transmitted to the Provincial Health Office.
13. Answer: (C) Target/client list. The MDT Client List is a record of clients
enrolled in MDT and other relevant data, such as dates when clients
collected their monthly supply of drugs.
14. Answer: (A) P.D. 651. P.D. 651 amended R.A. 3753, requiring the registry
of births within 30 days from their occurrence.
15. Answer: (D) Any of these health professionals. D. R.A. 3753 states that any
birth attendant may sign the certificate of live birth.
16. Answer: (C) Magnitude of the health problem. Magnitude of the problem
refers to the percentage of the population affected by a health problem.
The other choices are criteria considered in both family and community
health care.
17. Answer: (D) Its main strategy is certification of health centers able to comply
with standards. Sentrong Sigla Movement is a joint project of the DOH and
local government units. Its main strategy is certification of health centers
that are able to comply with standards set by the DOH.
18. Answer: (D) Those who just had a delivery within the past 15 months. The
ideal birth spacing is at least two years. 15 months plus 9 months of
pregnancy = 2 years.
19. Answer: (C) Adequate information for couples regarding the different
methods. To enable the couple to choose freely among different methods
of family planning, they must be given full information regarding the
different methods that are available to them, considering the availability of
quality services that can support their choice.
20. Answer: (B) Retinol 200,000 IU. Retinol 200,000 IU is a form of megadose
Vitamin A. This may have a teratogenic effect.
21. Answer: (A) Her OB score is G5P3. Only women with less than 5
pregnancies are qualified for a home delivery. It is also advisable for a
primigravida to have delivery at a childbirth facility.
22. Answer: (C) Folic acid. It is estimated that the incidence of neural tube
defects can be reduced drastically if pregnant women have an adequate
intake of folic acid.
23. Answer: (D) Note the interval, duration and intensity of labor
contractions.. Assessment of the woman should be done first to determine
whether she is having true labor and, if so, what stage of labor she is in.
24. Answer: (D) Explain to her that putting the baby to breast will lessen blood loss
after delivery. Suckling of the nipple stimulates the release of oxytocin by
the posterior pituitary gland, which causes uterine contraction. Lactation
begins 1 to 3 days after delivery. Nipple stretching exercises are done
when the nipples are flat or inverted. Frequent washing dries up the
nipples, making them prone to the formation of fissures.
25. Answer: (B) To stimulate milk production by the mammary acini. Suckling of
the nipple stimulates prolactin reflex (the release of prolactin by the
anterior pituitary gland), which initiates lactation.
26. Answer: (B) The mother does not feel nipple pain.. When the baby has
properly latched on to the breast, he takes deep, slow sucks; his mouth is
wide open; and much of the areola is inside his mouth. And, you’re right!
The mother does not feel nipple pain.
27. Answer: (B) 6 months. After 6 months, the baby’s nutrient needs,
especially the baby’s iron requirement, can no longer be provided by
mother’s milk alone.
28. Answer: (C) Retinol 200,000 I.U., 1 capsule. A capsule of Retinol 200,000
IU is given within 1 month after delivery. Potassium iodate is given during
pregnancy; malunggay capsule is not routinely administered after delivery;
and ferrous sulfate is taken for two months after delivery.
29. Answer: (C) Measles vaccine. Among the biologicals used in the Expanded
Program on Immunization, measles vaccine and OPV are highly sensitive
to heat, requiring storage in the freezer.
30. Answer: (B) 4. While the unused portion of other biologicals in EPI may
be given until the end of the day, only BCG is discarded 4 hours after
reconstitution. This is why BCG immunization is scheduled only in the
morning.
PNLE: Community Health Nursing
Exam 4
1. In immunizing school entrants with BCG, you are not obliged to secure
parental consent. This is because of which legal document?

A. P.D. 996
B. R.A. 7846
C. Presidential Proclamation No. 6
D. Presidential Proclamation No. 46
2. Which immunization produces a permanent scar?

A. DPT
B. BCG
C. Measles vaccination
D. Hepatitis B vaccination
3. A 4-week old baby was brought to the health center for his first
immunization. Which can be given to him?

A. DPT1
B. OPV1
C. Infant BCG
D. Hepatitis B vaccine 1
4. You will not give DPT 2 if the mother says that the infant had

A. Seizures a day after DPT 1.


B. Fever for 3 days after DPT 1.
C. Abscess formation after DPT 1.
D. Local tenderness for 3 days after DPT 1.
5. A 2-month old infant was brought to the health center for immunization.
During assessment, the infant’s temperature registered at 38.1°C. Which is the
best course of action that you will take?
A. Go on with the infant’s immunizations.
B. Give Paracetamol and wait for his fever to subside.
C. Refer the infant to the physician for further assessment.
D. Advise the infant’s mother to bring him back for immunization when he
is well.
6. A pregnant woman had just received her 4th dose of tetanus toxoid.
Subsequently, her baby will have protection against tetanus for how long?

A. 1 year
B. 3 years
C. 10 years
D. Lifetime
7. A 4-month old infant was brought to the health center because of cough.
Her respiratory rate is 42/minute. Using the Integrated Management of Child
Illness (IMCI) guidelines of assessment, her breathing is considered

A. Fast
B. Slow
C. Normal
D. Insignificant
8. Which of the following signs will indicate that a young child is suffering
from severe pneumonia?

A. Dyspnea
B. Wheezing
C. Fast breathing
D. Chest indrawing
9. Using IMCI guidelines, you classify a child as having severe pneumonia.
What is the best management for the child?

A. Prescribe an antibiotic.
B. Refer him urgently to the hospital.
C. Instruct the mother to increase fluid intake.
D. Instruct the mother to continue breastfeeding.
10. A 5-month old infant was brought by his mother to the health center
because of diarrhea occurring 4 to 5 times a day. His skin goes back slowly
after a skin pinch and his eyes are sunken. Using the IMCI guidelines, you will
classify this infant in which category?

A. No signs of dehydration
B. Some dehydration
C. Severe dehydration
D. The data is insufficient.
11. Based on assessment, you classified a 3-month old infant with the chief
complaint of diarrhea in the category of SOME DEHYDRATION. Based on IMCI
management guidelines, which of the following will you do?

A. Bring the infant to the nearest facility where IV fluids can be given.
B. Supervise the mother in giving 200 to 400 ml. of Oresol in 4 hours.
C. Give the infant’s mother instructions on home management.
D. Keep the infant in your health center for close observation.
12. A mother is using Oresol in the management of diarrhea of her 3-year old
child. She asked you what to do if her child vomits. You will tell her to

A. Bring the child to the nearest hospital for further assessment.


B. Bring the child to the health center for intravenous fluid therapy.
C. Bring the child to the health center for assessment by the physician.
D. Let the child rest for 10 minutes then continue giving Oresol more
slowly.
13. A 1 ½ year old child was classified as having 3rd degree protein energy
malnutrition, kwashiorkor. Which of the following signs will be most apparent
in this child?

A. Voracious appetite
B. Wasting
C. Apathy
D. Edema
14. Assessment of a 2-year old child revealed “baggy pants”. Using the IMCI
guidelines, how will you manage this child?

A. Refer the child urgently to a hospital for confinement.


B. Coordinate with the social worker to enroll the child in a feeding
program.
C. Make a teaching plan for the mother, focusing on menu planning for her
child.
D. Assess and treat the child for health problems like infections and
intestinal parasitism.
15. During the physical examination of a young child, what is the earliest sign
of xerophthalmia that you may observe?

A. Keratomalacia
B. Corneal opacity
C. Night blindness
D. Conjunctival xerosis
16. To prevent xerophthalmia, young children are given Retinol capsule every 6
months. What is the dose given to preschoolers?

A. 10,000 IU
B. 20,000 IU
C. 100,000 IU
D. 200,000 IU
17. The major sign of iron deficiency anemia is pallor. What part is best
examined for pallor?

A. Palms
B. Nailbeds
C. Around the lips
D. Lower conjunctival sac
18. Food fortification is one of the strategies to prevent micronutrient
deficiency conditions. R.A. 8976 mandates fortification of certain food items.
Which of the following is among these food items?

A. Sugar
B. Bread
C. Margarine
D. Filled milk
19. What is the best course of action when there is a measles epidemic in a
nearby municipality?

A. Give measles vaccine to babies aged 6 to 8 months.


B. Give babies aged 6 to 11 months one dose of 100,000 I.U. of Retinol
C. Instruct mothers to keep their babies at home to prevent disease
transmission.
D. Instruct mothers to feed their babies adequately to enhance their
babies’ resistance.
20. A mother brought her daughter, 4 years old, to the RHU because of cough
and colds. Following the IMCI assessment guide, which of the following is a
danger sign that indicates the need for urgent referral to a hospital?

A. Inability to drink
B. High grade fever
C. Signs of severe dehydration
D. Cough for more than 30 days
21. Management of a child with measles includes the administration of which
of the following?

A. Gentian violet on mouth lesions


B. Antibiotics to prevent pneumonia
C. Tetracycline eye ointment for corneal opacity
D. Retinol capsule regardless of when the last dose was given
22. A mother brought her 10 month old infant for consultation because of
fever, which started 4 days prior to consultation. To determine malaria risk,
what will you do?

A. Do a tourniquet test.
B. Ask where the family resides.
C. Get a specimen for blood smear.
D. Ask if the fever is present everyday.
23. The following are strategies implemented by the Department of Health to
prevent mosquito-borne diseases. Which of these is most effective in the
control of Dengue fever?

A. Stream seeding with larva-eating fish


B. Destroying breeding places of mosquitoes
C. Chemoprophylaxis of non-immune persons going to endemic areas
D. Teaching people in endemic areas to use chemically treated mosquito
nets
24. Secondary prevention for malaria includes

A. Planting of neem or eucalyptus trees


B. Residual spraying of insecticides at night
C. Determining whether a place is endemic or not
D. Growing larva-eating fish in mosquito breeding places
25. Scotch tape swab is done to check for which intestinal parasite?

A. Ascaris
B. Pinworm
C. Hookworm
D. Schistosoma
26. Which of the following signs indicates the need for sputum examination
for AFB?

A. Hematemesis
B. Fever for 1 week
C. Cough for 3 weeks
D. Chest pain for 1 week
27. Which clients are considered targets for DOTS Category I?

A. Sputum negative cavitary cases


B. Clients returning after a default
C. Relapses and failures of previous PTB treatment regimens
D. Clients diagnosed for the first time through a positive sputum exam
28. To improve compliance to treatment, what innovation is being
implemented in DOTS?

A. Having the health worker follow up the client at home


B. Having the health worker or a responsible family member monitor drug
intake
C. Having the patient come to the health center every month to get his
medications
D. Having a target list to check on whether the patient has collected his
monthly supply of drugs
29. Diagnosis of leprosy is highly dependent on recognition of symptoms.
Which of the following is an early sign of leprosy?
A. Macular lesions
B. Inability to close eyelids
C. Thickened painful nerves
D. Sinking of the nosebridge
30. Which of the following clients should be classified as a case of
multibacillary leprosy?

A. 3 skin lesions, negative slit skin smear


B. 3 skin lesions, positive slit skin smear
C. 5 skin lesions, negative slit skin smear
D. 5 skin lesions, positive slit skin smear
Answers and Rationales
1. Answer: (A) P.D. 996. Presidential Decree 996, enacted in 1976, made
immunization in the EPI compulsory for children under 8 years of age.
Hepatitis B vaccination was made compulsory for the same age group by
R.A. 7846.
2. Answer: (B) BCG. BCG causes the formation of a superficial abscess,
which begins 2 weeks after immunization. The abscess heals without
treatment, with the formation of a permanent scar.
3. Answer: (C) Infant BCG. Infant BCG may be given at birth. All the other
immunizations mentioned can be given at 6 weeks of age.
4. Answer: (A) Seizures a day after DPT 1. Seizures within 3 days after
administration of DPT is an indication of hypersensitivity to pertussis
vaccine, a component of DPT. This is considered a specific
contraindication to subsequent doses of DPT.
5. Answer: (A) Go on with the infant’s immunizations. In the EPI, fever up to
38.5°C is not a contraindication to immunization. Mild acute respiratory
tract infection, simple diarrhea and malnutrition are not contraindications
either.
6. Answer: (A) 1 year. The baby will have passive natural immunity by
placental transfer of antibodies. The mother will have active artificial
immunity lasting for about 10 years. 5 doses will give the mother lifetime
protection.
7. Answer: (C) Normal. In IMCI, a respiratory rate of 50/minute or more is
fast breathing for an infant aged 2 to 12 months.
8. Answer: (D) Chest indrawing. In IMCI, chest indrawing is used as the
positive sign of dyspnea, indicating severe pneumonia.
9. Answer: (B) Refer him urgently to the hospital. Severe pneumonia requires
urgent referral to a hospital. Answers A, C and D are done for a client
classified as having pneumonia.
10. Answer: (B) Some dehydration. Using the assessment guidelines of IMCI,
a child (2 months to 5 years old) with diarrhea is classified as having
SOME DEHYDRATION if he shows 2 or more of the following signs:
restless or irritable, sunken eyes, the skin goes back slow after a skin
pinch.
11. Answer: (B) Supervise the mother in giving 200 to 400 ml. of Oresol in 4
hours. In the IMCI management guidelines, SOME DEHYDRATION is
treated with the administration of Oresol within a period of 4 hours. The
amount of Oresol is best computed on the basis of the child’s weight (75
ml/kg body weight). If the weight is unknown, the amount of Oresol is
based on the child’s age.
12. Answer: (D) Let the child rest for 10 minutes then continue giving Oresol more
slowly. If the child vomits persistently, that is, he vomits everything that he
takes in, he has to be referred urgently to a hospital. Otherwise, vomiting is
managed by letting the child rest for 10 minutes and then continuing with
Oresol administration. Teach the mother to give Oresol more slowly.
13. Answer: (D) Edema. Edema, a major sign of kwashiorkor, is caused by
decreased colloidal osmotic pressure of the blood brought about by
hypoalbuminemia. Decreased blood albumin level is due a protein-
deficient diet.
14. Answer: (A) Refer the child urgently to a hospital for confinement. “Baggy
pants” is a sign of severe marasmus. The best management is urgent
referral to a hospital.
15. Answer: (D) Conjunctival xerosis. The earliest sign of Vitamin A deficiency
(xerophthalmia) is night blindness. However, this is a functional change,
which is not observable during physical examination.The earliest visible
lesion is conjunctival xerosis or dullness of the conjunctiva due to
inadequate tear production.
16. Answer: (D) 200,000 IU. Preschoolers are given Retinol 200,000 IU every
6 months. 100,000 IU is given once to infants aged 6 to 12 months. The
dose for pregnant women is 10,000 IU.
17. Answer: (A) Palms. The anatomic characteristics of the palms allow a
reliable and convenient basis for examination for pallor.
18. Answer: (A) Sugar. R.A. 8976 mandates fortification of rice, wheat flour,
sugar and cooking oil with Vitamin A, iron and/or iodine.
19. Answer: (A) Give measles vaccine to babies aged 6 to 8 months. Ordinarily,
measles vaccine is given at 9 months of age. During an impending
epidemic, however, one dose may be given to babies aged 6 to 8 months.
The mother is instructed that the baby needs another dose when the baby
is 9 months old.
20. Answer: (A) Inability to drink. A sick child aged 2 months to 5 years must
be referred urgently to a hospital if he/she has one or more of the
following signs: not able to feed or drink, vomits everything, convulsions,
abnormally sleepy or difficult to awaken.
21. Answer: (D) Retinol capsule regardless of when the last dose was given. An
infant 6 to 12 months classified as a case of measles is given Retinol
100,000 IU; a child is given 200,000 IU regardless of when the last dose
was given.
22. Answer: (B) Ask where the family resides. Because malaria is endemic, the
first question to determine malaria risk is where the client’s family resides.
If the area of residence is not a known endemic area, ask if the child had
traveled within the past 6 months, where he/she was brought and whether
he/she stayed overnight in that area.
23. Answer: (B) Destroying breeding places of mosquitoes. Aedes aegypti, the
vector of Dengue fever, breeds in stagnant, clear water. Its feeding time is
usually during the daytime. It has a cyclical pattern of occurrence, unlike
malaria which is endemic in certain parts of the country.
24. Answer: (C) Determining whether a place is endemic or not. This is
diagnostic and therefore secondary level prevention. The other choices
are for primary prevention.
25. Answer: (B) Pinworm. Pinworm ova are deposited around the anal orifice.
26. Answer: (C) Cough for 3 weeks. A client is considered a PTB suspect
when he has cough for 2 weeks or more, plus one or more of the following
signs: fever for 1 month or more; chest pain lasting for 2 weeks or more
not attributed to other conditions; progressive, unexplained weight loss;
night sweats; and hemoptysis.
27. Answer: (D) Clients diagnosed for the first time through a positive sputum
exam. Category I is for new clients diagnosed by sputum examination and
clients diagnosed to have a serious form of extrapulmonary tuberculosis,
such as TB osteomyelitis.
28. Answer: (B) Having the health worker or a responsible family member monitor
drug intake. Directly Observed Treatment Short Course is so-called
because a treatment partner, preferably a health worker accessible to the
client, monitors the client’s compliance to the treatment.
29. Answer: (C) Thickened painful nerves. The lesion of leprosy is not macular.
It is characterized by a change in skin color (either reddish or whitish) and
loss of sensation, sweating and hair growth over the lesion. Inability to
close the eyelids (lagophthalmos) and sinking of the nosebridge are late
symptoms.
30. Answer: (D) 5 skin lesions, positive slit skin smear. A multibacillary leprosy
case is one who has a positive slit skin smear and at least 5 skin lesions.

PNLE: Community Health Nursing


Exam 5
1. In the Philippines, which condition is the most frequent cause of death
associated with schistosomiasis?

A. Liver cancer
B. Liver cirrhosis
C. Bladder cancer
D. Intestinal perforation
2. What is the most effective way of controlling schistosomiasis in an
endemic area?

A. Use of molluscicides
B. Building of foot bridges
C. Proper use of sanitary toilets
D. Use of protective footwear, such as rubber boots
3. When residents obtain water from an artesian well in the neighborhood, the
level of this approved type of water facility is

A. I
B. II
C. III
D. IV
4. For prevention of hepatitis A, you decided to conduct health education
activities. Which of the following is IRRELEVANT?

A. Use of sterile syringes and needles


B. Safe food preparation and food handling by vendors
C. Proper disposal of human excreta and personal hygiene
D. Immediate reporting of water pipe leaks and illegal water connections
5. Which biological used in Expanded Program on Immunization (EPI) should
NOT be stored in the freezer?

A. DPT
B. Oral polio vaccine
C. Measles vaccine
D. MMR
6. You will conduct outreach immunization in a barangay with a population of
about 1500. Estimate the number of infants in the barangay.

A. 45
B. 50
C. 55
D. 60
7. In Integrated Management of Childhood Illness, severe conditions generally
require urgent referral to a hospital. Which of the following severe conditions
DOES NOT always require urgent referral to a hospital?

A. Mastoiditis
B. Severe dehydration
C. Severe pneumonia
D. Severe febrile disease
8. A client was diagnosed as having Dengue fever. You will say that there is
slow capillary refill when the color of the nailbed that you pressed does not
return within how many seconds?

A. 3
B. 5
C. 8
D. 10
9. A 3-year old child was brought by his mother to the health center because
of fever of 4-day duration. The child had a positive tourniquet test result. In the
absence of other signs, which is the most appropriate measure that the PHN
may carry out to prevent Dengue shock syndrome?

A. Insert an NGT and give fluids per NGT.


B. Instruct the mother to give the child Oresol.
C. Start the patient on intravenous fluids STAT.
D. Refer the client to the physician for appropriate management.
10. The pathognomonic sign of measles is Koplik’s spot. You may see
Koplik’s spot by inspecting the _____.

A. Nasal mucosa
B. Buccal mucosa
C. Skin on the abdomen
D. Skin on the antecubital surface
11. Among the following diseases, which is airborne?

A. Viral conjunctivitis
B. Acute poliomyelitis
C. Diphtheria
D. Measles
12. Among children aged 2 months to 3 years, the most prevalent form of
meningitis is caused by which microorganism?

A. Hemophilus influenzae
B. Morbillivirus
C. Steptococcus pneumoniae
D. Neisseria meningitidis
13. Human beings are the major reservoir of malaria. Which of the following
strategies in malaria control is based on this fact?

A. Stream seeding
B. Stream clearing
C. Destruction of breeding places
D. Zooprophylaxis
14. The use of larvivorous fish in malaria control is the basis for which
strategy of malaria control?

A. Stream seeding
B. Stream clearing
C. Destruction of breeding places
D. Zooprophylaxis
15. Mosquito-borne diseases are prevented mostly with the use of mosquito
control measures. Which of the following is NOT appropriate for malaria
control?

A. Use of chemically treated mosquito nets


B. Seeding of breeding places with larva-eating fish
C. Destruction of breeding places of the mosquito vector
D. Use of mosquito-repelling soaps, such as those with basil or citronella
16. A 4-year old client was brought to the health center with the chief
complaint of severe diarrhea and the passage of “rice water” stools. The client
is most probably suffering from which condition?

A. Giardiasis
B. Cholera
C. Amebiasis
D. Dysentery
17. In the Philippines, which specie of schistosoma is endemic in certain
regions?

A. S. mansoni
B. S. japonicum
C. S. malayensis
D. S. haematobium
18. A 32-year old client came for consultation at the health center with the
chief complaint of fever for a week. Accompanying symptoms were muscle
pains and body malaise. A week after the start of fever, the client noted
yellowish discoloration of his sclera. History showed that he waded in flood
waters about 2 weeks before the onset of symptoms. Based on his history,
which disease condition will you suspect?

A. Hepatitis A
B. Hepatitis B
C. Tetanus
D. Leptospirosis
19. MWSS provides water to Manila and other cities in Metro Manila. This is
an example of which level of water facility?

A. I
B. II
C. III
D. IV
20. You are the PHN in the city health center. A client underwent screening for
AIDS using ELISA. His result was positive. What is the best course of action
that you may take?

A. Get a thorough history of the client, focusing on the practice of high risk
behaviors.
B. Ask the client to be accompanied by a significant person before
revealing the result.
C. Refer the client to the physician since he is the best person to reveal the
result to the client.
D. Refer the client for a supplementary test, such as Western blot, since the
ELISA result may be false.
21. Which is the BEST control measure for AIDS?

A. Being faithful to a single sexual partner


B. Using a condom during each sexual contact
C. Avoiding sexual contact with commercial sex workers
D. Making sure that one’s sexual partner does not have signs of AIDS
22. The most frequent causes of death among clients with AIDS are
opportunistic diseases. Which of the following opportunistic infections is
characterized by tonsillopharyngitis?

A. Respiratory candidiasis
B. Infectious mononucleosis
C. Cytomegalovirus disease
D. Pneumocystis carinii pneumonia
23. To determine possible sources of sexually transmitted infections, which is
the BEST method that may be undertaken by the public health nurse?

A. Contact tracing
B. Community survey
C. Mass screening tests
D. Interview of suspects
24. Antiretroviral agents, such as AZT, are used in the management of AIDS.
Which of the following is NOT an action expected of these drugs.

A. They prolong the life of the client with AIDS.


B. They reduce the risk of opportunistic infections
C. They shorten the period of communicability of the disease.
D. They are able to bring about a cure of the disease condition.
25. A barangay had an outbreak of German measles. To prevent congenital
rubella, what is the BEST advice that you can give to women in the first
trimester of pregnancy in the barangay?

A. Advice them on the signs of German measles.


B. Avoid crowded places, such as markets and moviehouses.
C. Consult at the health center where rubella vaccine may be given.
D. Consult a physician who may give them rubella immunoglobulin.
26. You were invited to be the resource person in a training class for food
handlers. Which of the following would you emphasize regarding prevention
of staphylococcal food poisoning?

A. All cooking and eating utensils must be thoroughly washed.


B. Food must be cooked properly to destroy staphylococcal
microorganisms.
C. Food handlers and food servers must have a negative stool examination
result.
D. Proper handwashing during food preparation is the best way of
preventing the condition.
27. In a mothers’ class, you discussed childhood diseases such as chicken
pox. Which of the following statements about chicken pox is correct?

A. The older one gets, the more susceptible he becomes to the


complications of chicken pox.
B. A single attack of chicken pox will prevent future episodes, including
conditions such as shingles.
C. To prevent an outbreak in the community, quarantine may be imposed
by health authorities.
D. Chicken pox vaccine is best given when there is an impending outbreak
in the community.
28. Complications to infectious parotitis (mumps) may be serious in which
type of clients?

A. Pregnant women
B. Elderly clients
C. Young adult males
D. Young infants
Answers and Rationales
1. Answer: (B) Liver cirrhosis. The etiologic agent of schistosomiasis in the
Philippines is Schistosoma japonicum, which affects the small intestine
and the liver. Liver damage is a consequence of fibrotic reactions to
schistosoma eggs in the liver.
2. Answer: (C) Proper use of sanitary toilets. The ova of the parasite get out of
the human body together with feces. Cutting the cycle at this stage is the
most effective way of preventing the spread of the disease to susceptible
hosts.
3.  Answer: (B) II. A communal faucet or water standpost is classified as
Level II.
4. Answer: (A) Use of sterile syringes and needles. Hepatitis A is transmitted
through the fecal oral route. Hepatitis B is transmitted through infected
body secretions like blood and semen.
5.  Answer: (A) DPT. DPT is sensitive to freezing. The appropriate storage
temperature of DPT is 2 to 8° C only. OPV and measles vaccine are highly
sensitive to heat and require freezing. MMR is not an immunization in the
Expanded Program on Immunization.
6. Answer: (A) 45. To estimate the number of infants, multiply total
population by 3%.
7. Answer: (B) Severe dehydration. The order of priority in the management
of severe dehydration is as follows: intravenous fluid therapy, referral to a
facility where IV fluids can be initiated within 30 minutes,
Oresol/nasogastric tube, Oresol/orem. When the foregoing measures are
not possible or effective, tehn urgent referral to the hospital is done.
8. Answer: (A) 3. Adequate blood supply to the area allows the return of the
color of the nailbed within 3 seconds.
9. Answer: (B) Instruct the mother to give the child Oresol. Since the child does
not manifest any other danger sign, maintenance of fluid balance and
replacement of fluid loss may be done by giving the client Oresol.
10. Answer: (B) Buccal mucosa. Koplik’s spot may be seen on the mucosa of
the mouth or the throat.
11. Answer: (D) Measles. Viral conjunctivitis is transmitted by direct or
indirect contact with discharges from infected eyes. Acute poliomyelitis is
spread through the fecal-oral route and contact with throat secretions,
whereas diphtheria is through direct and indirect contact with respiratory
secretions.
12. Answer: (A) Hemophilus influenzae. Hemophilus meningitis is unusual
over the age of 5 years. In developing countries, the peak incidence is in
children less than 6 months of age. Morbillivirus is the etiology of
measles. Streptococcus pneumoniae and Neisseria meningitidis may
cause meningitis, but age distribution is not specific in young children.
13. Answer: (D) Zooprophylaxis. Zooprophylaxis is done by putting animals
like cattle or dogs close to windows or doorways just before nightfall. The
Anopheles mosquito takes his blood meal from the animal and goes back
to its breeding place, thereby preventing infection of humans.
14. Answer: (A) Stream seeding. Stream seeding is done by putting tilapia fry
in streams or other bodies of water identified as breeding places of the
Anopheles mosquito
15. Answer: (C) Destruction of breeding places of the mosquito vector. Anopheles
mosquitoes breed in slow-moving, clear water, such as mountain streams.
16. Answer: (B) Cholera. Passage of profuse watery stools is the major
symptom of cholera. Both amebic and bacillary dysentery are
characterized by the presence of blood and/or mucus in the stools.
Giardiasis is characterized by fat malabsorption and, therefore,
steatorrhea.
17. Answer: (B) S. japonicum. S. mansoni is found mostly in Africa and South
America; S. haematobium in Africa and the Middle East; and S. malayensis
only in peninsular Malaysia.
18. Answer: (D) Leptospirosis. Leptospirosis is transmitted through contact
with the skin or mucous membrane with water or moist soil contaminated
with urine of infected animals, like rats.
19. Answer: (C) III. Waterworks systems, such as MWSS, are classified as
level III.
20. Answer: (D) Refer the client for a supplementary test, such as Western blot,
since the ELISA result may be false. A client having a reactive ELISA result
must undergo a more specific test, such as Western blot. A negative
supplementary test result means that the ELISA result was false and that,
most probably, the client is not infected.
21. Answer: (A) Being faithful to a single sexual partner. Sexual fidelity rules out
the possibility of getting the disease by sexual contact with another
infected person. Transmission occurs mostly through sexual intercourse
and exposure to blood or tissues.
22. Answer: (B) Infectious mononucleosis. Cytomegalovirus disease is an
acute viral disease characterized by fever, sore throat and
lymphadenopathy.
23. Answer: (A) Contact tracing. Contact tracing is the most practical and
reliable method of finding possible sources of person-to-person
transmitted infections, such as sexually transmitted diseases.
24. Answer: (D) They are able to bring about a cure of the disease
condition. There is no known treatment for AIDS. Antiretroviral agents
reduce the risk of opportunistic infections and prolong life, but does not
cure the underlying immunodeficiency.
25. Answer: (D) Consult a physician who may give them rubella
immunoglobulin. Rubella vaccine is made up of attenuated German
measles viruses. This is contraindicated in pregnancy. Immune globulin, a
specific prophylactic against German measles, may be given to pregnant
women.
26. Answer: (D) Proper handwashing during food preparation is the best way of
preventing the condition. Symptoms of this food poisoning are due to
staphylococcal enterotoxin, not the microorganisms themselves.
Contamination is by food handling by persons with staphylococcal skin or
eye infections.
27. Answer: (A) The older one gets, the more susceptible he becomes to the
complications of chicken pox. Chicken pox is usually more severe in adults
than in children. Complications, such as pneumonia, are higher in
incidence in adults.
28. Answer: (C) Young adult males. Epididymitis and orchitis are possible
complications of mumps. In post-adolescent males, bilateral inflammation
of the testes and epididymis may cause sterility.
PNLE : Medical Surgical Nursing
Exam 1
SITUATION : Arthur, A registered nurse, witnessed an old woman hit by a motorcycle
while crossing a train railway. The old woman fell at the railway. Arthur rushed at
the scene.
1. As a registered nurse, Arthur knew that the first thing that he will do at the
scene is

A. Stay with the person, Encourage her to remain still and Immobilize the
leg while While waiting for the ambulance.
B. Leave the person for a few moments to call for help.
C. Reduce the fracture manually.
D. Move the person to a safer place.
2. Arthur suspects a hip fracture when he noticed that the old woman’s leg is

A. A. Lengthened, Abducted and Internally Rotated.


B. Shortened, Abducted and Externally Rotated.
C. Shortened, Adducted and Internally Rotated.
D. Shortened, Adducted and Externally Rotated.
3. The old woman complains of pain. John noticed that the knee is reddened,
warm to touch and swollen. John interprets that this signs and symptoms are
likely related to

A. Infection
B. Thrombophlebitis
C. Inflammation
D. Degenerative disease
4. The old woman told John that she has osteoporosis; Arthur knew that all of
the following factors would contribute to osteoporosis except

A. Hypothyroidism
B. End stage renal disease
C. Cushing’s Disease
D. Taking Furosemide and Phenytoin.
5. Martha, The old woman was now Immobilized and brought to the
emergency room. The X-ray shows a fractured femur and pelvis. The ER Nurse
would carefully monitor Martha for which of the following sign and
symptoms?

A. Tachycardia and Hypotension


B. Fever and Bradycardia
C. Bradycardia and Hypertension
D. Fever and Hypertension
SITUATION: Mr. D. Rojas, An obese 35 year old MS Professor of OLFU Lagro is
admitted due to pain in his weight bearing joint. The diagnosis was Osteoarthritis.
6. As a nurse, you instructed Mr. Rojas how to use a cane. Mr. Rojas has a
weakness on his right leg due to self immobilization and guarding. You plan to
teach Mr. Rojas to hold the cane

A. On his left hand, because his right side is weak.


B. On his left hand, because of reciprocal motion.
C. On his right hand, to support the right leg.
D. On his right hand, because only his right leg is weak.
7. You also told Mr. Rojas to hold the cane

A. 1 Inches in front of the foot.


B. 3 Inches at the lateral side of the foot.
C. 6 Inches at the lateral side of the foot.
D. 12 Inches at the lateral side of the foot.
8. Mr. Rojas was discharged and 6 months later, he came back to the
emergency room of the hospital because he suffered a mild stroke. The right
side of the brain was affected. At the rehabilitative phase of your nursing care,
you observe Mr. Rojas use a cane and you intervene if you see him

A. Moves the cane when the right leg is moved.


B. Leans on the cane when the right leg swings through.
C. keeps the cane 6 Inches out to the side of the right foot.
D. Holds the cane on the right side.
SITUATION: Alfred, a 40 year old construction worker developed cough, night
sweats and fever. He was brought to the nursing unit for diagnostic studies. He told
the nurse he did not receive a BCG vaccine during childhood
9. The nurse performs a Mantoux Test. The nurse knows that Mantoux Test is
also known as

A. PPD
B. PDP
C. PDD
D. DPP
10. The nurse would inject the solution in what route?

A. IM
B. IV
C. ID
D. SC
11. The nurse notes that a positive result for Alfred is

A. 5 mm wheal
B. 5 mm Induration
C. 10 mm Wheal
D. 10 mm Induration
12. The nurse told Alfred to come back after

A. a week
B. 48 hours
C. 1 day
D. 4 days
13. Mang Alfred returns after the Mantoux Test. The test result read
POSITIVE. What should be the nurse’s next action?

A. Call the Physician


B. Notify the radiology dept. for CXR evaluation
C. Isolate the patient
D. Order for a sputum exam
14. Why is Mantoux test not routinely done in the Philippines?

A. It requires a highly skilled nurse to perform a Mantoux test


B. The sputum culture is the gold standard of PTB Diagnosis and it will
definitively determine the extent of the cavitary lesions
C. Chest X Ray Can diagnose the specific microorganism responsible for
the lesions
D. Almost all Filipinos will test positive for Mantoux Test
15. Mang Alfred is now a new TB patient with an active disease. What is his
category according to the DOH?

A. I
B. II
C. III
D. IV
16. How long is the duration of the maintenance phase of his treatment?

A. 2 months
B. 3 months
C. 4 months
D. 5 months
17. Which of the following drugs is UNLIKELY given to Mang Alfred during the
maintenance phase?

A. Rifampicin
B. Isoniazid
C. Ethambutol
D. Pyridoxine
18. According to the DOH, the most hazardous period for development of
clinical disease is during the first

A. 6-12 months after


B. 3-6 months after
C. 1-2 months after
D. 2-4 weeks after
19. This is the name of the program of the DOH to control TB in the country

A. DOTS
B. National Tuberculosis Control Program
C. Short Coursed Chemotherapy
D. Expanded Program for Immunization
20. Susceptibility for the disease [ TB ] is increased markedly in those with the
following condition except

A. 23 Year old athlete with diabetes insipidus


B. 23 Year old athlete taking long term Decadron therapy and anabolic
steroids
C. 23 Year old athlete taking illegal drugs and abusing substances
D. Undernourished and Underweight individual who undergone
gastrectomy
21. Direct sputum examination and Chest X ray of TB symptomatic is in what
level of prevention?

A. Primary
B. Secondary
C. Tertiary
D. Quarterly
SITUATION: Michiel, A male patient diagnosed with colon cancer was newly put in
colostomy.
22. Michiel shows the BEST adaptation with the new colostomy if he shows
which of the following?

A. Look at the ostomy site


B. Participate with the nurse in his daily ostomy care
C. Ask for leaflets and contact numbers of ostomy support groups
D. Talk about his ostomy openly to the nurse and friends
23. The nurse plans to teach Michiel about colostomy irrigation. As the nurse
prepares the materials needed, which of the following item indicates that the
nurse needs further instruction?

A. Plain NSS / Normal Saline


B. K-Y Jelly
C. Tap water
D. Irrigation sleeve
24. The nurse should insert the colostomy tube for irrigation at approximately

A. 1-2 inches
B. 3-4 inches
C. 6-8 inches
D. 12-18 inches
25. The maximum height of irrigation solution for colostomy is

A. 5 inches
B. 12 inches
C. 18 inches
D. 24 inches
26. Which of the following behavior of the client indicates the best initial step
in learning to care for his colostomy?

A. Ask to defer colostomy care to another individual


B. Promises he will begin to listen the next day
C. Agrees to look at the colostomy
D. States that colostomy care is the function of the nurse while he is in the
hospital
27. While irrigating the client’s colostomy, Michiel suddenly complains of
severe cramping. Initially, the nurse would

A. Stop the irrigation by clamping the tube


B. Slow down the irrigation
C. Tell the client that cramping will subside and is normal
D. Notify the physician
28. The next day, the nurse will assess Michiel’s stoma. The nurse noticed
that a prolapsed stoma is evident if she sees which of the following?

A. A sunken and hidden stoma


B. A dusky and bluish stoma
C. A narrow and flattened stoma
D. Protruding stoma with swollen appearance
29. Michiel asked the nurse, what foods will help lessen the odor of his
colostomy. The nurse best response would be

A. Eat eggs
B. Eat cucumbers
C. Eat beet greens and parsley
D. Eat broccoli and spinach
30. The nurse will start to teach Michiel about the techniques for colostomy
irrigation. Which of the following should be included in the nurse’s teaching
plan?

A. Use 500 ml to 1,000 ml NSS


B. Suspend the irrigant 45 cm above the stoma
C. Insert the cone 4 cm in the stoma
D. If cramping occurs, slow the irrigation
31. The nurse knew that the normal color of Michiel’s stoma should be

A. Brick Red
B. Gray
C. Blue
D. Pale Pink
SITUATION: James, A 27 basketball player sustained inhalation burn that required
him to have tracheostomy due to massive upper airway edema.
32. Wilma, His sister and a nurse is suctioning the tracheostomy tube of
James. Which of the following, if made by Wilma indicates that she is
committing an error?

A. Hyperventilating James with 100% oxygen before and after suctioning


B. Instilling 3 to 5 ml normal saline to loosen up secretion
C. Applying suction during catheter withdrawal
D. Suction the client every hour
33. What size of suction catheter would Wilma use for James, who is 6 feet 5
inches in height and weighing approximately 145 lbs?

A. Fr. 5
B. Fr. 10
C. Fr. 12
D. Fr. 18
34. Wilma is using a portable suction unit at home, What is the amount of
suction required by James using this unit?

A. 2-5 mmHg
B. 5-10 mmHg
C. 10-15 mmHg
D. 20-25 mmHg
35. If a Wall unit is used, What should be the suctioning pressure required by
James?

A. 50-95 mmHg
B. 95-110 mmHg
C. 100-120 mmHg
D. 155-175 mmHg
36. Wilma was shocked to see that the Tracheostomy was dislodged. Both the
inner and outer cannulas was removed and left hanging on James’ neck. What
are the 2 equipment’s at james’ bedside that could help Wilma deal with this
situation?

A. New set of tracheostomy tubes and Oxygen tank


B. Theophylline and Epinephrine
C. Obturator and Kelly clamp
D. Sterile saline dressing
37. Which of the following method if used by Wilma will best assure that the
tracheostomy ties are not too tightly placed?

A. Wilma places 2 fingers between the tie and neck


B. The tracheotomy can be pulled slightly away from the neck
C. James’ neck veins are not engorged
D. Wilma measures the tie from the nose to the tip of the earlobe and to
the xiphoid process.
38. Wilma knew that James have an adequate respiratory condition if she
notices that

A. James’ respiratory rate is 18


B. James’ Oxygen saturation is 91%
C. There are frank blood suction from the tube
D. There are moderate amount of tracheobronchial secretions
39. Wilma knew that the maximum time when suctioning James is

A. 10 seconds
B. 20 seconds
C. 30 seconds
D. 45 seconds
SITUATION : Juan Miguel Lopez Zobel Ayala de Batumbakal was diagnosed with
Acute Close Angle Glaucoma. He is being seen by Nurse Jet.
40. What specific manifestation would nurse Jet see in Acute close angle
glaucoma that she would not see in an open angle glaucoma?

A. Loss of peripheral vision


B. Irreversible vision loss
C. There is an increase in IOP
D. Pain
41. Nurse jet knew that Acute close angle glaucoma is caused by

A. Sudden blockage of the anterior angle by the base of the iris


B. Obstruction in trabecular meshwork
C. Gradual increase of IOP
D. An abrupt rise in IOP from 8 to 15 mmHg
42. Nurse jet performed a TONOMETRY test to Mr. Batumbakal. What does
this test measures

A. It measures the peripheral vision remaining on the client


B. Measures the Intra Ocular Pressure
C. Measures the Client’s Visual Acuity
D. Determines the Tone of the eye in response to the sudden increase in
IOP.
43. The Nurse notices that Mr. Batumbakal cannot anymore determine RED
from BLUE. The nurse knew that which part of the eye is affected by this
change?

A. IRIS
B. PUPIL
C. RODS [RETINA]
D. CONES [RETINA]
44. Nurse Jet knows that Aqueous Humor is produce where?

A. In the sub arachnoid space of the meninges


B. In the Lateral ventricles
C. In the Choroids
D. In the Ciliary Body
45. Nurse Jet knows that the normal IOP is

A. 8-21 mmHg
B. 2-7 mmHg
C. 31-35 mmHg
D. 15-30 mmHg
46. Nurse Jet wants to measure Mr. Batumbakal’s CN II Function. What test
would Nurse Jet implement to measure CN II’s Acuity?

A. Slit lamp
B. Snellen’s Chart
C. Wood’s light
D. Gonioscopy
47. The Doctor orders pilocarpine. Nurse jet knows that the action of this drug
is to

A. Contract the Ciliary muscle


B. Relax the Ciliary muscle
C. Dilate the pupils
D. Decrease production of Aqueous Humor
48. The doctor orders timolol [timoptic]. Nurse jet knows that the action of
this drug is

A. Reduce production of CSF


B. Reduce production of Aquesous Humor
C. Constrict the pupil
D. Relaxes the Ciliary muscle
49. When caring for Mr. Batumbakal, Jet teaches the client to avoid

A. Watching large screen TVs


B. Bending at the waist
C. Reading books
D. Going out in the sun
50. Mr. Batumbakal has undergone eye angiography using an Intravenous dye
and fluoroscopy. What activity is contraindicated immediately after
procedure?
A. Reading newsprint
B. Lying down
C. Watching TV
D. Listening to the music
51. If Mr. Batumbakal is receiving pilocarpine, what drug should always be
available in any case systemic toxicity occurs?

A. Atropine Sulfate
B. Pindolol [Visken]
C. Naloxone Hydrochloride [Narcan]
D. Mesoridazine Besylate [Serentil]
SITUATION : Wide knowledge about the human ear, it’s parts and it’s functions will
help a nurse assess and analyze changes in the adult client’s health.

52. Nurse Anna is doing a caloric testing to his patient, Aida, a 55 year old
university professor who recently went into coma after being mauled by her
disgruntled 3rd year nursing students whom she gave a failing mark. After
instilling a warm water in the ear, Anna noticed a rotary nystagmus towards
the irrigated ear. What does this means?
A. Indicates a CN VIII Dysfunction
B. Abnormal
C. Normal
D. Inconclusive
53. Ear drops are prescribed to an infant, The most appropriate method to
administer the ear drops is

A. Pull the pinna up and back and direct the solution towards the eardrum
B. Pull the pinna down and back and direct the solution onto the wall of the
canal
C. Pull the pinna down and back and direct the solution towards the
eardrum
D. Pull the pinna up and back and direct the solution onto the wall of the
canal
54. Nurse Jenny is developing a plan of care for a patient with Menieres
disease. What is the priority nursing intervention in the plan of care for this
particular patient?
A. Air, Breathing, Circulation
B. Love and Belongingness
C. Food, Diet and Nutrition
D. Safety
55. After mastoidectomy, Nurse John should be aware that the cranial nerve
that is usually damage after this procedure is

A. CN I
B. CN II
C. CN VII
D. CN VI
56. The physician orders the following for the client with Menieres disease.
Which of the following should the nurse question?

A. Dipenhydramine [Benadryl]
B. Atropine sulfate
C. Out of bed activities and ambulation
D. Diazepam [Valium]
57. Nurse Anna is giving dietary instruction to a client with Menieres disease.
Which statement if made by the client indicates that the teaching has been
successful?

A. I will try to eat foods that are low in sodium and limit my fluid intake
B. I must drink atleast 3,000 ml of fluids per day
C. I will try to follow a 50% carbohydrate, 30% fat and 20% protein diet
D. I will not eat turnips, red meat and raddish
58. Peachy was rushed by his father, Steven into the hospital admission.
Peachy is complaining of something buzzing into her ears. Nurse Joemar
assessed peachy and found out It was an insect. What should be the first
thing that Nurse Joemar should try to remove the insect out from peachy’s
ear?

A. Use a flashlight to coax the insect out of peachy’s ear


B. Instill an antibiotic ear drops
C. Irrigate the ear
D. Pick out the insect using a sterile clean forceps
59. Following an ear surgery, which statement if heard by Nurse Oca from the
patient indicates a correct understanding of the post operative instructions?

A. Activities are resumed within 5 days


B. I will make sure that I will clean my hair and face to prevent infection
C. I will use straw for drinking
D. I should avoid air travel for a while
60. Nurse Oca will do a caloric testing to a client who sustained a blunt injury
in the head. He instilled a cold water in the client’s right ear and he noticed
that nystagmus occurred towards the left ear. What does this finding
indicates?

A. Indicating a Cranial Nerve VIII Dysfunction


B. The test should be repeated again because the result is vague
C. This is Grossly abnormal and should be reported to the neurosurgeon
D. This indicates an intact and working vestibular branch of CN VIII
61. A client with Cataract is about to undergo surgery. Nurse Oca is preparing
plan of care. Which of the following nursing diagnosis is most appropriate to
address the long term need of this type of patient?

A. Anxiety R/T to the operation and its outcome


B. Sensory perceptual alteration R/T Lens extraction and replacement
C. Knowledge deficit R/T the pre operative and post operative self care
D. Body Image disturbance R/T the eye packing after surgery
62. Nurse Joseph is performing a WEBERS TEST. He placed the tuning fork in
the patients forehead after tapping it onto his knee. The client states that the
fork is louder in the LEFT EAR. Which of the following is a correct conclusion
for nurse Josph to make?

A. He might have a sensory hearing loss in the left ear


B. Conductive hearing loss is possible in the right ear
C. He might have a sensory hearing loss in the right hear, and/or a
conductive hearing loss in the left ear.
D. He might have a conductive hearing loss in the right ear, and/or a
sensory hearing loss in the left ear.
63. Aling myrna has Menieres disease. What typical dietary prescription would
nurse Oca expect the doctor to prescribe?
A. A low sodium , high fluid intake
B. A high calorie, high protein dietary intake
C. low fat, low sodium and high calorie intake
D. low sodium and restricted fluid intake
SITUATION : [ From DEC 1991 NLE ] A 45 year old male construction worker was
admitted to a tertiary hospital for incessant vomiting. Assessment disclosed: weak
rapid pulse, acute weight loss of .5kg, furrows in his tongue, slow flattening of the
skin was noted when the nurse released her pinch.
Temperature: 35.8 C , BUN Creatinine ratio : 10 : 1, He also complains for postural
hypotension. There was no infection.

64. Which of the following is the appropriate nursing diagnosis?


A. Fluid volume deficit R/T furrow tongue
B. Fluid volume deficit R/T uncontrolled vomiting
C. Dehydration R/T subnormal body temperature
D. Dehydration R/T incessant vomiting
65. Approximately how much fluid is lost in acute weight loss of .5kg?

A. 50 ml
B. 750 ml
C. 500 ml
D. 75 ml
66. Postural Hypotension is

A. A drop in systolic pressure less than 10 mmHg when patient changes


position from lying to sitting.
B. A drop in systolic pressure greater than 10 mmHg when patient
changes position from lying to sitting
C. A drop in diastolic pressure less than 10 mmHg when patient changes
position from lying to sitting
D. A drop in diastolic pressure greater than 10 mmHg when patient
changes position from lying to sitting
67. Which of the following measures will not help correct the patient’s
condition

A. Offer large amount of oral fluid intake to replace fluid lost


B. Give enteral or parenteral fluid
C. Frequent oral care
D. Give small volumes of fluid at frequent interval
68. After nursing intervention, you will expect the patient to have

1. Maintain body temperature at 36.5 C


2. Exhibit return of BP and Pulse to normal
3. Manifest normal skin turgor of skin and tongue
4. Drinks fluids as prescribed
A. 1,3
B. 2,4
C. 1,3,4
D. 2,3,4
SITUATION: A 65 year old woman was admitted for Parkinson’s Disease. The charge
nurse is going to make an initial assessment.
69. Which of the following is a characteristic of a patient with advanced
Parkinson’s disease?

A. Disturbed vision
B. Forgetfulness
C. Mask like facial expression
D. Muscle atrophy
70. The onset of Parkinson’s disease is between 50-60 years old. This
disorder is caused by

A. Injurious chemical substances


B. Hereditary factors
C. Death of brain cells due to old age
D. Impairment of dopamine producing cells in the brain
71. The patient was prescribed with levodopa. What is the action of this drug?

A. Increase dopamine availability


B. Activates dopaminergic receptors in the basal ganglia
C. Decrease acetylcholine availability
D. Release dopamine and other catecholamine from neurological storage
sites
72. You are discussing with the dietician what food to avoid with patients
taking levodopa?
A. Vitamin C rich food
B. Vitamin E rich food
C. Thiamine rich food
D. Vitamin B6 rich food
73. One day, the patient complained of difficulty in walking. Your response
would be

A. You will need a cane for support


B. Walk erect with eyes on horizon
C. I’ll get you a wheelchair
D. Don’t force yourself to walk
SITUATION: Mr. Dela Isla, a client with early Dementia exhibits thought process
disturbances.
74. The nurse will assess a loss of ability in which of the following areas?

A. Balance
B. Judgment
C. Speech
D. Endurance
75. Mr. Dela Isla said he cannot comprehend what the nurse was saying. He
suffers from:

A. Insomnia
B. Aphraxia
C. Agnosia
D. Aphasia
76. The nurse is aware that in communicating with an elderly client, the nurse
will

A. Lean and shout at the ear of the client


B. Open mouth wide while talking to the client
C. Use a low-pitched voice
D. Use a medium-pitched voice
77. As the nurse talks to the daughter of Mr. Dela Isla, which of the following
statement of the daughter will require the nurse to give further teaching?

A. I know the hallucinations are parts of the disease


B. I told her she is wrong and I explained to her what is right
C. I help her do some tasks he cannot do for himself
D. Ill turn off the TV when we go to another room
78. Which of the following is most important discharge teaching for Mr. Dela
Isla

A. Emergency Numbers
B. Drug Compliance
C. Relaxation technique
D. Dietary prescription
SITUATION : Knowledge of the drug PROPANTHELINE BROMIDE [Probanthine]
Is necessary in treatment of various disorders.
79. What is the action of this drug?

A. Increases glandular secretion for clients affected with cystic fibrosis


B. Dissolve blockage of the urinary tract due to obstruction of cystine
stones
C. Reduces secretion of the glandular organ of the body
D. Stimulate peristalsis for treatment of constipation and obstruction
80. What should the nurse caution the client when using this medication

A. Avoid hazardous activities like driving, operating machineries etc.


B. Take the drug on empty stomach
C. Take with a full glass of water in treatment of Ulcerative colitis
D. I must take double dose if I missed the previous dose
81. Which of the following drugs are not compatible when taking Probanthine?

A. Caffeine
B. NSAID
C. Acetaminophen
D. Alcohol
82. What should the nurse tell clients when taking Probanthine?

A. Avoid hot weathers to prevent heat strokes


B. Never swim on a chlorinated pool
C. Make sure you limit your fluid intake to 1L a day
D. Avoid cold weathers to prevent hypothermia
83. Which of the following disease would Probanthine exert the much needed
action for control or treatment of the disorder?

A. Urinary retention
B. Peptic Ulcer Disease
C. Ulcerative Colitis
D. Glaucoma
SITUATION : Mr. Franco, 70 years old, suddenly could not lift his spoons nor speak
at breakfast. He was rushed to the hospital unconscious. His diagnosis was CVA.
84. Which of the following is the most important assessment during the acute
stage of an unconscious patient like Mr. Franco?

A. Level of awareness and response to pain


B. Papillary reflexes and response to sensory stimuli
C. Coherence and sense of hearing
D. Patency of airway and adequacy of respiration
85. Considering Mr. Franco’s conditions, which of the following is most
important to include in preparing Franco’s bedside equipment?

A. Hand bell and extra bed linen


B. Sandbag and trochanter rolls
C. Footboard and splint
D. Suction machine and gloves
86. What is the rationale for giving Mr. Franco frequent mouth care?

A. He will be thirsty considering that he is doesn’t drink enough fluids


B. To remove dried blood when tongue is bitten during a seizure
C. The tactile stimulation during mouth care will hasten return to
consciousness
D. Mouth breathing is used by comatose patient and it’ll cause oral
mucosa dying and cracking.
87. One of the complications of prolonged bed rest is decubitus ulcer. Which
of the following can best prevent its occurrence?

A. Massage reddened areas with lotion or oils


B. Turn frequently every 2 hours
C. Use special water mattress
D. Keep skin clean and dry
88. If Mr. Franco’s Right side is weak, What should be the most accurate
analysis by the nurse?

A. Expressive aphasia is prominent on clients with right sided weakness


B. The affected lobe in the patient is the Right lobe
C. The client will have problems in judging distance and proprioception
D. Clients orientation to time and space will be much affected
SITUATION : a 20 year old college student was rushed to the ER of PGH after he
fainted during their ROTC drill. Complained of severe right iliac pain. Upon
palpation of his abdomen, Ernie jerks even on slight pressure. Blood test was
ordered. Diagnosis is acute appendicitis.

89. Which result of the lab test will be significant to the diagnosis?
A. RBC : 4.5 TO 5 Million / cu. mm.
B. Hgb : 13 to 14 gm/dl.
C. Platelets : 250,000 to 500,000 cu.mm.
D. WBC : 12,000 to 13,000/cu.mm
90. Stat appendectomy was indicated. Pre op care would include all of the
following except?

A. Consent signed by the father


B. Enema STAT
C. Skin prep of the area including the pubis
D. Remove the jewelries
91. Pre-anesthetic med of Demerol and atrophine sulfate were ordered to :

A. Allay anxiety and apprehension


B. Reduce pain
C. Prevent vomiting
D. Relax abdominal muscle
92. Common anesthesia for appendectomy is

A. Spinal
B. General
C. Caudal
D. Hypnosis
93. Post op care for appendectomy include the following except

A. Early ambulation
B. Diet as tolerated after fully conscious
C. Nasogastric tube connect to suction
D. Deep breathing and leg exercise
94. Peritonitis may occur in ruptured appendix and may cause serious
problems which are

1. Hypovolemia, electrolyte imbalance


2. Elevated temperature, weakness and diaphoresis
3. Nausea and vomiting, rigidity of the abdominal wall
4. Pallor and eventually shock
A. 1 and 2
B. 2 and 3
C. 1,2,3
D. All of the above
95. If after surgery the patient’s abdomen becomes distended and no bowel
sounds appreciated, what would be the most suspected complication?

A. Intussusception
B. Paralytic Ileus
C. Hemorrhage
D. Ruptured colon
96. NGT was connected to suction. In caring for the patient with NGT, the
nurse must

A. Irrigate the tube with saline as ordered


B. Use sterile technique in irrigating the tube
C. advance the tube every hour to avoid kinks
D. Offer some ice chips to wet lips
97. When do you think the NGT tube be removed?

A. When patient requests for it


B. Abdomen is soft and patient asks for water
C. Abdomen is soft and flatus has been expelled
D. B and C only
Situation: Amanda is suffering from chronic arteriosclerosis Brain syndrome she fell
while getting out of the bed one morning and was brought to the hospital, and she
was diagnosed to have cerebrovascular thrombosis thus transferred to a nursing
home.
98. What do you call a STROKE that manifests a bizarre behavior?

A. Inorganic Stroke
B. Inorganic Psychoses
C. Organic Stroke
D. Organic Psychoses
99. The main difference between chronic and organic brain syndrome is that
the former

A. Occurs suddenly and reversible


B. Is progressive and reversible
C. tends to be progressive and irreversible
D. Occurs suddenly and irreversible
100. Which behavior results from organic psychoses?

A. Memory deficit
B. Disorientation
C. Impaired Judgement
D. Inappropriate affect
Answers
1. D. Move the person to a safer place.
2. D. Shortened, Adducted and Externally Rotated.
3. C. Inflammation
4. A. Hypothyroidism
5. A. Tachycardia and Hypotension
6. B. On his left hand, because of reciprocal motion.
7. c. 6 Inches at the lateral side of the foot.
8. A. Moves the cane when the right leg is moved.
9. A. PPD
10. C. ID
11. D. 10 mm Induration
12. B. 48 hours
13. A. Call the Physician
14. D. Almost all Filipinos will test positive for Mantoux Test
15. A. I
16. C. 4 months
17. C. Ethambutol
18. A. 6-12 months after
19. B. National Tuberculosis Control Program
20. A. 23 Year old athlete with diabetes insipidus
21. B. Secondary
22. B. Participate with the nurse in his daily ostomy care
23. A. Plain NSS / Normal Saline
24. B. 3-4 inches
25. C. 18 inches
26. C. Agrees to look at the colostomy
27. A. Stop the irrigation by clamping the tube
28. D. Protruding stoma with swollen appearance
29. C. Eat beet greens and parsley
30. B. Suspend the irrigant 45 cm above the stoma
31. A. Brick Red
32. D. Suction the client every hour
33. D. Fr. 18
34. C. 10-15 mmHg
35. C. 100-120 mmHg
36. C. Obturator and Kelly clamp
37. A. Wilma places 2 fingers between the tie and neck
38. A. James’ respiratory rate is 18
39. A. 10 seconds
40. D. Pain
41. A. Sudden blockage of the anterior angle by the base of the iris
42. B. Measures the Intra Ocular Pressure
43. D. CONES [RETINA]
44. D. In the Ciliary Body
45. A. 8-21 mmHg
46. B. Snellen’s Chart
47. A. Contract the Ciliary muscle
48. B. Reduce production of Aquesous Humor
49. B. Bending at the waist
50. A. Reading newsprint
51. A. Atropine Sulfate
52. C. Normal
53. B. Pull the pinna down and back and direct the solution onto the wall of the
canal
54. D. Safety
55. C. CN VII
56. C. Out of bed activities and ambulation
57. A. I will try to eat foods that are low in sodium and limit my fluid intake
58. A. Use a flashlight to coax the insect out of peachy’s ear
59. D. I should avoid air travel for a while
60. D. This indicates an intact and working vestibular branch of CN VIII
61. B. Sensory perceptual alteration R/T Lens extraction and replacement
62. C. He might have a sensory hearing loss in the right hear, and/or a conductive
hearing loss in the left ear.
63. D. low sodium and restricted fluid intake
64. B. Fluid volume deficit R/T uncontrolled vomiting
65. C. 500 ml
66. B. A drop in systolic pressure greater than 10 mmHg when patient changes
position from lying to sitting
67. A. Offer large amount of oral fluid intake to replace fluid lost
68. D. 2,3,4
69. C. Mask like facial expression
70. D. Impairment of dopamine producing cells in the brain
71. A. Increase dopamine availability
72. D. Vitamin B6 rich food
73. A. You will need a cane for support
74. B. Judgment
75. D. Aphasia
76. D. Use a medium-pitched voice
77. B. I told her she is wrong and I explained to her what is right
78. B. Drug Compliance
79. C. Reduces secretion of the glandular organ of the body
80. A. Avoid hazardous activities like driving, operating machineries etc.
81. D. Alcohol
82. A. Avoid hot weathers to prevent heat strokes
83. B. Peptic Ulcer Disease
84. D. Patency of airway and adequacy of respiration
85. D. Suction machine and gloves
86. D. Mouth breathing is used by comatose patient and it’ll cause oral mucosa
dying and cracking.
87. B. Turn frequently every 2 hours
88. A. Expressive aphasia is prominent on clients with right sided weakness
89. D. WBC : 12,000 to 13,000/cu.mm
90. B. Enema STAT
91. A. Allay anxiety and apprehension
92. A. Spinal
93. B. Diet as tolerated after fully conscious
94. D. All of the above
95. B. Paralytic Ileus
96. A. Irrigate the tube with saline as ordered
97. C. Abdomen is soft and flatus has been expelled
98. D. Organic Psychoses
99. C. tends to be progressive and irreversible
100. B. Disorientation

You might also like